Download as docx, pdf, or txt
Download as docx, pdf, or txt
You are on page 1of 229

1. E.G & I.

CONSTRUCTION
CORPORATION v. ANANIAS P.
SATO
G.R. No. 182070, February
16,2011
Topic: Illegal Dismissal
FACTS:
Sato was hired by E.G. &
I. Construction Corporation for
more than 13 years as a grader
operator. When Sato discovered
that E.G. & I had not been
remitting
his
premium
contributions to SSS, he kept on
telling E.G. & I to update his
premium contributions. Because
of this, he was removed as a
grader operator and made to
perform manual labor, such as
tilling the land in a private
cemetery or digging earthworks
in E.G. & Is construction
projects. E.G. & I told Sato to
look for employment in other
construction companies. Sato,
however, found difficulty in
finding a job because he had
been
blacklisted
in
other
construction companies and was
prevented from entering the
project sites of E.G. & I.
Respondent
filed
a
complaint with the Regional
Arbitration Branch of Cebu City
for illegal dismissal. The Labor
Arbiter rendered a decision
finding that respondent was
illegally
dismissed
from
employment.
On appeal, the
NLRC reversed the ruling of the
Labor Arbiter ratiocinating that,
other than respondents bare
allegation
that
he
was
dismissed, he failed to present a
2011- 2012 Labor Law Case Digest Compilations
UC LLB III-B
Page 1

written notice of dismissal.


Respondent filed a motion for
reconsideration. On October 24,
2007, the CA rendered a
Decision reinstating the decision
of the labor arbiter. The CA ruled
that respondent was illegally
dismissed. A written notice of
dismissal is not a pre-requisite
for a finding of illegal dismissal.
ISSUE:
Whether the CA erred in
declaring that respondents were
illegally
terminated
from
employment by E.G. & I, and
that respondents are entitled to
their monetary claims.
RULING:
We sustain the ruling of
the CA. E.G. & I failed to prove
that respondent was dismissed
for just or authorized cause. In
an illegal dismissal case, the
onus probandi rests on the
employer to prove that the
dismissal of an employee is for a
valid cause. For abandonment to
exist, it is essential (a) that the
employee must have failed to
report for work or must have
been absent without valid or
justifiable reason; and (b) that
there must have been a clear
intention to sever the employeremployee
relationship
manifested by some overt acts.
Mere absence is not sufficient.
There must be an unequivocal
intent on the part of the
employee to discontinue his
employment.
Respondents
filing of the case for illegal
dismissal barely four (4) days

from his alleged abandonment is


totally inconsistent with our
known
concept
of
what
constitutes abandonment.
2. NELSON CULILI
VS.EASTERN
TELECOMMUNICATIONS
PHILS.INC.
GR. NO. 165381, February
9,2011
Topic:
Illegal
Dismissal;
Unfair Labor Practices
Facts:
Eastern
Telecommunications Philippines,
Inc.
(ETPI)
is
a
telecommunications
company
engaged mainly in the business
of
establishing
commercial
telecommunications
systems
and leasing of international
datalines or circuits that pass
through
the
international
gateway facility (IGF). Culili was
employed
by
ETPI
as
a
Technician in its Field Operations
Department on January 27,
1981. On December 12, 1996,
Culili was promoted to Senior
Technician in the Customer
Premises
Equipment
Management Unit of the Service
Quality Department and his
basic salary was increased. ETPI
was required, under Republic
Act. No. 7925 and Executive
Order No. 109, to establish
landlines in Metro Manila and
certain provinces. However, due
to
interconnection
problems
with
the
Philippine
Long
Distance Telephone Company
2011- 2012 Labor Law Case Digest Compilations
UC LLB III-B
Page 2

(PLDT), poor subscription and


cancellation of subscriptions,
and other business difficulties,
ETPI was forced to halt its roll
out of one hundred twenty-nine
thousand (129,000) landlines
already allocated to a number of
its
employees.
ETPI
was
compelled to implement a RightSizing Program which consisted
of two phases: the first phase
involved the reduction of ETPIs
workforce
to
only
those
employees that were necessary
and which ETPI could sustain;
the second phase entailed a
company-wide
reorganization
which would result in the
transfer, merger, absorption or
abolition of certain departments
of ETPI. ETPI, on December 10,
1998, offered to its employees
who had rendered at least
fifteen years of service, the
Special Retirement Program,
which consisted of the option to
voluntarily retire at an earlier
age and a retirement package
equivalent to two and a half
(2) months salary for every
year of service. This offer was
initially rejected by the Eastern
Telecommunications Employees
Union
(ETEU),
ETPIs
duly
recognized bargaining agent,
which threatened to stage a
strike. ETPI explained to ETEU
the exact details of the RightSizing Program and the Special
Retirement Program and after
consultations
with
ETEUs
members, ETEU agreed to the
implementation
of
both
programs. On February 8, 1999,
ETPI re-offered the Special

Retirement Program and the


corresponding
retirement
package to the 102 employees
who qualified for the program.
Of all the employees who
qualified to avail of the program,
only Culili rejected the offer.
After
the
successful
implementation of the first
phase
of
the
Right-Sizing
Program, ETPI, on March 1, 1999
proceeded with the second
phase which necessitated the
abolition, transfer and merger of
a number of ETPIs departments.
Among
the
departments
abolished was the Service
Quality
Department.
The
functions of the Customer
Premises
Equipment
Management Unit, Culilis unit,
were absorbed by the Business
and
Consumer
Accounts
Department. The abolition of the
Service
Quality
Department
rendered
the
specialized
functions of a Senior Technician
unnecessary. As a result, Culilis
position was abolished due to
redundancy and his functions
were
absorbed
by
Andre
Andrada,
another
employee
already with the Business and
Consumer
Accounts
Department. In a letter dated
March 8, 1999, ETPI, through its
Assistant Vice President Stella
Garcia, informed Culili of his
termination from employment
effective April 8, 1999.
Culili alleged that neither
he nor the DOLE were formally
notified of his termination. Culili
asserted
that
ETPI
had
contracted out the services he
2011- 2012 Labor Law Case Digest Compilations
UC LLB III-B
Page 3

used to perform to a labor-only


contractor
which
not
only
proved that his functions had
not become unnecessary, but
which
also
violated
their
Collective Bargaining Agreement
(CBA) and the Labor Code. Culili
also alleged that he was
discriminated against when ETPI
offered
some
of
his
coemployees an additional benefit
in the form of motorcycles to
induce them to avail of the
Special Retirement Program,
while he was not. ETPI averred
that since Culili did not avail of
the Special Retirement Program
and
his
position
was
subsequently
declared
redundant, it had no choice but
to terminate Culili. On February
8, 2000, Culili filed a complaint
against ETPI and its officers for
illegal dismissal, unfair labor
practice, and money claims
before the Labor Arbiter. Labor
Arbiter rendered a decision
finding ETPI guilty of illegal
dismissal
and
unfair
labor
practice. On appeal, the NLRC
affirmed the Labor Arbiters
decision
but
modified
the
amount of moral and exemplary
damages awarded. ETPI filed a
Petition for Certiorari under Rule
65 of the Rules of Civil
Procedure before the Court of
Appeals on the ground of grave
abuse of discretion. ETPI prayed
that a Temporary Restraining
Order be issued against the
NLRC from implementing its
decision and that the NLRC
decision and resolution be set
aside.
The Court of Appeals

found that Culilis position was


validly
abolished
due
to
redundancy.
The
Court
of
Appeals said that ETPI had been
very candid with its employees
in implementing its Right-Sizing
Program, and that it was highly
unlikely that ETPI would effect a
company-wide
reorganization
simply for the purpose of getting
rid of Culili. The Court of
Appeals also held that ETPI
cannot be held guilty of unfair
labor
practice
as
mere
contracting out of services being
performed by union members
does not per se amount to
unfair labor practice unless it
interferes with the employees
right to self-organization. Culili
went to the Supreme Court.
Issues:
1. Whether the dismissal of
Culili was illegal.
2. Whether ETPI committed
unfair labor practice.
Ruling:
First Issue. There was an
illegal
dismissal.
The
determination of whether or not
an employees services are still
needed or sustainable properly
belongs
to
the
employer.
Provided there is no violation of
law or a showing that the
employer was prompted by an
arbitrary or malicious act, the
soundness or wisdom of this
exercise of business judgment is
not subject to the discretionary
review of the Labor Arbiter and
the NLRC. An employer cannot
simply declare that it has
2011- 2012 Labor Law Case Digest Compilations
UC LLB III-B
Page 4

become
overmanned
and
dismiss its employees without
producing adequate proof to
sustain
its
claim
of
redundancy. Among
the
requisites of a valid redundancy
program are: (1) the good faith
of the employer in abolishing
the redundant position; and (2)
fair and reasonable criteria in
ascertaining what positions are
to be declared redundant, such
as but not limited to: preferred
status, efficiency, and seniority.
ETPI was upfront with its
employees about its plan to
implement
a
Right-Sizing
Program. Even in the face of
initial opposition from and
rejection of the said program by
ETEU, ETPI patiently negotiated
with ETEUs officers to make
them
understand
ETPIs
business dilemma and its need
to reduce its workforce and
streamline its organization. This
evidently rules out bad faith on
the part of ETPI.
Second Issue. There was
no unfair labor practice. An
employer may only be held
liable for unfair labor practice if
it can be shown that his acts
affect in whatever manner the
right of his employees to selforganize. There is no showing
that ETPI, in implementing its
Right-Sizing
Program,
was
motivated by ill will, bad faith or
malice, or that it was aimed at
interfering with its employees
right to self-organize. In fact,
ETPI negotiated and consulted

with ETEU before implementing


its Right-Sizing Program.
3. Prince Transport vs.
Garcia
G.R. No. 167291, January 12,
2011
Topic: Piercing the Corporate
Veil; Unfair Labor Practice
Facts:
Garcia
et.
al.
were
employees of Prince Transport
Inc. (PTI),in addition to their
regular
monthly
income,
respondents
also
received
commissions equivalent to 8 to
10% of their wages; sometime
in October 1997, the said
commissions were reduced to 7
to 9%; this led Garcia and other
employees of PTI to hold a
series of meetings to discuss the
protection of their interests as
employees; these meetings led
petitioner Renato Claros, who is
the president of PTI, to suspect
that respondents are about to
form a union; he made known to
Garcia his objection to the
formation of a union; in order to
block the continued formation of
the union, PTI caused the
transfer of all union members
and sympathizers to one of its
sub-companies, Lubas Transport
(Lubas); despite such transfer,
the schedule of drivers and
conductors, as well as their
2011- 2012 Labor Law Case Digest Compilations
UC LLB III-B
Page 5

company identification cards,


were issued by PTI; the daily
time records, tickets and reports
of the respondents were also
filed at the PTI office; and, all
claims
for
salaries
were
transacted at the same office;
later, the business of Lubas
deteriorated because of the
refusal of PTI to maintain and
repair the units being used
therein, which resulted in the
virtual
stoppage
of
its
operations and respondents'
loss of employment. Garcia
et,al. then filed complaints
against PTI for illegal dismissal,
unfair labor practice. Labor
Arbiter ruled that PTI are not
guilty of unfair labor practice.
The Labor Arbiter also held that
Lubas
is
the
respondents
employer and that it (Lubas) is
an entity which is separate,
distinct and independent from
PTI so Lubas is guilty of illegally
dismissing respondents from
their
employment.
NLRC
partially modified the decision
on but when it reached the CA it
was reversed saying that there
was unfair labor practices then
it went to SC.
Issue:
1. Whether the doctrine
of piercing the corporate veil is
applicable in this case.

2. Whether there is unfair


labor practice.
Ruling:
First
Issue.It
is
not
applicable, Lubas is a mere
agent, conduit or adjunct of PTI.
A settled formulation of the
doctrine
of
piercing
the
corporate veil is that when two
business enterprises are owned,
conducted and controlled by the
same parties, both law and
equity will, when necessary to
protect the rights of third
parties, disregard the legal
fiction that these two entities
are distinct and treat them as
identical or as one and the
same.In the present case, it may
be true that Lubas is a single
proprietorship
and
not
a
corporation.
However,
petitioners attempt to isolate
themselves from and hide
behind the supposed separate
and distinct personality of Lubas
so as to evade their liabilities is
precisely what the classical
doctrine of piercing the veil of
corporate
entity
seeks
to
prevent and remedy.
Second Issue. There is
unfair
labor
practice,
respondents transfer of work
assignments to Lubas was
designed by PTI as a subterfuge
to foil the formers right to
2011- 2012 Labor Law Case Digest Compilations
UC LLB III-B
Page 6

organize themselves into a


union. Under Article 248 (a) and
(e) of the Labor Code, an
employer is guilty of unfair labor
practice if it interferes with,
restrains
or
coerces
its
employees in the exercise of
their right to self-organization or
if it discriminates in regard to
wages, hours of work and other
terms
and
conditions
of
employment
in
order
to
encourage
or
discourage
membership
in
any
labor
organization.
4. ISLRIZ TRADING VS.
CAPADA ET,AL.
GR. NO. 168501, January 31,
2011
Topic: Execution of Judgment
Ordering
Payment
of
Backwages Pending Appeal
Facts:
Capadaet,al.
were
employees of Islriz Trading, a
gravel and sand business owned
and operated by petitioner
Victor Hugo Lu. Claiming that
they were illegally dismissed,
Capadaet,al.
filed
a
Complaint for illegal dismissal
and non-payment of overtime
pay, holiday pay, rest day pay,
allowances and separation pay
against petitioner on August 9,
2000
before
the
Labor
Arbiter. On his part, petitioner
imputed abandonment of work

against
respondents. Labor
Arbiter
ruled
that
ISLRIZ
TRADING is guilty of illegal
dismissal, ordered to reinstate
complainants. On appeal, NLRC
reversed the decision of the
Labor Arbiter saying that there
was no illegal dismissal and
ordered the reinstatement of the
employees without backwages.
It was appealed to CA until it
reached the SC.
Issue:
Whether Garcia et,al. may
collect their wages during the
period
between
the
Labor
Arbiters order of reinstatement
pending appeal and the NLRC
Resolution overturning that of
the Labor Arbiter.
Ruling:
Garcia et,al. have the right
to collect their accrued salaries
during the period between the
Labor
Arbiters
Decision
ordering their reinstatement
pending appeal and the NLRC
Resolution overturning the same
because petitioners failure to
reinstate them either actually or
through payroll was due to
petitioners unjustified refusal to
effect
reinstatement.In
any
event, the decision of the Labor
Arbiter reinstating a dismissed
or separated employee, insofar
2011- 2012 Labor Law Case Digest Compilations
UC LLB III-B
Page 7

as the reinstatement aspect is


concerned, shall immediately be
executory, pending appeal. Even
if the order of reinstatement of
the Labor Arbiter is reversed on
appeal, it is obligatory on the
part of the employer to reinstate
and pay the wages of the
dismissed employee during the
period of appeal until reversal
by the higher court . In order to
enforce this, Labor Arbiter
Castillon thus correctly issued
the Writ of Execution dated
March 9, 2004 as well as the
Order dated June 3, 2004
denying petitioners Motion to
Quash Writ of Execution and
granting
respondents Urgent
Motion for Issuance of BreakOpen Order. Consequently, we
find no error on the part of the
CA in upholding these issuances
and in dismissing the petition
for certiorari before it.
5. LEGEND INTERNATIONAL
RESORTS LIMITED vs.
KILUSANG MANGGAGAWA NG
LEGENDA
G.R. No. 169754
February 23, 2011
Topic:
Registration and
Cancellation of Certificate of
Registration of Legitimate
Labor
Organizations;
Prospective
Effect
of
Cancellation of Registration
of
a
Legitimate
Labor
Organization

Facts:
KML filed with the MedArbitration Unit of the DOLE, a
Petition
for
Certification
Election. KML alleged that it is a
legitimate labor organization of
the rank and file employees of
Legend International Resorts
Limited. KML claimed that it
was issued its Certificate of
Registration
by
the
DOLE.
LEGEND moved to dismiss the
petition alleging that KML is not
a legitimate labor organization
because its membership is a
mixture of rank and file and
supervisory
employees
in
violation of Article 245 of the
Labor Code.
LEGEND also
claimed that KML committed
acts
of
fraud
and
misrepresentation when it made
it appear that certain employees
attended
its
general
membership meeting on April 5,
2001 when in reality some of
them were either at work; have
already resigned as; or were
abroad.
In its Comment, KML
argued that even if 41 of its
members
are
indeed
supervisory
employees
and
therefore excluded from its
membership, the certification
election could still proceed
because the required number of
the
total
rank
and
file
employees
necessary
for
certification purposes is still
sustained.
KML also claimed
that its legitimacy as a labor
union could not be collaterally
attacked in the certification
election proceedings but only
2011- 2012 Labor Law Case Digest Compilations
UC LLB III-B
Page 8

through
a
separate
and
independent
action
for
cancellation
of
union
registration. Finally, as to the
alleged
acts
of
misrepresentation, KML asserted
that
LEGEND
failed
to
substantiate its claim.
The Med-Arbiter rendered
judgment
dismissing
the
petition for certification election.
KML thus appealed to the DOLE.
DOLE held that KMLs legitimacy
as a union could not be
collaterally
attacked,
citing
Section 5, Rule V of D.O No. 9,
series of 1997. DOLE also
opined that Article 245 of the
Labor Code merely provides for
the prohibition on managerial
employees to form or join a
union and the ineligibility of
supervisors to join the union of
the rank and file employees and
vice versa. LEGEND filed its
Motion
for
Reconsideration.
DOLE denied LEGENDs motion
for reconsideration.
LEGEND
filed a Petition for Certiorari with
the Court of Appeals. The CA
rendered its decision finding no
grave abuse of discretion on the
part of DOLE.
The appellate
court held that the issue on the
legitimacy of KML as a labor
organization has already been
settled with finality Thus, having
already been settled that KML is
a legitimate labor organization,
the latter could properly file a
petition for certification election.
LEGEND filed a Motion for
Reconsideration. The appellate
court denied LEGENDs motion
for reconsideration.

ISSUES:
1. Whether or not the CA
grievously erred in ruling that
KMLs registration has already
become final and executory.
2. Whether or not KMLs
certificate of registration should
retroact to the time of its
issuance.
Ruling:
The petition is partly
meritorious.
First Issue. LEGEND has
timely appealed the March 26,
2002 Decision of the Bureau of
Labor Relations to the Court of
Appeals.
Records show that
LEGEND has timely filed a
petition for certiorari before the
CA.
In fact, KML received a
copy of said petition and has
filed its Comment.
Second
Issue.
The
cancellation of KMLs certificate
of
registration
should
not
retroact to the time of its
issuance. LEGEND claims that
KMLs petition for certification
election
filed
during
the
pendency of the petition for
cancellation and its demand to
enter into collective bargaining
agreement with LEGEND should
be dismissed due to KMLs lack
of
legal
personality.
A
certification election may be
conducted during the pendency
of the cancellation proceedings.
This is because at the time the
petition for certification was
filed, the petitioning union is
presumed to possess the legal
personality to file the same.
The legitimacy of the legal
2011- 2012 Labor Law Case Digest Compilations
UC LLB III-B
Page 9

personality of KML cannot be


collaterally attacked in a petition
for certification election. This is
in consonance with our ruling in
Laguna Autoparts Manufacturing
Corporation v. Office of the
Secretary, Department of Labor
and Employment that such
legal personality may not be
subject to a collateral attack but
only through a separate action
instituted particularly for the
purpose of assailing it. This is
categorically
prescribed
by
Section 5, Rule V of the
Implementing Rules of Book V,
which states as follows: SEC. 5.
Effect of registration. The
labor organization or workers
association shall be deemed
registered and vested with legal
personality on the date of
issuance of its certificate of
registration.
Such
legal
personality cannot thereafter be
subject to collateral attack but
may be questioned only in an
independent
petition
for
cancellation in accordance with
these Rules.
Equally important is Section 11,
Paragraph II, Rule IX of D.O. 9,
which provides for the dismissal
of a petition for certification
election based on the lack of
legal personality of a labor
organization
only
in
the
following
instances:
(1)
appellant is not listed by the
Regional Office or the BLR in its
registry of legitimate labor
organizations; or (2) appellants
legal personality has been
revoked
or
cancelled
with
finality. Since appellant is listed

in the registry of legitimate


labor organizations, and its
legitimacy has not been revoked
or cancelled with finality, the
granting of its petition for
certification election is proper.
6. Atlanta Industries v.
Sebolino,
GR No. 187320
26 January 2011
Topic: Apprenticeship; Illegal
Dismissal
Facts:
Sebolino,
Costales,
Almoite, and Sagun, along with
several others, filed complaints
for
illegal
dismissal,
regularization,
underpayment,
nonpayment of wages, and
other money claims against
Altanta Industries, Inc., which is
a domestic corporation engaged
in the manufacture of steel
pipes. The complainants alleged
that they were already regular
employees of Atlanta even
before they entered into two
separate,
subsequent
apprenticeship agreements with
the said company. The first
apprenticeship agreement was
for a period of 5 months, after
the expiration of which they
entered
into
a
second
apprenticeship agreement with
Atlanta for the training of a
second skill. Now, they claim to
have been illegally dismissed
when
the
apprenticeship
agreements expired.In defense,
Atlanta and Chan argued that
the workers were not entitled to
2011- 2012 Labor Law Case Digest Compilations
UC LLB III-B
Page 10

regularization and to their


money claims because they
were engaged as apprentices
under a government-approved
apprenticeship program. The
company offered to hire them as
regular employees in the event
vacancies for regular positions
occur in the section of the plant
where they had trained. They
also claimed that their names
did not appear in the list of
employees
prior
to
their
engagement as apprentices The
Labor Arbiter ruled that the
dismissal was indeed illegal.
While the case was on appeal
with the NLRC, Costales and
Almoite, together with other
workers allegedly entered into a
compromise agreement wherein
the workers were to be paid by
Atlanta a specified amount as
settlement and to acknowledge
them as regular employees. The
NLRC
approved
the
said
compromise
agreement,
withdrew the finding of illegal
dismissal
with
respect
to
Sebolino and Sagun, and denied
all the other claims. The CA
overturned the NLRC decision
and found that the complainants
were already employees of
Atlanta
even
before
they
entered into the apprenticeship
agreements; that the said
apprenticeship agreements were
executed in violation of the law
and the rules; that the positions
occupied by the complainants
are usually necessary and
desirable in the companys main
business;
and
that
the
compromise agreement entered

into by Costales and Almoite


were not binding because they
did not sign the agreement.
Issues:
1.Whether or not a second
apprenticeship agreement is
valid.
2.
Whether
or
not
complainants
were
illegally
dismissed.
Ruling:
First Issue. Even if the
companys need to train its
employees
through
apprenticeship is recognized,
only the first apprenticeship
agreement may be considered
for the purpose. With the
expiration of the first agreement
and the retention of the
employees, Atlanta had, to all
intents
and
purposes,
recognized the completion of
their
training
and
their
acquisition
of
a
regular
employee status. To foist upon
them the second apprenticeship
agreement for a second skill
which was not even mentioned
in the agreement itself, is a
violation of the Labor Codes
rules and regulations and is an
act manifestly unfair to the
employees.
Second Issue. The fact
that the complainants were
already rendering service to the
company when they were made
to
undergo
apprenticeship
renders
the
apprenticeship
agreements
irrelevant.
This
reality is highlighted by the
2011- 2012 Labor Law Case Digest Compilations
UC LLB III-B
Page 11

finding that the complainants


occupied positions that are
usually necessary and desirable
in Atlantas usual business or
trade, which characterize them
as regular employees under
Article 280 of the Labor Code.
Thus, when they were dismissed
without just or authorized cause,
without notice, and without the
opportunity to be heard, their
dismissal was illegal under the
law.
7. PRIMO E. CAONG, JR.,
ALEXANDER J. TRESQUIO,
and LORIANO D. DALUYON,
Petitioners v. AVELINO
REGUALOS, Respondent.
G.R. No. 179428 January 26,
2011
Topic: Jurisdiction of Labor
Arbiter;
Grounds
for
Appealling Decision of Labor
Arbiter; Illegal Dismissal
Facts:
Caong, Tresquio and Daluyon
were employed as jeepney
drivers in boundary sytem by
Regualos. The said petitioners
remitted deficient boundary.
Consequently, Regualos called a
meeting
to
inform
his
employees to strictly comply
with
the
policy
regarding
remittances and warned them
that they would not be allowed
to take out the jeepneys, which
are subject yet to amortization,
if they did not remit the full
amount
of
the
boundary.
Despite
the
warning,
the
petitioners remittances were
again short of the required

boundary fee on the immediate


following days. Thus, Regualos
barred them from driving his
jeepneys. Hence, the petitioners
filed an illegal dismissal case
with the Labor Arbiter which
decision found for Regualos. The
same was affirmed by the NLRC
and the CA.
Issues:
1. Whether the case is cognizable
by the Labor Arbiter.
2. Whether there is a grave abuse
of discretion to warrant appeal.
3. Whether the petitioners were
illegally dismissed.
Held:
First
Issue.the
Labor
Arbiter has jurisdiction over the
case. It is already settled that
the
relationship
between
jeepney owners/operators and
jeepney drivers under the
boundary system is that of
employer-employee and not of
lessor-lessee. The fact that the
drivers do not receive fixed
wages but only get the amount
in excess of the so-called
boundary that they pay to the
owner is not sufficient to negate
the relationship between them
as employer and employee.
Second Issue.Mere abuse
of discretion is not enough to
warrant
certiorari.
The
petitioners failed to establish
that the public respondent
exercised its power in an
arbitrary or despotic manner by
reason of passion or personal
hostility, and this must be so
2011- 2012 Labor Law Case Digest Compilations
UC LLB III-B
Page 12

patent and so gross as to


amount to an evasion of a
positive duty or to a virtual
refusal to perform the duty
enjoined or to act at all in
contemplation of law.
Third
Issue.The
petitioners suspension cannot
be categorized as dismissal,
considering that there was no
intent on the part of respondent
to sever the employer-employee
relationship. An employer has
free rein and enjoys wide
latitude of discretion to regulate
all aspects of employment,
including the prerogative to
instill
discipline
on
his
employees
and
to
impose
penalties
upon
erring
employees.
This
is
a
management
prerogative.
Having established that the case
does not involve termination of
employment, the strict, even
rigid, application of the twinnotice rule is not warranted. But
the due process safeguards are
nonetheless still available to
petitioners. The essence of due
process
is
simply
the
opportunity to be heard. A
formal or trial-type hearing is
not at all times and in all
instances essential, as the due
process
requirements
are
satisfied where the parties are
afforded fair and reasonable
opportunity to explain their side
of the controversy at hand. In
this case, a meeting was called
by Regualos regarding the
controversy.

8. ALEXANDER B. GATUS,
Petitioner, v. SOCIAL
SECURITY SYSTEM,
Respondent
G.R. No. 174725 January 26,
2011
Topic: Ailments Subject to
Compensation Under PD No.
626
Facts:
Gatus, a covered member
of the SSS, worked at the
Central Azucarera de Tarlac for
30 years. By the time of his
retirement, he held the position
of Tender. While employed, he
was confined at the Central
Luzon
Hospital
and
was
diagnosed to be suffering from
Coronary Artery Disease (CAD):
Triple Vessel and Unstable
Angina. His medical records
showed him to be hypertensive
for 10 years and a smoker. On
account of his CAD, he was
given by the SSS an EC/SSS
Permanent
Partial
Disability
benefits which the SSS later
sought to recover from Gatus on
the ground that his CAD, being
attributed
to
his
chronic
smoking, was not work-related.
Gatus however insisted that his
ailment was acquired through
exposure from wastes and
chemicals
at
work.
Such
contention however was not
given merit by the SSS and the
ECC. Upon appeal, the CA
decided in favour of SSS.
Issue:
Whether the petitioners
ailment is not compensable
2011- 2012 Labor Law Case Digest Compilations
UC LLB III-B
Page 13

under Presidential Decree No.


626.
Ruling:
No. Gatus was diagnosed
to have suffered from CAD;
Triple Vessel and Unstable
Angina, diseases or conditions
falling under the category of
Cardiovascular Diseases which
are not considered occupational
diseases under the Amended
Rules
on
Employees
Compensation. His disease not
being listed as an occupational
disease, he was expected to
show that the illness or the fatal
disease was caused by his
employment and the risk of
contracting the disease was
increased or aggravated by the
working conditions. His proof
would constitute a reasonable
basis for arriving at a conclusion
that the conditions of his
employment had caused the
disease or that such working
conditions had aggravated the
risk of contracting the illness or
the fatal disease, which in this
case, Gatus failed to prove.
Verily, his mere contention
of exposure to various smoke
emissions
in
the
working
environment for a period of time
does not ipso facto make the
resulting
disability
compensable.
Awards
of
compensation cannot rest on
speculations or presumptions,
for the claimant must prove a
positive proposition.

9. HOSPITAL MANAGEMENT
SERVICES, INC. - MEDICAL
CENTER MANILA, Petitioner,
v. HOSPITAL MANAGEMENT
SERVICES, INC. MEDICAL
CENTER MANILA EMPLOYEES
ASSOCIATION-AFW and EDNA
R. DE CASTRO,
Respondents.
G.R. No. 176287 January 31,
2011
Topic: Illegal Dismissal
Facts:
Nurse De Castro, an
employee of nine years, and
Gina, a ward clerk-orientee,
were the only ones stationed in
the ward when an 81 y/o patient
accidentally fell from her bed
while trying to reach for the
bedpan. Instead of personally
attending to the patient, De
Castro ordered Gina to check on
the patient since she is busy
attending one of her other
patients in one room. Nurse De
Castro did not, however, reassess the condition of the said
patient, neither did she record
the incident on the patients
chart nor report the incident to
the physician or endorse the
same to the next shift nurses.
The
incident
reached
the
Management;
hence,
an
investigation was conducted
and De Castro was then
dismissed. The Labor Arbiter
however found for De Castro,
which the NLRC opposed. Upon
further appeal, the CA affirmed
the Labor Arbiter and ordered
2011- 2012 Labor Law Case Digest Compilations
UC LLB III-B
Page 14

De Castros reinstatement with


full backwages.
Issue:
Whether De Castro was
illegally dismissed. If so, to what
extent
must
she
be
compensated as to her claims?
Ruling:
Yes. Neglect of duty, to be
a ground for dismissal, must be
both gross and habitual. Gross
negligence connotes want of
care in the performance of one's
duties. Habitual neglect implies
repeated failure to perform
one's duties for a period of time,
depending
upon
the
circumstances. A single or
isolated act of negligence does
not constitute a just cause for
the dismissal of the employee.
Despite the finding of culpability
against De Castro; there is no
any wrongful intent, deliberate
refusal, or bad faith on her part.
It was her judgment call, albeit
an error of judgment, being the
staff nurse with presumably
more work experience and
better learning curve, to send
the Nursing Assistant and wardclerk orientee to check on the
health condition of the patient,
as she deemed it best, under
the given situation, to attend to
a newly-admitted patient who
had more concerns that needed
to be addressed accordingly.
Being her first offense, De
Castro cannot be said to be
grossly negligent so as to justify
her termination of employment.
However, it is to be emphasized

that the nature of the business


of a hospital requires a higher
degree of caution. An act or
omission that falls short of the
required degree of care and
diligence amounts to serious
misconduct which constitutes a
sufficient ground for dismissal;
hence, suspension is proper. As
to dismissal, sanctioning an
erring
employee
with
suspension would suffice as the
extreme penalty of dismissal
would be too harsh. Thus, she
must be reinstated to her former
position without loss of seniority
rights, full backwages, inclusive
of
allowances
and
other
benefits, or their monetary
equivalent.
10. ROBINSONS
GALLERIA/ROBINSONS
SUPERMARKET
CORPORATION and/or JESS
MANUEL, Petitioners, v.
IRENE R. RANCHEZ,
Respondent.
G.R. No. 177937 January 19,
2011
Topic:
Illegal
Dismissal;
Security
of
Tenure
For
Probationary Employees
Facts:
Ranchez, a cashier who is
employed
on
probation,
reported to her supervisor the
loss of P 20, 229.00 which she
had secured in the company
locker.
Subsequently,
the
Operations
Manager
of
Robinsons,
ordered
that
Ranchez be strip-searched by
the company guards but such
2011- 2012 Labor Law Case Digest Compilations
UC LLB III-B
Page 15

yielded nothing. The incident


was then immediately reported
to the police and inquest
thereby ensued. Consequently,
a criminal action was filed
against her which led to her two
weeks
imprisonment.
Thus,
Ranchez
filed
an
illegal
dismissal case which the Labor
Arbiter dismissed. The NLRC,
however,
set
aside
such
decision, to which the CA
affirmed.
Issue:
Whether
Ranchez
was
illegally dismissed. If so, to what
extent of awards is she entitled
to?
Ruling:
Yes.
A
probationary
employee,
like
a
regular
employee, enjoys security of
tenure. The services of an
employee
who
has
been
engaged on probationary basis
may be terminated for any of
the following: (1) a just or (2)
an authorized cause; and (3)
when he fails to qualify as a
regular employee in accordance
with
reasonable
standards
prescribed by the employer.
Ranchez
was
constructively
dismissed by Robinsons. It
would
be
the
height
of
callousness to expect her to
return to work after suffering in
jail for two weeks. Work had
been rendered unreasonable,
unlikely,
and
definitely
impossible,
considering
the
treatment that was accorded
her by Robinsons. In a dismissal

case,
the
due
process
requirement under the Labor
Code is mandatory and may not
be
supplanted
by
police
investigation
or
court
proceedings.
The
criminal
aspect of the case is considered
independent
of
the
administrative aspect. Thus,
employers should not rely solely
on
the
findings
of
the
Prosecutors Office. They are
mandated to conduct their own
separate investigation, and to
accord the employee every
opportunity to defend himself. In
this case, Ranchez was divested
of such relief.
Due
to
the
strained
relations of the parties, the
payment of separation pay has
been considered an acceptable
alternative to reinstatement,
when the latter option is no
longer desirable or viable. On
the one hand, such payment
liberates the employee from
what
could
be
a
highly
oppressive work environment.
On the other, the payment
releases the employer from the
grossly unpalatable obligation of
maintaining in its employ a
worker it could no longer trust.
However, the backwages that
should
be
awarded
to
respondent shall be reckoned
from
the
time
of
her
constructive dismissal until the
date of the termination of her
employment.The
computation
should not cover the entire
period from the time her
compensation was withheld up
2011- 2012 Labor Law Case Digest Compilations
UC LLB III-B
Page 16

to the time of her actual


reinstatement. This is because
respondent was a probationary
employee, and the lapse of her
probationary
employment
without her appointment as a
regular employee of Robinsons
effectively
severed
the
employer-employee relationship
between the parties.
11. GREGORIO V. TONGKO,
PETITIONER, VS. THE
MANUFACTURERS LIFE
INSURANCE CO. (PHILS.),
INC. AND RENATO A. VERGEL
DE DIOS, RESPONDENTS.
G.R. No. 167622, January 25,
2011
Topic:
Employer-Employee
Relationship
Facts:
Tongko argues that for 19
years,
he
performed
administrative functions and
exercised supervisory authority
over employees and agents of
Manulife. In these 19 years, he
was designated as a Unit
Manager, a Branch Manager and
a Regional Sales Manager, and
posits that he was not only an
insurance agent for Manulife but
was its employee as well. This
case stemmed from a previous
Supreme Court decision which
declared that Tongko was an
insurance
agent,
not
the
employee, of Manulife; hence,
he is not entitled to backwages
and payment of benefits as a
result of, as claimed, his illegal
termination.

Issue:
Whether an employeremployee relationship exists
between Tongko and Manulife.
Ruling:
None.Control
over
the
performance of the task of one
providing service - both with
respect to the means and
manner, and the results of the
service - is the primary element
in determining whether an
employment
relationship
exists. Manulife's control fell
short of this norm and carried
only the characteristic of the
relationship
between
an
insurance company and its
agents, as defined by the
Insurance Code and by the law
of agency under the Civil Code.
The parties are bound by a
contract of agency that clearly
subsists notwithstanding the
successive
designation
of
Tongko as a unit manager, a
branch manager and a regional
sales manager. The petitioner
had always been governed by
the Agreement from the start
until the end of his relationship
with Manulife.
His agency
status never changed except to
the extent of being a lead agent.
Tongko simply progressed from
his individual agency to being a
lead agent who could use other
agents in selling insurance and
share in the earnings of these
other
agents.
Guidelines indicative of labor
law "control" do not merely
2011- 2012 Labor Law Case Digest Compilations
UC LLB III-B
Page 17

relate to the mutually desirable


result
intended
by
the
contractual relationship; they
must have the nature of
dictating
the
means
and
methods to be employed in
attaining the result. Manulife's
instructions
regarding
the
objectives and sales targets are
among the directives that the
principal may impose on the
agent to achieve the assigned
tasks. Also, Manulife's codes of
conduct are norms or standards
of
behavior
rather
than
employer directives into how
specific tasks are to be done.
These codes, as well as
insurance industry rules and
regulations, are not per se
indicative of labor law control
under
our
jurisprudence.
12. THE UNIVERSITY OF THE
IMMACULATE CONCEPTION
and MO. MARIA ASSUMPTA
DAVID, RVM, Petitioners, v.
NATIONAL LABOR RELATIONS
COMMISSION and TEODORA
AXALAN, Respondents.
G.R. No. 181146 January 26,
2011
Topic: Jurisdiction Of Labor
Arbiter;
Constructive
Dismissal
Facts:
Axalan is a regular faculty
member of the respondent
University and the President of
the employees union. On two
occasions, she attended out-oftown seminars without applying
for a leave. Thus, the University
sought for her explanation to

which she replied that she can


never be held absent since she
conducted on-line classes while
on seminar. In reply, the
University declared that she
must write a letter of admission
of absence and contrition in
order that no administrative
charge
would
be
pressed
against her but such was
ignored.
Consequently,
an
investigation was conducted
which found her guilty of AWOL
leading to her suspension. Thus,
she filed for an illegal dismissal
charge in the Labor Arbiter.
Upon expiration of the term of
suspension, Axalan resumed her
teaching job in the Univeristy.
Subsequently, the Labor Arbiter,
the NLRC and the CA found her
to be illegally terminated.
Issue:
1. Whether the voluntary
arbitrator had jurisdiction over
the labor dispute.
2. Whether Axalan was
constructively dismissed.
Ruling:
First Issue. Article 262 of
the Labor Code provides that
the Voluntary Arbitrator, upon
agreement of the parties, shall
also hear and decide all other
labor disputes. It is enough that
there is evidence on record
showing the parties have agreed
to
resort
to
voluntary
arbitration, which was attendant
in this case. Thus, the case
should have been referred to
the voluntary arbitrator. The
Constitution even declares as
2011- 2012 Labor Law Case Digest Compilations
UC LLB III-B
Page 18

state policy the preferential use


of voluntary modes in settling
disputes.
Second Issue.Constructive
dismissal occurs when there is
cessation of work because
continued
employment
is
rendered
impossible,
unreasonable, or unlikely as
when there is a demotion in
rank or diminution in pay or
when a clear discrimination,
insensibility, or disdain by an
employer becomes unbearable
to the employee leaving the
latter with no other option but to
quit. In this case however, there
was no cessation of employment
as Axalan promptly resumed
teaching at the university right
after the expiration of the
suspension. She cannot claim
that she was left with no choice
but to quit, a crucial element in
a
finding
of
constructive
dismissal.
There
being
no
constructive dismissal, there is
no legal basis for the order of
reinstatement
as
well
as
payment of backwages and
other claims.
The law protects both the
welfare of employees and the
prerogatives of management.
Courts will not interfere with
prerogatives of management on
the discipline of employees, as
long as they do not violate labor

laws,
collective
bargaining
agreements if any, and general
principles of fairness and justice.

13. PEOPLE OF THE


PHILIPPINES vs. TERESITA
"TESSIE" LAOGO
G.R. No. 176264, January 10,
2011
Topic: Illegal Recruitment in
Large Scale
Facts:
Teresita "Tessie" Laogo
was the proprietor and manager
of Laogo Travel Consultancy, a
travel agency firm located along
Padre Faura Street in Manila.
Susan invited Teodulo dela Cruz,
Billy dela Cruz, Jr., Dante Lopez,
Edwin
Enriquez,
Rogelio
Enriquez, and Gary Bustillos to
her house in Bulacan, Bulacan
to celebrate the town fiesta.
Gary
Bustillos
introduced
Teodulo Dela Cruz to Susan as
somebody who could help him
find work abroad. Susan told
him he can apply as assistant
cook and can work in Guam,
USA. Teodulo filled up an
application form and gave her
P3, 000.00 with a promise to
process his application to work
abroad. Teodulo and Susan went
to Teresitas travel agency and
he paid additional 15,000 as
placement fee.
A receipt
bearing the logo and name of
Laogo Travel Consultancy was
issued to him signed by Susan.
Susans promise to send him
abroad remained unfulfilled.
2011- 2012 Labor Law Case Digest Compilations
UC LLB III-B
Page 19

Billy Dela Cruz, Jr. also met


Susan
through
Gary,
who
himself was seeking help from
Susan to work in Guam. Billy
issued to Susan two Metrobank
checks, dated March 11 and
May 10, 2000, bearing the
amounts P23, 000.00 and P44,
000.00, respectively, as partial
payment for his placement fee.
Billy also went to appellants
travel agency in Ermita and
personally handed an additional
cash of P6, 000.00 to Susan and
gave the money to appellant.
Appellant
issued
a
corresponding receipt for the P6,
000.00
cash
bearing
her
signature and the name and
logo
of
Laogo
Travel
Consultancy.
After
several
months, no word was heard
from either Susan or appellant.
Sensing that something was
wrong, Billy decided to report
the matter to the authorities in
Bulacan, Bulacan and filed the
complaint against Susan and
appellant. Dante Lopez met
Susan through Gary and he paid
P6, 000.00 to process his
papers, covered by a receipt
dated May 19, 2000 showing
appellants
signature.
The
promise remained unfulfilled.
Rogelio Enriquez gave P3,
000.00 cash to Susan for the
processing of his visa and
employment documents.
He
gave an additional P900.00 to
Susan but no receipt issued to
both payments. Months passed
but Rogelio heard nothing from
either Susan or appellant.
Apprehensive, Rogelio verified

the status of the Laogo Travel


Consultancy with the Philippine
Overseas
Employment
Administration (POEA). From the
POEA, Rogelio learned that
neither of the accused nor
Laogo Travel was licensed to
recruit workers for employment
abroad.
Aggrieved,
Rogelio,
together
with
his
six
companions, filed the complaint
against Susan and appellant.
Edwin Enriquez also paid P12,
000.00 to Susan as processing
fee for his application to work in
Guam.
Issue:
Whether or not Susan and
Teresita committed large scale
illegal recruitment?
Ruling:
Yes.
Recruitment
and
placement refers to the act of
canvassing,
enlisting,
contracting,
transporting,
utilizing, hiring or procuring
workers, and includes referrals,
contract services, promising or
advertising for employment,
locally or abroad, whether for
profit or not. When a person or
entity, in any manner, offers or
promises for a fee employment
to two or more persons, that
person or entity shall be
deemed engaged in recruitment
and placement. Article 38(a) of
the Labor Code, as amended,
specifies
that
recruitment
activities undertaken by nonlicensees or non-holders of
authority are deemed illegal and
punishable by law. And when
2011- 2012 Labor Law Case Digest Compilations
UC LLB III-B
Page 20

the
illegal
recruitment
is
committed against three or
more persons, individually or as
a group, then it is deemed
committed in large scale and
carries with it stiffer penalties as
the same is deemed a form of
economic sabotage. To prove
illegal recruitment, it must be
shown that the accused, without
being duly authorized by law,
gave complainants the distinct
impression that he had the
power or ability to send them
abroad for work, such that the
latter were convinced to part
with their money in order to be
employed. It is important that
there must at least be a promise
or offer of an employment from
the person posing as a recruiter,
whether locally or abroad.
In this case, Teodulo, Billy,
Dante, Edwin, and Rogelio were
promised to be sent abroad by
Susan and Tessie as cooks and
assistant cooks. The follow up
transactions between Tessie and
the five persons were done
inside the said travel agency.
Moreover, all four receipts
issued to the said persons bear
the name and logo of Laogo
Travel Consultancy, with two of
the said receipts personally
signed by Tessie herself. Tessie
and Susan could thus be said to
have acted together in making
them believe that they were
transacting with a legitimate
recruitment agency and that
Laogo Travel Consultancy had
the authority to recruit them
and send them abroad for work

when in truth and in fact it had


none as certified by the POEA.
14. BERNADETH LONDONIO
AND JOAN CORCORO v. BIO
RESEARCH, INC. AND
WILSON Y. ANG
G.R. No. 191459, January 17,
2011
Topic: Validity of Quitclaims/
Compromise Agreements
Facts:
Petitioners Bernadeth E.
Londonio and Joan T. Corcoro
were hired by respondent Bio
Research Inc. as graphic/visual
artists on February 12 and
October 19, 2004, respectively.
In a Memorandum dated April
30, 2005 which petitioners
received on May 7, 2005, Bio
Research
informed
its
employees including petitioners
that pursuant to its plan to
reduce the workforce in order to
prevent losses, it would be
severing their employment with
the company. On May 9, 2005,
Bio
Research
filed
an
Establishment
Termination
Report with the Department of
Labor and Employment (DOLE)
stating that it was retrenching
18 of its employees including
petitioners due to redundancy
and
to
prevent
losses.
Bernadeth and Joan were in fact
retrenched on May 26 and May
18, 2005, respectively. Joan
accepted her retrenchment pay
in the sum of P9, 990.14 and
executed
a
Quitclaim
and
Waiver. Bernadeth refused to
accept hers. Petitioners filed a
2011- 2012 Labor Law Case Digest Compilations
UC LLB III-B
Page 21

complaint for illegal dismissal,


moral and exemplary damages
and attorney's fees against
respondent Bio Research and its
co-respondent
President/CEO
Wilson
Y.
Ang.
Petitioners
claimed that their dismissal was
done in bad faith and tainted
with malice, being retaliatory in
nature, following the filing by
Bernadeth
of
a
complaint
against Jose Ang, Jr. one of Bio
Research's managers, for a
sexual harassment incident that
occurred in his office on
February 19, 2005. To refute Bio
Research's claim that it had
been incurring business losses,
Joan cited the recommendation
for her regularization on April
12, 2005, 18 days before she
received
a
copy
of
the
Memorandum of April 30, 2005.
Bio Research, disclaiming that
the sexual harassment case had
anything to do with its decision
to terminate the services of
petitioners,
maintained
that
financial reverses prompted it to
take such drastic action. It went
on to stress that as Joan had
already received her separation
pay and had in fact signed a
waiver and quitclaim in its favor,
she
is
estopped
from
challenging the validity of her
dismissal.
Issue:
Whether an employees
execution of a final settlement
and receipt of amounts agreed
upon forecloses his right to pursue a
claim for illegal dismissal?

Ruling:
No, In determining that
petitioners
were
illegally
retrenched, the appellate court
pointed out that not only did Bio
Research fail to "submit in
evidence its audited financial
statements to show its financial
condition prior to and at the
time
it
enforced
its
retrenchment program"; it also
failed to show that it adopted
fair and reasonable standards in
ascertaining who would be
retained or dismissed among its
employees.
An
employee's
execution of a final settlement
and receipt of amounts agreed
upon do not foreclose his right
to pursue a claim for illegal
dismissal. Joan was illegally
retrenched. She is entitled to
reinstatement without loss of
seniority rights and privileges,
as well as to payment of full
back wages from the time of her
separation
until
actual
reinstatement, less the amount
of P9, 990.14 which she
received as retrenchment pay.
15. RENATO REAL V. SANGU
PHILIPPINES, INC. AND/ OR
KIICHI ABE
G.R. No. 168757, January 19,
2011
Topic: Jurisdiction of Labor
Arbiters; Test for
Intracorporate Acts
Facts:
Petitioner Renato Real was
the Manager of respondent
corporation Sangu Philippines,
Inc., a corporation engaged in
2011- 2012 Labor Law Case Digest Compilations
UC LLB III-B
Page 22

the
business
of
providing
manpower for general services,
like janitors, janitresses and
other maintenance personnel, to
various clients. Petitioner was
removed from his position as
Manager
through
Board
Resolution 2001-03 adopted by
Respondent Corporations Board
of Directors without notice given
to him. He just received a letter
stating that he has been
terminated from service for the
following
reasons:
(1)
continuous absences at his post
at Ogino Philippines Inc. for
several months which was
detrimental to the corporation's
operation; (2) loss of trust and
confidence; and, (3) to cut down
operational expenses to reduce
further losses being experienced
by
Respondent
Corporation.
Petitioner filed a complaint for
illegal dismissal case against
respondent.
Respondents
contended
that
petitioner
committed
gross
acts
of
misconduct detrimental to the
company.
Petitioner
always
absent
from
work
without
informing the corporation of his
whereabouts and he would
come to the office only to collect
his salaries. Petitioner neglected
to supervise the employees
resulting in complaints from
various clients about employees'
performance. In one instance,
petitioner together with a few
others, while drunk, went to the
premises of one of respondents'
clients, Epson Precision (Phils.)
Inc., and engaged in a heated
argument with the employees

therein.
Petitioner allegedly encouraged
the employees who had been
placed in the manpower pool to
file a complaint for illegal
dismissal against respondents.
He incited those assigned in
Epson Precision (Phils.) Inc.,
Ogino Philippines Corporation,
Hitachi Cable Philippines Inc.
and Philippine TRC Inc. to stage
a strike Petitioner together with
other employees barricaded the
premises
of
Respondent
Corporation.
Such
acts
respondents posited constitute
just
cause
for
petitioner's
dismissal and that same was
validly effected.
Issue:
Whether
petitioners
complaint for illegal dismissal
constitutes an intra-corporate
controversy and thus, beyond
the jurisdiction of the Labor
Arbiter
Ruling:
No. This case is not an
intra-corporate controversy but
a labor case cognizable by the
labor arbiter. To determine
whether a case involves an
intra-corporate controversy that
is to be heard and decided by
the branches of the RTC
specifically designated by the
Court to try and decide such
cases, two tests must be
applied: (a) the status or
relationship test, and (2) the
nature of the controversy test.
The first test requires that the
controversy arise out of intra2011- 2012 Labor Law Case Digest Compilations
UC LLB III-B
Page 23

corporate
or
partnership
relations
among
the
stockholders,
members
or
associates of the corporation,
partnership
or
association,
between any or all of them and
the corporation, partnership or
association of which they are
stockholders,
members
or associates; between such
corporation,
partnership,
or
association and the public or
between
such
corporation,
partnership, or association and
the State insofar as it concerns
its franchise, license or permit
to operate. The second test
requires that the dispute among
the parties be intrinsically
connected with the regulation of
the corporation. The Court in
this case held that petitioner is
not a corporate officer because
he was not validly appointed by
the Board, thus, failing the
relationship test, and that this is
a
case
of
employment
termination which is a labor
controversy and not an intracorporate dispute, thus failing
the nature of the controversy
test.
16. THE UNIVERSITY OF THE
IMMACULATE CONCEPTION
AND MO. MARIA ASSUMPTA
DAVID, RVM V. NATIONAL
LABOR RELATIONS
COMMISSION AND TEODORA
AXALAN
G.R. No. 181146, January 26,
2011

Topic:
Constructive
Dismissal;
Jurisdiction
of
Labor Arbiter
Facts:
University
of
the
Immaculate Conception is a
private educational institution
located in Davao City. Teodora C.
Axalan is a regular faculty
member
in
the
university
holding the position of Associate
Professor II. Axalan is the
elected
president
of
the
employees'
union.
Axalan
attended a seminar in Quezon
City on website development
then received a memorandum
from Dean Maria Rosa Celestial
asking her to explain in writing
why
she
should
not
be
dismissed for having been
absent without official leave.
Axalan claimed that she held
online classes while attending
the seminar. She explained that
she was under the impression
that faculty members would not
be marked absent even if they
were not physically present in
the classroom as long as they
conducted online classes. Dean
Celestial relayed to Axalan the
message of the university
president that no administrative
charge would be filed if Axalan
would admit having been absent
without official leave and write a
letter
of
apology
seeking
forgiveness. Axalan opted not to
write the letter of admission and
contrition
the
university
president requested. The Dean
wrote Axalan that the university
president had created an ad hoc
grievance
committee
to
2011- 2012 Labor Law Case Digest Compilations
UC LLB III-B
Page 24

investigate the AWOL charge.


Axalan attended a seminar in
Baguio City. Dean Celestial
wrote Axalan informing her that
her participation in the paralegal
seminar in Baguio City was the
subject of a second AWOL
charge. The dean asked Axalan
to explain in writing why no
disciplinary action should be
taken
against
her.
Axalan
explained that she sought the
approval of Vice-President for
Academics Alicia Sayson. VP
Sayson denied having approved
Axalan's application for official
leave. The VP stated in her
letter that it was the university
president,
Maria
Assumpta
David, who must approve the
application. After conducting
hearings
and
receiving
evidence,
the
grievance
committee found Axalan to have
incurred
AWOL
on
both
instances and recommended
that Axalan be suspended
without pay for six months on
each
AWOL
charge.
The
university president approved
the
committee's
recommendation.
The university president then
wrote Axalan informing her that
she incurred absences without
official leave in the 2 instances.
The
university
president
informed Axalan that the total
penalty of one-year suspension
without pay for both AWOL
charges would be effective
immediately. Axalan filed a
complaint against the University
for
Illegal
Suspension,
constructive
dismissal,

reinstatement with backwages,


and unfair labor practice with
prayer
for
damages and attorneys fee.
Issue:
1. Whether
Axalan
was
constructively dismissed
2. Whether the labor arbiter has
jurisdiction in the case at bar
Ruling:
First Issue.No. Respondent
cannot be considered to have
been constructively dismissed
by the petitioner during her
period
of
suspension.
Constructive dismissal occurs
when there is cessation of work
because continued employment
is
rendered
impossible,
unreasonable, or unlikely as
when there is a demotion in
rank or diminution in pay or
when a clear discrimination,
insensibility, or disdain by an
employer becomes unbearable
to the employee leaving the
latter with no other option but to
quit. In this case, there was no
cessation
of
employment
relations between the parties. It
is un refuted that respondent
promptly resumed teaching at
the university right after the
expiration of the suspension
period.
In
other
words
respondent never quit. Hence
she cannot claim to have been
left with no choice but to quit, a
crucial element in a finding of
constructive dismissal.
Second Issue.No. Although
Article 217 of the Labor Code
2011- 2012 Labor Law Case Digest Compilations
UC LLB III-B
Page 25

states that unfair labor practices


and termination disputes fall
within the original and exclusive
jurisdiction
of
the
Labor
Arbiter, Article 262 of the same
Code provides the exception.
For the exception to apply there
must be agreement between the
parties
clearly
conferring
jurisdiction to the voluntary
arbitrator. Such agreement may
be
stipulated
in
collective
bargaining
agreement.
However, in the absence of a
collective
bargaining
agreement, it is enough that
there is evidence on record
showing the parties have agreed
to
resort
to
voluntary
arbitration. As can be gleaned
from
the
transcript
of
stenographic
notes
of
the
administrative hearing held on
20 February 2003, the parties in
this case clearly agreed to resort
to voluntary arbitration.
17. Grand Plaza Hotel, Corp.
vs. National Union of
Workers in the Hotel,
Restaurant and Allied
Industries-Heritage Hotel
Manila Supervisors Chapter
(NUWHRAIN-HHMSC)
G.R. No. 178296, January 12,
2011
Topic:
Reportorial
Requirements As Ground for
Cancellation of Certificate of
Registration of a Legitimate
Labor Organization
Facts:
Respondent filed with the
Department
of
Labor
and

Employment-National
Capital
Region a petition for certification
election.
The
Med-Arbiter
granted the petition and ordered
the holding of a certification
election. After the pre-election,
petitioner
discovered
that
respondent had failed to submit
to the BLR its annual financial
report for several years and the
list of its members since it filed
its registration papers in 1995.
Petitioner
filed
a
Petition
for Cancellation of Registration
of respondent, on the ground of
non-submission of the said
documents. Petitioner prayed
that respondent's Certificate of
Creation of Local/Chapter be
cancelled and its name be
deleted
from
the
list
of
legitimate labor organizations. It
requested the suspension of the
certification
election
proceedings. The certification
election
pushed
through.
Respondent emerged as the
winner. Petitioner filed a Protest
with
Motion
to
Defer
Certification of Election Results
and Winner. Petitioner posits
that once it is determined that a
ground enumerated in Article
239 of the Labor Code is
present,
cancellation
of
registration should follow; it
becomes the ministerial duty of
the Regional Director to cancel
the registration of the labor
organization, hence, the use of
the word "shall." Petitioner
points out that the Regional
Director has admitted in its
decision that respondent failed
to
submit
the
required

documents for a number of


years; therefore, cancellation of
its registration should have
followed as a matter of course.
Issue:
1. Whether the DOLE Secretary
acted with grave abuse of
discretion in taking cognizance
of the appeal and affirming the
dismissal of its petition for
cancellation
of
respondent's
registration as such is conferred
to
the
Bureau
of
Labor
Relations.
2. Whether
the
respondent's
registration as a legitimate labor
union should be cancelled.

2011- 2012 Labor Law Case Digest Compilations


UC LLB III-B
Page 26

Ruling:
First Issue.No. Jurisdiction
remained with the BLR despite
the BLR Director's inhibition.
When the DOLE Secretary
resolved the appeal, she merely
stepped into the shoes of the
BLR Director and performed
function that the latter could not
himself perform. The DOLE
Secretary,
as
the
person
exercising
the
power
of
supervision and control over the
BLR, has the authority to
directly exercise the quasijudicial function entrusted by
law to the BLR Director. The
DOLE Secretary's act of taking
over the function of the BLR
Director was warranted and
necessitated by the latter's
inhibition from the case and the
objective to "maintain the
integrity of the decision, as well
as the Bureau itself.

Second
Issue.No.
ILO
Convention No. 87 provides that
workers
and
employers
organizations shall not be liable
to be dissolved or suspended by
administrative authority. The
ILO has expressed the opinion
that the cancellation of union
registration by the registrar of
labor unions, which in our case
is the BLR, is tantamount to
dissolution of the organization
by
administrative
authority
when such measure would give
rise to the loss of legal
personality of the union or loss
of advantages necessary for it
to carry out its activities, which
is true in our jurisdiction.
Although the ILO has allowed
such measure to be taken,
provided that judicial safeguards
are in place, the right to appeal
to a judicial body, it has
nonetheless
reminded
its
members that dissolution of a
union,
and
cancellation of
registration for that matter,
involve serious consequences
for occupational representation.
It is preferable if such actions
were to be taken only as a last
resort and after exhausting
other possibilities with less
serious
effects
on
the
organization. The appellee has
substantially complied with its
duty to submit its financial
report for that said period.
18. Nationwide Security
and Allied Services, Inc. v.
Ronald P. Valderama
G.R. No. 186614, February
23, 2011
2011- 2012 Labor Law Case Digest Compilations
UC LLB III-B
Page 27

Topic: Constructive Dismissal


Facts:
Respondent
Ronald
Valderama
was
hired
by
petitioner as security guard on
April 18, 2002. He was assigned
at the Philippine Heart Center
(PHC), Quezon City, until his
relief on January 30, 2006.
Valderama was not given any
assignment thereafter. Thus, on
August 2, 2006, he filed a
complaint
for
constructive
dismissal and non-payment of
13th month pay, with prayer for
damages against petitioner and
Romeo
Nolasco.
Petitioner
presented a different version. It
alleged that respondent was not
constructively
or
illegally
dismissed, but had voluntarily
resigned.
Petitioner averred
that respondent has committed
serious violations of the security
rules in the workplace. On
January 31, 2004, he was
charged
with
conduct
unbecoming for which he was
required to explain. He and four
(4) other co-security guards
failed to attend a mandatory
seminar. He was suspended for
seven (7) days. On January 22,
2005, seven security guards,
including
respondent,
were
made to explain their failure to
report
for
duty
without
informing the office despite the
instruction
during
their
formation day which was held a
day before. On January 31,
2006, Roy Datiles, Detachment
Commander,
reported
that
respondent
confronted
and

challenged him in a high pitch


and on top of his voice rudely
showing
discourtesy
and
rudeness. Being his superior,
Datiles recommended the relief
of
respondent
in
the
detachment effective January
31, 2006. By order of the
Operations Manager, he was
relieved from his post at the
Philippine Heart Center. He was
directed to report to the office.
On February 10, 2006, he got
his cash bond and firearm
deposit. Despite his voluntary
resignation, petitioner sent him
a letter through registered mail
to report for the office and give
information on whether or not
he was still interested for report
for duty or not. [Respondent] did
not bother to reply. Neither did
he report to the office.
Issue:
Whether the respondent
was constructively or illegally
dismissed
Ruling:
The
Court
held
that
respondent was constructively
dismissed. In cases involving
security guards, a relief and
transfer order in itself does not
sever employment relationship
between a security guard and
his agency. An employee has the
right to security of tenure, but
this does not give him a vested
right to his position as would
deprive the company of its
prerogative
to
change
his
assignment or transfer him
where his service, as security
2011- 2012 Labor Law Case Digest Compilations
UC LLB III-B
Page 28

guard, will be most beneficial to


the client. Temporary off-detail
or the period of time security
guards are made to wait until
they are transferred or assigned
to a new post or client does not
constitute
constructive
dismissal, so long as such status
does not continue beyond six
months. The onus of proving
that there is no post available to
which the security guard can be
assigned rests on the employer.
In the instant case, the failure of
petitioner to give respondent a
work assignment beyond the
reasonable six-month period
makes it liable for constructive
dismissal.
19. PAQUITO V. ANDO
vs.ANDRESITO Y. CAMPO, ET
AL.
G.R. No. 184007
February 16, 2011
Topic: RTCs Power To Enjoin
Execution
Of
A
Labor
Arbiters Decision/ Regular
Courts
Jurisdiction
Over
Labor
Cases;
NLRCs
Jurisdiction For Third Party
Claims
Facts:
Paquito Ando was the
president of Premier Allied and
Contracting
Services,
Inc.
(PACSI), an independent labor
contractor, which hired Campo,
et al as pilers or haulers.
Campo, et al. were dismissed
from employment, thus, they
filed a case for illegal dismissal
and some money claims. The
Labor Arbiter ruled that Campo,

et al. were illegally dismissed.


The NLRC affirmed the Labor
Arbiter decision.To answer for
the monetary award, NLRC
Acting Sheriff issued a Notice of
Sale on Execution of Personal
Property over the property in
the name of "Paquito V. Ando
married to Erlinda S. Ando."
Ando then file an action
for the issuance of a temporary
restraining order (TRO) before
the RTC, claiming that the
property belonged to him and
his wife, not to the corporation,
and, hence, could not be subject
of the execution sale. Since it is
the corporation that was the
judgment
debtor,
execution
should be made on the latters
properties. The RTC denied the
prayer for TRO for lack of
jurisdiction. The CA affirmed the
RTC decision. Hence, Ando filed
a
petition
for
review
on
certiorari before the SC. Ando
argues that he can choose
between filing a third-party
claim with the sheriff of the
NLRC or filing a separate action.
Issue:
1.Whether the RTC has
jurisdiction to
restrain
the
implementation of the writ of
execution issued by the Labor
Arbiter.
2. Whether case is a third
party-claim
under
the
jurisdiction of the NLRC.
Ruling:
First
Issue.None.
The
Court has long recognized that
regular
courts
have
no
2011- 2012 Labor Law Case Digest Compilations
UC LLB III-B
Page 29

jurisdiction to hear and decide


questions which arise from and
are
incidental
to
the
enforcement
of
decisions,
orders, or awards rendered in
labor cases by appropriate
officers and tribunals of the
Department
of
Labor
and
Employment. To hold otherwise
is to sanction splitting of
jurisdiction which is obnoxious
to the orderly administration of
justice.
Thus, it is, first and
foremost, the NLRC Manual on
the Execution of Judgment that
governs any question on the
execution of a judgment of that
body. Petitioner need not look
further than that. The Rules of
Court apply only by analogy or
in a suppletory character.
Second Issue.Yes. There is
no doubt in our mind that
petitioners complaint is a thirdparty
claim
within
the
cognizance
of
the
NLRC.
Petitioner
may
indeed
be
considered a "third party" in
relation to the property subject
of the execution vis--vis the
Labor Arbiters decision. There is
no question that the property
belongs to petitioner and his
wife, and not to the corporation.
It can be said that the property
belongs
to
the
conjugal
partnership, not to petitioner
alone.
Thus,
the
property
belongs to a third party, i.e., the
conjugal partnership. At the
very least, the Court can
consider that petitioners wife is

a
third
party
within
contemplation of the law.
Whatever
irregularities
attended
the
issuance
an
execution of the alias writ of
execution should be referred to
the same administrative tribunal
which rendered the decision.
This is because any court which
issued a writ of execution has
the inherent power, for the
advancement of justice, to
correct errors of its ministerial
officers and to control its own
processes.
The broad powers granted
to the Labor Arbiter and to the
National
Labor
Relations
Commission by Articles 217, 218
and 224 of the Labor Code can
only be interpreted as vesting in
them jurisdiction over incidents
arising from, in connection with
or relating to labor disputes, as
the
controversy
under
consideration, to the exclusion
of the regular courts. There is no
denying
that
the
present
controversy arose from the
complaint for illegal dismissal.
The
subject
matter
of
petitioners complaint is the
execution of the NLRC decision.
Execution is an essential part of
the proceedings before the
NLRC.
Jurisdiction,
once
acquired, continues until the
case is finally terminated, and
there can be no end to the
controversy without the full and
proper implementation of the
commissions directives.

2011- 2012 Labor Law Case Digest Compilations


UC LLB III-B
Page 30

20. E.G & I. CONSTRUCTION


CORPORATION vs. ANANIAS
P. SATO, et al.
February 16, 2011
Topic:
Illegal
DismissalBurden
of
Proof;
Abandonment
Facts:
Ananias P. Sato, et al.
were dismissed by E.G. & I.
Construction Corporation and
were barred from entering the
company premises. They filed
complaints for illegal dismissal,
underpayment of wages, and
others. E.G. & I. Construction
Corporation claims that Sato,et
al. abandoned their work when
they refused to report for work
for a certain period.
The Labor Arbiter ruled
that Sato, et al. were illegally
dismissed from employment,
and granted the monetary
claims because E.G. & I.
Construction Corporation failed
to effectively controvert the said
claims by not presenting proof
of payment, such as payrolls or
vouchers. The NLRC, however,
reversed the Labor Arbiter
decision. The CA, on the other
hand, reinstated the Labor
Arbiter
decision.
Hence,
a
petition for review on certiorari
was filed before the SC.
Issues:
1. Whether Sato, et al.
were illegally dismissed.
2. Whether Sato, et al.
have
abandoned
their
workplace.

3. Whether Sato, et al.


should be granted their money
claims.
Ruling:
First Issue.Yes. Petitioner
corporation failed to prove that
respondents were dismissed for
just or authorized cause. In an
illegal dismissal case, the onus
probandi rests on the employer
to prove that the dismissal of an
employee is for a valid cause.
Second Issue. No. For
abandonment to exist, it is
essential (a) that the employee
must have failed to report for
work or must have been absent
without valid or justifiable
reason; and (b) that there must
have been a clear intention to
sever the employer-employee
relationship manifested by some
overt acts. The employer has
the burden of proof to show the
employee's
deliberate
and
unjustified refusal to resume his
employment
without
any
intention of returning. Mere
absence is not sufficient. There
must be an unequivocal intent
on the part of the employee to
discontinue his employment.
In this case, the reason
why respondents failed to report
for work was because petitioner
corporation barred them from
entering its construction sites. It
is a settled rule that failure to
report for work after a notice to
return to work has been served
does not necessarily constitute
abandonment. The
intent
to
discontinue the employment
2011- 2012 Labor Law Case Digest Compilations
UC LLB III-B
Page 31

must be shown by clear proof


that it was deliberate and
unjustified. Petitioner
corporation failed to show overt
acts committed by respondents
from which it may be deduced
that they had no more intention
to work. Respondents filing of
the case for illegal dismissal
barely four (4) days from their
alleged abandonment is totally
inconsistent with our known
concept of what constitutes
abandonment.
Third Issue. We sustain
the ruling of the CA on
respondents money claims. As
a rule, one who pleads payment
has the burden of proving it.
Even as the employee must
allege non-payment, the general
rule is that the burden rests on
the employer to prove payment,
rather than on the employee to
prove non-payment. The reason
for the rule is that the pertinent
personnel
files,
payrolls,
records, remittances, and other
similar documents which will
show
that
overtime,
differentials, service incentive
leave, and other claims of the
worker have been paid are
not in the possession of the
worker but in the custody and
absolute
control
of
the
employer.
In
this
case,
the
submission
of
petitioner
corporation of the time records
and payrolls of respondents only
on their appeal before the NLRC
is
contrary
to
elementary
precepts of justice and fair play.

Respondents were not given the


opportunity
to
check
the
authenticity and correctness of
the same. Thus, we sustain the
ruling of the CA in the grant of
the
monetary
claims
of
respondents. We are guided by
the time-honored principle that
if doubts exist between the
evidence presented by the
employer and the employee, the
scales of justice must be tilted
in favor of the latter. It is the
rule in controversies between a
laborer and his master that
doubts reasonably arising from
the
evidence,
or
in
the
interpretation of agreements
and writing, should be resolved
in the former's favor.
21. INTERNATIONAL
CONSTRUCTION
CORPORATION vs.
GUILLERMO BISCOCHO, et al.
February 23, 2011
Topic:
Illegal
DismissalEvidence Needed to Prove
Acts Constituting Dismissal;
Abandonment;
Facts:
Exodus hired Biscocho, et
al. for a painting project. They
were reprimanded for absence
without leave, and eating during
their
hours
of
work.
Subsequently, they did not
return for work, and filed
complaints for illegal dismissal
and money claims. The Labor
Arbiter dismissed the case of
illegal dismissal as respondents
chose not to report for work.
The Labor Arbiter ruled that
2011- 2012 Labor Law Case Digest Compilations
UC LLB III-B
Page 32

since there is neither illegal


dismissal nor abandonment of
job, Biscocho, et al. should be
reinstated but without any
backwages, but allowed the
claims for holiday pay, service
incentive leave pay and 13th
month pay.
Exodus
appealed
the
monetary grant in the Labor
Arbiter decision. The NLRC
dismissed the appeal, ruling that
Exodus should have rebutted
the monetary claims against it,
which it did not, giving an
impression that it never paid
said
benefits.
The
CA
subsequently affirmed the NLRC
decision; hence, Exodus filed a
petition for review on certiorari
before the SC.
Issues:
1. Whether Biscocho, et
al. were illegally dismissed.
2. Whether Biscocho, et
al. abandoned their work.
3. Whether Biscocho, et
al. are entitled to payment of
holiday pay, service incentive
leave pay, and 13th month pay.
Ruling:
First Issue. No. In cases of
illegal dismissal, the employer
bears the burden of proof to
prove that the termination was
for a valid or authorized
cause. But before the employer
must bear the burden of proving
that the dismissal was legal, the
employee must first establish by
substantial evidence that indeed
they were dismissed. If there is
no dismissal, then there can be

no question as to the legality or


illegality thereof.
There was no dismissal in
this case, hence, there is no
question
that
can
be
entertained regarding its legality
or illegality. Absent any showing
of an overt or positive act
proving that petitioners had
dismissed
respondents,
the
latters claim of illegal dismissal
cannot be sustained. Indeed, a
cursory examination of the
records
reveal
no
illegal
dismissal to speak of.
Second Issue. It is a
settled rule that mere absence
or failure to report for work is
not enough to amount to
abandonment
of
work.
Abandonment is the deliberate
and unjustified refusal of an
employee
to
resume
his
employment.
It is the employer who has the
burden of proof to show a
deliberate
and
unjustified
refusal of the employee to
resume his employment without
any intention of returning. It is
therefore
incumbent
upon
petitioners to ascertain the
respondents interest or noninterest in the continuance of
their employment. However,
petitioners failed to do so.
Third
Issue:
Yes.
Respondents are also entitled to
their money claims such as the
payment of holiday pay, service
incentive leave pay, and 13th
month pay. Petitioners as the
employer of respondents and
2011- 2012 Labor Law Case Digest Compilations
UC LLB III-B
Page 33

having complete control over


the records of the company
could have easily rebutted the
monetary claims against it. All
that they had to do was to
present the vouchers or payrolls
showing payment of the same.
However, they decided not to
provide the said documentary
evidence.
Our
conclusion
therefore is that they never paid
said benefits and therefore they
must be ordered to settle their
obligation with the respondents.
22.
FILIPINAS
PALMOIL
PROCESSING, INC.vs. JOEL P.
DEJAPA
February 7, 2011
Topic: Finality of Judgment
Facts:
Joey Dejapa filed a Complaint for
illegal dismissal and money
claims against Filipinas Palmoil.
The Labor Arbiter dismissed the
complaint, and was affirmed by
the NLRC. The CA, on the other
hand,
reversed
the
NLRC
decision,
and
ordered
the
reinstatement of Dejapa and
payment of his monetary claims.
The decision became final and
executory after the denial of the
petition for review on certiorari
before the SC. Thereafter, the
CA rendered a nunc pro tunc
order clarifying the decretal
portion of its previous decision.
Before the SC now is a petition
for review filed by Filipinas
Palmoil
assailing
the
CA
Decision, which had already
become final and executory.
Issue:

Whether the CA decision


which had become final and
executory can be assailed.
Ruling:
No. No other procedural
law principle is indeed more
settled than that once a
judgment becomes final, it is no
longer
subject
to
change,
revision,
amendment
or
reversal,
except
only
for
correction of clerical errors, or
the making of nunc pro tunc
entries which cause no prejudice
to any party, or where the
judgment itself is void. The
underlying reason for the rule is
two-fold: (1) to avoid delay in
the administration of justice and
thus make orderly the discharge
of judicial business, and (2) to
put judicial controversies to an
end, at the risk of occasional
errors,
inasmuch
as
controversies cannot be allowed
to drag on indefinitely and the
rights and obligations of every
litigant must not hang in
suspense for an indefinite period
of time.
It is an important
fundamental principle in our
Judicial system that every
litigation must come to an end.
Access
to
the
courts
is
guaranteed. But there must be a
limit thereto. Once a litigant's
rights have been adjudicated in
a valid final judgment of a
competent court, he should not
be granted an unbridled license
to come back for another try.
The prevailing party should not
2011- 2012 Labor Law Case Digest Compilations
UC LLB III-B
Page 34

be harassed by subsequent
suits. For, if endless litigations
were to be encouraged, then
unscrupulous
litigants
will
multiply in number to the
detriment of the administration
of justice.
23. LEGEND INTERNATIONAL
RESORTS LIMITEDvs.
KILUSANG MANGGAGAWA NG
LEGENDA (KMLINDEPENDENT)
February 23, 2011
Topic: Acts Allowed
For
Legitimate
Labor
Organizations Whose Legal
Personality Is Questioned;
Collateral Attack On The
Legal
Personality
of
a
Legitimate
Labor
Organization
Facts:
KML filed with the MedArbitration Unit a Petition for
Certification Election, alleging
that it is a legitimate labor
organization of the rank and file
employees
of
Legend
International Resorts Limited
(LEGEND). LEGEND moved to
dismiss the petition alleging that
KML is not a legitimate labor
organization
because
its
membership is a mixture of rank
and
file
and
supervisory
employees in violation of Article
245 of the Labor Code. LEGEND
also
claimed
that
KML
committed acts of fraud and
misrepresentation when it made
it appear that certain employees
attended
its
general
membership meeting when in

reality some of them were either


at work; have already resigned;
or were abroad.
The Med-Arbiter dismissed
the petition for certification
election for lack of merit. The
Med-Arbiter found that indeed
there were several supervisory
employees
in
KMLs
membership. Since Article 245
of the Labor Code expressly
prohibits supervisory employees
from joining the union of rank
and file employees, the MedArbiter concluded that KML is
not
a
legitimate
labor
organization. KML thus appealed
to the Office of the Secretary of
the DOLE. The Secretary of
DOLE reversed the Med-Arbiters
Decision, ruling that KMLs
legitimacy as a union could not
be collaterally attacked. The
Court of Appeals held that the
issue on the legitimacy of KML
has already been settled, thus,
the latter could properly file a
petition for certification election.
LEGEND then filed a petition for
review on certiorari before the
SC.
Issues:
1. Whether the pendency
of the cancellation proceedings
bar
collective
bargaining
between LEGEND and KML.
2. Whether the legal
personality of KML can be
collaterally attacked in a petition
for certification election
Ruling:
First Issue. No. An order to
hold a certification election is
2011- 2012 Labor Law Case Digest Compilations
UC LLB III-B
Page 35

proper despite the pendency of


the petition for cancellation of
the registration certificate of the
respondent union. The rationale
for this is that at the time the
respondent
union
filed
its
petition, it still had the legal
personality to perform such act
absent an order directing a
cancellation. Based on the
foregoing jurisprudence, it is
clear that a certification election
may be conducted during the
pendency of the cancellation
proceedings. This is because at
the time the petition for
certification was filed, the
petitioning union is presumed to
possess the legal personality to
file the same. There is therefore
no basis for LEGENDs assertion
that the cancellation of KMLs
certificate of registration should
retroact to the time of its
issuance or that it effectively
nullified all of KMLs activities,
including its filing of the petition
for certification election and its
demand to collectively bargain.
Second Issue. No. The
legitimacy
of
the
legal
personality of KML cannot be
collaterally attacked in a petition
for certification election. Such
legal personality may not be
subject to a collateral attack but
only through a separate action
instituted particularly for the
purpose of assailing it. Once a
certificate of registration is
issued to a union, its legal
personality cannot be subject to
a collateral attack. It may be
questioned
only
in
an

independent
petition
for
cancellation in accordance with
Section 5 of Rule V, Book V of
the Implementing Rules.
24. AVELINA F. SAGUN
vs.
SUNACE INTERNATIONAL
MANAGEMENT SERVICES,
INC.
February 23, 2011
Topic:
Collection
Of
Excessive Placement Fees
Facts:
Avelina F. Sagun filed a
complaint
for
excessive
placement
fees
against
respondent Sunace International
Management
Services,
Inc.
before
the
POEA.
Sagun
contends that Sunace promised
her a caretaker position in
Taiwan in consideration of
placement and employment
fees
in
the
amounts
of
P30,000.00 cash,P10,000.00 in
the form of a promissory note,
and
NT$60,000.00
through
salary deduction. She, however,
worked as a domestic helper in
Taiwan.
Sunace maintained that it only
collected P20,840.00,
the
amount authorized by the POEA
and for which the corresponding
official receipt was issued. The
POEA
Administrator
6
dismissed the complaint, ruling
that Sagun failed to establish
facts showing a violation of
Article 32, since it was proven
that the amount received by
respondent as placement fee
2011- 2012 Labor Law Case Digest Compilations
UC LLB III-B
Page 36

was covered by an official


receipt.
The Secretary of Labor
ruled that SUNACE is liable for
collection
of
excessive
placement fee. The Office of the
President
(OP)
affirmed the
Order of the Secretary of Labor.
The CA reversed the decision of
the OP, ruling that their
conclusions were based not on
evidence but on speculation,
conjecture, possibilities, and
probabilities. Hence,
Sagun
filed a petition for review on
certiorari before the SC.
Issue:
Whether SUNACE is liable
for
collection
of
excess
placement fee.
Ruling:
No.
The
pieces
of
evidence presented by Sagun
are not substantial to show that
SUNACE collected from her more
than the allowable placement
fee. There is no single piece of
document or receipt showing
that deductions have in fact
been made, nor is there any
proof that these deductions
from the salary formed part of
the subject placement fee.
To be sure, mere general
allegations
of
payment
of
excessive
placement
fees
cannot be given merit as the
charge of illegal exaction is
considered a grave offense
which
could
cause
the
suspension or cancellation of
the agencys license. They
should
be
proven
and

substantiated by clear, credible,


and competent evidence.
25. NELSON CULILI
VS.EASTERN
TELECOMMUNICATIONS
PHILS.INC., GR. NO. 165381,
February 9, 2011
Topic: Illegal Dismissal;
Unfair Labor Practice
Facts:
Eastern
Telecommunications Philippines,
Inc.
(ETPI)
is
a
telecommunications
company
engaged mainly in the business
of
establishing
commercial
telecommunications
systems
and leasing of international
datalines or circuits that pass
through
the
international
gateway facility (IGF). Nelson A.
Culili (Culili) was employed by
ETPI as a Technician in its Field
Operations
Department
on
January 27, 1981. On December
12, 1996, Culili was promoted to
Senior
Technician
in
the
Customer Premises Equipment
Management Unit of the Service
Quality Department and his
basic salary was increased. As a
telecommunications
company
and an authorized IGF operator,
ETPI
was
required,
under
Republic Act. No. 7925 and
Executive Order No. 109, to
establish landlines in Metro
Manila and certain provinces.
However, due to interconnection
problems with the Philippine
Long
Distance
Telephone
Company
(PLDT),
poor
2011- 2012 Labor Law Case Digest Compilations
UC LLB III-B
Page 37

subscription and cancellation of


subscriptions,
and
other
business difficulties, ETPI was
forced to halt its roll out of one
hundred twenty-nine thousand
(129,000)
landlines
already
allocated to a number of its
employees. ETPI was compelled
to implement a Right-Sizing
Program which consisted of two
phases: the first phase involved
the
reduction
of
ETPIs
workforce
to
only
those
employees that were necessary
and which ETPI could sustain;
the second phase entailed a
company-wide
reorganization
which would result in the
transfer, merger, absorption or
abolition of certain departments
of ETPI. ETPI, on December 10,
1998, offered to its employees
who had rendered at least
fifteen years of service, the
Special Retirement Program,
which consisted of the option to
voluntarily retire at an earlier
age and a retirement package
equivalent to two and a half
(2) months salary for every
year of service. This offer was
initially rejected by the Eastern
Telecommunications Employees
Union
(ETEU),
ETPIs
duly
recognized bargaining agent,
which threatened to stage a
strike. ETPI explained to ETEU
the exact details of the RightSizing Program and the Special
Retirement Program and after
consultations
with
ETEUs
members, ETEU agreed to the
implementation
of
both
programs. On February 8, 1999,
ETPI re-offered the Special

Retirement Program and the


corresponding
retirement
package to the one hundred two
(102) employees who qualified
for the program. Of all the
employees who qualified to avail
of the program, only Culili
rejected the offer. After the
successful implementation of
the first phase of the RightSizing Program, ETPI, on March
1, 1999 proceeded with the
second
phase
which
necessitated
the
abolition,
transfer and merger of a
number of ETPIs departments.
Among
the
departments
abolished was the Service
Quality
Department.
The
functions of the Customer
Premises
Equipment
Management Unit, Culilis unit,
were absorbed by the Business
and
Consumer
Accounts
Department. The abolition of the
Service
Quality
Department
rendered
the
specialized
functions of a Senior Technician
unnecessary. As a result, Culilis
position was abolished due to
redundancy and his functions
were
absorbed
by
Andre
Andrada,
another
employee
already with the Business and
Consumer
Accounts
Department. In a letter dated
March 8, 1999, ETPI, through its
Assistant Vice President Stella
Garcia, informed Culili of his
termination from employment
effective April 8, 1999.
Culili alleged that neither
he nor the Department of Labor
and Employment (DOLE) were
formally
notified
of
his
2011- 2012 Labor Law Case Digest Compilations
UC LLB III-B
Page 38

termination. Culili asserted that


ETPI had contracted out the
services he used to perform to a
labor-only contractor which not
only proved that his functions
had not become unnecessary,
but which also violated their
Collective Bargaining Agreement
(CBA) and the Labor Code. Aside
from these, Culili also alleged
that he was discriminated
against when ETPI offered some
of
his
co-employees
an
additional benefit in the form of
motorcycles to induce them to
avail of the Special Retirement
Program, while he was not. ETPI
averred that since Culili did not
avail of the Special Retirement
Program and his position was
subsequently
declared
redundant, it had no choice but
to terminate Culili. On February
8, 2000, Culili filed a complaint
against ETPI and its officers for
illegal dismissal, unfair labor
practice, and money claims
before the Labor Arbiter. Labor
Arbiter rendered a decision
finding ETPI guilty of illegal
dismissal
and
unfair
labor
practice. On appeal, the NLRC
affirmed the Labor Arbiters
decision
but
modified
the
amount of moral and exemplary
damages awarded. ETPI filed a
Petition for Certiorari under Rule
65 of the Rules of Civil
Procedure before the Court of
Appeals on the ground of grave
abuse of discretion. ETPI prayed
that a Temporary Restraining
Order be issued against the
NLRC from implementing its
decision and that the NLRC

decision and resolution be set


aside.
The Court of Appeals
found that Culilis position was
validly
abolished
due
to
redundancy.
The
Court
of
Appeals said that ETPI had been
very candid with its employees
in implementing its Right-Sizing
Program, and that it was highly
unlikely that ETPI would effect a
company-wide
reorganization
simply for the purpose of getting
rid of Culili. The Court of
Appeals also held that ETPI
cannot be held guilty of unfair
labor
practice
as
mere
contracting out of services being
performed by union members
does not per se amount to
unfair labor practice unless it
interferes with the employees
right to self-organization. Culili
went to the Supreme Court.
Issues:
1. Whether the dismissal
of Culili was illegal.
2.
Whether
ETPI
committed unfair labor practice.
Ruling:
First Issue. There was an
illegal
dismissal.
The
determination of whether or not
an employees services are still
needed or sustainable properly
belongs
to
the
employer.
Provided there is no violation of
law or a showing that the
employer was prompted by an
arbitrary or malicious act, the
soundness or wisdom of this
exercise of business judgment is
not subject to the discretionary
review of the Labor Arbiter and
2011- 2012 Labor Law Case Digest Compilations
UC LLB III-B
Page 39

the NLRC. An employer cannot


simply declare that it has
become
overmanned
and
dismiss its employees without
producing adequate proof to
sustain
its
claim
of
redundancy. Among
the
requisites of a valid redundancy
program are: (1) the good faith
of the employer in abolishing
the redundant position; and (2)
fair and reasonable criteria in
ascertaining what positions are
to be declared redundant, such
as but not limited to: preferred
status, efficiency, and seniority.
ETPI was upfront with its
employees about its plan to
implement
a
Right-Sizing
Program. Even in the face of
initial opposition from and
rejection of the said program by
ETEU, ETPI patiently negotiated
with ETEUs officers to make
them
understand
ETPIs
business dilemma and its need
to reduce its workforce and
streamline its organization. This
evidently rules out bad faith on
the part of ETPI.
Second Issue: There was
no unfair labor practice. An
employer may only be held
liable for unfair labor practice if
it can be shown that his acts
affect in whatever manner the
right of his employees to selforganize. There is no showing
that ETPI, in implementing its
Right-Sizing
Program,
was
motivated by ill will, bad faith or
malice, or that it was aimed at
interfering with its employees
right to self-organize. In fact,

ETPI negotiated and consulted


with ETEU before implementing
its Right-Sizing Program.
26. PLASTIMER INDUSTRIAL
CORP. VS. NATALIA GOPO
GR. NO. 183390, FEBRUARY
16, 2011
Topic: Invalid Retrenchment;
Validity
of
Quitclaims/Compromise
Agreements
Facts:
The
Personnel
and
Administration
Manager
of
Plastimer Industrial Corporation
(Plastimer)
issued
a
Memorandum informing all its
employees of the decision of the
Board of Directors to downsize
and reorganize its business
operations due to withdrawal of
investments and shares of
stocks which resulted in the
change
of
its
corporate
structure. The employees of
Plastimer, including Natalia C.
Gopo et,al. were served written
notices of their termination.
Plastimer
and
Plastimer
Industrial Corporation Christian
Brotherhood
(PICCB),
the
incumbent sole and exclusive
collective
bargaining
representative of all rank and
file employees, entered into a
Memorandum
of
Agreement
(MOA) relative to the terms and
conditions that would govern
the retrenchment of the affected
employees. Plastimer submitted
to the Department of Labor and
Employment
(DOLE)
an
Establishment
Termination
2011- 2012 Labor Law Case Digest Compilations
UC LLB III-B
Page 40

Report containing the list of the


employees affected by the
reorganization and downsizing.
On 28 May 2004, the affected
employees,
including
respondents, signed individual
"Release Waiver and Quitclaim."
Gopo et,al. filed a complaint
against
Plastimer
and
its
President
Teo
Kee
Bin
(petitioners) before the Labor
Arbiter for illegal dismissal with
prayer for reinstatement and full
backwages, underpayment of
separation pay, moral and
exemplary
damages
and
attorneys fees. Gopo alleged
that they did not voluntarily
relinquish their jobs and that
they were required to sign the
waivers and quitclaims without
giving them an opportunity to
read
them
and
without
explaining
their
contents.
Respondents further alleged
that Plastimer failed to establish
the causes/valid reasons for the
retrenchment and to comply
with the one-month notice to
the DOLE as well as the
standard prescribed under the
Collective Bargaining Agreement
between Plastimer and the
employees. Plastimer countered
that the retrenchment was a
management prerogative and
that respondents got their
retrenchment or separation pay
even before the effective date of
their separation from service.
Labor arbiter decided in favor of
plastimer, Gopo then appealed
to the NLRC. NLRC affirmed the
decision of the labor arbiter,
Gopo then went to CA via

certiorari. CA reversed the NLRC


decision, CA ruled that there
was
no
valid
cause
for
retrenchment. Plastimer went to
the Supreme Court.
Issues:
1. Whether respondents
were illegally retrenched by
Plastimer.
2. Whether the quitclaims
and waivers were valid.
Ruling:
First Issue. There was
retrenchment. The fact that
there was a net income in 2003
does not justify the Court of
Appeals ruling that there was
no
valid
reason
for
the
retrenchment. Records showed
that
the
net
income
of P6,185,707.05 for 2003 was
not even enough for petitioners
to
recover
from
the P52,904,297.88
loss
in
2002. Article 283 of the Labor
Code recognizes retrenchment
to prevent losses as a right of
the management to meet clear
and continuing economic threats
or during periods of economic
recession
to
prevent
losses. There is no need for the
employer to wait for substantial
losses to materialize before
exercising ultimate and drastic
option to prevent such losses.
Second Issue. Quitclaims
and waivers were valid. The
Labor Arbiter and the NLRC were
correct that respondents were
sufficiently apprised of their
rights under the waivers and
2011- 2012 Labor Law Case Digest Compilations
UC LLB III-B
Page 41

quitclaims that they signed.


Each document contained the
signatures of Edward Marcaida
(Marcaida), PICCB President, and
Atty. Bayani Diwa, the counsel
for the union, which proved that
respondents were duly assisted
when they signed the waivers
and
quitclaims.
Further,
Marcaidas letter to Teo Kee Bin,
dated 28 May 2004, proved that
proper assistance was extended
upon respondents.
27. HARPOON MARINE
SERVICES, INC VS. FERNAN
FRANCISCO
GR. NO. 167751, March 2,
2011
Topic: Illegal Dismissal
Facts:
Harpoon Marine Services
Inc., a company engaged in ship
building and ship repair, with
petitioner Rosit as its President
and Chief Executive Officer
(CEO), originally hired Francisco
in 1992 as its Yard Supervisor
tasked to oversee and supervise
all projects of the company. In
1998,
Francisco
left
for
employment elsewhere but was
rehired by petitioner Harpoon
and assumed his previous
position a year after. On June 15,
2001, Francisco averred that he
was unceremoniously dismissed
by petitioner Rosit. He was
informed that the company
could no longer afford his salary
and that he would be paid his
separation pay and accrued
commissions.
Francisco
nonetheless continued to report

for work. A few days later,


however, he was barred from
entering the company premises.
Relying on the promise of
petitioner Rosit, Francisco went
to the office on June 30, 2001 to
receive his separation pay and
commissions,
but
petitioner
Rosit offered only his separation
pay. Francisco refused to accept
it and also declined to sign a
quitclaim.
After
several
unheeded requests, respondent,
through his counsel, sent a
demand letter dated September
24, 2001 to petitioners asking
for
payment
of P70,000.00,
which
represents
his
commissions for the seven
boats constructed and repaired
by the company under his
supervision. In a letter-reply
dated
September
28,
2001, petitioners denied that it
owed
respondent
any
commission, asserting that they
never entered into any contract
or agreement for the payment
of commissions. Hence, on
October 24, 2001, Francisco
filed
an
illegal
dismissal
complaint
praying
for
the
payment of his backwages,
separation
pay,
unpaid
commissions,
moral
and
exemplary
damages
and
attorneys fees. The Labor
Arbiter ruled that Francisco was
validly dismissed. Both parties
appealed to NLRC, NLRC decided
that there was illegal dismissal.
Harpoon filed a certiorari with
the CA but it also affirmed the
decision of the NLRC. Harpoon
went to SC.
2011- 2012 Labor Law Case Digest Compilations
UC LLB III-B
Page 42

Issue:
Whether there was illegal
dismissal.
Ruling:
Francisco
was
illegally
dismissed
for
failure
of
petitioners
to
prove
the
existence of a just cause for his
dismissal.Harpoon
failed
to
prove that it was respondent
who voluntarily refused to report
back for work by his defiance
and refusal to accept the
memoranda and the notices of
absences sent to him. Harpoon
failed to present evidence that
they sent these notices to
respondents last known address
for the purpose of warning him
that his continued failure to
report would be construed as
abandonment of work. The
affidavit of petitioner Harpoons
liaison
officer
that
the
memoranda/notices were duly
sent to respondent is insufficient
and self-serving. Despite being
stamped
as
received,
the
memoranda do not bear any
signature of respondent to
indicate
that
he
actually
received the same. There was
no proof on how these notices
were given to respondent.
Neither was there any other
cogent evidence that these were
properly received by Francisco.
28. SAMAHANG
MANGGAGAWA SA CHARTER
CHEMICAL SOLIDARITY OF
UNIONS IN THE PHILIPPINES
FOR EMPOWERMENT AND

REFORMS (SMCC-SUPER)VS.
CHARTER CHEMICAL AND
COATING CORPORATION.GR.
NO. 169717, March 16, 2011
Topic: Collateral Attack On
The Legal Personality Of A
Legitimate
Labor
Organization
Facts:
On February 19, 1999,
Samahang
Manggagawa
sa
Charter Chemical Solidarity of
Unions in the Philippines for
Empowerment
and
Reforms
(petitioner union) filed a petition
for certification election among
the
regular
rank-and-file
employees of Charter Chemical
and
Coating
Corporation
(respondent company) with the
Mediation Arbitration Unit of the
DOLE,
National
Capital
Region.On
April
14,
1999,
respondent company filed an
Answer
with
Motion
to
Dismisson the ground that
petitioner union is not a
legitimate labor organization
because of (1) failure to comply
with
the
documentation
requirements set by law, and (2)
the inclusion of supervisory
employees
within
petitioner
union. Med-Arbiter dismissed
the petition on the ground that
the labor organization not being
a legitimate has no right to file a
petition for certification election
for the purpose of collective
bargaining. On appeal DOLE
revised the decision of the MedArbiter
and
allowed
the
certification of election but the
2011- 2012 Labor Law Case Digest Compilations
UC LLB III-B
Page 43

CA set aside the formers


decision. They went to SC.
Issue:
Whether
the
legal
personality
of
SMCC-SUPER
union
can
be
collaterally
attacked by Charter Chemical in
the
certification
election
proceedings.
Ruling:
The legal personality of
petitioner union cannot be
collaterally
attacked
by
respondent company in the
certification
election
proceedings. Except when it is
requested
to
bargain
collectively, an employer is a
mere bystander to any petition
for certification election; such
proceeding is non-adversarial
and merely investigative, for the
purpose thereof is to determine
which
organization
will
represent the employees in their
collective bargaining with the
employer. The choice of their
representative is the exclusive
concern of the employees; the
employer cannot have any
partisan interest therein; it
cannot interfere with, much less
oppose, the process by filing a
motion to dismiss or an appeal
from it; not even a mere
allegation that some employees
participating in a petition for
certification election are actually
managerial employees will lend
an employer legal personality to
block the certification election.
The employer's only right in the

proceeding is to be notified or
informed thereof.
29. PFIZER VS. GERALDINE
VELASCO, GR.NO. 177467,
MARCH 9, 2011
Facts: Reckoning Date For
Payment Of Backwages.
Geraldine L. Velasco was
employed
with
petitioner
PFIZER, INC. as Professional
Health
Care
Representative
since 1 August 1992. Sometime
in April 2003, Velasco had a
medical work up for her highrisk
pregnancy
and
was
subsequently advised bed rest
which resulted in her extending
her leave of absence. Velasco
filed her sick leave for the
period from 26 March to 18 June
2003, her vacation leave from
19 June to 20 June 2003, and
leave without pay from 23 June
to 14 July 2003. On 26 June
2003, while Velasco was still on
leave, PFIZER through its Area
Sales
Manager
Ferdinand
Cortez,
personally
served
Velasco a "Show-cause Notice"
dated 25 June 2003. Aside from
mentioning
about
an
investigation on her possible
violations of company work
rules regarding "unauthorized
deals and/or discounts in money
or samples and unauthorized
withdrawal and/or pull-out of
stocks" and instructing her to
submit her explanation on the
matter within 48 hours from
receipt of the same, the notice
also advised her that she was
being placed under "preventive
2011- 2012 Labor Law Case Digest Compilations
UC LLB III-B
Page 44

suspension" for 30 days or from


that day to 6 August 2003 and
consequently
ordered
to
surrender
the
following
"accountabilities;" 1) Company
Car, 2) Samples and Promats, 3)
CRF/ER/VEHICLE/SOA/POSAP/MP
OA and other related Company
Forms, 4) Cash Card, 5) Caltex
Card,
and
6)
MPOA/TPOA
Revolving Travel Fund. Velasco
sent a letter addressed to
Cortez dated 28 June 2003
denying the charges. In her
letter, Velasco claimed that the
transaction with Mercury Drug,
Magsaysay Branch covered by
her check (no. 1072) in the
amount
of P23,980.00
was
merely to accommodate two
undisclosed patients of a certain
Dr. Renato Manalo. In support
thereto, Velasco attached the
Doctors letter and the affidavit
of the latters secretary. On 12
July 2003, Velasco received a
"Second Show-cause Notice"
informing her of additional
developments
in
their
investigation. Velasco filed a
complaint for illegal suspension
with money claims before the
Regional Arbitration Branch. On
29 July 2003, PFIZER informed
Velasco of its "Management
Decision"
terminating
her
employment.
Labor
Arbiter
rendered its decision declaring
the dismissal of Velasco illegal.
PFIZER appealed to the National
Labor Relations Commission
(NLRC) but its appeal was
denied via the
NLRC
Decision dated
October
20,
2004, which affirmed the Labor

Arbiters ruling but deleted the


award for damages. PFIZER filed
with the Court of Appeals a
special civil action for the
issuance
of
a
writ
of certiorariunder Rule 65 of the
Rules of Court to annul and set
aside the aforementioned NLRC
issuances. Court of Appeals
upheld
the
validity
of
respondents
dismissal
from
employment. Velasco went to
SC via rule 45 it was denied bud
Pfizer filed a petition with SC.
Issue:
Whether Pfizer is liable to
pay Velasco wages from the
date of the Labor Arbiters
decision
ordering
her
reinstatement until November
23, 2005, when the Court of
Appeals rendered its decision
declaring Velascos dismissal
valid.
Ruling:
Yes. Under Article 223 of
the Labor Code, an employee
entitled to reinstatement "shall
either be admitted back to
work under the same terms and
conditions prevailing prior to his
dismissal or separation or, at
the option of the employer,
merely
reinstated
in
the
payroll."It is established in
jurisprudence
that
reinstatement
means
restoration to a state or
condition from which one had
been removed or separated. The
person reinstated assumes the
position he had occupied prior
to his dismissal. Reinstatement
2011- 2012 Labor Law Case Digest Compilations
UC LLB III-B
Page 45

presupposes that the previous


position from which one had
been removed still exists, or
that there is an unfilled position
which is substantially equivalent
or of similar nature as the one
previously occupied by the
employee. The Court reiterates
the principle that reinstatement
pending appeal necessitates
that it must be immediately selfexecutory without need for a
writ of execution during the
pendency of the appeal, if the
law is to serve its noble
purpose, and any attempt on
the part of the employer to
evade or delay its execution
should
not
be
allowed.
Furthermore, we likewise restate
our ruling that an order for
reinstatement
entitles
an
employee to receive his accrued
backwages from the moment
the reinstatement order was
issued up to the date when the
same was reversed by a higher
court without fear of refunding
what he had received. It cannot
be denied that, under our
statutory and jurisprudential
framework,
respondent
is
entitled to payment of her
wages for the period after
December 5, 2003 until the
Court of Appeals Decision dated
November
23,
2005,
notwithstanding
the
finding
therein that her dismissal was
legal and for just cause. Thus,
the payment of such wages
cannot be deemed as unjust
enrichment
on respondents
part.

30. LORES REALTY


ENTERPRISES, INC. VS.
VIRGINIA PACIA
GR. NO. 171189, MARCH 9,
2011
Topic:
Illegal
DismissalConstruction Of Doubts In
Terms Of Evidence
Facts:
Virginia E. Pacia was hired
by LREI. At the time of her
dismissal, she was the assistant
manager and officer-in-charge of
LREIs Accounting Department
under
the
Finance
Administrative
Division.
On
October 28, 1998, LREIs acting
general
manager,
petitioner
Sumulong, through Ms. Julie
Ontal, directed Pacia to prepare
Check
Voucher
No.
16477
worth P150,000.00 as partial
payment for LREIs outstanding
obligation to the Bank of the
Philippine
Islands-Family
Bank (BPI-FB). Pacia did not
immediately comply with the
instruction. After two repeated
directives,
Pacia
eventually
prepared
Check
No.
0000737526 in the amount
ofP150,000.00. Later, Sumulong
again directed Pacia to prepare
Check Voucher No. 16478 in the
amount ofP175,000.00 to settle
the
balance
of
LREIs
outstanding indebtedness with
BPI-FB. Pacia once again was
slow in obeying the order. Due
to the insistence of Sumulong,
however,
Pacia
eventually
prepared
Check
No.
0000737527 in the amount
of P175,000.00. To explain her
2011- 2012 Labor Law Case Digest Compilations
UC LLB III-B
Page 46

refusal to immediately follow


the directive, Pacia reasoned out
that the funds in LREIs account
were not sufficient to cover the
amounts to be indicated in the
checks. October 29, 1998,
Sumulong
issued
a
memorandum ordering Pacia to
explain in writing why she
refused to follow a clear and
lawful directive. Pacia replied in
writing and explained that her
initial refusal to prepare the
checks
was
due
to
the
unavailability of funds to cover
the amounts and that she only
wanted to protect LREI from
liability under the Bouncing
Checks Law.On November 6,
1998, Pacia received a notice of
termination stating,
among
others, that she was being
dismissed because of her willful
disobedience and their loss of
trust and confidence in her.Pacia
then filed a Complaint for Unfair
Labor
Practice
due
to
Harassment,
Constructive
Dismissal, Moral and Exemplary
Damagesagainst
LREI
and
Sumulong. Subsequently, Pacia
filed an Amended Complaint to
include the charges of illegal
dismissal and non-payment of
salaries. Labor Arbiter rendered
a decision finding that the
dismissal of Pacia was for a just
and valid cause but ordering
payment of what was due her.
On appeal, the NLRC in its March
31, 2000 Decision reversed the
LAs Decision and found LREI
and Sumulong guilty of illegal
dismissal. LREI and Sumulong
elevated the case to the CA, CA

found no merit in the petition


and dismissed it. LREI went to
SC.
Issue:
Whether Pacias dismissal
was
justified
under
the
circumstances.
Ruling:
It was not justified. The
Court is guided by the timehonored principle that if doubt
exists between the evidence
presented by the employer and
the employee, the scales of
justice must be tilted in favor of
the
latter.
The
rule
in
controversies between a laborer
and his master distinctly states
that doubts reasonably arising
from the evidence, or in the
interpretation of agreements
and writing, should be resolved
in the former's favor.
31. SLL INTERNATIONAL
CABLES SPECIALIST VS.
NLRC
G.R. No. 172161, March 02,
2011
Topic:
Test
For
Regular
Employees; Venue For Illegal
Dismissal
Cases;
WagesWhat Does It Include; Illegal
Dismissal
Facts:
Lopez, Canete and Zuniga
(respondents) from Cebu, were
hired by Lagon as apprentices or
trainee cable/ lineman in Manila.
They were minimum wage but
are not required to work
regularly because they are
2011- 2012 Labor Law Case Digest Compilations
UC LLB III-B
Page 47

substitutes. Respondents got


terminated after the completion
of the project. Zuniga and
Canete later on engaged Lagon
as
project
employees
for
another
project
and
after
termination they re-engaged
Lagon again for another project,
which they were hired. Again
respondents sought their 4rth
employment with Lagon and
they were hired. During their
latest project Lagons project
had delays resulting Lagon to
cut down some workers, which
included respondents.
Respondents
filed
a
complaint of illegal dismissal,
non-payment of wages, holiday
pay, 13th month pay and
damages.
Lagon admit employment of
respondents but claimed that
the latter were only project
employees, for their services
were merely engaged for a
specific project or undertaking
and the same were covered by
contracts duly signed by private
respondents.
Lagon
further
alleged that the food allowance
of P63.00 per day as well as
respondents
allowance
for
lodging house, transportation,
electricity, water and snacks
allowance should be added to
their basic pay. Lagon claimed
that
respondents
received
higher wage rate than that
prescribed in Rizal and Manila.
Also, Lagon alleged that since
the workplaces of respondents
were all in Manila, the complaint
should be filed there. LA ruled in
favor of the respondents but

ruled that respondents are not


illegally dismissed. NLRC and CA
affirmed LA.
Issue:
1. Whether the respondents are
regular employees.
2. Whether the case should be
filed in the workplace.
3. Whether
facilities/
accommodations are included
as wage.
4. Whether
there
is
illegal
dismissal.
Ruling:
Fist Issue.Respondents are
regular employees because they
were repeatedly hired by Lagon
and they performed activities
which were usual, necessary
and desirable in the business or
trade of the employer.
Second Issue.The LA had
jurisdiction
because
the
"workplace," as defined in the
said rule, included the place
where
the
employee
was
supposed to report back after a
temporary detail, assignment or
travel, which in this case was
Cebu.
ThirdIssue.Free board and
lodging, electricity, water, and
food enjoyed by respondents
could not be included in the
computation of their wages
because these were given
without their written consent.
Fourth Issue.Lagon is not
liable for illegal dismissal. The
2011- 2012 Labor Law Case Digest Compilations
UC LLB III-B
Page 48

LA viewed respondents' act of


going home as an act of
indifference
when
Lagon
decided to prohibit overtime
work.
Also, the appeal by Lagon
generally
involves
factual
issues, such as, whether or not
there is evidence on record to
support the findings of the LA,
the NLRC and the CA that
respondents were project or
regular employees and that
their salary differentials had
been paid.This calls for a reexamination of the evidence,
which the SC cannot entertain.
Settled is the rule that factual
findings of labor officials, who
are deemed to have acquired
expertise in matters within their
respective
jurisdiction,
are
generally accorded not only
respect but even finality, and
bind the Court when supported
by substantial evidence. It is not
the Court's function to assess
and evaluate the evidence
32. JAMES JERUSALEM VS
KEPPEL MONTE BANK, HOE
ENG HOCK, SUNNY YAP AND
JESEFINA PICART
G.R. 169564, APRIL 6, 2011
Topic: Illegal Dismissal
Facts:
James was employed by
Keppel
as
Assistant
VicePresident.
Later
he
was
assigned as Head of the newly
created
VISA
Credit
Card
Department.
The
bank
subsequently re-organized the
VISA Credit Card Department

and reduced it to a mere unit.


James, carrying the same rank
was reassigned as Head of the
Marketing and Operations of the
Jewelry Department. The VISA
Credit Card Unit was then
headed by Senior Vice President
Roberto.
James received from Jorge,
a Visa Card Holder, a sealed
envelope said to be containing
VISA Card application forms.
James
forwarded
the
application to the Credit Card
Unit and was later on approved.
It turned out, all the accounts
under
these
approved
applications became past due.
A letter was sent to Jorge
asking to assist the bank in the
collection of his referred VISA
accounts which have already an
accumulated principal balance
of P6m. James upon knowing the
status of the accounts referred
by Jorge, sent a Memorandumto
Roberto
recommending
the
filing of a criminal case for
estafa against Jorge.
James
sent
a
Memorandum to Napoleon, VicePresident of Audit Department.
He summarized in the said
Memorandum the events that
transpired during the meeting
with Jorge and reiterated his
suggestion for Keppel to file a
case against Jorge. He further
suggested that Keppel look into
the inside job angle of the
approval of the VISA cards and
that all key officers and staff
should be probed for possible
involvement. Napoleon replied
advisingJames to coordinate
2011- 2012 Labor Law Case Digest Compilations
UC LLB III-B
Page 49

with Roberto and not with him.


Furthermore,
James
was
requested not to interfere with
the
audit
process
being
undertaken
by
the
Audit
Department.
James received a Notice to
Explainfrom
Keppels
Vice
President for Operations, why no
disciplinary action should be
taken
against
him
for
referring/endorsing
fictitious
VISA card applicants. The said
referrals resulted in substantial
financial losses to Keppel.
James pointed out that he had
no
participation
in
the
processing of the VISA card
applications since he was no
longer connected with the VISA
Credit Card Unit at the time of
such transactions.
James
was
later
on
terminated because of breach of
trust
and
confidence
for
knowingly
and
maliciously
referring,
endorsing
and
vouching
for
VISA
card
applicants who later turned out
to be impostors resulting in
financial loss to Keppel. This
prompted James to file before
the Labor Arbiter a complaint for
illegal dismissal.
The LA and NLRC sided
with James but was reversed by
the CA
Issue:
Whether there is illegal
dismissal.
Ruling:
There is illegal dismissal.

Keppel has the burden of


proof
to
discharge
its
allegations.Unlike in other cases
where the complainant has the
burden of proof to discharge its
allegations,
the
burden
of
establishing facts as bases for
an employers loss of confidence
in an employee facts which
reasonably generate belief by
the employer that the employee
was connected with some
misconduct and the nature of
his participation therein is such
as to render him unworthy of
trust and confidence demanded
of his position is on the
employer.
While it is true that loss of
trust and confidence is one of
the just causes for termination,
such
loss
of
trust
and
confidence must, however, have
some
basis.
Proof
beyond
reasonable
doubt
is
not
required. It is sufficient that
there must only be some basis
for such loss of confidence or
that there is reasonable ground
to believe, if not to entertain,
the moral conviction that the
concerned
employee
is
responsible for the misconduct
and that the nature of his
participation therein rendered
him absolutely unworthy of trust
and confidence demanded by
his position.
33. WILFREDO ANTIQUINA
VS MAGSAYSAY MARITIME
CO AND MASTERBULK
G.R. 168922, April 13, 2011

2011- 2012 Labor Law Case Digest Compilations


UC LLB III-B
Page 50

Topic:
Compensation
For
Work-Related
Illnesses
Incurred By Seamen
Facts:
Antiquina
was
hired,
through
manning
agency
Magsaysay
Maritime
Corporation (MMC), to serve as
Third Engineer on the vessel,
M/T Star Langanger, which was
owned
and
operated
by
Masterbulk. Almost 7 months
later,
during
a
routine
maintenance of the vessels
purifier,
Antiquinasuffered
a
fracture on his lower left arm
after a part fell down on him.
After first aid treatment was
given to Antiquina, he was
brought
to
a
hospital
in
Constanza, Romania where the
vessel happened to be at the
time of the accident.
Antiquinawas signed off
the vessel at Port Said, Egypt
and was repatriated to the
Philippines.
He immediately
reported to the office of MMC
and
was
referred
to
the
Metropolitan
Hospital.
Antiquinawas examined at the
Hospital and the doctor issued a
medical report confirming that
Antiquinahas an undisplaced
fracture of the left ulna.After 1
month,
Antiquinascast
was
removed and he was advised to
undergo
physical
therapy
sessions.
Despite
several
months of physical therapy,
Antiquinas arm still did not heal
and
he
had
difficulty
straightening his arm. Another
company designated doctor,

evaluated Antiquinascondition
and advised to undergo a bone
grafting procedure whereby a
piece of metal would be
attached to the fractured bone.
Antiquinareacted with fear and
decided not to have the
operation.
Antiquinafiled a complaint
for
permanent
disability
benefits, sickness allowance,
damages and attorneys fees
against herein respondents.
Antiquinaasserted that he
is entitled to the purportedly
superior benefits provided for
under respondents CBA
Respondents
contended
that Antiquina monetary claims
were premature by reason of the
latters refusal to undergo the
operation recommended by the
company designated physician.
LA and NLRC ruled in favor
of Antiquina
CA found that it was
undisputed
that
petitioner
suffered a work-related injury,
the CA still saw fit to award
medical
unfitness
benefits,
based on the POEA Standard
Contract of Employment and the
finding of Antiquinas
own
physician
that
the
proper
disability grade for Antiquinas
injury was Grade 11 or 14.93%.
Antiquina claimed that it
was only by inadvertence that
he previously failed to attach a
copy of the CBA.
Respondents objected to
the annexes of Antiquinas
motion for reconsideration on
the grounds that his belated
2011- 2012 Labor Law Case Digest Compilations
UC LLB III-B
Page 51

filing violated their right to due


process and that the list of
monthly
contributions
he
presented did not prove he was
a member of AMOSUP since the
said list did not contain any
validation/signature
of
an
AMOSUP officer.
CA denied Antiquinas motion
for reconsideration.
Issue:
Whether
Antiquinacan
may avail to the benefits
Ruling:
No. In line with the
objective
of
dispensing
substantial justice, this Court
has examined the evidence
belatedly
submitted
by
Antiquina
to
the
CA.
Unfortunately, even with this
procedural concession in favor
of Antiquina, the Court does not
find any sufficient basis to
overturn the CA. The burden of
proof shifts to Antiquina and
unable to prove his claim.
In all, the CA committed
no error in ruling that the LAs
award of US$80,000.00 in
disability
benefits
was
unsupported by the evidence on
record, even if we take into
consideration petitioners late
documentary
submissions.
There is no cogent reason to
disturb the appellate courts
finding that the only credible
and competent bases for an
award of disability benefits to
petitioner
are
the
POEA
Standard
Contract
of
Employment and petitioners

own medical evidence that his


disability grade is Grade 11
(14.93%). Thus, the Court of
Appeals
computation
of
petitioners permanent medical
unfitness benefits in the amount
of US$7,465.00must stand.

Further, the fact that


Lapid was employed by the
PCSO as a casual employee,
means that she does not enjoy
security of tenure. Lapids
services
are
terminable
anytime, there being no need to
show cause. Lapids allegations
that there is no substantial
evidence to sustain the finding
of her guilt for Grave Misconduct
and her dismissal from the
service is irrelevant in the
present case as she is a casual
employee, without any security
of tenure. Hence, she may be
separated from service at any
time
Lapid filed a petition for
review (under Rule 43) before
the CA presenting the sole issue
of whether the ruling must
concentrate
more
on
due
process rather than status of
employment
The CA agreed with Lapid

34. PCSO VS MARTIN AND


LAPID
G.R. 191940, April 12, 2011
Topic: Due Process And
Dismissal
Of
A
Casual
Employee
Facts:
PCSO found Lapid, a clerk
of PCSO, guilty of Discourtesy in
the Course of Official Duties and
Grave Misconduct and imposed
on her the penalty of Dismissal
from the Service. She allegedly
confronted, badmouthed and
shouted
invectives
at
Mr.
Guemo, in the presence of other
employees and patients seeking
assistance from the PCSOBataan Provincial District Office.
Another
incident
involving
respondent-appellant took place
where the latter caused a scene
in the office.
When Lapidreceived her
Notice of Termination
from
PCSO
she
moved
for
reconsideration but was denied
thus appealing to the CSC. The
CSC dismissed the appeal.
Records clearly show that
Lapid
was
never
formally
charged for the administrative
offense of Discourtesy in the
Course of Official Duties and
Grave Misconduct, for which she
was dismissed from service.

Issues:
1. Whether termination is justified
for being a casual employee.
2. Whether due process for casual
employee is needed.

2011- 2012 Labor Law Case Digest Compilations


UC LLB III-B
Page 52

Ruling:
First
Issue.
Lapids
termination is unjustified. The
rule
is
still
that
casual
employment
will
cease
automatically at the end of the
period unless renewed as stated
in the Plantilla of Casual
Employment. Casual employees
may also be terminated anytime
though
subject
to
certain
conditions
or
qualifications.

Thus, they may be laid-off


anytime before the expiration of
the employment period provided
any of the following occurs: (1)
when their services are no
longer needed; (2) funds are no
longer available; (3) the project
has
already
been
completed/finished; or (4) their
performance are below par.
Second Issue.Due Process
is also applicable for casual
employees.Equally
important,
they are entitled to due process
especially if they are to be
removed
for
more
serious
causes or for causes other than
the reasons mentioned in CSC
Form No. 001. This is pursuant
to Section 2, Article IX(B) of the
Constitution and Section 46 of
the Civil Service Law.
The
reason for this is that their
termination from the service
could carry a penalty affecting
their
rights
and
future
employment in the government.
In the case at bench, the
action of PCSO clearly violated
Lapids basic rights as a casual
employee. As pointed out by the
CSC itself, Lapid was NEVER
formally
charged
with
the
administrative
offenses
of
Discourtesy in the Course of
Official
Duties
and
Grave
Misconduct. According to the
CSC, the Formal Charge, was
even
unsigned,
and
it
categorically stated that PCSO
failed to observe due process.
Section 3(2), Article XIII of the
Constitution
guarantees
the
rights of all workers not just in
2011- 2012 Labor Law Case Digest Compilations
UC LLB III-B
Page 53

terms
of
self-organization,
collective bargaining, peaceful
concerted activities, the right to
strike
with
qualifications,
humane conditions of work, and
a living wage but also to
security of tenure. Likewise,
Section 2(3), Article IX-B of the
Constitution provides that no
officer or employee of the civil
service shall be removed or
suspended except for cause
provided by law.
The Civil Service Law
echoes this constitutional edict
of security of tenure of the
employees in the civil service.
Thus, Section 46 (a) of the Civil
Service Law provides that no
officer or employee in the Civil
Service shall be suspended or
dismissed except for cause as
provided by law after due
process.
35. YAP VS THENAMARIS
SHIPS MANAGEMENT AND
INTERMARE MARITIME
AGENCIES INC
G.R. 179532, May 30, 2011
Topic: Constitutionality of
Laws
Providing
The
Employer
of
A
Migrant
Worker
The
Option
To
Choose And Award Lesser
Benefits In Case Of Dismissal
Facts:
Yap was employed as
electrician of a vessel, Intermare
Maritime Agencies, Inc. The
contract of employment entered
into by Yap and Capt. Adviento,
was for a duration of 12
months.
Yap boarded and

commenced
his
job
as
electrician. However, later the
vessel was sold. POEA was
informed about the sale in a
letter signed by Capt. Adviento.
Yap, along with the other
crewmembers, was informed by
the Master of their vessel that
the same was sold and will be
scrapped.
They were also
informed about the Advisory
sent by Capt. Constatinou,
which states if the crew wishes
to be transferred.
Yap received his bonuses.
However, with respect to the
payment of his wage, he refused
to accept the payment of onemonth basic wage. He insisted
that he was entitled to the
payment of the unexpired
portion of his contract since he
was illegally dismissed from
employment. He alleged that
he opted for immediate transfer
but none was made.
Respondents, contended
that Yap was not illegally
dismissed. They alleged that
the sale of the vessel, Yap
signed off from the vessel and
was
paid
his
wages
corresponding to the months he
worked
pluscertain bonuses.
They further alleged that Yaps
employment
contract
was
validly terminated due to the
sale of the vessel and no
arrangement was made for
Yaps transfer to Thenamaris
other vessels.
Yap
filed
for
illegal
dismissal in the LA which was
favored.
LA
found
that
respondents acted in bad faith
2011- 2012 Labor Law Case Digest Compilations
UC LLB III-B
Page 54

when they assured Yap of reembarkation and required him


to
produce
an
electrician
certificate during the period of
his contract, but actually he was
not able to board one despite of
respondents numerous vessels.
Petitioner made several followups for his re-embarkation but
respondents failed to heed his
plea; thus, Yap was forced to
litigate in order to vindicate his
rights. Lastly, the LA opined that
since the unexpired portion of
Yaps contract was less than one
year, Yap was entitled to his
salaries
for
the
unexpired
portion of his contract for a
period of nine months.
NLRC affirmed. The CA
affirmed
but
modified
the
penalty where Yap is to received
3 months of basic salary and not
the entire date stipulated in the
contract.
Issue:
Whether the 3 month
Section 10 of R.A. [No.] 8042, to
the extent that it affords an
illegally
dismissed
migrant
worker the lesser benefit of
salaries for [the] unexpired
portion of his employment
contract or for three (3) months
for every year of the unexpired
term, whichever is less is
constitutional.
Ruling:
It is unconstitutional
It is not disputed the Yaps
dismissal is illegal. Likewise not
disputed
is
the
tribunals

unanimous finding of bad faith


on the part of respondents.
What
remains
in
issue,
therefore, is the constitutionality
of the 5th paragraph of Section
10 of R.A. No. 8042 and,
necessarily,
the
proper
computation of the lump-sum
salary to be awarded to
petitioner by reason of his illegal
dismissal.
In the Serrano case, that
the clause or for three months
for every year of the unexpired
term,
whichever
is
less
provided in the 5th paragraph of
Section 10 of R.A. No. 8042 is
unconstitutional
for
being
violative of the rights of OFWs to
equal protection of the laws.
Moreover, this Court held
therein that the subject clause
does not state or imply any
definitive
governmental
purpose; hence, the same
violates
not
just
therein
petitioners
right
to
equal
protection, but also his right to
substantive
due
process
Consequently, Yap therein was
accorded his salaries for the
entire unexpired period of nine
months and 23 days of his
employment contract, pursuant
to law and jurisprudence prior to
the enactment of R.A. No. 8042.
36. APACIBLE VS MULTIMED
INDUSTIES INC.
G.R. 178903, May 30, 2011
Topics: Illegal Dismissal
Facts:
2011- 2012 Labor Law Case Digest Compilations
UC LLB III-B
Page 55

Apacible
was
hired
sometime byMultimed Industries
Incorporated (the company) as
Hospital Sales Representative.
She rose from the ranks to
become Assistant Area Sales
Manager for Cebu Operations,
the position she held at the time
she was separated from the
service.
Apaciblewas informed that
she would be transferred to the
companys main office in Pasig
City on account of the ongoing
reorganization. As the transfer
would entail major adjustments,
Apaciblerequested
that
her
transfer be made effective in
October or November 2003 and
that she be given time to
discuss it with her husband and
daughter.
A week later, Apacible was
informed that her transfer would
be effective August 18, 2003.
On even date, she was placed
under investigation for the
delayed released of BCRs (cash
budget
for
customer
representation
in
sealed
envelopes which are given to
loyal clients) which she received
for distribution earlier in July
2003.
In
her
written
explanation, admits that the
delay constituted a violation of
company policies, averred that
she forgot to endorse the BCRs
because she was thinking about
her impending transfer; and
that she did not misappropriate
the money as she had already
released the BCRs.
Finding that the delay in
releasing the BCRs amounted to

loss of trust and confidence,


Apaciblewas given the option to
resign. She reported to the
head office .
Thehuman resources gave her
four
options:
resignation,
termination, availment of an
early retirement package worth
P40,000, or transfer to Pasig
City. Without availing of any
option, Apacible took a leave of
absence.
Apaciblesent lettersto the
company
denouncing
their
meeting
as
illegal,
insensitive, inhumane and
Apaciblesdismissals
a
unilateral arrangement and
ruthless display of power.
Apacibledemanded payment of
separation pay and stated that
he had advised petitioner to
remain in her current position in
Cebu.
The
company
sent
Apaciblea
memorandumdirective for her to immediately
report to the head office in Pasig
City and to return the company
vehicle assigned to her to the
Cebu Office within 24 hours.
Apacibledid
not
heed
the
directive, however. She instead
filed an application for sick
leave.
Apaciblerequested that she be
given her daily work assignment
in Cebu, which request was later
to be denied by.Apaciblewas
given a show cause noticefor
her to explain in writing why she
should not be sanctioned for
insubordination for failure to
comply with the transfer order.
2011- 2012 Labor Law Case Digest Compilations
UC LLB III-B
Page 56

Apaciblethrough
Atty.
Montenegro,
wrote
the
respondent
company,
maintaining that she was not
transferring to Manila and that
if
the
company
want[ed]
petitioner out of the company,
separation pay must be paid.
The
company
denied
having pressured Apacibleas it
stressed that the transfer was
based on business demands and
did not entail a demotion in rank
nor diminution of benefits.
The company later on sent
Apaciblea
notice
of
termination,prompting
Apacibleto file a complaint for
illegal dismissal before the LA.
LA dismissed petition. NLRC
affirmed
Issue:
Whether there is illegal
dismissal.
Ruling:
There
is
no
illegal
dismissal.
Apacible was dismissed
for wilfully disobeying the lawful
order of her employer to
transfer from Cebu to Pasig
City.
The
appellate
court
says,Apacibleknew
and
accepted respondent companys
policy on transfers when she
was hired and was in fact even
transferred many times from
one area of operations to
another.
Apaciblesadamant refusal
to transfer, coupled with her
failure to heed the order for her
return the company vehicle

assigned to her and, more


importantly,
allowing
her
counsel to write letters couched
in harsh language to her
superiors unquestionably show
that
she
was
guilty
of
insubordination,
hence,
not
entitled to the award of
separation pay.
37. BANAHAW
BROADCASTINGCORPORATIO
N, Petitioner, vs.
CAYETANO PACANA III, NOE
U. DACER et al, Respondents
G.R. No. 171673,May 30,
2011
Topic: Perfection Of AppealPosting Of Appeal Bond By
GOCCs
Performing
Governmental Or Proprietal
Functions.
Facts:
DXWG is a radio station
owned by petitioner Banahaw
Broadcasting Corporation (BBC),
a corporation managed by
Intercontinental
Broadcasting
Corporation (IBC). On August 29,
1995, the DXWG personnel filed
complaint for illegal dismissal,
unfair
labor
practice,
reimbursement
of
unpaid
Collective Bargaining Agreement
(CBA) benefits, and attorneys
fees against IBC and BBC. On
June 21, 1996, the Labor Arbiter
rendered his Decisionawarding
the DXWG personnel a total of
P12,002,157.28 as unpaid CBA
benefits consisting of unpaid
wages and increases, 13th
month pay, longevity pay, sick
leave cash conversion, rice and
2011- 2012 Labor Law Case Digest Compilations
UC LLB III-B
Page 57

sugar subsidy, retirement pay,


loyalty reward and separation
pay.Other claims were denied.
Both parties appealed to the
NLRC. A motion to dismiss with
quitclaim was jointly filed by IBC
and the employees on the
ground that IBC is not their
employer as it does not own
DXWG. This was granted. So a
motion for reconsideration was
filed by BBC on the ground that
neither
BBC
or
its
duly
authorized representatives or
officers
were
served
with
summons or copy of the
complaint while the case was
pending before the labor arbiter.
As a result, the NLRC issued a
resolution vacating the decision
of the Labor Arbiter and
remanding the case to the
arbitration branch of origin. The
labor arbiter decided that BBC is
liable for the same amount.
Both the employees and
BBC appealed to the NLRC but it
denied BBCs appeal which
incorporated
a
Motion
for
recomputation of the monetary
award. NLRC ordered BBC to
post the required bond with a
warning that noncompliance will
cause dismissal or appeal for
non perfection. The BBC filed a
motion for reconsideration on
the ground that it is wholly
owned by the republic so it need
not to post a bond. The NLRC
dismissed both appeals. BBC
appealed to the Court of
Appeals.
The Court of appeals
denied BBCs appeal on the
ground that, though owned by

the
government,
it
is
a
corporation with a personality
distinct from the republic and
therefore do not partake in the
Republics
exception
from
posting of appeal bond.
Issue:
Whether BBC is exempt
from posting an appeal bond.
Ruling:
No. As a general rule, the
government
and
all
the
attached agencies with no legal
personality distinct from the
former are exempt from posting
appeal
bonds,
whereas
government-owned
and
controlled corporations (GOCCs)
are not similarly exempted. This
distinction is brought about by
the very reason of the appeal
bond itself: to protect the
presumptive judgment creditor
against the insolvency of the
presumptive judgment debtor.
When the State litigates, it is not
required to put up an appeal
bond because it is presumed to
be
always
solvent.
This
exemption, however, does not,
as a general rule, apply to
GOCCs for the reason that the
latter has a personality distinct
from its shareholders. Thus,
while
a
GOCCs
majority
stockholder, the State, will
always be presumed solvent,
the
presumption
does
not
necessarily extend to the GOCC
itself. However, when a GOCC
becomes
a
"government
machinery to carry out a
2011- 2012 Labor Law Case Digest Compilations
UC LLB III-B
Page 58

declared government policy,"it


becomes similarly situated as its
majority stockholder as there is
the
assurance
that
the
government
will
necessarily
fund its primary functions. Thus,
a GOCC that is sued in relation
to its governmental functions
may be, under appropriate
circumstances, exempted from
the payment of appeal fees.
In the case at bar, BBC
was organized as a private
corporation, sequestered in the
1980s and the ownership of
which
was
subsequently
transferred to the government in
a
compromise
agreement.
Further, it is stated in its
Amended
Articles
of
Incorporation that BBC has the
following primary function: To
engage in commercial radio and
television broadcasting, and for
this
purpose, to establish,
operate and maintain such
stations, both terrestrial and
satellite or interplanetary, as
may
be
necessary
for
broadcasting on a network wide
or international basis.
It is therefore crystal clear
that BBCs function is purely
commercial or proprietary and
not governmental. As such, BBC
cannot be deemed entitled to an
exemption from the posting of
an appeal bond.
In case of a judgment
involving a monetary award, an
appeal by the employer may be
perfected only upon the posting
of a cash or surety bond issued
by
a
reputable
bonding
company duly accredited by the

Commission in the amount


equivalent to the monetary
award in the judgment appealed
from.
The posting of the appeal
bond within the period provided
by law is not merely mandatory
but jurisdictional. The failure on
the part of BBC to perfect the
appeal thus had the effect of
rendering the judgment final
and executory.
38. RODOLFO LUNA,
Petitioner,
vs.ALLADO CONSTRUCTION
CO., INC., and/or RAMON
ALLADO, Respondents.
G.R. No. 175251, May 30,
2011
Topic: New Matters/ Issues
Raised On Appeal
Facts:
Luna alleged that he was
an
employee
of
Allado
consrtruction, having been a
part of the construction pool of
personnel. He had continuously
rendered
services
as
a
warehouseman
and
a
timekeeper
in
every
construction project undertaken.
Sometime in the afternoon of
November 24, 2001, while at
the construction site in Maasim,
Sarangani Province, he was
given a travel order dated
November 24, 2001 to proceed
to the main office in Davao City
for reassignment. Upon arrival
at the main office on November
26, 2001, he was told by one
Marilou
Matilano,
personnel
manager, to sign several sets of
2011- 2012 Labor Law Case Digest Compilations
UC LLB III-B
Page 59

"Contract
of
Project
Employment". He refused to
sign the said contracts. Because
of his refusal, he was not given
a reassignment or any other
work. These incidents prompted
him to file the complaint.
Allado Construction Co.,
on the other hand, alleged that
on November 29, 2001, Luna
applied for a leave of absence
until December 6, 2001, which
was granted. Upon expiration of
his leave, he was advised to
report to the companys project
in Kablacan, Sarangani Province.
However, he refused to report to
his new assignment and claimed
instead that he had been
dismissed illegally.
The
Labor
Arbiter
dismissed petitioners complaint
for illegal dismissal against
respondents, because it found
that Luna should be deemed to
have resigned. It ordered Allado
Construction to pay Luna the
amount of P18,000.00 by way of
financial assistance.
Only
respondents
interposed an appeal with the
National
Labor
Relations
Commission (NLRC), purely for
the purpose of questioning the
validity of the grant of financial
assistance made by the Labor
Arbiter. In its Resolutionof May
9, 2003, the NLRC reversed the
June 26, 2002 Decision of the
Labor Arbiter and declared
respondents guilty of illegal
dismissal and ordered them to
pay petitioner one-month salary
for every year of service as
separation pay, computed at

P170.00 per day and full


backwages from November 21,
2001 up to the finality of the
decision.
The respondents elevated
their cause to the Court of
Appeals via a petition for
certiorari under Rule 65 of the
Rules of Court. In its Decision
dated July 28, 2006, the Court of
Appeals granted respondents
petition for certiorari ruled that
the
assailed
Orders
of
respondent Commission are set
aside and the decision of the
labor arbiter is reinstated with
the modification that the award
of
financial
assistance
is
deleted.
The Court of Appeals
decision was based on the
ground that it was grave abuse
of discretion for the NLRC to rule
on the issue of illegal dismissal
when the only issue raised to it
on appeal was the propriety of
the
award
of
financial
assistance. The Court of Appeals
further ruled that financial
assistance may not be awarded
in
cases
of
voluntary
resignation.
Issue:
Whether NLRC, in the
exercise of its inherent powers
could still review issues not
brought to it before appeal.
Ruling:
No. Section 4(c), Rule VI
of the 2002 Rules of Procedure
of the NLRC, which was in effect
at
the
time
respondents
appealed the Labor Arbiters
2011- 2012 Labor Law Case Digest Compilations
UC LLB III-B
Page 60

decision, expressly provided


that, on appeal, the NLRC shall
limit itself only to the specific
issues that were elevated for
review, to wit:
RULEVI.
Appeals.Section
4.Requisites for Perfection of
Appeal.x x x.
(c) Subject to the provisions of
Article 218, once the appeal is
perfected in accordance with
these Rules, the Commission
shall limit itself to reviewing and
deciding specific issues that
were elevated on appeal.
As a testament to its
effectivity and the NLRCs
continued implementation of
this procedural policy, the same
provision
was
retained
as
Section 4(d), Rule VI of the 2005
Revised Rules of Procedure of
the NLRC.
In the case at bar, the
NLRC evidently went against its
own rules of procedure when it
passed upon the issue of illegal
dismissal although the question
raised by respondents in their
appeal was concerned solely
with the legality of the labor
arbiters award of financial
assistance despite the finding
that petitioner was lawfully
terminated.
39. MIGUEL DELA PENA
BARAIRO, Petitioner,
vs.OFFICE OF THE
PRESIDENT and MST MARINE
SERVICES (PHILS.), INC.
Respondent.
G.R. No. 189314, June 15,
2011

Topic: Appealing
Of The Secretary
Who
has
Jurisdiction Over
of the POEA.

Decisions
Of Labor/
Appellate
Decisions

Facts:
Miguel Barairo was hired
on June 29, 2004 by MST Marine
Services Inc., (MST) for its
principal,
TSM
International,
Ltd., as Chief Mate of the vessel
Maritina, for a contract period of
six months. He boarded the
vessel and discharged his duties
on July 23, 2004, but was
relieved on August 28, 2004
ostensibly
for
transfer
to
another vessel, Solar. Petitioner
thus disembarked in Manila on
August 29, 2004.
Barairo was later to claim
that he was not paid the
promised "stand-by fee" in lieu
of salary that he was to receive
while
awaiting
transfer
to
another vessel as in fact the
transfer never materialized.
On October 20, 2004,
Barairo signed a new Contract of
Employmentfor
a
six-month
deployment as Chief Mate in a
newly-built Japanese vessel, M/T
Haruna. He was paid a onemonth
"standby
fee"
in
connection with the Maritina
contract.
Barairo boarded the M/T
Haruna on October 31, 2004 but
he disembarked a week later as
MST claimed that his boarding
of M/T Haruna was a "sea trial"
which, MST maintains, was
priorly made known to him on a
"stand-by"
fee.
MST
soon
2011- 2012 Labor Law Case Digest Compilations
UC LLB III-B
Page 61

informed Barairo that he would


be redeployed to the M/T
Haruna on November 30, 2004,
but
petitioner
refused,
prompting
MST
to
file
a
complaintfor breach of contract
against
him
before
the
Philippine
Overseas
Employment
Administration
(POEA).
Petitioner
claimed,
however, that he was placed on
"forced vacation" when he was
made to disembark from the M/T
Haruna, and that not wanting to
experience a repetition of the
previous "termination" of his
employment
aboard
the
Maritina, he refused to be
redeployed to the M/T Haruna.
By Order of April 5, 2006,
then
POEA
Administrator
Rosalinda D. Baldoz penalized
Barairo
with
one
year
suspension
from
overseas
deployment upon a finding that
his refusal to complete his
contract aboard the M/T Haruna
constituted a breach thereof.
Barairo appealed to the
Secretary of Labor, by Orderof
September 22, 2006, noting that
it was petitioners first offense,
modified the POEA Order by
shortening
the
period
of
suspension from one year to six
months.
The Office of the President
(OP), by Decisionof November
26, 2007, dismissed petitioners
appeal for lack of jurisdiction.
The OP held that appeals to it in
labor
cases,
except
those
involving national interest, have
been eliminated.

Issue:
Whether it is proper for
the petitioner to question the
decision or orders of the
Secretary of Labor via an appeal
to the Office of the President.
Ruling:
No.Following
settled
jurisprudence,
the
proper
remedy
to
question
the
decisions or orders of the
Secretary of Labor is via Petition
for Certiorari under Rule 65, not
via an appeal to the OP. For
appeals to the OP in labor cases
have indeed been eliminated,
except those involving national
interest
over
which
the
President
may
assume
jurisdiction.
It cannot be gainsaid that
petitioners
case does not
involve national interest.
Petitioners appeal of the
Secretary of Labors Decision to
the Office of the President did
not toll the running of the
period, hence, the assailed
Decisions of the Secretary of
Labor are deemed to have
attained finality.
Although appeal is an
essential part of our judicial
process, it has been held, time
and again, that the right thereto
is not a natural right or a part of
due process but is merely a
statutory privilege. Thus, the
perfection of an appeal in the
manner and within the period
prescribed by law is not only
mandatory
but
also
jurisdictional and failure of a
2011- 2012 Labor Law Case Digest Compilations
UC LLB III-B
Page 62

party to conform to the rules


regarding appeal will render the
judgment final and executory.
Once a decision attains finality,
it becomes the law of the case
irrespective of whether the
decision is erroneous or not and
no court - not even the Supreme
Court - has the power to revise,
review, change or alter the
same.
40. UNIVERSITY PLANS
INCORPORATED, Petitioner,
vs.BELINDA P. SOLANO, et
al., Respondents.
G.R. No. 170416, June 22,
2011
Topic: Perfection Of Appeal
Facts:
Respondents Belinda P.
Solano et al. filed before the
Labor Arbiter complaints for
illegal
dismissal,
illegal
deductions,
overriding
commissions,
unfair
labor
practice, moral and exemplary
damages, and actual damages
against petitioner University
Plans Incorporated.
The Labor Arbiter found
petitioner
guilty
of
illegal
dismissal
and
ordered
respondents reinstatement as
well as the payment of their full
backwages, proportionate 13th
month pay, moral/exemplary
damages, and attorneys fees.
University
Plans
filed
before
the
NLRC
its
Memorandum on Appeal as well
as a Motion to Reduce Bond.
Simultaneous with the filing of

said pleadings, it posted a cash


bond
in
the
amount
of
P30,000.00. In its Motion to
Reduce Bond, petitioner alleged
that it was under receivership
and that it cannot dispose of its
assets at such a short notice.
Because of this, it could not post
the required bond. Nevertheless,
it has P30,000.00 available for
immediate disposition and thus
prayed that said amount be
deemed sufficient to satisfy the
required bond for the perfection
of its appeal.
The
NLRC
denied
University Plans Motion to
Reduce Bond and directed it to
post an additional appeal bond
in the amount of P3,013,599.50
within an unextendible period of
10 days from notice, otherwise
the appeal shall be dismissed
for non-perfection. In resolving
the motion, the NLRC held that
the amount of the appeal bond
is fixed by law pursuant to
Article 223 of the Labor Code.
University Plans Motion
for Reconsideration was denied
by the NLRC, so it resorted to an
appeal to the Court of Appeals
which was denied. The CA held
that the NLRC in meritorious
cases and upon motion by the
appellant
may
reduce
the
amount of the bond but the
petitioner should show veritable
proof that it is entitled to the
same. Since petitioner failed to
provide the NLRC with sufficient
basis to determine its incapacity
to post the required appeal
bond, the CA opined that the
NLRCs denial of petitioners
2011- 2012 Labor Law Case Digest Compilations
UC LLB III-B
Page 63

Motion to Reduce Bond was


justified.
Issues:
1. Whether posting of
bond is indispensable to the
perfection of an appeal in cases
involving monetary awards from
the Decision of the Labor
Arbiter.
2. What are the instances
when the NLRC may reduce the
amount of bond?
RULING:
First Issue.Yes.Article 223
of the Labor Code provides in
part:
Article 223.Appeal. Decisions,
awards, or orders of the Labor
Arbiter are final and executory
unless
appealed
to
the
Commission by any or both
parties within ten (10) calendar
days from receipt of such
decisions, awards, or orders. x x
x
x x x xIn case of a judgment
involving a monetary award, an
appeal by the employer may be
perfected only upon the posting
of a cash or surety bond issued
by
a
reputable
bonding
company duly accredited by the
Commissionin
the
amount
equivalent to the monetary
award in the judgment appealed
from.
Second
Issue.Under
Section 6, Rule VI of the NLRCs
Revised Rulesof Procedure, the
bond may be reduced albeit
only on meritorious grounds and

upon posting of a partial bond in


a reasonable amount in relation
to the monetary award. Suffice
it to state that while said Rules
"allows the Commission to
reduce the amount of the bond,
the exercise of the authority is
not a matter of right on the part
of the movant, but lies within
the sound discretion of the NLRC
upon a showing of meritorious
grounds."
In
Nicol
v.
Footjoy
Industrial Corporation,the Court
reviewed
the
jurisprudence
respecting
the
bond
requirement
for
perfecting
appeal and summarized the
guidelines under which the
NLRC
must
exercise
its
discretion in considering an
appellants motion for reduction
of bond, viz:
The bond requirement on
appeals
involving
monetary
awards has been and may be
relaxed in meritorious cases.
These cases include instances in
which (1) there was substantial
compliance with the Rules, (2)
surrounding
facts
and
circumstances
constitute
meritorious grounds to reduce
the
bond,
(3)
a
liberal
interpretation
of
the
requirement of an appeal bond
would
serve
the
desired
objective
of
resolving
controversies on the merits, or
(4) the appellants, at the very
least, exhibited their willingness
and/or good faith by posting a
partial
bond
during
the
reglementary period.
2011- 2012 Labor Law Case Digest Compilations
UC LLB III-B
Page 64

41. GENERAL MILLING


CORPORATION,
Petitioner,vs.GENERAL
MILLING CORPORATIONINDEPENDENT LABOR UNION
(GMC-ILU), ET.
AL,Respondents.
G.R. No. 183889, June 15,
2011
Topic: Duration/Efficacy Of A
Collective
Bargaining
Agreement
FACTS:
On 28 April 1989, GMC
and the Union entered into a
collective bargaining agreement
(CBA) which provided that the
unions representation of the
collective bargaining unit for a
three-year term is made to
retroact to 1 December 1988.
On 29 November 1991 or
one day before the expiration of
the subject CBA, the Union sent
a draft CBA proposal to GMC,
with a request for counterproposals from the latter, for the
purpose of renegotiating the
existing
CBA
between
the
parties. GMC failed to comply
with said request. This resulted
to a complaint for unfair labor
practice by the union but which
was dismissed for lack of merit
in
a
decision
dated
21
December 1993 issued by the
Regional Arbitration Branch-VII
(RAB-VII) of the National Labor
Relations Commission (NLRC).
The Union appealed to the
NLRC. On its January 30, 1998
decision, it reversed and set
aside the dismissal. The NLRC
ruled that unions draft CBA

proposal be imposed for the


remaining two years duration of
the original CBA which is from
December 1, 1991 to November
30, 1993. GMC filed a motion for
reconsideration
which
was
granted in an NLRC decision of
October 6, 1998. In a decision
dated 19 July 2000, the CA
reversed and set aside the
NLRCs resolution and reinstated
the 30 January 1998 decision. A
motion for Reconsideration by
GMC was denied by the CA so
GMC elevated the case to the
Supreme Court via the petition
for review on certiorari. In a
decision dated February 11,
2004 by the SC, the CAs
January 30, 1998 decision (draft
proposal to be imposed for the
remaining 2 yrs.) was affirmed.
It also held that the Company
was liable for unfair labor
practice for violating its duty to
bargain collectively.
The Union filed for the
issuance of a motion for
execution to enforce the claims
of the employees which was
computed at P 433, 786,786.36.
GMC opposed. Later on the
Labor Arbiter issued an order
limiting the computation of the
benefits to the remaining 2 yrs.
of the duration of the original
CBA. The Union filed a partial
appeal. The NLRC on its July 20,
2006 decision affirmed the
October 27, 2005 order of
execution.
GMC and the union file separate
rule 65 petitions before the CA.
the
Unions
petition
was
partially granted. The CA refered
2011- 2012 Labor Law Case Digest Compilations
UC LLB III-B
Page 65

the case to the Grievance


Machinery under the imposed
CBA and directed the exclusion
of
some
items
from
the
computation. The Union filed a
motion for reconsideration but
this was denied so it resorted to
a Petition for Review under Rule
45 before the Supreme Court.
Issue:
Whether the CBA should
be imposed for a period of 5
years.
Ruling:
Yes. Anent its period of
effectivity, Article XIV of the
imposed CBA provides that
"(t)his Agreement shall be in full
force and effect for a period of
five (5) years from 1 December
1991, provided that sixty (60)
days prior to the lapse of the
third year of effectivity hereof,
the
parties
shall
open
negotiations
on
economic
aspect for the fourth and fifth
years
effectivity
of
this
Agreement." Considering that
no new CBA had been, in the
meantime, agreed upon by GMC
and the Union, we find that the
CAs Special Twentieth Division
correctly ruled in CA-G.R. CEBSP No. 02226 that, pursuant to
Article 253 of the Labor Code,
the provisions of the imposed
CBA continues to have full force
and effect until a new CBA has
been entered into by the
parties. Article 253 mandates
the parties to keep the status
quo and to continue in full force
and effect the terms and

conditions
of
the
existing
agreement during the 60-day
period prior to the expiration of
the old CBA and/or until a new
agreement is reached by the
parties. In the same manner
that it does not provide for any
exception nor qualification on
which economic provisions of
the existing agreement are to
retain its force and effect,the
law
does
not
distinguish
between a CBA duly agreed
upon by the parties and an
imposed CBA like the one under
consideration.
42. WILLIAM ENDELISEO
BARROGA, Petitioner,
vs.DATA CENTER COLLEGE
OF THE PHILIPPINES and
WILFRED
BACTAD,Respondents
Topic: Perfection Of AppealCompliance
With
Formal
Requisites/
Substantial
Compliance
With
Formal
Requisites Of An Appeal
FACTS:
On November 11, 1991,
Barogga was employed as an
Instructor
in
Data
Center
College Laoag City branch in
Ilocos Norte then he
was
transferred him to University of
Northern Philippines (UNP) in
Vigan, Ilocos Sur where the
school had a tie-up program. In
1994, he was recalled to Laoag
campus. On October 3, 2003, he
was transferred again to Data
Center Abra branch as Head for
Education/Instructor
but
he
declined to accept his transfer.
2011- 2012 Labor Law Case Digest Compilations
UC LLB III-B
Page 66

On November 10, 2003,


Barogga filed a Complaint for
constructive dismissal against
Data Center. He alleged that his
proposed
transfer
to
Abra
constitutes a demotion in rank
and diminution in pay and would
cause personal inconvenience
and hardship.
On September 24, 2004,
the Labor Arbiter rendered a
Decisiondismissing
the
Complaint for lack of merit. The
Labor Arbiter ruled that there
was no demotion in rank.
Barroga appealed to the
NLRC. In a Decisiondated August
25, 2005, the NLRC affirmed the
findings of the Labor Arbiter that
there was no constructive
dismissal. It ruled that the
management
decision
to
transfer petitioner was well
within the rights of respondents
in consonance with petitioners
contract of employment and
which was not sufficiently shown
to
have
been
exercised
arbitrarily by respondents.
Both parties filed petitions
for certiorari before the CA. On
May 15, 2006, the CA dismissed
Baroggas
petition
in
a
Resolution due to the following
infirmities:
1. there is no statement of
material dates as to when the
petitioner received the assailed
decision dated August 25, 2005
and when he filed a Motion for
Reconsideration thereof;
2. there is no affidavit of
service attached to the petition;
3.
these
initiatory
pleadings and the respondents

Motion for Reconsideration of


the Decision dated August 25,
2005 are not attached to the
petition.
Barogga filed a Motion for
Reconsiderationalleging that the
material dates of receipt of the
NLRC Decision and the filing of
his motion for reconsideration
are explicitly stated in his Partial
Motion
for
Reconsideration
which was attached as an annex
to the petition and was made an
integral part thereof. As to the
absence of the affidavit of
service, he argued that there is
no legal impediment for the
belated
admission
of
the
affidavit of service as it was duly
filed before the dismissal of the
petition. As for his failure to
attach respondents motion for
reconsideration,
petitioner
manifested that a separate
petition for certiorari has been
filed by respondents and is
pending with the CA, docketed
as CA-G.R. SP No. 94205, where
the denial of said motion is at
issue.
On August 4, 2006, the CA
denied Barogas motion for lack
of merit due to non-compliance
despite opportunity afforded to
comply, hence a petition with
the Supreme Court.
Issue:
Whether an appeal may
be dismissed for failure to
comply
with
formal
requirements.
Ruling:
2011- 2012 Labor Law Case Digest Compilations
UC LLB III-B
Page 67

No. Petitioners substantial


compliance
callsfor
the
relaxation
of
the
rules.
Therefore, the CA should have
given due course to the petition.
The three material dates
which should be stated in the
petition for certiorari under Rule
65 are the dates when the
notice of the judgment was
received, when a motion for
reconsideration was filed and
when the notice of the denial of
the motion for reconsideration
was
received.These
dates
should be reflected in the
petition to enable the reviewing
court to determine if the petition
was filed on time. In Acaylar, Jr.
v. Harayo, we held that failure to
state these two dates in the
petition may be excused if the
same are evident from the
records of the case. It was
further ruled by this Court that
the more important material
date which must be duly alleged
in the petition is the date of
receipt of the resolution of
denial of the motion for
reconsideration. In the case at
bar,
petitioner
has
duly
complied with this rule.
Next, the CA dismissed
the petition for failure to attach
an affidavit of service. However,
records show that petitioner
timely rectified this omission by
submitting the required affidavit
of service even before the CA
dismissed his petition.
The Court has time and
again upheld the theory that the
rules of procedure are designed
to secure and not to override

substantial justice.These are


mere tools to expedite the
decision or resolution of cases,
hence, their strict and rigid
application which would result in
technicalities
that
tend
to
frustrate rather than promote
substantial justice must be
avoided. The CA thus should not
have
outrightly
dismissed
petitioners petition based on
these procedural lapses.
43. JOEB M. ALIVIADO et al,
Petitioners, vs PROCTER &
GAMBLE PHILS., INC., and
PROMM-GEM INC.,
Respondents.
G.R. No. 160506 June 6,
2011
Topic:
Labor-Only
Contracting;
EmployerEmployee Relationship
Facts:
Procter & Gamble Phils.
Inc. is principally engaged in the
manufacture and production of
different consumer and health
products which sells on a
wholesale basis to various
supermarkets and distributors.
To
enhance
consumer
awareness and acceptance of
the products, P&G entered into
contracts with Promm-Gem and
SAPS for the promotion and
merchandising of its products.
Aliviado et al. worked as
merchandisers of P&G from
various
dates.
They
all
individually signed employment
contracts with either PrommGem Inc. (Promm-Gem) or Sales
and Promotions Services (SAPS)
2011- 2012 Labor Law Case Digest Compilations
UC LLB III-B
Page 68

for periods of more or less five


months at a time. They were
assigned at different outlets,
supermarkets and stores where
they handled all the products of
P&G. They received their wages
from Promm-Gem or SAPS.
SAPS and Promm-Gem imposed
disciplinary measures on erring
merchandisers for reasons such
as
habitual
absenteeism,
dishonesty or changing day-off
without prior notice. Aliviado et
al. filed a complaint against P&G
for
regularization,
service
incentive leave pay and other
benefits with damages. As a
result of filing the complaint,
Aliviado
et
al.
were
subsequently dismissed, and
eventually their complaint was
amended
to
include
their
dismissal.
The Labor Arbiter
dismissed the complaint for lack
of merit and ruled that there
was
no
employer-employee
relationship between Aliviado et
al. and P&G. He found that the
selection and engagement of
the Aliviado et al., the payment
of their wages, the power of
dismissal and control with
respect to the means and
methods by which their work
was accomplished, were all
done and exercised by PrommGem/SAPS. He further found
that Promm-Gem and SAPS were
legitimate
independent
job
contractors. Upon appeal, the
NLRC affirmed the decision of
the
Labor
Arbiter.
The
subsequent
Motion
for
Reconsideration
was
also
denied. Aliviado et al. then filed

a petition for certiorari with the


Court of Appeals, alleging grave
abuse of discretion amounting
to lack or excess of jurisdiction
on the part of the Labor Arbiter
and the NLRC; but the same was
denied by the CA.
Contention of aliviado et
al.: They insist that they are
employees of P&G. They claim
that they were recruited by the
salesmen of P&G and were
engaged
to
undertake
merchandising chores for P&G
long before the existence of
Promm-Gem and/or SAPS. They
further assert that Promm-Gem
and
SAPS
are
labor-only
contractors providing services of
manpower to their client. They
claim that the contractors have
neither substantial capital nor
tools
and
equipment
to
undertake independent labor
contracting. They insist that
since they had been engaged to
perform activities which are
necessary or desirable in the
usual business or trade of P&G,
then they are its regular
employees.
Contention Of Procter &
Gamble: , P&G argues that there
is no employment relationship
between it and Aliviado et al. It
was Promm-Gem or SAPS that
selected
petitioners
and
engaged their services; paid
their salaries; wielded the power
of dismissal; and had the power
of control over their conduct of
work.
Issue:
2011- 2012 Labor Law Case Digest Compilations
UC LLB III-B
Page 69

1. Whether Promm-Gem
and SAPS are labor-only
contractors or legitimate job
contractors.
2. Whether the
employees were illegally
dismissed.
Ruling:
First Issue. Promm-gem is
a legitimate job contractor,
while saps is a labor-only
contractor. There is "labor-only"
contracting where the person
supplying
workers
to
an
employer
does
not
have
substantial capital or investment
in the form of tools, equipment,
machineries, work premises,
among others, and the workers
recruited and placed by such
person are performing activities
which are directly related to the
principal
business
of
such
employer. In such cases, the
person or intermediary shall be
considered merely as an agent
of the employer who shall be
responsible to the workers in the
same manner and extent as if
the
latter
were
directly
employed by him. In legitimate
contracting, there exists a
trilateral
relationship
under
which there is a contract for a
specific job, work or service
between the principal and the
contractor or subcontractor, and
a contract of employment
between the contractor or
subcontractor and its workers.
The current labor rules expressly
prohibit labor-only contracting.
Promm-Gem has substantial
investment which relates to the

work to be performed.
The
records also show that PrommGem supplied its complainantworkers
with
the
relevant
materials necessary for them to
perform their work, as well as
their uniforms. It is also relevant
to mention that Promm-Gem
already
considered
the
complainants working under it
as its regular, not merely
contractual
or
project,
employees.
Under
the
circumstances,
Promm-Gem
cannot be considered as a laboronly contractor, rather it is a
legitimate
independent
contractor. On the other hand,
SAPS has no substantial capital,
has no evidence presented to
show how much its working
capital
and
assets
are.
Furthermore,
there
is
no
showing
of
substantial
investment in tools, equipment
or other assets. Furthermore,
the
petitioners
have
been
charged with the merchandising
and promotion of the products
of P&G, an activity that has
already been considered by the
Court as doubtlessly directly
related to the manufacturing
business, which is the principal
business of P&G. Considering
that SAPS has no substantial
capital or investment and the
workers
it
recruited
are
performing activities which are
directly related to the principal
business of P&G, it is found that
the former is engaged in "laboronly contracting". Where laboronly contracting exists, the
Labor Code itself establishes an
2011- 2012 Labor Law Case Digest Compilations
UC LLB III-B
Page 70

employer-employee relationship
between the employer and the
employees of the labor-only
contractor."
The
statute
establishes this relationship for
a comprehensive purpose: to
prevent a circumvention of labor
laws.
The
contractor
is
considered merely an agent of
the principal employer and the
latter is responsible to the
employees of the labor-only
contractor as if such employees
had been directly employed by
the principal employer.
Second Issue. Yes, the
employees
were
illegally
dismissed. In cases of regular
employment, the employer shall
not terminate the services of an
employee except for a just or
authorized cause. In the instant
case, the termination letters
given by Promm-Gem to its
employees uniformly specified
the cause of dismissal as grave
misconduct and breach of trust.
Misconduct has been defined as
improper or wrong conduct; the
transgression
of
some
established and definite rule of
action, a forbidden act, a
dereliction of duty, unlawful in
character
implying
wrongful
intent and not mere error of
judgment. The misconduct to be
serious must be of such grave
and aggravated character and
not
merely
trivial
and
unimportant. To be a just cause
for dismissal, such misconduct
(a) must be serious; (b) must
relate to the performance of the
employees duties; and (c) must

show that the employee has


become
unfit
to
continue
working for the employer. A
misconduct which is not serious
or grave cannot be a valid basis
for dismissing an employee.
Meanwhile, loss of trust and
confidence, as a ground for
dismissal, must be based on the
willful breach of the trust
reposed in the employee by his
employer. Ordinary breach will
not suffice. A breach of trust is
willful if it is done intentionally,
knowingly
and
purposely,
without justifiable excuse, as
distinguished from an act done
carelessly,
thoughtlessly,
heedlessly or inadvertently. Loss
of trust and confidence, as a
cause
for
termination
of
employment, is premised on the
fact
that
the
employee
concerned holds a position of
responsibility or of trust and
confidence. As such, he must be
invested with confidence on
delicate
matters,
such
as
custody, handling or care and
protection of the property and
assets of the employer. And, in
order to constitute a just cause
for
dismissal,
the
act
complained of must be workrelated and must show that the
employee is unfit to continue to
work for the employer. In the
instant case, the petitionersemployees of Promm-Gem have
not been shown to be occupying
positions of responsibility or of
trust and confidence. Neither is
there any evidence to show that
they are unfit to continue to
2011- 2012 Labor Law Case Digest Compilations
UC LLB III-B
Page 71

work as merchandisers
Promm-Gem.

for

44. ROMEO VILLARUEL,


Petitioner, vs. YEO HAN
GUAN, doing business under
the name and style YUHANS
ENTERPRISES, Respondent.
G.R. No. 169191 June 1,
2011
Topic: Illegal Dismissal;
Separation Pay
Facts:
Villaruel filed with the
NLRC, National Capital Region,
Quezon City a Complaint for
payment of separation pay
against
Yuhans
Enterprises.
Villaruel was employed as a
machine operator by Ribonette
Manufacturing Company, an
enterprise engaged in the
business of manufacturing and
selling PVC pipes and is owned
and managed by Yeo Han Guan.
Over twenty (20) years, the
company changed its name four
times. From 1993 to 1999, the
company was operating under
the name of Yuhans Enterprises.
Despite the changes in the
company's
name,
petitioner
remained in the employ of
Guan. Villaruel further alleged
that on he got sick and was
confined in a hospital and he
subsequently reported for work
but was no longer permitted to
go back because of his illness.
He asked that Guan allow him to
continue
working
but
be
assigned a lighter kind of work
but his request was denied.

Instead, he was offered a sum of


P15,000.00 as his separation
pay. However, the amount
corresponds only to the period
between
1993
and
1999.
Villaruel prayed that he be
granted
separation
pay
computed from his first day of
employment in June 1963, but
Guan
refused.
Aside
from
separation pay, Villaruel prayed
for the payment of service
incentive leave for three years
as well as attorney's fees. On
the other hand, Guan contended
that Villaruel was hired as
machine operator from March 1,
1993 until he stopped working
sometime in February 1999 on
the ground that he was suffering
from illness. After his recovery,
Villaruel was directed to report
for work, but he never showed
up. Guan was later caught by
surprise when Villaruel filed the
instant case for recovery of
separation pay. Guan claimed
that he never terminated the
services of Villaruel and that
during
their
mandatory
conference, he even told the
latter that he could go back to
work anytime. Villaruel clearly
manifested that he was no
longer interested in returning to
work and instead asked for
separation pay.
The Labor
Arbiter
handling
the
case
rendered judgment in favor of
Villaruel.
During appeal, the
NLRC dismissed Guans appeal
and its subsequent motion for
reconsideration, and affirmed
the Labor Arbiters decision.
Guan then filed a petition for
2011- 2012 Labor Law Case Digest Compilations
UC LLB III-B
Page 72

certiorari under Rule 65 to the


Court of Appeals, and the CA
partially grants the petition
deleting
the
award
for
separation pay. Villaruel filed a
Motion for Reconsideration to
the CA but the same was
denied.
Issue:
1. Whether Villaruel was
illegally dismissed from his
employment.
2. Whether Villaruel is
entitled to his separation pay.
Ruling:
First Issue. No, There Was
No Dismissal Involved. Guan did
not
terminate
Villaruel's
employment.
First, the only
cause of action in Villaruel's
original complaint is that he was
"offered a very low separation
pay".
Second, there was no
allegation of illegal dismissal,
both in Villaruel's original and
amended
complaints
and
position paper. Lastly, there was
no prayer for reinstatement.
The Court finds that Villaruel
was the one who initiated the
severance of his employment
relations with Guan. It is evident
from the various pleadings filed
by Villaruel that he never
intended to return to his
employment with Guan on the
ground that his health is failing.
Indeed, Villaruel did not ask for
reinstatement.
In
fact,
he
rejected Guan's offer for him to
return
to
work.
This
is
tantamount
to
resignation.
Resignation is defined as the

voluntary act of an employee


who finds himself in a situation
where he believes that personal
reasons cannot be sacrificed in
favor of the exigency of the
service and he has no other
choice but to disassociate
himself from his employment. It
may not be amiss to point out at
this juncture that aside from
Article 284 of the Labor Code,
the award of separation pay is
also authorized in the situations
dealt with in Article 283 of the
same Code and under Section 4
(b), Rule I, Book VI of the
Implementing
Rules
and
Regulations of the said Code
where there is illegal dismissal
and reinstatement is no longer
feasible. By way of exception,
this Court has allowed grants of
separation pay to stand as "a
measure of social justice" where
the
employee
is
validly
dismissed for causes other than
serious misconduct or those
reflecting
on
his
moral
character.
Second Issue. No, he is
not entitled to separation pay.
There is no provision in the
Labor
Code
which
grants
separation pay to voluntarily
resigning employees. In fact, the
rule is that an employee who
voluntarily
resigns
from
employment is not entitled to
separation pay, except when it
is stipulated in the employment
contract or CBA, or it is
sanctioned
by
established
employer practice or policy. In
the present case, neither the
2011- 2012 Labor Law Case Digest Compilations
UC LLB III-B
Page 73

abovementioned provisions of
the
Labor
Code
and
its
implementing
rules
and
regulations nor the exceptions
apply because Villaruel was not
dismissed from his employment
and there is no evidence to
show
that
payment
of
separation pay is stipulated in
his employment contract or
sanctioned
by
established
practice or policy of herein
Guan, his employer. Since
Villaruel was not terminated
from his employment and,
instead, is deemed to have
resigned therefrom, he is not
entitled to separation pay under
the provisions of the Labor
Code.
**
However,
it
is
noteworthy that considering
Villaruels length of service and
his failing physical condition, the
Supreme Court granted him
financial assistance (amounting
to P50,000), citing Eastern
Shipping Lines vs Sedan and
Eastern
Shipping Lines
vs
Antonio as their legal bases;
which is just and equitable
under the circumstances.

45. YOLITO FADRIQUELAN ET


AL, Petitioners, vs.
MONTEREY FOODS
CORPORATION, Respondent.
G.R. No. 178434
x-------------------- - -x
MONTEREY FOODS
CORPORATION, Petitioner,

vs. BUKLURAN NG MGA


MANGGAGAWA SA
MONTEREY-ILAW AT BUKLOD
NG MANGGAGAWA, YOLITO
FADRIQUELAN, ET AL.
Respondents.
G.R. No. 178409
June 8, 2011
Topic:
Illegal
Dismissal;
Illegal Acts Of Union Officers
As
Valid
Grounds
For
Dimissal
Facts:
On April 30, 2002 the
three-year collective bargaining
agreement or CBA between the
union Bukluran ng Manggagawa
sa Monterey-Ilaw at Buklod ng
Manggagawa (the union) and
Monterey Foods Corporation
(the company) expired. On
March 28, 2003 after the
negotiation for a new CBA
reached a deadlock, the union
filed a notice of strike with the
National
Conciliation
and
Mediation Board (NCMB). To
head off the strike, on April 30,
2003 the company filed with the
DOLE a petition for assumption
of jurisdiction over the dispute
in view of its dire effects on the
meat industry. In an Order dated
May 12, 2003, the DOLE
Secretary assumed jurisdiction
over the dispute and enjoined
the union from holding any
strike. It also directed the union
and the company to desist from
taking any action that may
aggravate the situation. On May
21, 2003 the union filed a
second notice of strike before
the NCMB on the alleged ground
2011- 2012 Labor Law Case Digest Compilations
UC LLB III-B
Page 74

that the company committed


unfair labor practices. On June
10, 2003 the company sent
notices to the union officers,
charging them with intentional
acts of slowdown. Six days later,
the company sent new notices
to the union officers, informing
them of their termination from
work for defying the DOLE
Secretarys assumption order.
On June 23, 2003, acting on
motion of the company, the
DOLE Secretary included the
unions second notice of strike
in his earlier assumption order.
But, on the same day, the union
filed a third notice of strike
based on allegations that the
company had engaged in union
busting and illegal dismissal of
union officers. On July 7, 2003
the company filed a petition for
certification of the labor dispute
to the National Labor Relations
Commission
(NLRC)
for
compulsory arbitration but the
DOLE Secretary denied the
motion. He, however, subsumed
the third notice of strike under
the first and second notices. On
November 20, 2003 the DOLE
rendered a decision that upheld
the companys termination of
the 17 union officers. The union
and its officers appealed the
decision to the Court of Appeals
(CA) and upheld the validity of
the companys termination of 10
union officers but declaring
illegal that of the other seven.
Both parties sought recourse to
the Supreme Court, the union in
G.R. 178409 and the company
in G.R. 178434.

Issue:
1. Whether the slowdowns
actually
transpired
at
the
companys farms.
2. Whether the union
officers committed illegal acts
that warranted their dismissal
from work.
3. Whether there was
illegal dismissal.
Ruling:
First Issue.
Yes, the
slowdowns transpired. The law
is explicit: no strike shall be
declared after the Secretary of
Labor has assumed jurisdiction
over a labor dispute. A strike
conducted
after
such
assumption is illegal and any
union officer who knowingly
participates in the same may be
declared as having lost his
employment. Here, what is
involved is a slowdown strike.
Unlike other forms of strike, the
employees
involved
in
a
slowdown do not walk out of
their jobs to hurt the company.
They need only to stop work or
reduce the rate of their work
while generally remaining in
their assigned post. It is found
that the union officers and
members in this case held a
slowdown
strike
at
the
companys farms despite the
fact that the DOLE Secretary
had on May 12, 2003 already
assumed jurisdiction over their
labor dispute. The evidence
sufficiently shows that union
officers
and
members
simultaneously stopped work at
2011- 2012 Labor Law Case Digest Compilations
UC LLB III-B
Page 75

the companys Batangas and


Cavite farms at 7:00 a.m. on
May 26, 2003.
There is no
allegation that the company
prevented
the
union
from
holding meetings after working
hours.
Second Issue.
Yes. A
distinction
exists,
however,
between the ordinary workers
liability for illegal strike and that
of the union officers who
participated in it. The ordinary
worker cannot be terminated for
merely participating in the
strike. There must be proof that
he committed illegal acts during
its conduct. On the other hand,
a
union
officer
can
be
terminated upon mere proof
that he knowingly participated
in the illegal strike. The Court
sustains the validity of the
termination of the rest of the
union officers. The identity and
participations
of
Arturo
Eguna,Armando
Malaluan,
Danilo Alonso, Romulo Dimaano,
Roel Mayuga, Wilfredo Rizaldo,
Romeo
Suico,
Domingo
Escamillas, and Domingo Bautro
in the slowdowns were properly
established.
These
officers
simply refused to work or they
abandoned their work to join
union assemblies.
Third Issue. Yes, in some
of the named employees. The
court held that the dismissal of
Alberto Castillo, Nemesio Agtay,
Carlito
Abacan,
and
Yolito
Fadriquelan are illegal.
In
termination
cases,
the

dismissed employee is not


required to prove his innocence
of the charges against him. The
burden of proof rests upon the
employer to show that the
employees dismissal was for
just cause. The employers
failure to do so means that the
dismissal was not justified. Here,
the company failed to show that
all 17 union officers deserved to
be dismissed. Ordinarily, the
illegally dismissed employees
are entitled to two reliefs:
reinstatement and backwages.
Still, the Court has held that the
grant of separation pay, instead
of reinstatement, may be proper
especially when as in this case
such reinstatement is no longer
practical or will be for the best
interest of the parties.
46. CIRTEK EMPLOYEES
LABOR UNION-FEDERATION
OF FREE WORKERS
Petitioner, vs.
CIRTEK ELECTRONICS, INC.,
Respondent.
G.R. No. 190515, June 6,
2011
Topic: Adjudicative Powers
Of The Secretary Of Dole
Facts:
[From the November 15,
2010 case, the facts are as
follows: Cirtek Electronics, Inc.,
an
electronics
and
semiconductor firm, had an existing
Collective Bargaining Agreement
(CBA) with Cirtek Employees
Labor Union-Federation of Free
Workers (Cirtek EE Union) from
2011- 2012 Labor Law Case Digest Compilations
UC LLB III-B
Page 76

January 1, 2001 up to December


31, 2005. Prior to the 3rd year of
the
CBA,
the
parties
renegotiated
its
economic
provisions but failed to reach a
settlement, particularly on the
issue of wage increases. Cirtek
EE Union declared a bargaining
deadlock and filed a Notice of
Strike with the NCMB on April
26, 2004. Cirtek Electronics,
upon the other hand, filed a
Notice of Lockout on June 16,
2004.
While the conciliation
proceedings
were
ongoing,
Cirtek Electronics placed seven
union officers including the
President, a Vice President, the
Secretary and the Chairman of
the Board of Directors under
preventive
suspension
for
allegedly
spearheading
a
boycott of overtime work. The
officers
were
eventually
dismissed from employment,
prompting Cirtek EE Union to file
another Notice of Strike which
was, after conciliation meetings,
converted
to
a
voluntary
arbitration case. The dismissal
of the officers was later found to
be legal, hence, Cirtek EE Union
appealed. In the meantime, as
amicable settlement of the CBA
was deadlocked, Cirtek EE Union
went on strike on June 20, 2005.
By Order dated June 23, 2005,
the Secretary of Labor assumed
jurisdiction over the controversy
and issued a Return to Work
Order which was complied with.
Before the Secretary of Labor
could rule on the controversy,
Cirtek Electronics created a
Labor
Management
Council

(LMC)
through
which
it
concluded with the remaining
officers of Cirtek EE Union a
Memorandum
of
Agreement
(MOA) providing for daily wage
increases of P6.00 per day
effective January 1, 2004 and
P9.00 per day effective January
1, 2005. Cirtek EE Union
submitted the MOA via Motion
and
Manifestation
to
the
Secretary of Labor, alleging that
the remaining officers signed
the
MOA
under
Cirtek
Electronicss
assurance
that
should the Secretary order a
higher award of wage increase,
Cirtek Electronics would comply.
On March 16, 2006, the
Secretary of Labor resolved the
CBA deadlock by awarding a
wage increase of from P6.00 to
P10.00 per day effective January
1, 2004 and from P9.00 to
P15.00 per day effective January
1, 2005, and adopting all other
benefits as embodied in the
MOA. Cirtek Electronics moved
for a reconsideration of the
Decision as Cirtek EE Unions
vice-president
submitted
a
"Muling
Pagpapatibay
ng
Pagsang-ayon sa Kasunduan na
may Petsang ika-4 ng Agosto
2005," stating that the union
members were waiving their
rights and benefits under the
Secretarys Decision, however
the same was denied. Cirtek
Electronics filed a petition for
certiorari before the Court of
Appeals.
The appellate court
ruled
in
favor
of
Cirtek
Electronics and accordingly set
aside the Decision of the
2011- 2012 Labor Law Case Digest Compilations
UC LLB III-B
Page 77

Secretary of Labor. It held that


the Secretary of Labor gravely
abused his discretion in not
respecting the MOA. It did not
give credence to the minutes of
the meeting that attended the
forging of the MOA as it was not
verified, nor to the "Paliwanag"
submitted by Cirtek Electronics
union members explaining why
they signed the MOA as it was
not notarized. Cirtek EE Unions
motion
for
reconsideration
having
been
denied
by
Resolution of December 2, 2009,
the present petition was filed,
maintaining that the Secretary
of Labors award is in order,
being in accord with the parties
CBA history Cirtek Electronics
having already granted P15.00
per day for 2001, P10.00 per
day for 2002, and P10.00 per
day for 2003, and that the
Secretary has the power to
grant awards higher than what
are
stated
in
the
CBA.
Respecting the MOA, Cirtek EE
Union posits that it was
"surreptitiously entered into in
bad faith," it having been forged
without the assistance of the
Federation of Free Workers or
counsel, adding that Cirtek
Electronics could have waited
for the Secretarys resolution of
the pending CBA deadlock or
that the MOA could have been
concluded
before
representatives of the Secretary
of Labor. The relevant issues for
resolution are 1) whether the
Secretary of Labor is authorized
to give an award higher than
that agreed upon in the MOA,

and 2) whether the MOA was


entered into and ratified by the
remaining officers of Cirtek EE
Union under the condition,
which was not incorporated in
the MOA, that Cirtek Electronics
would honor the Secretary of
Labors award in the event that
it is higher. The Supreme Court,
in this case resolved both issues
in the affirmative.] The June 6,
2011
case
presents
the
primordial issue of whether the
Secretary
of
Labor
is
empowered to give arbitral
awards in the exercise of his
authority to assume jurisdiction
over labor disputes. The issue
involves a determination and
application of existing law, the
provisions of the Labor Code,
and prevailing jurisprudence.
Furthermore, the issue regarding
the alleged disaffiliation of the
Union from the FFW during the
pendency of the case which
resulted in the FFW losing its
personality to represent the
Union, was also raised in this
forum, in relation to the
Secretary of Labors authority to
give arbitral awards.
Issue:
Whether the Secretary of
Labor is empowered to give
arbitral awards in the exercise of
his
authority
to
assume
jurisdiction over labor disputes.
Ruling:
Yes. The then Acting
Secretary of Labor Manuel G.
Imson acted well within his
jurisdiction in ruling that the
2011- 2012 Labor Law Case Digest Compilations
UC LLB III-B
Page 78

wage increases to be given are


P10 per day effective January 1,
2004 and P15 per day effective
January 1, 2005, pursuant to his
power to assume jurisdiction
under Art. 263 (g) of the Labor
Code. While an arbitral award
cannot per se be categorized as
an
agreement
voluntarily
entered into by the parties
because
it
requires
the
interference
and
imposing
power of the State thru the
Secretary of Labor when he
assumes jurisdiction, the award
can be considered as an
approximation of a collective
bargaining agreement which
would otherwise have been
entered into by the parties.
Hence, it has the force and
effect of a valid contract
obligation between the parties.
In determining arbitral awards
then, aside from the MOA,
courts considered other factors
and documents including, as in
this
case,
the
financial
documents submitted by Cirtek
Electronics,
as well as its
previous bargaining history and
financial
outlook
and
improvements as stated in its
own website.
As for the
contention that the alleged
disaffiliation of the Union from
the FFW during the pendency of
the case resulted in the FFW
losing
its
personality
to
represent the Union, the same
does not affect the Courts
upholding of the authority of the
Secretary of Labor to impose
arbitral awards higher than what
was supposedly agreed upon in

the MOA. Contrary to Cirtek


Electronics
assertion,
the
"unavoidable
issue
of
disaffiliation"
bears
no
significant legal repercussions to
warrant the reversal of the
Courts Decision. Whether there
was a valid disaffiliation is a
factual issue. At all events, the
issue of disaffiliation is an intraunion dispute which must be
resolved in a different forum in
an action at the instance of
either or both the FFW and the
Union
or
a
rival
labor
organization, not the employer.
An intra-union dispute refers to
any conflict between and among
union
members,
including
grievances arising from any
violation of the rights and
conditions
of
membership,
violation of or disagreement
over any provision of the unions
constitution and by-laws, or
disputes arising from chartering
or disaffiliation of the union.
Sections 1 and 2, Rule XI of
Department Order No. 40-03,
Series of 2003 of the DOLE
enumerate
the
following
circumstances as inter/intraunion disputes. Indeed, as Cirtek
Electronics itself argues, a local
union may disaffiliate at any
time from its mother federation,
absent any showing that the
same is prohibited under its
constitution
or
rule.
Such,
however, does not result in it
losing its legal personality
altogether. A local labor union is
a separate and distinct unit
primarily designed to secure
and maintain an equality of
2011- 2012 Labor Law Case Digest Compilations
UC LLB III-B
Page 79

bargaining power between the


employer and their employeemembers. A local union does not
owe its existence to the
federation with which it is
affiliated. It is a separate and
distinct voluntary association
owing its creation to the will of
its members. The mere act of
affiliation does not divest the
local
union
of
its
own
personality, neither does it give
the mother federation the
license to act independently of
the local union. It only gives rise
to a contract of agency where
the
former
acts
in
representation of the latter.
Whether
then,
as
Cirtek
Electronics claims, FFW "went
against the will and wishes of its
principal"
(the
memberemployees) by pursuing the
case despite the signing of the
MOA, is not for the Court, nor for
Cirtek Electronics to determine,
but for the Union and FFW to
resolve on their own pursuant to
their
principal-agent
relationship.
47. SAN MIGUEL FOODS,
INCORPORATED, Petitioner,
vs.SAN MIGUEL
CORPORATION SUPERVISORS
and EXEMPT UNION,
Respondent
G.R. No. 146206
August
1, 2011
Topic: Test For A Singleness
Of
A
Bargaining
Unit;
Confidential Employees
Facts:

DOLE-NCR conducted preelection conferences. However,


there was a discrepancy in the
list of eligible voters, i.e., San
Miguel Foods Inc. submitted a
list of 23 employees for the San
Fernando plant and 33 for the
Cabuyao plant, while SMCSEU
listed 60 and 82, respectively.
On August 31, 1998, the MedArbiter issued an order directing
the election officer to proceed
with the conduct of certification
election. On September 30,
1998, a certification election
was conducted. On the date of
the election, San Miguel Foods
Inc.
filed
the
Omnibus
Objections and Challenge to
Voters, questioning the eligibility
to vote by some of its
employees on the grounds that
some employees do not belong
to the bargaining unit which
SMCSEU seeks to represent or
that there is no existence of
employer-employee relationship
with San Miguel Foods Inc.
Specifically, it argued that
certain employees should not be
allowed to vote as they are: (1)
confidential
employees;
(2)
employees assigned to the live
chicken operations, which are
not covered by the bargaining
unit; (3) employees whose job
grade is level 4, but are
performing managerial work and
scheduled to be promoted; (4)
employees who belong to the
Barrio Ugong plant; (5) non-SMFI
employees; and (6) employees
who are members of other
unions. On October 21, 1998,
the Med-Arbiter issued an Order
2011- 2012 Labor Law Case Digest Compilations
UC LLB III-B
Page 80

directing SMCSEU to submit


proof
showing
that
the
employees in the submitted list
are covered by the original
petition for certification election
and belong to the bargaining
unit it seeks to represent and,
likewise, directing San Miguel
Foods Inc. to substantiate the
allegations contained in its
Omnibus
Objections
and
Challenge to Voters. SMCSEU
averred that (1) the bargaining
unit contemplated in the original
petition is the Poultry Division of
San Miguel Corporation, now
known as San Miguel Foods,
Inc.;
(2)
it
covered
the
operations in Calamba, Laguna,
Cavite, and Batangas and its
home base is either in Cabuyao,
Laguna
or
San
Fernando,
Pampanga; and (3) it submitted
individual
and
separate
declarations of the employees
whose votes were challenged in
the election. The Med-Arbiter
issued the Resolution directing
the parties to appear before the
Election Officer of the Labor
Relations Division on March 9,
1999, 10:00 a.m., for the
opening of the segregated
ballots. On April 12, 1999, the
segregated ballots were opened,
showing that out of the 76
segregated votes, 72 were cast
for "Yes" and 3 for "No," with
one "spoiled" ballot. Based on
the results, the Med-Arbiter
issued the order stating that
since the "Yes" vote received
97% of the valid votes cast,
SMCSEU is certified to be the
exclusive bargaining agent of

the supervisors and exempt


employees of San Miguel Foods
Inc.'s Magnolia Poultry Products
Plants
in
Cabuyao,
San
Fernando, and Otis. On appeal,
the
then
Acting
DOLE
Undersecretary, affirmed the
order with modification that
George C. Matias, Alma Maria M.
Lozano, Joannabel T. Delos
Reyes, and Marilyn G. Pajaron be
excluded from the bargaining
unit which SMCSEU seeks to
represent. She opined that the
challenged voters should be
excluded from the bargaining
unit, because Matias and Lozano
are
members
of
Magnolia
Poultry
Processing
Plants
Monthly Employees Union, while
Delos Reyes and Pajaron are
employees
of
San
Miguel
Corporation, which is a separate
and distinct entity from San
Miguel Foods Inc. San Miguel
Foods Inc.s Partial Motion for
Reconsideration was denied by
the
then
Acting
DOLE
Undersecretary. In the decision
on San Miguel Foods, Inc. v. The
Honorable
Office
of
the
Secretary of Labor, Bureau of
Labor Relations, and San Miguel
Corporation Supervisors and
Exempt Union, the Court of
Appeals (CA) affirmed with
modification
the
Resolution
dated July 30, 1999 of the DOLE
Undersecretary, stating that
those holding the positions of
Human Resource Assistant and
Personnel
Assistant
are
excluded from the bargaining
unit. San Miguel Foods Inc.s
Motion
for
Partial
2011- 2012 Labor Law Case Digest Compilations
UC LLB III-B
Page 81

Reconsideration was denied by


the CA.
Issue:
1.
Whether
a
single
bargaining unit should exist in
the different poultry plants of
San Miguel Inc.
2. Whether the payroll
master and all other positions
with access to salary and
compensation data, in the
context
of
confidential
employees, should be excluded
from the bargaining unit.
3. Whether the Human
Resource
Assistant
and
Personnel Assistant belong to
the category of confidential
employees, and should be
excluded from the bargaining
unit.
Ruling:
First Issue. Yes. In G.R. No.
110399, the Court explained
that the employees of San
Miguel Corporation Magnolia
Poultry
Products
Plants
of
Cabuyao, San Fernando, and
Otis
constitute
a
single
bargaining unit, which is not
contrary to the one-company,
one-union policy. An appropriate
bargaining unit is defined as a
group of employees of a given
employer, comprised of all or
less than all of the entire body
of
employees,
which
the
collective interest of all the
employees,
consistent
with
equity to the employer, indicate
to be best suited to serve the
reciprocal rights and duties of
the parties under the collective

bargaining provisions of the law.


In National Association of Free
Trade Unions v. Mainit Lumber
Development Company Workers
Union United Lumber and
General Workers of the Phils, the
Court, it held that while the
existence of a bargaining history
is a factor that may be reckoned
with
in
determining
the
appropriate bargaining unit, the
same
is
not
decisive
or
conclusive. Other factors must
be considered. The test of
grouping
is
community
or
mutuality of interest. This is so
because the basic test of an
asserted
bargaining
units
acceptability is whether or not it
is
fundamentally
the
combination which will best
assure to all employees the
exercise of their collective
bargaining
rights.
Their
functions
mesh
with
one
another. One group needs the
other in the same way that the
company needs them both.
There may be differences as to
the nature of their individual
assignments,
but
the
distinctions are not enough to
warrant the formation of a
separate bargaining unit. Thus,
applying the ruling to the
present case, the Court affirms
the finding of the CA that there
should be only one bargaining
unit for the employees in
Cabuyao, San Fernando, and
Otis
of
Magnolia
Poultry
Products
Plant
involved
in
"dressed" chicken processing
and Magnolia Poultry Farms
engaged
in
"live"
chicken
2011- 2012 Labor Law Case Digest Compilations
UC LLB III-B
Page 82

operations. Certain factors, such


as specific line of work, working
conditions, location of work,
mode of compensation, and
other relevant conditions do not
affect
or
impede
their
commonality
of
interest.
Although they seem separate
and distinct from each other,
the specific tasks of each
division are actually interrelated
and there exists mutuality of
interests which warrants the
formation of a single bargaining
unit.
Second
Issue.
No.
Confidential
employees
are
defined as those who (1) assist
or act in a confidential capacity,
in regard (2) to persons who
formulate,
determine,
and
effectuate management policies
in the field of labor relations.
The two criteria are cumulative,
and both must be met if an
employee is to be considered a
confidential employee - that is,
the
confidential
relationship
must
exist
between
the
employee and his supervisor,
and the supervisor must handle
the prescribed responsibilities
relating to labor relations. The
exclusion from bargaining units
of employees who, in the normal
course of their duties, become
aware of management policies
relating to labor relations is a
principal objective sought to be
accomplished
by
the
"confidential employee rule." A
confidential employee is one
entrusted with confidence on
delicate, or with the custody,

handling or care and protection


of the employers property.
Confidential employees, such as
accounting personnel, should be
excluded from the bargaining
unit,
as
their
access
to
confidential information may
become the source of undue
advantage. However, such fact
does not apply to the position of
Payroll Master and the whole
gamut of employees who, as
perceived by petitioner, has
access
to
salary
and
compensation data. The CA
correctly held that the position
of Payroll Master does not
involve dealing with confidential
labor relations information in the
course of the performance of his
functions. Since the nature of
his work does not pertain to
company rules and regulations
and confidential labor relations,
it follows that he cannot be
excluded from the subject
bargaining unit.
Third
Issue.
Yes.
Jurisprudence has extended this
prohibition
to
confidential
employees or those who by
reason of their positions or
nature of work are required to
assist or act in a fiduciary
manner
to
managerial
employees and, hence, are
likewise privy to sensitive and
highly
confidential
records.
Confidential employees are thus
excluded from the rank-and-file
bargaining unit. The rationale
for their separate category and
disqualification to join any labor
organization is similar to the
2011- 2012 Labor Law Case Digest Compilations
UC LLB III-B
Page 83

inhibition
for
managerial
employees, because if allowed
to be affiliated with a union, the
latter might not be assured of
their loyalty in view of evident
conflict of interests and the
union
can
also
become
company-denominated with the
presence
of
managerial
employees
in
the
union
membership. Having access to
confidential
information,
confidential employees may also
become the source of undue
advantage. Said employees may
act as a spy or spies of either
party to a collective bargaining
agreement. The CA correctly
ruled that the positions of
Human Resource Assistant and
Personnel Assistant belong to
the category of confidential
employees and, hence, are
excluded from the bargaining
unit,
considering
their
respective positions and job
descriptions.
As
Human
Resource Assistant, the scope of
ones work necessarily involves
labor relations, recruitment and
selection of employees, access
to employees' personal files and
compensation
package,
and
human resource management.
As
regards
a
Personnel
Assistant, one's work includes
the recording of minutes for
management during collective
bargaining
negotiations,
assistance
to
management
during grievance meetings and
administrative
investigations,
and securing legal advice for
labor
issues
from
the
petitioners team of lawyers,

and implementation of company


programs. Therefore, in the
discharge of their functions,
both gain access to vital labor
relations
information
which
outrightly disqualifies them from
union membership.
48. MARTICIO SEMBLANTE
and DUBRICK PILAR,
Petitioners, vs. COURT OF
APPEALS, 19th DIVISION,
now SPECIAL FORMER 19th
DIVISION, GALLERA DE
MANDAUE / SPOUSES
VICENTE and MARIA LUISA
LOOT, Respondents.
G.R. No. 196426
August
15, 2011
Topic:
Employer-Employee
Relationship; Perfection Of
Appeal.
Facts:
Semblante and Pilar were
hired by Vicente and Maria Luisa
Loot, the owners of Gallera de
Mandaue (the cockpit), as the
official
masiador
and
sentenciador, respectively, of
the cockpit sometime in 1993.
As the masiador, Semblante
calls and takes the bets from the
gamecock owners and other
bettors and orders the start of
the
cockfight.
He
also
distributes the winnings after
deducting the arriba, or the
commission for the cockpit.
Meanwhile, as the sentenciador,
Pilar oversees the proper gaffing
of fighting cocks, determines
the fighting cocks physical
condition and capabilities to
2011- 2012 Labor Law Case Digest Compilations
UC LLB III-B
Page 84

continue the cockfight, and


eventually declares the result of
the cockfight. For their services
as masiador and sentenciador,
Semblante receives P2,000 per
week or a total of P8,000 per
month, while Pilar gets P3,500 a
week or P14,000 per month.
They work every Tuesday,
Wednesday,
Saturday,
and
Sunday every week, excluding
monthly derbies and cockfights
held on special holidays. Their
working days start at 1:00 p.m.
and last until 12:00 midnight, or
until the early hours of the
morning depending on the
needs of the cockpit. Semblante
and Pilar had both been issued
employees identification cards
that they wear every time they
report for duty. They alleged
never having incurred any
infraction and/or violation of the
cockpit rules and regulations.
On
November
14,
2003,
however, Semblante and Pilar
were denied entry into the
cockpit upon the instructions of
Spouses
Loot,
and
were
informed of the termination of
their services effective that
date. This prompted Semblante
and Pilar to file a complaint for
illegal dismissal against Spouses
Loot. Spouses Loot denied that
Semblante and Pilar were their
employees and alleged that
they were associates of Spouses
Loots independent contractor,
Tomas Vega. Spouses Loot
claimed that Semblante and
Pilar have no regular working
time or day and they are free to
decide for themselves whether

to report for work or not on any


cockfighting day. In times when
there are few cockfights in
Gallera de Mandaue, Semblante
and Pilar go to other cockpits in
the vicinity. Lastly, Semblante
and Pilar, so Spouses Loot
assert,
were
only
issued
identification cards to indicate
that they were free from the
normal entrance fee and to
differentiate them from the
general public.
The Labor
Arbiter found Semblante and
Pilar to be regular employees of
Spouses
Loot
since
they
performed
work
that
was
necessary and indispensable to
the usual trade or business of
Spouses Loot for a number of
years. The Labor Arbiter also
ruled that Semblante and Pilar
were illegally dismissed, and so
ordered Spouses Loot to pay
Semblante
and
Pilar
their
backwages and separation pay.
Spouses Loots counsel received
the Labor Arbiters Decision and
filed an appeal on September
24, 2004 but without posting a
cash or surety bond equivalent
to the monetary award granted
by the Labor Arbiter. It was only
on October 11, 2004 that
Spouses Loot filed an appeal
bond dated October 6, 2004. On
August 25, 2005, the NLRC
denied the appeal for its nonperfection. The NLRC, acting on
Spouses
Loot
Motion
for
Reconsideration, reversed its
Resolution on the ground that
their appeal was meritorious
and the filing of an appeal bond,
albeit belated, is a substantial
2011- 2012 Labor Law Case Digest Compilations
UC LLB III-B
Page 85

compliance with the rules. The


NLRC held that there was no
employer-employee relationship
between Semblante and Pilar
and Spouses Loot, Spouses Loot
having no part in the selection
and engagement of Semblante
and Pilar, and that no separate
individual contract with Spouses
Loot was ever executed by
Semblante and Pilar. The Motion
for
Reconsideration
was
subsequently
denied.
Semblante and Pilar went to the
CA on a petition for certiorari. In
support
of
their
petition,
Semblante and Pilar argued that
the NLRC gravely abused its
discretion in entertaining an
appeal that was not perfected in
the first place. On the other
hand, Spouses Loot argued that
the NLRC did not commit grave
abuse of discretion, since they
eventually posted their appeal
bond and that their appeal was
so meritorious warranting the
relaxation of the rules in the
interest
of
justice.
The
appellate court ruled in favor of
Spouses
Loot,
noting
that
referees and bet-takers in a
cockfight need to have the kind
of
expertise
that
is
characteristic of the game to
interpret messages conveyed by
mere
gestures.
Hence,
Semblante and Pilar are akin to
independent contractors who
possess unique skills, expertise,
and talent to distinguish them
from
ordinary
employees.
Further, Spouses Loot did not
supply Semblante and Pilar with
the tools and instrumentalities

they needed to perform work.


Semblante
and
Pilar
only
needed their unique skills and
talents to perform their job as
masiador and sentenciador.
Issue:
1. Whether the employeremployee relationship between
the
Spouses
Loot
and
Semblante & Pilar exists.
2.
Whether the appeal
has been perfected despite the
non-posting of the required
bond.
Ruling:
First Issue. No, there is no
existing
er-ee
relationship.
Semblante and Pilar are duly
licensed
"masiador"
and
"sentenciador" in the cockpit
owned
by
Lucia
Loot.
Cockfighting, which is a part of
our cultural heritage, has a
peculiar set of rules. It is a game
based on the fighting ability of
the game cocks in the cockpit.
The referees and bet-takers
need to have that kind of
expertise that is characteristic
of the cockfight gambling who
can interpret the message
conveyed
even
by
mere
gestures. They ought to have
the talent and skill to get the
bets
from
numerous
cockfighting aficionados and
decide which cockerel to put in
the arena. They are placed in
that elite spot where they can
control the game and the crowd.
They are not given salaries by
cockpit
owners
as
their
compensation is based on the
2011- 2012 Labor Law Case Digest Compilations
UC LLB III-B
Page 86

"arriba". In fact, they can offer


their
services
everywhere
because they are duly licensed
by the GAB. They are free to
choose which cockpit arena to
enter and offer their expertise.
Spouses Loot cannot even
control over the means and
methods of the manner by
which they perform their work.
In this light, they are akin to
independent contractors who
possess unique skills, expertise
and talent to distinguish them
from
ordinary
employees.
Furthermore, Spouses Loot did
not supply Semblante and Pilar
with
the
tools
and
instrumentalities they needed to
perform their work. Semblante
and Pilar only needed their
talent and skills to be a
"masiador" and "sentenciador".
As such, they had all the tools
they needed to perform their
work.
Second Issue. Yes, appeal
has been perfected. Considering
the substantial merits of the
case, has relaxed this rule on,
and excused the late posting of,
the appeal bond when there are
strong and compelling reasons
for the liberality, such as the
prevention of miscarriage of
justice extant in the case, or the
special circumstances in the
case combined with its legal
merits or the amount and the
issue involved. While Spouses
Loot had failed to post their
bond within the 10-day period
provided, it is evident, that
Semblante and Pilar are NOT

employees of Spouses Loot,


since their relationship fails to
pass the four-fold test of
employment: (1) the selection
and
engagement
of
the
employee; (2) the payment of
wages; (3) the power of
dismissal; and (4) the power to
control the employees conduct,
which is the most important
element. As found by both the
NLRC and the CA, Spouses Loot
had no part in Semblante and
Pilars
selection
and
management; Semblante and
Pilars compensation was paid
out of the arriba (which is a
percentage deducted from the
total bets), and Semblante and
Pilar performed their functions
as masiador and sentenciador
free from the direction and
control of Spouses Loot. In the
conduct
of
their
work,
Semblante and Pilar relied
mainly on their "expertise that is
characteristic of the cockfight
gambling," and were never
given by Spouses Loot any tool
needed for the performance of
their work.
49. PEOPLE OF THE
PHILIPPINES, PLAINTIFFAPPELLEE, VS. ROSARIO
"ROSE" OCHOA, ACCUSEDAPPELLANT
G.R. No. 173792
August 31, 2011
Topic: Illegal Recruitment In
Large Scale

2011- 2012 Labor Law Case Digest Compilations


UC LLB III-B
Page 87

Facts:
Accused Rosario Ochoa
alleges that she is an employee
of AXIL. On February 1997
accused Rosario Ochoa recruited
Robert Gubat, Junior Agustin,
Cesar Aquino, Richard Luciano,
Fernando Rivera, Mariano R.
Mislang,
Helen
B.
Palogo,
Joebert Decolongon, Corazon S.
Austria, Cristopher A. Bermejo,
Letecia D. Londonio, Alma
Borromeo, Francisco Pascual,
Raymundo A. Bermejo and
Rosemarie A. Bermejo for a
consideration
ranging
from
P2,000.00 to P32,000.00 or a
total amount of P124,000.00 as
placement fee
for their job
application for Taiwan, which the
complainants paid to herein
accused without the accused
having secured the necessary
license from the Department of
Labor and Employment.
The RTC convicted her of
illegal recruitment and the CA
affirmed the decision.
Issue:
Whether Ochoa not being
a licensed recruiter may be
convicted of illegal recruitment
under the Labor Code.
Ruling:

Yes.As found in the offices


records,
appellant,
in
her
personal capacity, is neither
licensed nor authorized to
recruit workers for overseas
employment.
More importantly, Ochoa
could still be convicted of illegal
recruitment even if we disregard
the POEA certification, for
regardless of whether or not
Ochoa was a licensee or holder
of authority, she could still have
committed illegal recruitment.
Section 6 of Republic Act No.
8042 clearly provides that any
person, whether a non-licensee,
non-holder, licensee or holder of
authority may be held liable for
illegal recruitment for certain
acts
as
enumerated
in
paragraphs (a) to (m) thereof.
Among such acts, under Section
6(m) of Republic Act No. 8042, is
the "[f]ailure to reimburse
expenses incurred by the worker
in
connection
with
his
documentation and processing
for purposes of deployment, in
cases where the deployment
does not actually take place
without the workers fault."
Ochoa
committed
illegal
recruitment as described in the
said provision by receiving
placement and medical fees
from
private
complainants,
evidenced by the receipts
2011- 2012 Labor Law Case Digest Compilations
UC LLB III-B
Page 88

issued by her, and failing to


reimburse
the
private
complainants the amounts they
had paid when they were not
able to leave for Taiwan and
Saudi Arabia, through no fault of
their own.
We are not convinced.
Ochoas
claim
was
not
supported by any corroborating
evidence. The POEA verification
dated September 23, 1998, also
signed by Dir. Mateo, and
presented by Ochoa during trial,
pertains only to the status of
AXIL as a placement agency
with
a
"limited
temporary
authority" which had already
expired. Said verification did not
show whether or not Ochoa was
employed by AXIL. Strangely, for
an alleged employee of AXIL,
Ochoa was not able to present
the most basic evidence of
employment,
such
as
appointment
papers,
identification card (ID), and/or
payslips. The receipts presented
by
some
of
the
private
complainants were issued and
signed by Ochoa herself, and
did not contain any indication
that Ochoa issued and signed
the same on behalf of AXIL.
Also, Ochoa was not able to
present any proof that private
complainants
money
were

actually turned
received by AXIL.

over

to

or

Under the last paragraph


of Section 6 of Republic Act No.
8042, illegal recruitment shall
be
considered
an
offense
involving economic sabotage if
committed in a large scale, that
is, committed against three or
more persons individually or as
a group. Here, there are eight
private
complainants
who
convincingly
testified
on
Ochoas
acts
of
illegal
recruitment.
In
view
of
the
overwhelming
evidence
presented by the prosecution,
we uphold the verdict of the
RTC, as affirmed by the Court of
Appeals, that Ochoa is guilty of
illegal recruitment constituting
economic sabotage.

50. NIPPON HOUSING PHIL.


INC., and/or TADASHI OTA,
HOROSHI TAKADA,
YUSUHIRO KAWATA, MR.
NOBOYUSHI and JOEL REYES
Petitioners, vs. MAIAH
ANGELA LEYNES,
Respondent.
G.R. No. 177816
3, 2011

August

2011- 2012 Labor Law Case Digest Compilations


UC LLB III-B
Page 89

Topic:
Redundancy;
Constructive Dismissal
Facts:
From its original business
of
providing
building
maintenance, it appears that
petitioner
Nippon
Housing
Philippines, Inc. (NHPI) ventured
into
building
management,
providing such services as
handling of the lease of
condominium units, collection of
dues and compliance with
government
regulatory
requirements. Having gained
the Bay Gardens Condominium
Project (the Project) of the Bay
Gardens
Condominium
Corporation (BGCC) as its first
and only building maintenance
client, NHPI hired respondent
Maiah Angela Leynes (Leynes)
on 26 March 2001 for the
position of Property Manager,
with a salary of P40,000.00 per
month. Tasked with surveying
the
requirements
of
the
government and the client for
said project, the formulation of
house rules and regulations and
the preparation of the annual
operating
and
capital
expenditure budget, Leynes was
also responsible for the hiring
and deployment of manpower,
salary
and
position
determination as well as the
assignment of the schedules

and
responsibilities
employees.

of

Leynes
had
a
misunderstanding with certain
Engr. Cantuba, casuing Leynes
to
apprised
Cantuba
for
disrespect and insubordination.
The alleged misunderstanding
was
considered
by
the
Management (NHPI) as simple
personal
differences.
Unsatisfied
of
the
managements action, Leynes
signified his resignation. The
Human Resource Management
thereafter hired a substitute in
the
absence
of
Leynes.
However, Leynes reported back
to his work after days of
absences. He was informed that
someone has taken over his
position and he is in floating
status.
Leynes filed for or illegal
dismissal,
unpaid
salaries,
benefits, and damages.
Issue:
Whether NHPIs act of putting
Leynes on floating status was
equivalent to termination from
employment without just cause
and compliance with the twin
requirements of notice and
hearing.
Ruling:
2011- 2012 Labor Law Case Digest Compilations
UC LLB III-B
Page 90

Considering
that
even
labor laws discourage intrusion
in the employers judgment
concerning the conduct of their
business, courts often decline to
interfere in their legitimate
business
decisions,
absent
showing of illegality, bad faith or
arbitrariness. Indeed, the right
of employees to security of
tenure does not give them
vested rights to their positions
to the extent of depriving
management of its prerogative
to change their assignments or
to transfer them. The record
shows that Leynes filed the
complaint for actual illegal
dismissal from which the case
originated on 22 February 2002
or immediately upon being
placed on floating status as a
consequence of NHPIs hiring of
a new Property Manager for the
Project. The rule is settled,
however, that "off-detailing" is
not equivalent to dismissal, so
long as such status does not
continue beyond a reasonable
time and that it is only when
such a "floating status" lasts for
more than six months that the
employee may be considered to
have
been
constructively
dismissed. A complaint for
illegal dismissal filed prior to the
lapse of said six-month and/or
the actual dismissal of the

employee
is
generally
considered as prematurely filed.
Viewed in the light of the
foregoing factual antecedents,
we find that the CA reversibly
erred in holding petitioners
liable
for
constructively
dismissing Leynes from her
employment. There is said to be
constructive dismissal when an
act of clear discrimination,
insensitivity or disdain on the
part of the employer has
become so unbearable as to
leave an employee with no
choice but to forego continued
employment.
Constructive
dismissal exists where there is
cessation of work because
continued
employment
is
rendered
impossible,
unreasonable or unlikely, as an
offer involving a demotion in
rank and a diminution in pay.
Stated otherwise, it is a
dismissal in disguise or an act
amounting to dismissal but
made to appear as if it were not.
In constructive dismissal cases,
the employer is, concededly,
charged with the burden of
proving that its conduct and
action or the transfer of an
employee are for valid and
legitimate grounds such as
genuine business necessity. To
our mind, respondents have
more than amply discharged
2011- 2012 Labor Law Case Digest Compilations
UC LLB III-B
Page 91

this burden with proof of the


circumstances surrounding Engr.
Carlos employment as Property
Manager for the Project and the
consequent unavailability of a
similar position for Leynes.
With no other client aside
from BGCC for the building
management
side
of
its
business, we find that NHPI was
acting
well
within
its
prerogatives when it eventually
terminated Leynes services on
the ground of redundancy. One
of the recognized authorized
causes for the termination of
employment, redundancy exists
when the service capability of
the workforce is in excess of
what is reasonably needed to
meet the demands of the
business
enterprise.
A
redundant
position
is
one
rendered superfluous by any
number of factors, such as
overhiring of workers, decreased
volume of business, dropping of
a
particular
product
line
previously manufactured by the
company or phasing out of
service
activity
priorly
undertaken by the business. It
has been held that the exercise
of
business
judgment
to
characterize
an
employees
service as no longer necessary
or sustainable is not subject to
discretionary review where, as

here, it is exercised there is no


showing of violation of the law
or arbitrariness or malice on the
part of the employer. An
employer has no legal obligation
to keep more employees than
are necessary for the operation
of its business.
Considering that Leynes
was terminated from service
upon an authorized cause, we
find that the CA likewise erred in
faulting NHPI for supposedly
failing to notify said employee of
the particular act or omission
leveled against her and the
ground/s for which she was
dismissed from employment.
Where dismissal, however, is for
an
authorized
cause
like
redundancy, the employer is,
instead, required to serve a
written notice of termination on
the worker concerned and the
DOLE, at least one month from
the intended date thereof. Here,
NHPI specifically made Leynes
termination
from
service
effective 22 August 2002, but
only informed said employee of
the same on 8 August 2002 and
filed with the DOLE the required
Establishment
Termination
Report only on 16 August 2002.
For its failure to comply strictly
with
the 30-day
minimum
requirement for said notice and
effectively
violating
Leynes
2011- 2012 Labor Law Case Digest Compilations
UC LLB III-B
Page 92

right to due process, NHPI


should be held liable to pay
nominal damages in the sum of
P50,000.00. The penalty should
understandably
be
stiffer
because the dismissal process
was initiated by the employer's
exercise of its management
prerogative.
Having
been
validly
terminated on the ground of
redundancy, Leynes is entitled
to separation pay equivalent to
one month salary for every year
of service but not to the
backwages adjudicated in her
favor by the Labor Arbiter. Hired
by NHPI on 26 March 2001 and
terminated effective 22 August
2002, Leynes is entitled to a
separation pay in the sum of
P40,000.00, in addition to her
last pay which, taking into
consideration her proportionate
13th month pay, tax refund and
SILP, was computed by NHPI at
P28,188.16. For lack of showing
of
bad
faith,
malice
or
arbitrariness on the part of
NHPI, there is, however, no
justifiable ground for an award
of
moral
and
exemplary
damages.
51. FRANCIS BELLO,
REPRESENTED HEREIN BY
HIS DAUGHTER AND
ATTORNEY-IN-FACT,
GERALDINE BELLO-ONA,

PETITIONER, VS. BONIFACIO


SECURITY SERVICES, INC.
AND SAMUEL TOMAS,
RESPONDENTS.
G.R. No. 188086
August 3, 2011
Topic:
Dismissal;

Constructive

Facts:
Respondent
Bonifacio
Security Services, Inc. (BSSI) is a
domestic private corporation
engaged in the business of
providing security services. In
July 2001, the BSSI hired Bello
as a roving traffic marshal to
manage traffic and to conduct
security
and
safety-related
operations in the Bonifacio
Global City (BGC). In August
2001, Bello was posted at the
Negros Navigation Company in
Pier
2,
North
Harbor,
to
supervise sectoral operations. In
November
2001,
he
was
assigned at BGC as assistant
detachment commander. After a
week, he was transferred to
Pacific Plaza Towers as assistant
detachment commander and
later
as
detachment
commander. In June 2002, he
was assigned at Pier 2, North
Harbor as assistant detachment
commander,
but
later
reassigned to BGC. In August
2011- 2012 Labor Law Case Digest Compilations
UC LLB III-B
Page 93

2002, the BSSI hired a new


operations manager, resulting in
the reorganization of posts. In
October
2002,
Bello
was
assigned
as
roving
traffic
marshal at the BGC. On October
25, 2002, he filed an indefinite
leave of absence when his new
assignment took effect.
On November 5, 2002,
Bello filed a complaint against
the BSSI and its General
Manager, respondent Samuel
Tomas, with the National Labor
Relations Commission (NLRC),
claiming that he had been
constructively dismissed when
he was demoted from a
detachment commander to a
mere traffic marshal. He alleged
that he received a series of
promotions from 2001 to 2002,
from
traffic
marshal
to
supervisor,
to
assistant
detachment commander, and to
detachment commander.
The BSSI denied Bellos claim of
constructive dismissal, arguing
that no promotion took place;
Bellos designation as assistant
detachment
commander
or
detachment commander was
not an employment position but
a duty-related assignment; Bello
abandoned his job when he
went on an indefinite leave of
absence and did not report for
work.

The Labor Arbiter held that Bello


was illegally dismissed, while
the CA ruled on the other hand.
Issue:
Whether Bellos demotion
to a mere traffic marshal after
having been promoted to the
positions of supervisor, assistant
detachment commander, and
detachment
commanderconstitutes
constructive dismissal.
Ruling:
On the merits of the case,
we find no reason to disturb the
CA conclusion that there was no
constructive dismissal. Case law
defines constructive dismissal
as a cessation of work because
continued
employment
has
been
rendered
impossible,
unreasonable, or unlikely, as
when there is a demotion in
rank or diminution in pay, or
both,
or
when
a
clear
discrimination, insensibility, or
disdain
by
an
employer
becomes unbearable to the
employee.
We note that, other than
his
bare
and
self-serving
allegations, Bello has not offered
any evidence that he was
promoted in a span of four
months since his employment
as traffic marshal in July 2001 to
2011- 2012 Labor Law Case Digest Compilations
UC LLB III-B
Page 94

a detachment commander in
November 2001. During his sixmonth probationary period of
employment,
it
is
highly
improbable that Bello would be
promoted after just a month of
employment, from a traffic
marshal
in
July
2001
to
supervisor in August 2001, and
three months later to assistant
detachment commander and to
detachment
commander
in
November 2001. At most, the
BSSI
merely
changed
his
assignment or transferred him
to the post where his service
would be most beneficial to its
clients.
The
management's
prerogative of transferring and
reassigning employees from one
area of operation to another in
order to meet the requirements
of the business is generally not
constitutive
of
constructive
dismissal. We see this to be the
case in the present dispute so
that
the
consequent
reassignment of Bello to a traffic
marshal post was well within the
scope
of
the
BSSIs
management prerogative.
52. JOBEL ENTERPRISES
and/or MR. BENEDICT LIM,
Petitioners, vs. NATIONAL
LABOR RELATIONS
COMMISSION (Seventh
Division, Quezon City) and

ERIC MARTINEZ, SR.,


Respondents.
G.R. No. 194031
August 8, 2011
Topic: Perfecting An Appeal
Facts:
The
petitioner
Jobel
Enterprises (the company) hired
respondent Eric Martinez, Sr. as
driver
in
2004.
Martinez
allegedly performed well during
the first few months of his
employment, but later became
stubborn, sluggish and often
came late to work.
On January 27, 2005,
Martinez had a fight with one of
his co-employees and nephew,
Roderick
Briones.
The
companys proprietor, Benedict
Lim, pacified the two and
instructed Martinez to come
early the next day for an
important
delivery.
Martinez
allegedly did not report for work
the
following
day.
The
companys efforts to contact
Martinez,
through
Briones,
failed.
On March 6, 2006, the
company received a notice of
hearing from the Department of
Labor
and
Employment
in
Region IV-A (DOLE-RO-IV-A) in
relation to an illegal dismissal
2011- 2012 Labor Law Case Digest Compilations
UC LLB III-B
Page 95

complaint filed by Martinez. The


DOLE-RO-IV-A failed to effect an
amicable settlement between
the parties; Martinez allegedly
asked
for
P300,000.00
as
settlement and manifested that
he did not want to work
anymore. Thereafter, Martinez
formally filed an illegal dismissal
complaint, with money claims,
against the company and Lim.
On compulsory arbitration,
Labor Arbiter Danna M. Castillon
ruled that Martinez had been
illegally dismissed. She awarded
him backwages and separation
pay amounting to P479,529.49,
and wage differentials and 13th
month pay in the combined
amount of P53,363.44.
On May 16, 2008, the
petitioners appealed to the
National
Labor
Relations
Commission (NLRC), filing a
notice
of
appeal,
a
memorandum of appeal and a
motion to reduce bond. They
likewise
deposited
a
Rizal
Commercial
Banking
Corporation managers check for
P100,000.00. In its order of
September 15, 2008, the NLRC
denied the companys motion to
reduce bond and directed the
posting of an additional cash or
surety bond for P432,892.93
within ten (10) days.

The company complied by


posting a surety bond in the
required amount, but Martinez
moved
for
the
immediate
dismissal of the appeal; he
questioned the effectivity of the
surety bond and the legal
standing of the surety company.
In answer, the company asked
for a denial of the motion and
submitted a copy of the joint
declaration by the companys
authorized representative and
the Executive Vice-President of
the surety company9 that the
posted surety bond is genuine
and shall be effective until final
disposition of the case. It also
submitted a copy of a certificate
of authority issued by the
Insurance Commission, and a
certificate of accreditation and
authority issued by this Court.
Issue:
Whether the Company
was able to comply with the
requirements on appeal?
Ruling:
We note that this case was
dismissed on purely technical
grounds at both the NLRC and
the CA levels, in total disregard
of the merits of the case. The
NLRC dismissed the companys
appeal for non-perfection for its
failure "to substantially address
2011- 2012 Labor Law Case Digest Compilations
UC LLB III-B
Page 96

the issue of failure to post the


required appeal bond pursuant
to Section 6, Rule VI of the 2005
Revised Rules of Procedure of
the
NLRC."
In
summarily
throwing out the appeal, the
NLRC apparently forgot that
earlier, or on September 15,
2008, it gave the company "ten
(10) unextendible days xxx
within which to file an additional
cash or surety bond in the
amount of FOUR HUNDRED
THIRTY TWO THOUSAND EIGHT
HUNDRED NINETY TWO PESOS
and
93/100
(P432,892.93)"
when it denied the companys
motion to reduce bond. The
NLRC even warned that "[t]heir
failure to post the required bond
shall result in the dismissal of
the appeal for non-perfection."
As earlier mentioned, the
company complied with the
NLRC directive by posting a
surety bond in the required
amount within the 10-day
period; it received a copy of the
NLRC resolution directing it to
post an additional cash or surety
bond on October 13, 2008 and
posted the bond on October 23,
2008. The company likewise
submitted a joint declaration
between
the
company
representative and the surety
company on the period of
effectivity of the bond, and the

documents on the legal status of


the surety company. The NLRC
grossly erred, therefore, in
declaring that the company
failed to address the issue of its
failure to post the required
bond. The CA grossly failed to
consider this lapse.
We note, too, that the
CAs refusal to consider the
petition was the absence of a
duplicate original or certified
true copy of the assailed NLRC
decision, in violation of Section
3, Rule 46 of the Rules of Court
(in relation to Section 1, Rule
65).
The
company though
corrected the procedural lapse
by attaching a certified copy of
the NLRC decision to its motion
for reconsideration. At this
point, the CA should have at
least considered the merits of
the petitioners case as we did
in Gutierrez v. Secretary of the
Department
of
Labor
and
Employment. We held in that
case that while "what [were]
submitted
were
mere
photocopies[,]
there
was
substantial compliance with the
Rules since petitioner attached
to her Supplemental Motion for
Reconsideration certified true
copies of the questioned DOLE
Orders."
53. ATOK BIG WEDGE
COMPANY, INC., Petitioner,
2011- 2012 Labor Law Case Digest Compilations
UC LLB III-B
Page 97

vs. JESUS P. GISON,


Respondent.
G.R. No. 169510
August 8, 2011
Topic:
Employer-Employee
Relationship
Facts:
Sometime
in
February
1992, respondent Jesus P. Gison
was engaged as part-time
consultant on retainer basis by
petitioner Atok Big Wedge
Company, Inc. through its then
Asst. Vice-President and Acting
Resident Manager, Rutillo A.
Torres. As a consultant on
retainer
basis,
respondent
assisted petitioner's retained
legal counsel with matters
pertaining to the prosecution of
cases against illegal surface
occupants within the area
covered by the company's
mineral claims. Respondent was
likewise tasked to perform
liaison
work
with
several
government agencies, which he
said was his expertise.
Petitioner did not require
respondent to report to its office
on a regular basis, except when
occasionally requested by the
management to discuss matters
needing his expertise as a
consultant. As payment for his
services, respondent received a

retainer fee of P3,000.00 a


month, which was delivered to
him either at his residence or in
a local restaurant. The parties
executed a retainer agreement,
but
such
agreement
was
misplaced and can no longer be
found.

On
February
21,
2003,
respondent filed a Complaint for
illegal dismissal, unfair labor
practice,
underpayment
of
wages, non-payment of 13th
month pay, vacation pay, and
sick leave pay with the National
Labor Relations Commission
(NLRC), against petitioner, Mario
D. Cera, and Teofilo R. Asuncion,
Jr.

The
said
arrangement
continued for the next eleven
years.
Sometime
thereafter,
since
respondent was getting old, he
requested that petitioner cause
his registration with the Social
Security System (SSS), but
petitioner did not accede to his
request. He later reiterated his
request but it was ignored by
respondent considering that he
was only a retainer/consultant.
On
February
4,
2003,
respondent filed a Complaint
with the SSS against petitioner
for the latter's refusal to cause
his registration with the SSS.

Issue:
1. Whether Gisons eleven-year
service as a consultant to the
company creates an employeremployee relationship.
2. If so, whether there was an
illegal dismissal

On the same date, Mario


D. Cera, in his capacity as
resident manager of petitioner,
issued a Memorandum advising
respondent that within 30 days
from receipt thereof, petitioner
is terminating his retainer
contract with the company since
his services are no longer
necessary.

2011- 2012 Labor Law Case Digest Compilations


UC LLB III-B
Page 98

Ruling:
First
Issue.
Wellentrenched is the doctrine that
the existence of an employeremployee
relationship
is
ultimately a question of fact and
that the findings thereon by the
Labor Arbiter and the NLRC shall
be accorded not only respect
but
even
finality
when
supported
by
substantial
evidence. Being a question of
fact, the determination whether
such
a
relationship
exists
between
petitioner
and
respondent was well within the
province of the Labor Arbiter
and the NLRC. Being supported

by substantial evidence, such


determination should have been
accorded great weight by the CA
in resolving the issue.
To ascertain the existence
of
an
employer-employee
relationship jurisprudence has
invariably adhered to the fourfold test, to wit: (1) the selection
and
engagement
of
the
employee; (2) the payment of
wages; (3) the power of
dismissal; and (4) the power to
control the employee's conduct,
or the so-called "control test." Of
these four, the last one is the
most important. The so-called
"control test" is commonly
regarded as the most crucial
and determinative indicator of
the presence or absence of an
employer-employee relationship.
Under the control test, an
employer-employee relationship
exists where the person for
whom
the
services
are
performed reserves the right to
control not only the end
achieved, but also the manner
and means to be used in
reaching that end.
Applying
the
aforementioned
test,
an
employer-employee relationship
is apparently absent in the case
at bar. Among other things,
respondent was not required to
report everyday during regular
2011- 2012 Labor Law Case Digest Compilations
UC LLB III-B
Page 99

office
hours
of
petitioner.
Respondent's monthly retainer
fees were paid to him either at
his
residence
or
a
local
restaurant. More importantly,
petitioner did not prescribe the
manner in which respondent
would accomplish any of the
tasks in which his expertise as a
liaison officer was needed;
respondent was left alone and
given
the
freedom
to
accomplish the tasks using his
own
means
and
method.
Respondent was assigned tasks
to perform, but petitioner did
not control the manner and
methods by which respondent
performed these tasks. Verily,
the absence of the element of
control on the part of the
petitioner
engenders
a
conclusion that he is not an
employee of the petitioner.
Moreover, the absence of
the
parties'
retainership
agreement
notwithstanding,
respondent clearly admitted
that petitioner hired him in a
limited capacity only and that
there will be no employeremployee relationship between
them.
As
averred
in
respondent's Position Paper:
Second Issue. For the
participation
of
complainant
regarding
this
particular
problem of Atok, Mr. Torres

offered him a pay in the amount


of Php3,000.00 per month plus
representation expenses. It was
also agreed by Mr. Torres and
the
complainant
that
his
participation on this particular
problem
of
Atok
will
be
temporary since the problem
was then contemplated to be
limited in nature, hence, there
will be no employer-employee
relationship between him and
Atok. Complainant agreed on
this arrangement. It was also
agreed
that
complainant's
compensations,
allowances,
representation expenses and
reimbursement of companyrelated
expenses
will
be
processed and paid through
disbursement vouchers;
Respondent
was
well
aware of the agreement that he
was hired merely as a liaison or
consultant of the petitioner and
he agreed to perform tasks for
the petitioner on a temporary
employment
status
only.
However, respondent anchors
his claim that he became a
regular
employee
of
the
petitioner
based
on
his
contention that the "temporary"
aspect of his job and its
"limited" nature could not have
lasted for eleven years unless
some time during that period,
he became a regular employee
2011- 2012 Labor Law Case Digest Compilations
UC LLB III-B
Page 100

of the petitioner by continually


performing services for the
company.
Contrary to the conclusion
of the CA, respondent is not an
employee, much more a regular
employee of petitioner. The
appellate court's premise that
regular employees are those
who perform activities which are
desirable and necessary for the
business of the employer is not
determinative in this case. In
fact,
any
agreement
may
provide that one party shall
render services for and in behalf
of another, no matter how
necessary
for
the
latter's
business, even without being
hired as an employee. Hence,
respondent's length of service
and petitioner's repeated act of
assigning
respondent
some
tasks to be performed did not
result
to
respondent's
entitlement to the rights and
privileges
of
a
regular
employee.
Furthermore, despite the
fact that petitioner made use of
the services of respondent for
eleven years, he still cannot be
considered
as
a
regular
employee of petitioner. Article
280 of the Labor Code, in which
the lower court used to buttress
its findings that respondent
became a regular employee of

the petitioner, is not applicable


in the case at bar. Indeed, the
Court has ruled that said
provision is not the yardstick for
determining the existence of an
employment
relationship
because it merely distinguishes
between
two
kinds
of
employees,
i.e.,
regular
employees
and
casual
employees, for purposes of
determining the right of an
employee to certain benefits, to
join or form a union, or to
security of tenure; it does not
apply where the existence of an
employment relationship is in
dispute.
It
is,
therefore,
erroneous on the part of the
Court of Appeals to rely on
Article 280 in determining
whether an employer-employee
relationship
exists
between
respondent and the petitioner
Considering that there is
no
employer-employee
relationship
between
the
parties, the termination of
respondent's services by the
petitioner after due notice did
not constitute illegal dismissal
warranting his reinstatement
and
the payment
of
full
backwages,
allowances
and
other benefits.

2011- 2012 Labor Law Case Digest Compilations


UC LLB III-B
Page 101

54. COASTAL SAFEWAY


MARINE SERVICES INC.,
Petitioner, vs. ELMER T.
ESGUERRA, Respondent
G.R. No. 185352
August 10, 2011
Topic: Disability Claims Of
Migrant Seafarers
Facts:
A seafarer since 1991,
respondent Elmer T. Esguerra
(Esguerra)
applied
for
placement
with
petitioner
Coastal
Safeway
Marine
Services, Inc. (CSMSI) sometime
in 2003. Found fit for work
during
the
pre-employment
medical examination conducted
by
the
company-designated
physician, Esguerra was hired
by the CSMSI as Third Mate for
the M/V Mr. Nelson, an oceangoing vessel under the flag of
the United Arab Emirates (UAE)
owned by its foreign principal,
Canada & Middle East General
Trading (CMEGT). Subject to the
provisions of the POEA-SEC, the
contract
of
employment
executed by the parties on 9
May 2003 provided a term of
one (1) year and a basic
monthly salary of US$800.00 for
a 48-hour work-week, with
provisions for overtime pay and
vacation leave with pay. Rather

than the aforesaid


vessel,
however, it appears that, on 13
May 2003, Esguerra, as Second
Officer, eventually boarded the
vessel M/V Gondwana which was
likewise manned by CSMSI on
behalf
of
Nabeel
Shipmanagement
Ltd.
Fze.
(NSLF).
On 28 June 2003 or after
forty six (46) days of shipboard
employment,
Esguerra
requested medical attention for
back and chest pains while M/V
Gondwana was docked at Port
Jebel Ali, UAE. Examined on 5
July 2003 at the Jebel Ali Medical
Centre, Esguerra was declared
"not fit for work until complete
cardiac evaluation is done" and
"advised to rest until then" by
Dr. Zarga S. Tulmar. Despite the
normal results of the serology,
hematology, biochemistry and
x-ray tests administered upon
him, however, Esguerra insisted
on going home on the ground
that he had been rendered unfit
for work. Alleging that he had
yet to receive his salary for June
2003 and that his employer was
making
him
shoulder
his
repatriation expenses as a
consequence of his failure to
finish his contract, Esguerra also
sought assistance from the Jebel
Ali police/coastguard regarding
his predicament. Subsequent to
2011- 2012 Labor Law Case Digest Compilations
UC LLB III-B
Page 102

his arrival in the Philippines on 7


July 2003, Esguerra went to the
Philippine Heart Center (PHC),
the
Philippine
Orthopedic
Hospital
(POH)
and
the
Philippine
General
Hospital
(PGH) for medical evaluation
and treatment.
Having consulted with Dr.
Efren R. Vicaldo, a Doctor of
Internal
Medicine
and
Cardiology at the PHC as well as
Dr. Rimando C. Saguin, an
Orthopedic Surgeon at the POH,
Esguerra
further
underwent
diagnostic
tests
and
was
prescribed various medications
at the PGH for "chronic stable
angina."
Esguerra,
being
constrained to shell out from his
pocket
in
seeking
outside
medical
examinations
attentions, filed his complaint
for disability benefits, sickness
allowance,
damages
and
attorneys fees.
Issue:
Whether the Company
was able to comply with its
obligation
to
provide
the
statutory mandate of providing
and
securing
good
health
conditions
and
welfare
of
seafarers.
Ruling:

Viewed in light of the fact


that Esguerras contract of
employment was executed on 9
May 2003, CSMSI correctly faults
the CA for applying POEA
Memorandum Circular No. 05596 instead of the 2000 POEASEC which took effect on 25
June 2000. Deemed written in
the
seafarer's
contract
of
employment, the 2000 POEASEC like its predecessor was
designed primarily for the
protection and benefit of Filipino
seamen in the pursuit of their
employment on board oceangoing
vessels.
Anent
a
seafarers
entitlement
to
compensation and benefits for
injury and illness, Section 20-B
(3) thereof provides as follows:
"Section
20B.Compensation and Benefits
for Injury and Illness.
xxxx
Upon sign-off from the
vessel for medical treatment,
the seafarer is entitled to
sickness allowance equivalent to
his basic wage until he is
declared fit to work or the
degree of permanent disability
has been assessed by the
company-designated physician,
but in no case shall this period
exceed one hundred twenty
(120) days.E
2011- 2012 Labor Law Case Digest Compilations
UC LLB III-B
Page 103

For this purpose, the


seafarer shall submit himself to
a
post-employment
medical
examination by a companydesignated
physician
within
three working days upon his
return except when he is
physically incapacitated to do
so, in which case, a written
notice to the agency within the
same period is deemed as
compliance.
Failure
of
the
seafarer to comply with the
mandatory
reporting
requirement shall result in his
forfeiture of the right to claim
the above benefits.
If a doctor appointed by
the seafarer disagrees with the
assessment, a third doctor may
be agreed jointly between the
employer and the seafarer. The
third doctor's decision shall be
final and binding on both
parties.
The foregoing provision
has been interpreted to mean
that
it
is
the
companydesignated physician who is
entrusted with the task of
assessing
the
seaman's
disability, whether total or
partial, due to either injury or
illness, during the term of the
latter's
employment.
Concededly, this does not mean
that the assessment of said
physician is final, binding or

conclusive on the claimant, the


labor tribunal or the courts.
Should he be so minded, the
seafarer has the prerogative to
request a second opinion and to
consult a physician of his choice
regarding his ailment or injury,
in which case the medical report
issued by the latter shall be
evaluated by the labor tribunal
and the court, based on its
inherent merit. For the seamans
claim to prosper, however, it is
mandatory that he should be
examined
by
a
companydesignated
physician
within
three days from his repatriation.
Failure to comply with this
mandatory
reporting
requirement without justifiable
cause shall result in forfeiture of
the
right
to
claim
the
compensation
and
disability
benefits provided under the
POEA-SEC.
There
is
no
dispute
regarding the fact that Esguerra
had altogether failed to comply
with
the
above-discussed
mandatory
reporting
requirement. Beyond his bare
assertion, however, that CSMSI
"never gave him referrals to
continue his medications as
recommended by the foreign
doctor" despite his call on 8 July
2003 "to inform them that he
will report the next day in order
2011- 2012 Labor Law Case Digest Compilations
UC LLB III-B
Page 104

to submit his medical evaluation


abroad,"
Esguerra
did
not
present any evidence to prove
justification for his inability to
submit himself to a postemployment
medical
examination by a companydesignated
physician.
If
a
written notice is required of a
seafarer
who
is
physically
incapacitated for purposes of
compliance
with
said
requirement, we fail to see why
a more tangible proof should not
likewise
be
expected
of
Esguerra who, after his arrival
on 7 July 2003, appears to have
been well enough to consult
with Dr. Vicaldo and Dr. Saguin
on 9 July 2003. Indeed, selfserving and unsubstantiated
declarations are insufficient to
establish a case before quasijudicial
bodies
where
the
quantum of evidence required to
establish a fact is substantial
evidence Often described as
more than a mere scintilla,
substantial evidence is such
relevant
evidence
as
a
reasonable mind might accept
as adequate to support a
conclusion, even if other equally
reasonable
minds
might
conceivably opine otherwise.
To our mind, Esguerras
compliance with the mandatory
reporting requirement under the

POEA-SEC was made even more


imperative by the fact that his
repatriation for medical reasons
was categorically disputed by
CSMSI. Consistent with the 5
July 2003 diagnosis made by Dr.
Tulmar at the Jebel Ali Medical
Centre declaring him "not fit for
work until complete cardiac
evaluation is done" and advising
him "to rest until then," it
appears
that
Esguerra
underwent
serology,
hematology, biochemistry and
x-ray diagnostic tests which
yielded no significant findings
relative to the back and chest
pains he claims to have
suffered. Although the 5 July
2003 notation made on the M/V
Gondwana
Chief
Officers
Logbook states that he was
"advised to be repatriated" on
the same day and "to continue
his
medication
in
the
Philippines,"
no
less
than
Esguerra himself confirmed in
his 6 July 2003 letter to the Jebel
Ali police/coastguard that he
had yet to undergo a compete
cardiac evaluation and that
CSMSIs foreign principal, NSLF,
had refused to shoulder his
repatriation expenses on the
ground that he was unable to
finish his contract.

2011- 2012 Labor Law Case Digest Compilations


UC LLB III-B
Page 105

55. Pamela Florentina P.


Jumuad vs HiFlyer Food Inc.
and/or Jesus R. Montemayor
GR No. 187887, September
7, 2011
Topic:
Illegal
Dismissal;
When Findings Of Facts Of
Labor Arbiters and the NLRC
May Be Reviewed; Issues
Not Cognizable In A Labor
Case
Facts:
Pamela Jumuad started
out as a manager-trainee for
HiFlyer Food Inc, a corporation
licensed to operate KFC. until
she was eventually promoted to
Area Manager for the VisayasMindanao Operations. She was
highly efficient and excellent
even being recognized as the
top 3 manager. She also availed
of a 40% car loan granted by
HiFlyer to its managers.
On
one
inspection
however,
one
branchesin
Visayas
failed
sanitary
inspections
and
it
was
discovered that some of their
essential
equipment
were
broken, there were lapses in
inventories and delay in sales.
One year later, another branch
in Visayas was found to be n the
same condition as such, it was
closed. Jumuad was issued a
written notice to explain the
situation as she was held
accountable. Jumuad gave a
written
explanation
but
seemingly unsatisfied, HiFlyer
still
conducted
and
administrative hearing where
Jumuad appeared with a lawyer.

After which, Jumuad was issued


a notice of dismissal on the
ground of gross and habitual
neglect of duty.
Jumuad filed a case for
illegal dismissal with prayer for
backwages,
reinstatement,
damages, and reimbursement of
the car loan.
The LA ruled that Jumuad
was illegally dismissed since her
dismissal
was
too
harsh
considering
that
the
circumstances leading to her
dismissal was not entirely her
fault. Jumuad was awarded
separation
pay
and
reimbursement of her car loan.
Montemayor was found to be
solidarily liable with HiFlyer.
Both Jumuad and HiFlyer
appealed to the NLRC. Jumuad
claims for backwages and
damages. HiFlyer questions the
finding of illegal dismissal and
the monetary awards. The NLRC
sustained in toto the decision of
the LA especially the finding
that the dismissal is too harsh,
citing email messages between
Officers of HiFlyer where they
stated that dismissal is the
proper sanction for Jumuad and
anything less is light.
HiFlyer
appealed
the
NLRCs decision to the CA and
the CA reversed the LA and
NLRCs decision. The CA ruled
that the requisites of due
process were fulfilled. The CA
ruled
that
failure
of
the
branches
to
pass
sanitary
inspections,
lapses
in
inventories, and delays in sales
were enough to constitute loss
2011- 2012 Labor Law Case Digest Compilations
UC LLB III-B
Page 106

of trust and confidence as a


valid ground for dismissal. As to
the reimbursement of the Car
loan,
the
CA
refused
to
adjudicate
whether
reimbursement is proper or not,
and merely ordered the filing of
the case in the appropriate
court since the jurisdiction to do
so belongs to regular courts.
Issue:
1. Whether the factual
findings of the LA and NLRC may
not be disturbed.
2. Whether Jumuad was
illegally dismissed.
3. Whether HiFlyer is
liable to reimburse the car loan.
Ruling:
First Issue.In this case,
yes. As a rule, factual findings of
administrative or quasi-judicial
bodies, which are deemed to
have acquired expertise in
matters within their respective
jurisdictions,
are
generally
accorded not only respect but
even finality, and bind the Court
when supported by substantial
evidence. However there are
exceptions to this rule. These
include: (1) when there is grave
abuse of discretion; (2) when
the findings are grounded on
speculation;
(3)
when
the
inference made is manifestly
mistaken;
(4)
when
the
judgment of the Court of
Appeals
is
based
on
a
misapprehension of facts; (5)
when the factual findings are
conflicting; (6) when the Court
of Appeals went beyond the

issues of the case and its


findings are contrary to the
admissions of the parties; (7)
when the Court of Appeals
overlooked undisputed facts
which, if properly considered,
would
justify
a
different
conclusion; (8) when the facts
set forth by the petitioner are
not disputed by the respondent;
and (9) when the findings of the
Court of Appeals are premised
on the absence of evidence and
are
contradicted
by
the
evidence on record.
Second
Issue.No.
Circumstances
suggest
that
Jumuad may not be dismissed
due to neglect of duty. The
neglect must be gross and
habitual. Even if her neglect of
duty may be considered gross, it
is not habitual, since the time
interval between the first and
second inspection is more than
1 year. There can be no showing
of habituality in such ocassion.
However, despite saying this, it
cannot be denied that Jumuad
willfully breached her duties as
to be unworthy of the trust and
confidence
of
Hi-Flyer.First,
there is no denying that Jumuad
was a managerial employee
with the power to discipline
other lower ranked employees.
She did not discipline erring
employees, recommend, plan,
or execute reasonable policies
which eventually led to closure
of one branch. The mere
existence of the grounds for the
loss of trust and confidence
justifies petitioners dismissal.
2011- 2012 Labor Law Case Digest Compilations
UC LLB III-B
Page 107

Pursuant to the Courts ruling in


Lima Land, Inc. v. Cuevas,as
long as there is some basis for
such loss of confidence, such as
when
the
employer
has
reasonable ground to believe
that the employee concerned is
responsible for the purported
misconduct, and the nature of
his participation therein renders
him unworthy of the trust and
confidence demanded of his
position, a managerial employee
may be dismissed.
Third Issue.Jumuads claim
for the reimbursement of the
40% of the value of the car loan
subsidized by Hi-Flyer under its
car loan policy must be denied.
The rights and obligations of the
parties to a car loan agreement
is not a proper issue in a labor
dispute but in a civil one. It
involves the relationship of
debtor and creditor rather than
employee-employer
relations.
Jurisdiction, therefore, lies with
the regular courts in a separate
civil action.
56. Jose Mel Bernante vs
PBA et al
GR No. 192084, September
14, 2011
Topic:
When
Service
Of
Decisions
Is
Deemed
Effected Giving Rise To The
Reglementary Period To File
An
Appeal;
Independent
Labor
Contracting;
Employer-Employee
Relationship
Facts:

Jose Mel Bernante et al


claim that they were invited the
PBA to become referees. During
the term of Commissioner
Bernardino, they were made to
sign
year-to-year
contracts.
Things
changed
during
Commissioner Ealas term. Most
of the time, Bernante et al were
not made to sign contracts, and
when they do, it mostly covers 1
and a half month for the
duration
of
conference.
Bernante et al were issued a
notice in 2004 that their
contract will no longer be
renewed
due
to
their
unsatisfactory performance on
court.
A complaint for illegal
dismissal was filed.
PBA reasoned that their
contracts are mere retainers, so
they are not employees of the
PBA. Their contracts were simply
not renewed. Bermonte et al
countered by stating that they
were required to attend all PBA
games, that PBA was the one
assigning what games the
referees would officiate and
requiring them to follow a
certain work ethic and some
rules, and the power to impose
sanctions among them.
The LA ruled that there is
indeed illegal dismissal and
ordered the reinstatement of the
referees along with payment of
backwages,
damages,
and
attorneys fees. PBA appealed
the decision. Bermonte et al
opposed the appeal on the
ground that it was filed beyond
the reglementary period. The
2011- 2012 Labor Law Case Digest Compilations
UC LLB III-B
Page 108

Decision of the LA was sent


through registered mail to PBA
in March 2005 while the appeal
was filed in March 2006. For its
part, PBA claims that there it
was never served the registered
mail. The Postmaster, however
issued 3 notices when it couldnt
serve the mail and thus,
returned the mail to its sender.
The last notice was issued on
August 2005.
The NLRC affirmed the
LAs Decision. The CA however
reversed both the LA and the
NLRC. The CA ruled that PBA
was an independent contractor
and that there can be no
employer-employee relationship
between the referees and the
PBA since the latter does not
exercise control over the former.
The CA opined that the PBA
does not control the whistles
and the calls that the referees
make.
Issue:
1. Whether the appeal was
filed on time.
2. Whether Bernante et al
were illegally dismissed.
3. Whether there exists
employer-employee relationship
between the referees and the
PBA.
Ruling:
First Issue.The appeal was
filed on time.
Sec 10 of the Rules of
Court provides : SEC. 10.
Completeness of service.
Personal service is complete
upon actual delivery. Service by

ordinary mail is complete upon


the expiration of ten (10) days
after mailing, unless the court
otherwise provides. Service by
registered mail is complete upon
actual receipt by the addressee,
or after five (5) days from the
date he received the first notice
of the postmaster, whichever
date is earlier.
The rule on service by
registered mail contemplates
two
situations:
(1)
actual
service the completeness of
which
is
determined upon
receipt by the addressee of the
registered
mail;
and
(2)
constructive
service
the
completeness
of
which
is
determined upon expiration of
five days from the date the
addressee received the first
notice of the postmaster.
Insofar as constructive
service is concerned, there must
be conclusive proof that a first
notice was duly sent by the
postmaster
to
the
addressee.Not only is it required
that notice of the registered
mail be issued but that it should
also be delivered to and
received by the addressee.
Notably, the presumption that
official duty has been regularly
performed is not applicable in
this situation. It is incumbent
upon a party who relies on
constructive service to prove
that the notice was sent to, and
received by, the addressee. In
this case however, Bermonte et
al did not present any proof that
the notice and certification by
2011- 2012 Labor Law Case Digest Compilations
UC LLB III-B
Page 109

the Postmaster was received by


PBA.
Second Issue and Third
Issue. Bermonte et al were not
illegally dismissed since there is
no
employer-empolyee
relationship between them and
PBA. The element is control is
lacking.
PBA is not the one
controlling when and how the
whistles are blown, and calls are
made. What the PBA merely
required are guidelines for the
referees in performing their job.
In Sonza v. ABS-CBN
Broadcasting Corporation, which
determined
the
relationship
between a television and radio
station and one of its talents,
the Court held that not all rules
imposed by the hiring party on
the hired party indicate that the
latter is an employee of the
former. The Court held:
We find that these general
rules are merely guidelines
towards the achievement of the
mutually desired result, which
are top-rating television and
radio programs that comply with
standards of the industry. We
have ruled that: Further, not
every form of control that a
party reserves to himself over
the conduct of the other party in
relation to the services being
rendered may be accorded the
effect
of
establishing
an
employer-employee relationship.
57. University of the East vs
University of the East
Employees Association et al

GR No. 179593, September


14, 2011
Topic: Exceptions To The
Prohibition On A Second
Motion For Reconsideratoin;
Company Practice As Basis
For Dimunition Of Benefits
Facts:
University of the East
Employees Association (UEEA)
is the registered labor union for
rank and file employees of UE.
When there was a 70% increase
in tuition fee for the School year
1994, the UEEA and UE came to
an agreement regarding its
distribution scheme. The 70%
incremental proceeds from the
tuition
fee
increase
was
distributed by UE to its covered
employees based on a new
formula of percentage of salary.
The distribution scheme agreed
upon is based on 5% of the
wages of UEs employees. To
this, UEEA President, Ernesto
Verceles later objected and
demanded that the old manner
provided for in their CBA shall
be resorted to, which is 100 Php
per month for rank and file
employees for every 1 Php
share of a full load teaching
personnel. To support their
stand, UEEA presented the
former agreement on the old
distribution scheme where it
expressly provides UE the
discretion to distribute tuition
fee
incremental
proceeds,
except where it forms part of an
existing CBA.
When another meeting
was conducted to determine the
2011- 2012 Labor Law Case Digest Compilations
UC LLB III-B
Page 110

distribution scheme, it was


agreed that the distribution
scheme shall be based on salary
percentage. This was attested to
by the UEEA officers. However,
5 years later, in 1999 UEEA filed
a case before the NLRC for
underpayment of underpayment
of the rank-and-file employees
share
of
the
tuition
fee
increases against UE pursuant
to P.D. No. 451, as amended,
and Republic Act (R.A.) No. 6728
otherwise known as Government
Assistance to Students and
Teachers in Private Education
Act.
UE reasoned that the law
is silent to the manner of
distribution and that it allows
distribution according to the
Universitys discretion. Further it
reasoned
that
the
new
distribution scheme was validly
agreed upon, and that UEEA
officers even attested to it the
second time that a meeting was
called
to
determine
the
distribution scheme. As a result,
the 5% salary base as basis for
distribution was increased to
9.9%.
Therefore,
UEEA
is
estopped from questioning the
distribution
scheme.
Furthermore, since the case is a
money claim arising out of an
employer-employee relationship,
the right to bring about an
action prescribes in 3 years from
the moment that a cause of
action arises. UEEA waited for 5
years
before
assailing
the
agreement.
The LA ruled in favor of
the UEEA. It ruled that indeed,

the old distribution scheme had


become a practice of UE since
1983, had become part of the
CBA between UEEA and UE, and
that the tripartite agreement
providing for the old distribution
scheme expressly recognized
that UEs distribution may not
override CBA provisions. It
further ruled that no new
tripartite agreement has taken
place over the old agreement.
On appeal to the NLRC,
the NLRC sustained the LAs
finding.
However,
Upon
2
motions for reconsideration by
UE, the NLRC reversed the LAs
findings and ruled in favor of UE.
Upon appeal to the CA, the CA
overruled the NLRC reasoning
that
a
2nd
motion
for
reconsideration is prohibited.
Issue:
1. Whether a 2nd motion
for reconsideration is valid.
2. Whether the change in
the distribution scheme of the
incremental increase of tuition
fees goes against company
tradition
and
constitutes
dimunition of benefits, thus
becoming invalid.
Ruling:
First. In this case, the 2nd
motion for reconsideration filed
by UE is valid. As a rule, a 2nd
motion for reconsideration is a
prohibited pleading. However,
since rules of courts are
designed to expedite disposition
of cases and not to impede
attainment
of
substantial
justice, rules may be relaxed.
There are valid exceptions to
2011- 2012 Labor Law Case Digest Compilations
UC LLB III-B
Page 111

the 2nd MR rule in cases of


extraordinarily
persuasive
reasons such a when the
assailed decision is patently
void. The exceptional reasons
for this case include the
following:
1. Prescription has set in
on the right of the employees
(UEEA) to bring about their
action on a money claim arising
from
employer-employee
relationship.
2. A new agreement
providing for the new scheme
has
overridden
the
old
agreement prescribing the old
scheme.
3. The new agreement
was voluntarily entered into.
4. The law indeed provides
discretion on the part of the
University
to
distribute
incremental increase in tuition
fees.
Second
Issue.The
old
distribution scheme has not
ripened into company practice,
thus there is no dimunition of
benefits if the new distribution
scheme will be implemented.
Generally, employees have a
vested
right
over
existing
benefits voluntarily granted to
them by their employer, thus,
said benefits cannot be reduced,
diminished,
discontinued
or
eliminated by the latter. This
principle against diminution of
benefits, however, is applicable
only if the grant or benefit is
founded on an express policy or
has ripened into a practice over
a long period of time which is
consistent and deliberate. It

does
not
contemplate
the
continuous
grant
of
unauthorized
or
irregular
compensation
but
it
presupposes that a company
practice, policy and tradition
favourable to the employees has
been clearly established; and
that the payments made by the
company pursuant to it have
ripened into benefits enjoyed by
them. The test or rationale of
this rule on long practice
requires an indubitable showing
that the employer agreed to
continue giving the benefits
knowing fully well that said
employees are not covered by
the law requiring payment
thereof.
In this case, even if the
old distribution scheme has
been practiced since 1983,
there is no showing that the UE
is well aware that it need not
give in to that old scheme but
agreed to such scheme anyway.
UEs act is not deliberate and
voluntary and that no law or
jurisprudence provides for a
minimum
years
before
an
employers gratuitous act will be
considered a company practice.
58. UNITED LABORATORIES
VS JAIME DOMINGO ET AL.
GR No. 186209, September
21, 2011
Topic: Redundancy;
Constructive Dismissal
Facts:
Jaime Domingo et al, are
former accountants of Unilab
manning
16
Distribution
2011- 2012 Labor Law Case Digest Compilations
UC LLB III-B
Page 112

Centers. Unilab soon merged


these Distribution Centers all
into Manila District under a
Physical
Distribution
Master
Plan. Jaime Domingo et als
positions were not declared
redundant. What they want is to
transfer to another position
which is considered redundant
to avail of the 2 and a half
month separation pay per year
of service. When it cant be
granted,
Domingo
et
al
requested for separation or
retirement from service under a
separation package similar or
equivalent to that of the
redundant employees in the
provincial depots, which is much
higher
than
theirs.
They
reasoned that they occupied
similar positions warranting the
same treatment. When Unilab
did not give in to their demands,
Domingo et al filed a complaint
for constructive dismissal and
nonpayment of separation pay.
They also pointed to the
Distribution
Plan
as
an
unreasonable
cost
cutting
measure.
Curiously
enough,
during the pendency of their
case, Domingo et al worked for
Unilab until they received their
full benefits and executed a
quitclaim until they eventually
separated from the company.
Thus the LA dismissed the
case. The NLRC affirmed the
LAs ruling. The CA however
found that there is indeed
Constructive
Dismissal
but
during the pendency of the
proceedings with the CA, a
quitclaim was executed by

Domingo et al. Some of the


counsels for the employees do
not want to withdraw and fulfill
the quitclaim.
Issue:
1. Whether the acts of the
employees
counsel
is
appropriate.
2. Whether there is constructive
dismissal.
Ruling:
First Issue.No. A lawyer
owes candor, fairness and good
faith to the Court. Specifically,
Rule 10.01: A lawyer shall not
do any falsehood, nor consent to
the doing of any in Court; nor
shall he mislead, or allow the
Court to be misled by any
artifice.
Second
Issue.There
is
no
constructive dismissal. In this
case, the employees want to
transfer to another position
which has been declared as
redundant, to avail of benefits
under the separation pay. There
is even no dismissal to speak of.
The concept of constructive
dismissal is inapplicable to
respondents.
59. Elmer Lopez Vs Keppel
Bank Philippines, et al.
GR No. 176800, September
5, 2011
Topic: Dismissal For Just
Cause; Breach Of Trust And
Confidence As Grounds For
Dismissal.
Facts:
2011- 2012 Labor Law Case Digest Compilations
UC LLB III-B
Page 113

Elmer Lopez was a branch


manager of Keppel Bank in
Iloilo. He enticed Hertz Cars
Exclusive to be one of the
banks clients.
He was later sent a notice
by Keppel Bank to explain why
he issued Purchase Orders to
Hertz Cars for 13 brand new
cars without authority. Elmer
Lopez sent a written explanation
which was given no credit.
Lopez
was
terminated
thereafter.
Lopez
moved
for
reconsideration and a meeting/
hearing was conducted. Lopez
attended with a lawyer and a
military man. But still, the bank
affirmed its decision. Lopez then
filed a case for illegal dismissal.
Lopez claimed that he issued
Purchase Orders
to
entice
clients and promote growth of
the bank. Lopez claimed that
through his Purchase Orders, the
bank got gains. The Bank
reiterated that Purchase Orders
may only be made with express
authority from the Bank.
The LA ruled that Lopez
was illegally dismissed. The
NLRC reversed the LA as it ruled
that issued Purchase Orders
without
express
authority
constitutes willful disobedience
against his superiors. The NLRC
also
ruled
that
the
twin
requisites of Notice and Hearing
were complied with by the Bank.
The CA also affirmed the NLRCs
decision. Lopez assails the CAs
cognizance of the case for
failure of Keppel bank to file a
notice of appeal together with

the memorandum of appeal,


and that the bank did not post
an appeal bond.
Issue:
1. Whether
the
appeal
was
perfected
2. Whether Lopez was dismissed
for just cause due to willful
disobedience of superiors.
Ruling:
First Issue. The matter
raised by Lopez is a new issue
not proper for cognizance. He
never raised the issue before
the CA. His willful participation
in the proceedings before the
CA cures any defect in the
appeal.
Second Issue.Lopez was
dismissed for just cause. He
maintains that there is inherent
authority in his position as bank
manager to issue Purchase
Orders even without express
authority. That is wrong. He can
not be said to have been
illegally dismissed as notice and
hearing were given to him. As a
branch manager, Lopez clearly
occupies a position of trust. His
continuous issuance of Purchase
Orders despite clear orders not
to justifies breach of trust and
confidence.
60. Pamela Jumuad vs
HiFlyer et al., (see case No.
55)

2011- 2012 Labor Law Case Digest Compilations


UC LLB III-B
Page 114

61.ALERT SECURITY AND


INVESTIGATION AGENCY,
INC. vs. .PASAWILAN, et al.
GR No. 182397, September
14, 2011
Topics:
Illegal
Dismissal;
Abandonment; Valid Transfer
Facts:
Pasawilan,
et
al.,
all
security guards of Alert Security
and Investigation Agency, Inc.
,filed a complaint for money
claims against Alert Security as
they were receiving P165.00
only a day. As a result of their
complaint, they were relieved
from their posts and were not
given new assignments despite
the lapse of six months.
Thereafter, they filed a joint
complaint for illegal dismissal
against Alert Security. Alert
Security
contends
that
Pasawilan, et al. were merely
transferred and they refused to
report to their new posts. The
Labor
Arbiter
ruled
that
Pasawilan, et al. were illegally
dismissed. The NLRC reversed
the decision, ruling that the
generalization
that
the
complainants
were
constructively dismissed is not
sufficient
to
establish
the
existence of illegal dismissal.
The CA reversed the NLRC
decision and ruled that Alert
Security, as an employer, failed
to discharge its burden to show
that the employees separation
from employment was not
motivated by discrimination,
made in bad faith, or effected as
a form of punishment or

demotion
without
sufficient
cause. Alert Security then filed a
petition for review on certiorari
before the SC. Alert Security
contends that Pasawilan, et al.
were merely transferred to new
posts and they voluntarily
abandoned their jobs when they
failed to report for duty in the
new location.
Issues:
1.Whether
respondents
were illegally dismissed.
2.Whether
respondents
abandoned their jobs.
3.Whether the transfer of
respondents to a new post is
valid.
Ruling:
First Issue. Yes. Although
we recognize the right of
employers to shape their own
work force, this management
prerogative must not curtail the
basic right of employees to
security of tenure. There must
be a valid and lawful reason for
terminating the employment of
a worker. Otherwise, it is illegal
and would be dealt with by the
courts accordingly.
In the case at bar,
respondents were relieved from
their posts because they filed
with the Labor Arbiter a
complaint
against
their
employer for money claims due
to underpayment of wages. This
reason is unacceptable and
illegal. Nowhere in the law
providing for the just and
authorized
causes
of
2011- 2012 Labor Law Case Digest Compilations
UC LLB III-B
Page 115

termination of employment is
there any direct or indirect
reference to filing a legitimate
complaint for money claims
against the employer as a valid
ground for termination.
Second Issue. No. For
abandonment of work to fall
under
Article
282
(b)
of
the Labor Code, as amended, as
gross and habitual neglect of
duties there must be the
concurrence of two elements.
First, there should be a failure of
the employee to report for work
without a valid or justifiable
reason, and second, there
should be a showing that the
employee intended to sever the
employer-employee relationship,
the second element being the
more determinative factor as
manifested by overt acts.
As regards the second
element of intent to sever the
employer-employee relationship,
the CA correctly ruled that the
fact that petitioners filed a
complaint for illegal dismissal is
indicative of their intention to
remain employed with private
respondent considering that one
of their prayers in the complaint
is for re-instatement. Further,
respondents continued to report
for work after the alleged
transfer order was issued, which
makes
it
unlikely
that
respondents
have
clear
intention
of
leaving
their
respective jobs.
On the element of the
failure of the employee to report
for work, we also cannot accept

the allegations of petitioners


that respondents unjustifiably
refused to report for duty in
their new posts. A careful review
of the records reveals that there
is no showing that respondents
were notified of their new
assignments. Granting that the
"Duty Detail Orders" were
indeed issued, they served no
purpose unless the intended
recipients of the orders are
informed of such.
Third
Issue.
No.
The
managerial
prerogative
to
transfer personnel must be
exercised without grave abuse
of discretion, bearing in mind
the basic elements of justice
and fair play. Having the right
should not be confused with the
manner in which that right is
exercised. Thus, it cannot be
used as a subterfuge by the
employer to rid himself of an
undesirable worker. In particular,
the employer must be able to
show that the transfer is not
unreasonable, inconvenient or
prejudicial to the employee; nor
does it involve a demotion in
rank or a diminution of his
salaries, privileges and other
benefits.
In addition to these tests
for a valid transfer, there should
be proper and effective notice to
the employee concerned. It is
the employers burden to show
that the employee was duly
notified of the transfer. Verily,
an employer cannot reasonably
expect an employee to report
for work in a new location
2011- 2012 Labor Law Case Digest Compilations
UC LLB III-B
Page 116

without first informing said


employee of the transfer. Alert
Securitys insistence on the
sufficiency of mere issuance of
the transfer order is indicative of
bad faith on their part.
62. NISSAN MOTORS PHILS.,
INC. vs VICTORINO ANGELO
G.R. No. 164181, September
14, 2011
Topic: Illegal Dismissal
Facts:
Victorino
Angelo
was
dismissed by Nissan on the
grounds of serious misconduct,
willful disobedience and gross
neglect of duties. He failed to
prepare
payrolls
on
two
occasions without valid reasons,
and despite reminders from
Nissan. Further, Angelo wrote an
explanation letter to Nissan,
using inflammatory language.
Angelo filed a complaint for
illegal dismissal. The Labor
Arbiter dismissed the complaint
for lack of merit. It was affirmed
by the NLRC. The CA, however,
ruled that Angelo was illegally
dismissed. Hence, Nissan filed a
petition for review on certiorari
before the SC.
Issue:
WhetherAngelo
illegally dismissed.

was

Ruling:
No.
The
Labor
Code
provides that an employer may
terminate the services of an
employee
for
a
just
cause. Nissan dismissed Angelo

based on allegations of serious


miscounduct,
willful
disobedience and gross neglect.
Misconduct is improper or
wrong conduct. It is the
transgression
of
some
established and definite rule of
action, a forbidden act, a
dereliction of duty, willful in
character, and implies wrongful
intent and not mere error in
judgment.
Such misconduct,
however
serious,
must
nevertheless be in connection
with the employee's work to
constitute just cause for his
separation. Past decisions of this
Court have been one in ruling
that
accusatory
and
inflammatory language used by
an employee to the employer or
superior can be a ground for
dismissal or termination.
Disobedience, to be a just
cause for termination, must be
willful or intentional, willfulness
being
characterized
by
a
wrongful and perverse mental
attitude
rendering
the
employees act inconsistent with
proper subordination. A willful or
intentional disobedience of such
rule, order or instruction justifies
dismissal only where such rule,
order or instruction is (1)
reasonable and lawful, (2)
sufficiently
known
to
the
employee, and (3) connected
with the duties which the
employee has been engaged to
discharge.
Neglect of duty, to be a
ground for dismissal, must be
both gross and habitual.
In
finding that petitioner was able
2011- 2012 Labor Law Case Digest Compilations
UC LLB III-B
Page 117

to adduce evidence that would


justify
its
dismissal
of
respondent, the NLRC correctly
ruled that the latter's failure to
turn over his functions to
someone capable of performing
the vital tasks which he could
not
effectively
perform
or
undertake because of his heart
ailment or condition constitutes
gross neglect.
It must be emphasized
at this point that the onus
probandi to prove the lawfulness
of the dismissal rests with the
employer. In termination cases,
the burden of proof rests upon
the employer to show that the
dismissal is for just and valid
cause. Failure to do so would
necessarily mean that the
dismissal was not justified and,
therefore, was illegal. In this
case, both the Labor Arbiter and
the NLRC were not amiss in
finding that the dismissal of
respondent was legal or for a
just cause based on substantial
evidence
presented
by
petitioner.
Substantial
evidence, which is the quantum
of proof required in labor cases,
is that amount of relevant
evidence which a reasonable
mind might accept as adequate
to justify a conclusion.
63. BPI EMPLOYEES UNION
METRO MANILA vs. BANK OF
THE PHILIPPINE ISLANDS
G.R. No. 178699, September
21, 2011
Topic: Basis Of Computation
Of Backwages/ Definition Of
Full Backwages

Facts:
Zenaida Uys services as a
bank teller in BPIs Escolta Branch
was terminated on grounds of
gross
disrespect/discourtesy
towards an officer, insubordination
and absence without leave. Uy,
together with the Union, thus filed
a case for illegal dismissal. The
case reached the Supreme
Court which ordered BPI to pay
petitioner Uy backwages from the
time of her illegal dismissal until
her actual reinstatement; and to
reinstate petitioner Uy to her
former
position,
or
to
a
substantially
equivalent
one,
without loss of seniority right and
other benefits attendant to the
position.
Uy computed the amount of
her back wages based on the
current wage level and included all
the increases in wages and
benefits under the CBA that were
granted during the entire period of
her illegal dismissal. BPI disputed
Uy's/Unions computation arguing
that the basis for the computation
of back wages should be the
employees wage rate at the time
of dismissal.
The Voluntary
Arbitrator
agreed
with
Uys/Unions contention that the
payment of full back wages
computed from the time of
dismissal
until
actual
reinstatement
including
all
benefits under the CBA.
The CA ruled that the
computation of Uys full back
wages, as defined under Republic
Act No. 6715, should be based on
the basic salary at the time of her
2011- 2012 Labor Law Case Digest Compilations
UC LLB III-B
Page 118

dismissal
plus
the
regular
allowances that she had been
receiving likewise at the time of
her dismissal. It held that any
increase in the basic salary
occurring after Uys dismissal as
well as all benefits given after said
dismissal should not be awarded
to her in consonance with settled
jurisprudence on the matter.
Issue:
Whether the increases in
wages and benefits under the CBA
that were granted during the
entire period of her illegal
dismissal should be included in the
computation of backwages.
Ruling:
No.The base figure in
computing the award of back
wages to an illegally dismissed
employee is the employees basic
salary plus regular allowances and
benefits received at the time of
dismissal, unqualified by any wage
and benefit increases granted in
the interim. Full backwages
means backwages without any
deduction
or
qualification,
including
benefits
or
their
monetary
equivalent
the
employee is enjoying at the time
of his dismissal.
Clearly, it is the intention of
the Supreme Court to grant unto
Private
Respondent
Uy
full
backwages as defined under RA
6715. Consequently, any benefit
or allowance over and above that
allowed and provided by said law
is deemed excluded under said SC
Decision.
The CBA benefits
awarded by Public Respondent is

not within the benefits under RA


6715. Said benefits are not to be
included in the backwages.
64. EMIRATE SECURITY AND
MAINTENANCE SYSTEMS,
INC.
vs.GLENDA M. MENESE
GR Nos. 175699, 178735,
October 5, 2011
Topic: Illegal Dismissal; Valid
Transfer
Facts:
Glenda Menese, a payroll
and billing clerk of Emirate
Security, filed a complaint for
constructive dismissal; illegal
reduction
of
salaries
and
allowances, etc. While still an
employee, she was required to
work daily with a low salary, and
then her position was given to
another. She was asked to
become a lady guard of Emirate
Security, but refused as her rank
and salary will be reduced. She
was harassed and humiliated in
the work place, and her salary
was withheld. Thereafter, she
was dismissed; with threats that
she will be physically removed
from the establishment in case
she refuses to leave.
The Labor Arbiter ruled in
favor of Menese and ordered her
reinstatement.
The
NLRC
reversed the judgment, ruling
that
Menese
was
merely
transferred and is a valid
management prerogative. The
CA reinstated the Labor Arbiter
decision, ruling that while it is
the managements prerogative
2011- 2012 Labor Law Case Digest Compilations
UC LLB III-B
Page 119

to transfer an employee from


one office to another within the
business establishment, it is not
without limitation. It must be
exercised in such a way that
there is no demotion in rank or
diminution in pay, benefits and
other privileges. Otherwise, the
transfer
amounts
to
a
constructive dismissal.
Issue:
Whether
Menese
illegally dismissed.

was

Ruling:
Yes.
The
managerial
prerogative
to
transfer
personnel must be exercised
without abuse of discretion,
bearing in mind the basic
elements of justice and fair play.
Having the right should not be
confused with the manner in
which that right is exercised.
Thus, it should not be used as a
subterfuge by the employer to
get rid of "an undesirable
worker." Measured against this
basic precept, the petitioners
undoubtedly
abused
their
discretion
or
authority
in
transferring Menese to the
agencys head office. She had
become "undesirable" because
she stood in the way of Claros
entry into the PGH detachment.
Menese had to go, thus the
need for a pretext to get rid of
her. The request of a client for
the
transfer
became
the
overriding
command
that
prevailed over the lack of basis
for the transfer.

We cannot blame Menese


for refusing Yans offer to be
transferred. Not only was the
transfer arbitrary and done in
bad faith, it would also result, as
Menese feared, in a demotion in
rank and a diminution in pay. In
these lights, Meneses transfer
constituted
a
constructive
dismissal as it had no justifiable
basis and entailed a demotion in
rank and a diminution in pay for
her. For a transfer not to be
considered
a
constructive
dismissal, the employer must be
able to show that the transfer is
for a valid reason, entails no
diminution in the terms and
conditions of employment, and
must
be
unreasonably
inconvenient or prejudicial to
the employee. If the employer
fails to meet these standards,
the employees transfer shall
amount, at the very least, to
constructive
dismissal.
The
petitioners, unfortunately for
them, failed to come up to these
standards.
In
declaring
Meneses
transfer to be in the valid
exercise of the petitioners
management prerogative, the
NLRC grossly misappreciated
the evidence and, therefore,
gravely abused its discretion in
closing its eyes to the patent
injustice committed on Menese.
It completely disregarded the
obvious presence of bad faith in
Meneses transfer. Labor justice
demands
that
Menese
be
awarded moral and exemplary
damages and, for having been
constrained to litigate in order
2011- 2012 Labor Law Case Digest Compilations
UC LLB III-B
Page 120

to protect her rights, attorneys


fees.
65. MARITIME FACTORS INC.
vs BIENVENIDO R. HINDANG
GR No. 151993, October 19,
2011
Topic: Weight/ Credibility Of
Findings Of Facts Of NLRC;
Authenticity Of Documents
Attached
As
Evidence;
Claims For Compensation For
Death Of Seamen
Facts:
Danilo R. Hindang, a
seaman under the employ of
Maritime Factors, was found
hanging by a strap on his neck
in a kneeling position inside his
locker. The Medical Examiner of
Saudi Arabia conducted an
autopsy on Danilo's remains and
concluded
that
Danilo
committed suicide by hanging
himself. Danilo's remains were
repatriated to the Philippines
where
an
autopsy
was
conducted by the National
Bureau of Investigation (NBI)
and concluded that the cause of
Danilo's death was Asphyxia by
Strangulation, Ligature.
Thereafter, Bienvenido R.
Hindang,
brother
of
the
deceased,
filed
for
death
compensation benefits pursuant
to
the
POEA
Standard
Employment Contract Governing
the Employment of All Filipino
Seamen on Board Ocean-Going
Vessels.
Maritime
Factors
contend that Danilo committed
suicide by hanging himself;
thus,
his
death
is
not

compensable.
Bienvenido
contended that the NBI autopsy
report categorically declared
that the cause of Danilo's death
was Asphyxia by strangulation,
ligature. The Saudi Arabian
medical report cannot be given
legal effect, since the report was
a mere photocopy of a fax
transmission
from
Maritime
Factors foreign principal, hence,
the document was unreliable as
to its due execution and
genuineness.
The Labor Arbiter ruled
that Danilo did not commit
suicide
and
ordered
the
payment of death benefits. It
found that the NBI autopsy
report concluding that Danilo
died
of
Asphyxia
by
strangulation should be given
credence as against petitioner's
evidence which consisted of a
mere photocopy of the fax
transmission of the alleged
medical jurisprudence report
from Saudi Arabia.The NLRC
affirmed the Labor Arbiter
decision. Subsequently, the CA
affirmed the NLRC decision.
Hence, Maritime Factors filed a
petition for review on certiorari
before the SC.

Ruling:
First
Issue.
Yes.
Elementary is the principle that
this Court is not a trier of facts.
Judicial review of labor cases
does not go beyond the
evaluation of the sufficiency of
the evidence upon which its
labor officials findings rest. As
such, the findings of fact and
conclusion of the NLRC are
generally accorded not only
great weight and respect but
even clothed with finality and
deemed binding on this Court as
long as they are supported by
substantial evidence. This is
because it is not the function of
this Court to analyze or weigh
all over again the evidence
already
considered
in
the
proceedings
below;
or
reevaluate the credibility of
witnesses; or substitute the
findings
of
fact
of
an
administrative tribunal which
has expertise in its special
field. In this case, we adopt the
factual findings of the LA, as
affirmed by the NLRC and the
CA, as the same was duly
supported
by
substantial
evidence.

Issues:
1. Whether the decision of
the NLRC as to the cause of
death should be accorded
respect.
2. Whether the photocopy
of the Saudi Arabian medical
report should be disregarded.
3. Whether Hindang is
entitled to the death benefits.

Second Issue. Yes. The


existence of the original medical
report, which was written in the
Arabic
language,
was
not
attached to the records and has
not been proved. Moreover, the
identity of the person who made
the translation and whether the
translator has the recognized
competence in both English and

2011- 2012 Labor Law Case Digest Compilations


UC LLB III-B
Page 121

the language the medical report


was originally written are not
established. Thus, there is no
clear
assurance
that
the
translated
words
are
the
accurate translation of the
original medical report of the
Saudi
Arabian
medical
examiner.More importantly, the
alleged
translated
medical
report was not even signed
which creates doubt as to its
authenticity.
The
unsigned
translated medical report is
nothing
but
a
self-serving
document which ought to be
treated as a mere scrap of
paper devoid of any evidentiary
value even in administrative
proceedings.
Thus,
based
on
the
foregoing, the photocopy of the
fax transmission of an alleged
translated medical report was
correctly denied consideration,
since it is required that there be
some proof of authenticity or
reliability as condition for the
admission of documents.
Third Issue. Yes. In order
to avail of death benefits, the
death of the employee should
occur during the effectivity of
the employment contract. The
death of a seaman during the
term of employment makes the
employer liable to his heirs for
death compensation benefits.
Once it is established that the
seaman
died
during
the
effectivity of his employment
contract,
the
employer
is
liable. This rule, however, is not
absolute. The employer may be
2011- 2012 Labor Law Case Digest Compilations
UC LLB III-B
Page 122

exempt from liability if he can


successfully prove that the
seaman's death was caused by
an injury directly attributable to
his
deliberate
or
willful
act. Clearly,
respondent's
entitlement to any death benefit
depends on whether petitioner's
evidence suffices to prove that
Danilo committed suicide, and
the burden of proof rests on
petitioner.
In the case at bar, we find that
the records are bereft of any
substantial evidence showing
that
petitioner
employer
successfully
discharged
its
burden of proving that Danilo
committed suicide so as to
evade its liability for death
benefits under POEA's Standard
Employment Contract for Filipino
Seaman.
66. ENRIQUE U. BETOY vs.
THE BOARD OF DIRECTORS,
NATIONAL POWER
CORPORATION
GR Nos. 156556-57, October
4, 2011
Topic: Locus Standi To Raise
Issues Of Constitutionality;
Validity Of Reorganization;
Facts:
The
Electric
Power
Industry Reform Act of 2001
(EPIRA)
was
enacted
by
Congress with the goal of
restructuring the electric power
industry and privatization of the
assets of the National Power
Corporation (NPC). Section 63 of
the
EPIRA
provides
for
separation benefits to officials

and employees who would be


affected by the restructuring of
the electric power industry and
the privatization of the assets of
the NPC. Displaced or separated
personnel as a result of the
privatization, if qualified, shall
be given preference in the hiring
of the manpower requirements
of the privatized companies.
Pursuant to sec. 63 of the EPIRA,
Enrique U. Betoy, together with
thousands of his co-employees
from the NPC were terminated.
Hence, Betoy filed a special civil
action
for certiorari1 and
supplemental
petition
for mandamus before the SC
assailing sec 63, of the said law
and other provisions thereof.
Issues:
1. Whether Betoy may
directly
question
the
constitutionality of the EPIRA
before this Court.
2.
Whether
the
reorganization of NPC is valid.
3. Whether Sec. 63 of the
EPIRA Law is constitutional.
Ruling:
First Issue. No. It has long
been established that this Court
will not entertain direct resort to
it unless the redress desired
cannot be obtained in the
appropriate courts, or where
exceptional
and
compelling
circumstances justify availment
of a remedy within and call for
the exercise of our primary
jurisdiction. Thus, herein petition
should already be dismissed at
the outset; however, since
2011- 2012 Labor Law Case Digest Compilations
UC LLB III-B
Page 123

similar petitions have already


been resolved by this Court
tackling the validity of NPB
Resolutions No. 2002-124 and
No. 2002-125, as well as the
constitutionality
of
certain
provisions of the EPIRA, this
Court
shall
disregard
the
procedural defect.
Second
Issue.
Yes.
Reorganization
involves
the
reduction
of
personnel,
consolidation of offices, or
abolition thereof by reason of
economy or redundancy of
functions. It could result in the
loss of ones position through
removal or abolition of an office.
However, for a reorganization
for the purpose of economy or
to make the bureaucracy more
efficient to be valid, it must pass
the test of good faith; otherwise,
it is void ab initio.
It is undisputed that NPC
was in financial distress and the
solution found by Congress was
to pursue a policy towards its
privatization. The privatization
of NPC necessarily demanded
the
restructuring
of
its
operations. To carry out the
purpose, there was a need to
terminate employees and re-hire
some
depending
on
the
manpower requirements of the
privatized
companies.
The
privatization and restructuring
of the NPC was, therefore, done
in good faith as its primary
purpose was for economy and to
make the bureaucracy more
efficient.

Petitioner's allegation that


the reorganization was merely
undertaken to accommodate
new appointees is at most
speculative and bereft of any
evidence on record. It is settled
that bad faith must be duly
proved
and
not
merely
presumed. It must be proved by
clear
and
convincing
evidence, which is absent in the
case at bar.
Third Issue. Yes. Section
63 of the EPIRA and Section 33
of the IRR of the EPIRA did not
impair the vested rights of NPC
personnel to claim benefits
under existing laws. Neither
does the EPIRA cut short the
years
of
service
of
the
employees concerned. If an
employee
availed
of
the
separation
pay
and
other
benefits in accordance with
existing laws or the superior
separation pay under the NPC
restructuring plan, it is but
logical that those who availed of
such privilege will start their
government service anew if they
will later be employed by any
government-owned
successor
company
or
government
instrumentality.
The receipt of retirement
benefits does not bar the retiree
from receiving separation pay.
Separation pay is a statutory
right designed to provide the
employee with the wherewithal
during the period that he/she is
looking
for
another
employment. On the other hand,
retirement benefits are intended
2011- 2012 Labor Law Case Digest Compilations
UC LLB III-B
Page 124

to help the employee enjoy the


remaining years of his life,
lessening
the
burden
of
worrying about his financial
support, and are a form of
reward for his loyalty and
service to the employer. A
separation pay is given during
ones employable
years, while
retirement benefits are given
during
ones unemployable
years. Hence, they are not
mutually exclusive
67. CARMELITO N.
VALENZONA, PETITIONER VS
FAIRSHIPPING CORPORATION
AND/OR SEJIN LINES
COMPANY LIMITED,
RESPONDENTS
GR No. 176884, October 19,
2011
Topic: Claims For Disability
By Migrant Workers Working
In Vessels
Facts:
On
May
5,
2001,
respondent
Fair
Shipping
Corporation, for and on behalf of
its principal, respondent Sejin
Lines Company Limited, hired
petitioner
as
2nd Assistant
Engineer aboard its vessel M/V
Morelos for a duration of nine
months. Before his embarkation
on May 23, 2001, he was
declared medically fit to work.
However, while aboard the
vessel on September 29, 2001,
petitioner complained of chest
pain. He was thus brought to
Centro
Medico
Quirurgico
Echauri in Mexico where he was
confined up to October 6, 2001

and
diagnosed
with
hypertensive crisis, high blood
pressure.
Petitioner
was
then
examined by Dr. Cruz who
diagnosed
his
illness
as
hypertension. He was then
treated for 6 months by Dr.
Cruz. The petitioner, however,
consulted another Doctor who
diagnosed
him
with
Hypertensive
Cardiovascular
Disease. Due to his prolonged
illness, he demanded payment
of his balance of his sickness
allowance
and
permanent
disability
benefits
to
the
company but was unheeded.
After which, Dr. Cruz issued a
certification declaring petitioner
as fit to work.
The
petitioner
was
unconvinced so he consulted
another doctor who diagnosed
him
with
Ischemic
Heart
Disease and declared him
unfit to work in any capacity.
Hence, petitioner filed a claim
for
recovery
of
disability
benefits, sickness allowance and
other damages.
Issue:
Whether the petitioner is
entitled to receive permanent
disability benefits.
Ruling:
Petitioner is entitled to
permanent disability benefits.
a)
The certification by
the
company-designated
physician that petitioner is fit to
work was issued after 199 days or
more than 120 days from the time
2011- 2012 Labor Law Case Digest Compilations
UC LLB III-B
Page 125

he was medically repatriated to


the Philippines.
Petitioner
herein
was
medically repatriated to the
Philippines
on
October
8,
2001. However, it was only on
April 25, 2002 or after a lapse of
199 days that Dr. Cruz issued a
certification declaring him fit to
work. Thus, we declare herein,
just
as
we
pronounced
in Quitoriano, that
petitioners
disability
is
considered permanent
and
total because the fit to work
certification was issued by Dr.
Cruz only on April 25, 2002, or
more than 120 days after he
was medically repatriated on
October 8, 2001.
b.)
The
company

designated
physicians
certification that petitioner is fit to
work does not make him ineligible
for permanent total disability
benefits.
Considering
the
circumstances prevailing in the
instant case, we likewise rule
that it does not matter that the
company-designated
physician
assessed petitioner as fit to
work. It is undisputed that from
the time petitioner was repatriated
on October 8, 2001, he was
unable to work for more than 120
days as he was only certified fit to
work
on
April
25,
2002. Consequently, petitioner's
disability is considered permanent
and total. In fact, from his
repatriation until the filing of his
petition before this Court on March
21, 2007, or for more than five
years, petitioner claims that he

was unable to resume his job as a


seaman which thus strongly
indicates that his disability is
permanent and total. Also, we
note that the certification was
issued
only
after
petitioner
consulted a private physician (Dr.
Mapapala) and after he formally
demanded from the respondents,
through his lawyer, the payment
of
his
sickness
allowance,
disability benefits and attorney's
fees.
Consequently, we find it
irrelevant to discuss at this
juncture as to which prognosis,
that of Dr. Cruz or petitioner's
private physicians', is more
accurate.
68. MARITIME FACTORS INC.,
PETITIONER, VS.
BIENVENIDO R. HINDANG,
RESPONDENT.
G.R. No. 151993, October 19,
2011
Topic:Weight/Credibility
Of
Findings Of Facts Of Labor
Arbiters And NLRC; Claims
For
Death
Benefits
Of
Migrant Workers
Facts:
On
June
10,
1994,
petitioner Maritime Factors Inc.,
a domestic manning agency, for
and in behalf of its foreign
principal
Bahrain
Marine
Contracting/Panama,
engaged
the services of Danilo R.
Hindang (Danilo) to work as
GP/Deckhand on board the M/T
"Reya,"
a
Panamanianregistered ocean-going vessel.
Danilo's contract of employment
2011- 2012 Labor Law Case Digest Compilations
UC LLB III-B
Page 126

was for a period of 12 months


with a basic monthly salary of
US$230.00.
On July 27, 1994, while in
the Kingdom of Saudi Arabia
and on board the vessel, Chief
Mate Marcial Lauron, Jr., AB
Jaime Aguinaldo and Oiler Allan
P. Sarabia forced open Danilo's
cabin door by taking out the
screws on the door lock with a
screw
driver.
They
found
Danilo's body inside the locker
(wardrobe) of his cabin. Danilo
was found hanging by a strap on
his neck in a kneeling position.
Upon arriving at West Pier, Ras
Tanurah, they turned over
Danilo's body to the Saudi police
authorities.
The
Medical
examiner concluded that Danilo
committed suicide.
Danilos body was then
repatriated to the Philippines
where his family filed for death
compensation benefits with the
POEA. The petitioner claims that
Danilos cause of death is
suicide
therefore,
noncompensable. The respondents
argued that as per NBI autopsy
report, the cause of death was
Asphyxia by strangulation.
Issue:
Whether
Danilo
committed suicide during the
term
of
his
employment
contract which would exempt
petitioner from paying Danilo's
death compensation benefits to
his beneficiaries.
Ruling:

The Supreme Court held


the decision of the Labor
Arbiter, the NLRC and the CA
that Danilos death was not
suicide therefore, compensable.
Elementary is the principle
that this Court is not a trier of
facts. Judicial review of labor
cases does not go beyond the
evaluation of the sufficiency of
the evidence upon which its
labor officials' findings rest. As
such, the findings of fact and
conclusion of the NLRC are
generally accorded not only
great weight and respect but
even clothed with finality and
deemed binding on this Court as
long as they are supported by
substantial evidence. This is
because it is not the function of
this Court to analyze or weigh
all over again the evidence
already
considered
in
the
proceedings
below;
or
reevaluate the credibility of
witnesses; or substitute the
findings
of
fact
of
an
administrative tribunal which
has expertise in its special field.
In this case, we adopt the
factual findings of the LA, as
affirmed by the NLRC and the
CA, as the same was duly
supported
by
substantial
evidence.
Under Part II, Section C,
Nos. 1 and 6 of the POEA
"Standard Employment Contract
Governing the Employment of
All Filipino Seamen on Board
Ocean-Going Vessels," it is
provided that:
1. In case of death of the
seaman during the term of this
2011- 2012 Labor Law Case Digest Compilations
UC LLB III-B
Page 127

Contract, the employer shall pay


his beneficiaries the Philippine
Currency equivalent to the
amount of U.S.$50,000.00 and
an additional amount of U.S.
$7,000.00 to each child under
the age of twenty-one (21) but
not exceeding four children at
the exchange rate prevailing
during the time of payment. xxx
6. No compensation shall
be payable in respect of any
injury, incapacity, disability or
death resulting from a willful act
on his own life by the seaman,
provided, however, that the
employer can prove that such
injury, incapacity, disability or
death is directly attributable to
him.
In order to avail of death
benefits, the death of the
employee should occur during
the
effectivity
of
the
employment contract. The death
of a seaman during the term of
employment
makes
the
employer liable to his heirs for
death compensation benefits.
Once it is established that the
seaman
died
during
the
effectivity of his employment
contract, the employer is liable.
This rule, however, is not
absolute. The employer may be
exempt from liability if he can
successfully prove that the
seaman's death was caused by
an injury directly attributable to
his deliberate or willful act.
Clearly,
respondent's
entitlement to any death benefit
depends on whether petitioner's
evidence suffices to prove that
Danilo committed suicide, and

the burden of proof rests on


petitioner.
The LA, the NLRC and the
CA found that Danilo died of
Asphyxia by strangulation as
proved by the NBI post-mortem
findings and certification issued
by the medico-legal officer, Dr.
Reyes. These three tribunals did
not consider the photocopy of
the
fax
transmission
of
the purported
English
translation of Dr. Hameed's
medical report to prove that
Danilo committed suicide, since
the
medical
report's
genuineness and due execution
were unverifiable.
69. GOVERNMENT SERVICE
INSURANCE SYSTEM (GSIS),
ET.AL PETITIONERS,
VS. COMMISSION ON AUDIT
(COA), ET AL RESPONDENTS.
G.R. No. 162372
October 19, 2011
Topic:
Validity
Of
GSIS
Retirement Plans
Facts:
GSIS Act was enacted on
May 30, 1997 expanding and
increasing the coverage and
benefits of the GSIS. Pursuant to
this, Board Resolution No. 326
was passed wherein it adopted
the GSIS Employees Loyalty
Incentive Plan (ELIP). On Review,
ELP
was
renamed
GSIS
Retirement Financial Plan (FRP).
Upon assumption of the new
GSIS President and General
Manager, Garcia, Dimagiba, the
Auditor requested Garcia to
review the RFP again. Garcia,
2011- 2012 Labor Law Case Digest Compilations
UC LLB III-B
Page 128

however denied the request.


Dimagiba sought the assistance
of COA wherein it was opined
that the RFP is prohibited under
RA
4968
and
it
was
subsequently pronounced that
such was null and void.
The COA decision was
forwarded to Garcia but he
claimed that such disallowance
is highly irregular as COA had no
authority. Garcia then asked
Dimagiba to withdraw the notice
of disallowance in the interest of
industrial peace of the GSIS.
Dimagiba did not heed to
Garcias request. Petitioner now
comes before the court arguing
that their disallowance of their
retirement
plan
would
be
tantamount to a violation of the
constitutional right to equal
protection of laws.
Issue:
Whether the GSIS RFP is
null and void
Ruling:
It is null and void.
It is true that under
Section 41(n) of Republic Act No.
8291, GSIS is expressly granted
the power to adopt a retirement
plan and/or financial assistance
for its employees, but a closer
look at the provision readily
shows that this power is not
absolute. It is qualified by the
words early, incentive, and
for the purpose of retirement.
The retirement plan must be an
early retirement incentive plan
and such early retirement
incentive plan or financial

assistance must be for the


purpose of retirement.
It is clear from the
foregoing that Section 41(n) of
Republic
Act
No.
8291
contemplates
a
situation
wherein
GSIS,
due
to
a
reorganization, a streamlining of
its organization, or some other
circumstance, which calls for the
termination of some of its
employees, must design a plan
to
encourage,
induce,
or
motivate these employees, who
are not yet qualified for either
optional
or
compulsory
retirement under our laws, to
instead voluntarily retire. This is
the very reason why under the
law, the retirement plan to be
adopted is in reality an incentive
scheme
to
encourage
the
employees to retire before their
retirement age.
The above interpretation applies
equally to the phrase financial
assistance, which, contrary to
the
petitioners
assertion,
should
not
be
read
independently of the purpose of
an early retirement incentive
plan.
Under the doctrine of
noscitur
a
sociis,
the
construction of a particular word
or phrase, which is in itself
ambiguous,
or
is
equally
susceptible of various meanings,
may be made clear and specific
by considering the company of
words in which it is found or
with which it is associated. In
other words, the obscurity or
doubt of the word or phrase
may be reviewed by reference
to associated words. Thus, the
2011- 2012 Labor Law Case Digest Compilations
UC LLB III-B
Page 129

phrase financial assistance, in


light of the preceding words with
which it is associated, should
also be construed as an
incentive scheme to induce
employees to retire early or as
an assistance plan to be given
to employees retiring earlier
than their retirement age.
Such is not the case with
the GSIS RFP. Its very objective,
[t]o motivate and reward
employees
for
meritorious,
faithful,
and
satisfactory
service, contradicts the nature
of an early retirement incentive
plan, or a financial assistance
plan,
which
involves
a
substantial amount that is given
to motivate employees to retire
early. Instead, it falls exactly
within
the
purpose
of
a
retirement benefit, which is a
form
of
reward
for
an
employees loyalty and lengthy
service, in order to help him or
her enjoy the remaining years of
his life.
Furthermore, to be able to
apply for the GSIS RFP, one
must be qualified to retire under
Republic Act No. 660 or Republic
Act No. 8291, or must have
previously retired under our
existing retirement laws. This
only means that the employees
covered by the GSIS RFP were
those who were already eligible
to retire or had already retired.
Certainly, this is not included in
the
scope
of
an
early
retirement incentive plan or
financial assistance for the
purpose of retirement.

The
fact
that
GSIS
changed
the
name
from
Employees Loyalty Incentive
Plan to Retirement/Financial
Plan does not change its
essential nature. A perusal of
the plan shows that its purpose
is not to encourage GSISs
employees to retire before their
retirement age, but to augment
the retirement benefits they
would receive under our present
laws. Without a doubt, the GSIS
RFP
is
a
supplementary
retirement
plan,
which
is
prohibited
by
the
Teves
Retirement Law.
70. MAGDALA
MULTIPURPOSE &
LIVELIHOOD COOPERATIVE
and SANLOR MOTORS CORP.,
Petitioners, vs. KILUSANG
MANGGAGAWA NG LGS,
MAGDALA MULTIPURPOSE &
LIVELIHOOD CORPERATIVE
(KMLMS) and UNION
MEMBERS/ STRIKERS
G.R. Nos. 191138-39,
October 19, 2011
Topic: Illegal Strikes
Facts:
KMLMS
is
a
union
operating
in
Magdala
Multipurpose
and
Livelihood
Cooperative and Sanlor Motors
Corporation. KMLMS filed a
notice of strike on March 5,
2002 and conducted its strike on
April 8, 2002. The KMLMS only
acquired legal personality when
its registration was granted on
April 19, 2002 by the DOLE. On
2011- 2012 Labor Law Case Digest Compilations
UC LLB III-B
Page 130

the same day, it became


officially affiliated as a local
chapter of the Pambansang
Kaisahan ng Manggagawang
Pilipino.
On May 6, 2002, KMLMS
staged a strike where several
prohibited and illegal acts were
committed by its participating
members.
Petitioners
filed
their
petition to declare the strike
illegal further praying that
officers and members of KMLMS
who participated in the illegal
strike be declared to have lost
or forfeited their employment
status.
The Labor Arbiter declared
the strike illegal and forfeited
the employment status of erring
members. The NLRC and the CA
likewise
affirmed
the
LAs
decision.
Issue:
Whether the strike was
illegal.
Ruling:
There is no question that
the May 6, 2002 strike was
illegal, first, because when
KMLMS filed the notice of strike
on March 5 or 14, 2002, it had
not
yet
acquired
legal
personality and, thus, could not
legally represent the eventual
union and its members. And
second, similarly when KMLMS
conducted the strike-vote on
April 8, 2002, there was still no
union to speak of, since KMLMS
only acquired legal personality
as an independent LLO only on

April 9, 2002 or the day after it


conducted the strike-vote. These
factual findings are undisputed
and borne out by the records.

2. Whether the affirmation


of the Court is in violation of the
security of tenure of the other
employees.

71. BANK OF THE PHILIPPINE


ISLANDS, PETITIONER, VS.
BPI EMPLOYEES UNIONDAVAO CHAPTERFEDERATION OF UNIONS IN
BPI UNIBANK, RESPONDENT.
G.R. No. 164301, October 19,
2011
Topic:
Absorption
Of
Employees
Of
Former
Company In Case Of Merger;
Validity
Of
Union
Shop
Clause In Case Of Merger;
Security Of Tenure In Case
Of Merger

Ruling:
Merger leads to absorption
of the former company to the
new one. Absorption of the
former companys employees
does not violate Security of
Tenure of Other Employees.
First and Second Issue. By
upholding
the
automatic
assumption of the non-surviving
corporations
existing
employment contracts by the
surviving
corporation
in
a
merger, the Court strengthens
judicial protection of the right to
security of tenure of employees
affected by a merger and avoids
confusion regarding the status
of their various benefits which
were among the chief objections
of our dissenting colleagues.
However,
nothing
in
this
Resolution shall impair the right
of an employer to terminate the
employment of the absorbed
employees for a lawful or
authorized cause or the right of
such an employee to resign,
retire or otherwise sever his
employment, whether before or
after the merger, subject to
existing contractual obligations.
In this manner, Justice Brions
theory of automatic assumption
may be reconciled with the
majoritys concerns with the
successor
employers
prerogative
to
choose
its
employees and the prohibition
against involuntary servitude.

Facts:
On August 10, 2010, the
SC issued a decision holding
that the former employees of
the Far East Bank and Trust
Company (FEBTC) is absorbed
by BPI pursuant to the two
banks merger in 2000 were
covered by the Union Shop
Clause in the then existing CBA
of BPI with BPI Employees Union
Davao Chapter Federation of
Unions in BPI Unibank. The
Petitioners however moved for a
motion
for
reconsideration,
hence, this petition.
Issue:
1. Whether the merger of
the
two
companies
automatically
led
to
the
absorption of the employees of
the former company to the new
one.
2011- 2012 Labor Law Case Digest Compilations
UC LLB III-B
Page 131

Again, it is worthwhile to
highlight
that
a
contrary
interpretation of the Union Shop
Clause would dilute its efficacy
and put the certified union that
is supposedly being protected
thereby at the mercy of
management. For if the former
FEBTC employees had no say in
the merger of its former
employer with another bank, as
petitioner BPI repeatedly decries
on their behalf, the Union
likewise could not prevent BPI
from
proceeding
with
the
merger
which
undisputedly
affected
the
number
of
employees in the bargaining
unit that the Union represents
and may negatively impact on
the Unions majority status. In
this instance, we should be
guided by the principle that
courts must place a practical
and realistic construction upon a
CBA, giving due consideration to
the context in which it is
negotiated and purpose which it
is intended to serve.
In Rance v. National Labor
Relations Commission, we held
that: It is the policy of the state
to assure the right of workers to
"security of tenure" (Article XIII,
Sec. 3 of the New Constitution,
Section 9, Article II of the 1973
Constitution). The guarantee is
an act of social justice. When a
person has no property, his job
may possibly be his only
possession
or
means
of
livelihood. Therefore, he should
be
protected
against
any
arbitrary deprivation of his job.
Article 280 of the Labor Code
2011- 2012 Labor Law Case Digest Compilations
UC LLB III-B
Page 132

has construed security of tenure


as meaning that "the employer
shall not terminate the services
of an employee except for a just
cause or when authorized by"
the Code.
We have also previously
held that the fundamental
guarantee of security of tenure
and due process dictates that no
worker
shall
be
dismissed
except for a just and authorized
cause provided by law and after
due process is observed. Even
as we now recognize the right to
continuous,
unbroken
employment of workers who are
absorbed into a new company
pursuant to a merger, it is but
logical that their employment
may be terminated for any
causes provided for under the
law or in jurisprudence without
violating their right to security
of tenure. As Justice Carpio
discussed in his dissenting
opinion, it is well-settled that
termination of employment by
virtue of a union security clause
embodied in a CBA is recognized
in our jurisdiction. In Del Monte
Philippines, Inc. v. Saldivar, we
explained the rationale for this
policy in this wise:
Article 279 of the Labor
Code ordains that "in cases of
regular
employment,
the
employer shall not terminate
the services of an employee
except for a just cause or when
authorized by [Title I, Book Six
of the Labor Code]." Admittedly,
the enforcement of a closedshop or union security provision
in the CBA as a ground for

termination finds no extension


within any of the provisions
under Title I, Book Six of the
Labor Code. Yet jurisprudence
has consistently recognized,
thus: "It is State policy to
promote unionism to enable
workers
to
negotiate
with
management
on
an
even
playing field and with more
persuasiveness than if they
were
to
individually
and
separately bargain with the
employer. For this reason, the
law has allowed stipulations for
'union shop' and 'closed shop'
as
means
of
encouraging
workers to join and support the
union of their choice in the
protection of their rights and
interests vis-a-vis the employer."
72. AGG TRUCKING AND/OR
ALEX ANG GAEID,
PETITIONERS, VS. MELANIO
B. YUAG, RESPONDENT.
G.R. No. 195033, October 12,
2011
Topic: Regular Employees
Facts:
Petitioner
employed
respondent as a driver since
February 2002 in his trucking
business. Respondent received
his salary on commission basis
at 9% of his gross delivery per
trip. After sometime, petitioner
noticed that respondent started
incurring substantial shortages
in his deliveries. There were also
reports that respondent illegally
sold bags of sugar along the
way at a lower price.
2011- 2012 Labor Law Case Digest Compilations
UC LLB III-B
Page 133

Alarmed at the delivery


shortages, petitioner took it
upon himself to monitor all his
drivers, including respondent,
by instructing them to report to
him their location from time to
time
through
their
mobile
phones. He also required them
to make their delivery trips in
convoy, in order to avoid illegal
sale of cargo along the way. The
respondent, in one instance, did
not join the convoy and in his
delivery, there was an enormous
shortage, as alleged by one of
the
petitioners
clients.
Respondent was told by the
petitioner to take a rest from
work, so respondent
went
directly to the DOLE.
Issue:
Whether respondent was a
regular employee hence, there
was illegal dismissal.
Ruling:
The Respondent is a
regular employee but there is no
finding of illegal dismissal.
To us, private respondent's
"advice to take a rest" theory is
nothing but a mere ploy to
reinforce his hypothesis that the
petitioner is not a regular
employee. What makes this
worse is that the NLRC bought
private respondent's aforesaid
theory hook, line and sinker and
ruled that the petitioner was
neither dismissed from work, he
(the
petitioner)
being
considered merely on "leave of
absence without pay", nor is he
(the petitioner) entitled to

separation pay on the ground


that he was paid on purely
"commission" or "pakyaw" basis
which is in legal parlance, in
effect,
implies
that
the
petitioner is not a regular
employee
of
the
private
respondent,
but
a
mere
seasonal worker or independent
contractor.
It is most disturbing to see
how the CA regarded labor
terms "paid on commission,"
"pakyaw" and "seasonal worker"
as one and the same. In labor
law, they are different and have
distinct meanings, which we do
not need to elaborate on in this
Petition as they are not the
issue here. It should also be
remembered that a regular
status of employment is not
based on how the salary is paid
to an employee. An employee
may
be
paid
purely
on
commission
and
still
be
considered a regular employee.
Moreover, a seasonal employee
may also be considered a
regular employee.
Further, the appreciation
by the CA of the NLRC
Resolution was erroneous. The
fact is that the refusal by the
NLRC to grant separation pay
was merely consistent with its
ruling that there was no
dismissal. Since respondent was
not
dismissed,
much
less
illegally dismissed, separation
pay was unnecessary. The CA
looked at the issue differently
and erroneously, as it held that
the NLRC refused to grant the
award
of
separation
pay
2011- 2012 Labor Law Case Digest Compilations
UC LLB III-B
Page 134

because respondent had not


been found to be a regular
employee. The NLRC had in fact
made no such ruling. These are
flagrant
errors
that
are
reversible by this Court. They
should be corrected for the sake
not only of the litigants, but also
of the CA, so that it would
become more circumspect in its
appreciation of the records
before it.
73. KAISAHAN AT KAPATIRAN
NG MGA MANGGAGAWA AT
KAWANI SA MWC-EAST ZONE
UNION AND EDUARDO
BORELA, ETC. VS. MANILA
WATER COMPANY, INC.
G.R. No. 174179. November
16, 2011
Topic: Award Of Attorneys
Fees
Facts:
Manila Water Company,
Inc (MWCI) and Metropolitan
Waterworks
and
Sewerage
System (MWSS) entered into a
Concession Agreement where
the benefits of the employees of
the MWSS shall also be granted
to the employees of MWCI.
The
amelioration
allowance (AA) was paid by
integrating it to the monthly
payroll
of
the
employees.
However, it was not paid which
prompted
KAISAHAN
AT
KAPATIRAN
NG
MGA
MANGGAGAWA AT KAWANI SA
MWC-EAST ZONE UNION and
EDUARDO BORELA,the union, to
file a complaint against the
company for payment of AA and

COLA plus attorneys fee. The


LA ruled in favour of the Union
and ordered the Company to
pay AA and COLA plus 10 %
attorneys fee.
The Company appealed to
NLRC
which
affirmed
the
decision of LA with modification.
The NLRC ordered the payment
of AA and not the COLA and
affirmed the 10 % attorneys
fee.
Consequently,
a
Memorandum of Agreement was
agreed upon by the Company
and the Union that the 10 %
attorneys fee is separated and
distinct from the award of 10 %
attorneys feed by the NLRC.
The Company filed its motion for
reconsideration but was denied,
prompting the latter to elevate
the case to the CA which the CA
modified the NLRC decision by
deleting the order of payment of
the 10 % attorneys fee for it is
unconscionable.
The
Union
moved for reconsideration but
was denied. Hence, the appeal.
Issues:
Whether the award of 10
% attorneys fee is proper in the
case.
Ruling:
Yes. The 10 % attorneys
fee is proper in the case.
There
are
two
commonly
accepted concepts of attorneys
fees

the
ordinary
and
extraordinary. In its ordinary
concept, an attorneys fee is the
reasonable compensation paid
to a lawyer by his client for the
legal
services
the
former
2011- 2012 Labor Law Case Digest Compilations
UC LLB III-B
Page 135

renders; compensation is paid


for the cost and/or results of
legal services per agreement or
as may be assessed. In its
extraordinary
concept,
attorneys fees are deemed
indemnity for damages ordered
by the court to be paid by the
losing party to the winning
party. This is payable not to the
lawyer but to the client, unless
the client and his lawyer have
agreed that the award shall
accrue
to
the
lawyer
as
additional
or
part
of
his
compensation. Article 111 of
the Labor Code, as amended,
contemplates the extraordinary
concept of attorneys fees.
Although an express finding of
facts and law is still necessary
to prove the merit of the award,
there need not be any showing
that
the
employer
acted
maliciously or in bad faith when
it withheld the wages. Thus the
SC concluded that the CA erred
in ruling that a finding of the
employers malice or bad faith
in withholding wages must
precede an award of attorneys
fees under Article 111 of the
Labor Code. To reiterate, a plain
showing that the lawful wages
were
not
paid
without
justification is sufficient.
74. FIL-STAR MARITIME
CORPORATION, CAPTAIN
VICTORIO S. MIGALLOS AND
GRANDSLAM ENTERPRISE
CORPORATION,
PETITIONERS, VS. HANZIEL
O. ROSETE, RESPONDENT

GR No. 192686, November


23, 2011
Topic: Basis For Disability
Claims By OFWs
Facts:
In 2005, petitioner Fil-Star
Maritime Corporation (Fil-Star),
the local manning agency of copetitioner Grandslam Enterprise
Corporation (Grandslam), hired
Hanziel Rosete, respondent, as
third officer on board the oceangoing vessel M/V Ansac Asia.
He was in charge of the loading
and unloading operations of the
vessels
cargo
primarily
consisting of soda ash in bulk.
Respondent stated that the
nature of his work exposed him
to minute particles of soda ash
during
the
loading
and
unloading
operations.
On
November 22, 2005, respondent
finished
his
contract
and
returned to the Philippines.
Sometime in 2006, a little
over
a
month
from
his
embarkation,
respondent
experienced an abrupt blurring
of his left eye. He was checked
up and diagnosed with Central
Retinal Vein Occlusion. He
undergone several laser surgery.
Thereafter, he was declared fit
to work.
When he was repatriated
to the Philippines, he also
underwent
series
of
laser
surgery.
Consequently,
Dr.
Antonio Say declared his left eye
to be legally blind. The medical
certificate read; a) left eye is
legally blind; and b) partial
permanent disability.
2011- 2012 Labor Law Case Digest Compilations
UC LLB III-B
Page 136

Respondent went after for


a claim for permanent total
disability. He argues that under
their CBA he is entitled to
US$105,000.00 for being unable
to perform his job for more than
120 day from his repatration.
The company denied his
claim averring that the disability
is only Grade 7, prompting
respondent to file a complaint
against the company with the
Labor Arbiter which favors the
former
and
ordered
the
company to pay the claim.
The company applealed to
NLRC
which
modified
the
resolution by reducing the claim
to US$20,900 which is the
proper amount equivalent not
Grade 7 under the POEA
Standard Employment Contract.
Respondent, not satisfied,
elevated to the CA which
reversed the NLRC decision and
reinstated the LA decision. The
company
moved
for
reconsideration but was denied.
Hence, the appeal.
Issue:
Whether Rosete is entitled
to the disability benefit based
on the CBA or based on POEA
Standard Employment Contract.
Ruling:
A total disability does not
require that the employee be
completely disabled, or totally
paralyzed. What is necessary is
that the injury must be such
that the employee cannot
pursue his or her usual work and
earn from it. On the other hand,

a total disability is considered


permanent
if
it
lasts
continuously for more than 120
days. What is crucial is whether
the employee who suffers from
disability could still perform his
work
notwithstanding
the
disability he incurred. Evidently,
respondent was not able to
return to his job as a seafarer
after his left eye was declared
legally blind. Records showed
that the petitioners did not give
him a new overseas assignment
after his disability. This only
proved
that
his
disability
effectively barred his chances to
be deployed abroad as an officer
of an ocean-going vessel.
Hence, the Supreme Court
found it fitting that respondent
be entitled to permanent total
disability benefits considering
that he would not be able to
resume his position as a
maritime
officer,
and
the
probability that he would be
hired
by
other
maritime
employers would be close to
impossible.
Respondent
is
entitled
to
disability benefit based on the
schedule of disability under
Section 32 of the 2000 POEA
AMENDED STANDARD TERMS
AND CONDITIONS. Permanent
total disability is classified as
Grade 1. Thus, Respondents
disability benefit should be
US$60,000.
75. PHILIPPINE NATIONAL
BANK VS. DAN PADAO,
G.R. Nos. 180849 and
187143. November 16, 2011.
2011- 2012 Labor Law Case Digest Compilations
UC LLB III-B
Page 137

Topic: Separation Pay; No


Separation Pay In Case Of
Valid Dismissal
Facts:
On August 21, 1981, Dan
Padao was hired by Philippine
National Bank (PNB) as a clerk
and was later promoted to the
position of Loan and Credit
Officer IV.
Sometime in 1994, PNB
became embroiled in a scandal
involving behest loans which
some of its officers were
charged and convicted of grave
misconduct, gross neglect of
duty and gross violation of bank
rules
and
regulations
and
criminal cases.
On June 1996, Padau was
also involved in similar charges,
administratively and criminal.
PNB found Padao guilty
which led to his dismissal. Padao
filed a complaint with Executive
Labor Arbiter (ELA) against PNB
for Reinstatement, Backwages,
Illegal
Dismissal,
Treachery/Badfaith and Palpable
Discrimination.
ELA
found
Padaos
dismissal valid, but awarded his
separation pay.
Padao appealed to NLRC
which reversed ELAs decision
an ordered PNB for Padaos
reistatement. PNB moved for
reconsideration
which
was
denied.
PNB appealed to CA but
was also denied. Hence, the
appeal.
Issue:

WhetherPadao is entitled
to separation pay even if he was
validly dismissed.
Ruling:
No. Padao is not entitled
to separation pay.
Padao is not entitled to financial
assistance. The rule regarding
separation pay as a measure of
social justice is that it shall be
paid only in those instances
where the employee is validly
dismissed for causes other than
serious
misconduct,
willful
disobedience,
gross
and
habitual neglect of duty, fraud
or willful breach of trust,
commission of a crime against
the employer or his family, or
those reflecting on his moral
character. In this case, Padao
was guilty of gross and habitual
neglect of duty.
His dismissal was for gross
and habitual neglect of duties.
His repeated failure to discharge
his
duties
as
a
credit
investigator
of
the
bank
amounted to gross and habitual
neglect of duties under ART.
282(b) of the Labor Code. He
not only failed to perform what
he was employed to do, but also
did so repetitively and habitually
causing millions of pesos in
damage to PNB. Thus, PNB
acted within the bounds of the
law by meting out the penalty of
dismissal. Therefore, Padao is
not entitled to separation pay.
76. Gilbert Quizora vs.
Denholm Crew Management
(Philippines), Inc.
2011- 2012 Labor Law Case Digest Compilations
UC LLB III-B
Page 138

G.R. No. 185412. November


16, 2011.
Topic: Claims For Disability
Benefits By Seamen
Facts:
Gilbert Quizora was hired
by Denholm Crew Management
as messman for 8 years. After a
month of the expiration of his
last
employment
with
MV
Leopard he was diagnosed to be
suffering from varicose veins.
He
was
disqualified
for
employment and declared unfit
for sea duty.
Subsequently,
he
demanded from the Company
the payment of disability benefit
but the latter denied his claim.
He filed with the Labor Arbiter a
complaint
for
payment
of
disability
benefits,
medical
expenses,
separation
pay,
damages, and attorneys fees.
The LA dismissed the complaint
for lack of merit.
On appeal, the NLRC
reversed LAs decision and
award Gilbert the amount of
US$60,000.00
as
disability
compensation. The Company
filed its motion which was
denied, thus, elevated to the CA
which
reversed
the
NLRC
decision and reinstated the LAs
decision.
Motion
for
reconsideration
was
denied.
Hence, the appeal.
Issue:
Whether Gilbert is entitled
to the disability benefit.
Ruling:

No. Gilbert is not entitled


to the said benefit.
Granting
that
the
provisions of the 2000 POEASEC
apply,
the
disputable
presumption provision in Section
20 (B) does not allow petitioner
to just sit down and wait for
respondent company to present
evidence to overcome the
disputable presumption of workrelatedness of the illness.
Contrary to his position, the
seafarer still has to substantiate
his claim in order to be entitled
to disability compensation. He
has to prove that the illness he
suffered was work-related and
that it must have existed during
the term of his employment
contract. For disability to be
compensable under Section 20
(B) of the 2000 POEA-SEC, two
elements must concur: (1) the
injury or illness must be workrelated; and (2) the work-related
injury or illness must have
existed during the term of the
seafarers
employment
contract. In other words, to be
entitled to compensation and
benefits under this provision, it
is not sufficient to establish that
the seafarers illness or injury
has rendered him permanently
or partially disabled; it must also
be shown that there is a causal
connection
between
the
seafarers illness or injury and
the work for which he had been
contracted. Unfortunately for
petitioner, he failed to prove
that his varicose veins arose out
of
his
employment
with
respondent company.
2011- 2012 Labor Law Case Digest Compilations
UC LLB III-B
Page 139

77. Government Service


Insurance System vs. Manuel
P. Besitan
G.R. No. 178901. November
23, 2011.
Topic: Claims Sickness
Compensation Benefit By
Local Employees Under PD
No. 626
Facts:
On January 21, 1976,
Manuel Besitan was employed
as
Bank
Examiner
and
eventually promoted as Bank
Officer III by the Bangko Sentral
ng Pilipinas.
On October 2005, after
almost 30 years in service,
Besitan was diagnosed with End
State Renal Disease and had to
undergone kidney transplant
where he incurred medical
expenses
amounting
to
P817,455.50.
Believing that his illness
was work related, he filed a
claim for compensation benefits
under Presidential Decree No.
626
with
the
Government
Service
Insurance
System
(GSIS). The latter denied the
claim.
Manuel
elevated
to
Employment
Compensation
Commission (ECC)where the
same was also denied.
On appeal to the CA, the
decision of ECC is reversed and
set aside. GSIS moved for
reconsideration but was denied.
Hence, the appeal.
Issue:

Whether Manuel Besitan is


entitled to compensation benefit
under PD No. 626.
Ruling:
Yes. For a sickness or
resulting disability or death to
be compensable, the claimant
must prove either (1) that the
employees sickness was the
result
of
an
occupational
disease listed under Annex A
of the Amended Rules on
Employees Compensation, or
(2) that the risk of contracting
the disease was increased by his
working conditions. Under the
increased risk theory, there
must be a reasonable proof that
the
employees
working
condition increased his risk of
contracting the disease, or that
there is a connection between
his work and the cause of the
disease. In this case, since
Besitans ailment, End Stage
Renal Disease secondary to
Chronic Glomerulonephritis is
not among those listed under
Annex A, of the Amended
Rules
on
Employees
Compensation, he needs to
show by substantial evidence
that his risk of contracting the
disease was increased by his
working condition.
Direct and clear evidence,
is not necessary to prove a
compensable claim. Strict rules
of evidence do not apply as PD
No.
626
only
requires
substantial evidence. The SC
found
that
Besitan
has
sufficiently proved that his
working condition increased his
2011- 2012 Labor Law Case Digest Compilations
UC LLB III-B
Page 140

risk
of
contracting
Glomerulonephritis,
which
according to GSIS may be
caused by bacterial, viral, and
parasitic
infection.
When
Besitan entered the government
service in 1976, he was given a
clean bill of health. In 2005, he
was diagnosed with End Stage
Renal Disease secondary to
Chronic Glomerulonephritis. It
would appear therefore that the
nature of his work could have
increased his risk of contracting
the
disease.
His
frequent
travels to remote areas in the
country could have exposed him
to certain bacterial, viral, and
parasitic infection, which in turn
could have caused his disease.
Delaying his urination during his
long trips to the provinces could
have also increased his risk of
contracting the disease. As a
matter of fact, even the Bank
Physician of Bangko Sentral ng
Pilipinas, Dr. Gregorio Suarez II,
agreed that Besitans working
condition
could
have
contributed to the weakening of
his kidneys, which could have
caused
the
disease.
This
Medical Certificate is sufficient
to prove that the working
condition of Besitan increased
his
risk
of
contracting
Glomerulonephritis. In claims
for compensation benefits, a
doctors certification as to the
nature of a claimants disability
deserves full credence because
no medical practitioner would
issue
certifications
indiscriminately.

78. CESAR C. LIRIO, DOING


BUSINESS UNDER THE NAME
AND STYLE OF CELKOR AD
SONIMIX VS. WILMER D.
GENOVIA,
G.R. No. 169757. November
23, 2011.
Topic:
Employer-Employee
Relationship
Facts:
On
August
15,2001,
Wilmer D Genovia was hired as
studio manager of Celkor Ad
Sonimix Recording Studio owned
by Cesar Lirio. He was employed
to manage and operate Celkor
and to promote and sell the
recording studio's services to
music enthusiasts and other
prospective clients. He received
a monthly salary ofP7,000.00.
They also agreed that he was
entitled
to
an
additional
commission of P100.00 per hour
as
recording
technician
whenever a client uses the
studio for recording, editing or
any related work. He was made
to report for work from Monday
to Friday from 9:00 a.m. to 6
p.m. On Saturdays, he was
required to work half-day only,
but most of the time, he still
rendered eight hours of work or
more. All the employees of
petitioner, including respondent,
rendered overtime work almost
everyday, but petitioner never
kept a daily time record to avoid
paying the employees overtime
pay.
A few days after he
started working as a studio
manager, petitioner approached
2011- 2012 Labor Law Case Digest Compilations
UC LLB III-B
Page 141

him and told him about his


project to produce an album for
his 15-year-old daughter, Celine
Mei Lirio, a former talent of ABSCBN Star Records. Petitioner
asked respondent to compose
and arrange songs for Celine
and promised that he (Lirio)
would draft a contract to assure
respondent of his compensation
for such services. As agreed
upon, the additional services
that respondent would render
included
composing
and
arranging musical scores only,
while the technical aspect in
producing the album, such as
digital editing, mixing and sound
engineering would be performed
by respondent in his capacity as
studio manager for which he
was paid on a monthly basis.
Wilmer finished what was
agreed upon. An album was
released and generated profit.
When he reminded Cesar about
their agreement, the latter
disacknowledged
the
claim.
Wilmer insisted that he be
properly compensated for the
services rendered. Thereafter,
he was verbally terminated
which prompted him to file a
complaint with the Labor Arbiter
for illegal dismissal and prayed
for reinstatement, be paid
separation pay, backwages and
that he be awarded unpaid
commission in the amount
of P2,000.00
for
services
rendered as a studio technician
as well as moral and exemplary
damages.
On October 31, 2003, the Labor
Arbiter ruled in favor of Wilmer.

There
was
an
employeremployee relationship and that
he was illegally dismissed.
On appeal, the NLRC
reversed and set aside the
decision of the Labor Arbiter. Is
ruled that Wilmer failed to prove
his
employment
tale
with
substantial evidence. Wilmer
moved for reconsideration but
was denied. He then elevated to
the CA which reversed and set
aside NLRCs decision and
reinstated that of the Labor
Arbiters. Cesar moved for
reconsideration but was denied.
Hence, the petition.
Issue:
Whether there exists
employer-employee relationship.
Ruling:
The
elements
to
determine the existence of an
employment relationship are: (a)
the selection and engagement
of the employee; (b) the
payment of wages; (c) the
power of dismissal; and (d) the
employers power to control the
employees conduct. In this
case, the documentary evidence
presented by respondent to
prove that he was an employee
of petitioner are as follows: (a)
a document denominated as
payroll (dated July 31, 2001 to
March 15, 2002) certified correct
by petitioner, which showed that
respondent received a monthly
salary of P7,000.00 with the
corresponding deductions due to
absences
incurred
by
respondent; and (2) copies of
2011- 2012 Labor Law Case Digest Compilations
UC LLB III-B
Page 142

petty cash vouchers, showing


the amounts he received and
signed for in the payrolls. These
documents
showed
that
petitioner hired respondent as
an employee and he was paid
monthly wages of P7,000.00.
Additionally, as to the existence
of the power of control, it is not
essential for the employer to
actually
supervise
the
performance of duties of the
employee. It is sufficient that
the former has a right to wield
the power.
In this case,
petitioner even stated in his
Position Paper that it was agreed
that he would help and teach
respondent how to use the
studio equipment. In such case,
petitioner certainly had the
power to check on the progress
and work of respondent.
80. TAMSON'S ENTERPRISES,
INC., NELSON LEE, LILIBETH
ONG and JOHNSON NG,
Petitioners, vs.COURT OF
PPEALS and ROSEMARIE L.
SY, Respondents.
G.R. No. 192881,
November 16, 2011
Topic: Illegal Dismissal
Facts:
This case stemmed from a
complaint for illegal dismissal
with money claims filed by
respondent
Rosemarie
L.
SyTamsons Enterprises, Inc.
Rosemarie L. Sy was hired by
Tamsons enterprise as an
assistant to the president,
although she actually worked as
a payroll clerk. She is a

probationary employee. Four


days before her sixth month of
work, her employment was
terminated. The company said
that she did not qualify as a
regular
employee,
however
there were no evaluation or
appraisal reports regarding her
alleged inefficient performance.
The
Executive
Labor
Arbiter (ELA) ruled in favor of
Sy, NLRC reversed the decision
of ELA, CA reinstated ELAs
decision.
Issue:
Whether the termination
of Sy, a probationary employee,
was valid.
Ruling:
No. As a probationary
employee,
her
employment
status was only temporary.
Although a probationary or
temporary employee with a
limited tenure, she was still
entitled to a security of tenure.
It is settled that even if
probationary employees do not
enjoy permanent status, they
are accorded the constitutional
protection of security of tenure.
This means they may only be
terminated for a just cause or
when they otherwise fail to
qualify as regular employees in
accordance
with
reasonable
standards made known to them
by the employer at the time of
their engagement.
In this case, the justification
given by the petitioners for Sys
dismissal was her alleged failure
to qualify by the companys
2011- 2012 Labor Law Case Digest Compilations
UC LLB III-B
Page 143

standard.
Other
than
the
general allegation that said
standards were made known to
her at the time of her
employment,
however,
no
evidence,
documentary
or
otherwise, was presented to
substantiate the same. Neither
was there any performance
evaluation presented to prove
that
indeed
hers
was
unsatisfactory.
Thus,
the
dismissal was not valid.One of
the
conditions
before
an
employer can terminate a
probationary
employee
is
dissatisfaction on the part of the
employer which must be real
and in good faith, not feigned so
as to circumvent the contract or
the law. In the case at bar,
absent any proof showing that
the
work
performance
of
petitioner was unsatisfactory,
We
cannot
conclude
that
petitioner failed to meet the
standards of performance set by
private
respondents.
This
absence of proof, in fact, leads
us
to
infer
that
their
dissatisfaction with her work
performance was contrived so
as
not
to
regularize
her
employment.
81. PHILIPPINE NATIONAL
BANK, v DAN PADAO
G.R. Nos. 180849 and
GR187143
November 16, 2011
Topic:
Valid
Dismissal;
Employees Monetary Claims
In Case Of Valid Dismissal
Facts:

Padaois a Loan and Credit


Officer IV of PNB. Sometime in
1994, PNB became embroiled in
a scandal involving behest
loans. The expos triggered
the
conduct
of
separate
investigations by the COA and
PNBs Internal Audit Department
(IAD).
Both
investigations
confirmed that the collateral
provided in numerous loan
accommodations were grossly
over-appraised.
The
credit
standing of the loan applicants
was also fabricated, allowing
them to obtain larger loan
portfolios from PNB. These
borrowers eventually defaulted
on the payment of their loans,
causing PNB to suffer millions in
losses.
Padao was accused of
having
over-appraised
the
collateral of the certain clients.
After due investigation, PNB
found Padao guilty of gross and
habitual neglect of duty and
ordered him dismissed from the
bank. Padao appealed to the
banks Board of Directors.
Due
to
the
boards
inaction, Padao filed a case of
illegal dismissal with ELA. the
ELA found Padaos dismissal
valid. Despite the finding of
legality, the ELA still awarded
separation pay of one-half (1/2)
months pay for every year of
service.Padao appealed to the
NLRC, He was thereby ordered
reinstated
to
his
previous
position without loss of seniority
rights and PNB was ordered to
pay him full backwages and
attorneys fees equivalent to ten
2011- 2012 Labor Law Case Digest Compilations
UC LLB III-B
Page 144

percent (10%) of the total


monetary award.
During the pendency of
G.R. No. 180849 before the
Court, the NLRC certified that
there decision become final and
executory.
NLRC
granted
Padaos motion for execution.
Issue:
1. Whether Padao was
validly dismissed
2. Whether he is entitled to
monetary award.
Ruling:
First Issue.Yes,The reasons
for valid dismissal are:
(a) Serious misconduct or
willful disobedience by the
employee of the lawful orders of
his employer or representative
in connection with his work;
(b)Gross
and
habitual
neglect by the employee of his
duties;
(c)Fraud or willful breach
by the employee of the trust
reposed in him by his employer
or
duly
authorized
representative;
(d) Commission of a crime
or offense by the employee
against the person of his
employer or any immediate
family member of his family or
his
duly
authorized
representative; and
(e)Other causes analogous
to the foregoing.
A just and authorized
cause and requirement of notice
containing a statement of the
causes for termination and shall
afford
the
latter
ample

opportunity to be heard and to


defend himself.
In this case, Padao was
dismissed by PNB for gross and
habitual neglect of duties under
Article 282 (b) of the Labor
Code.Gross negligence connotes
want of care in the performance
of ones duties, while habitual
neglect implies repeated failure
to perform ones duties for a
period of time, depending on
the
circumstances.
Gross
negligence has been defined as
the want or absence of or failure
to exercise slight care or
diligence, or the entire absence
of care. It evinces a thoughtless
disregard
of
consequences
without exerting any effort to
avoid them.
In the case at bench, Padao was
accused of having presented a
fraudulently positive evaluation
of
the
business,
credit
standing/rating and financial
capability of some clients. Padao
over-appraised the collateral of
three clients.Padaos repeated
failure to discharge his duties as
a credit investigator of the bank
amounted to gross and habitual
neglect of duties under Article
282 (b) of the Labor Code. He
not only failed to perform what
he was employed to do, but also
did
so
repetitively
and
habitually, causing millions of
pesos in damage to PNB. Thus,
PNB acted within the bounds of
the law by meting out the
penalty of dismissal, which it
deemed appropriate given the
circumstances.
2011- 2012 Labor Law Case Digest Compilations
UC LLB III-B
Page 145

Second Issue.Padao is not


entitled to financial assistance.
In Toyota Motor Phils. Corp.
Workers Association v. NLRC, the
Court reaffirmed the general
rule that separation pay shall be
allowed as a measure of social
justice only in those instances
where the employee is validly
dismissed for causes other than
serious
misconduct,
willful
disobedience,
gross
and
habitual neglect of duty, fraud
or willful breach of trust,
commission of a crime against
the employer or his family, or
those reflecting on his moral
character. Padao, however, was
dismissed due to the mentioned
grounds.
The
court
must
be
judicious and circumspect in
awarding separation pay or
financial assistance as the
constitutional policy to provide
full protection to labor is not
meant to be an instrument to
oppress the employers.
The
court should be more cautions in
awarding financial assistance to
the undeserving and those who
are unworthy of the liberality of
the law.
82. TIMOTEO H. SARONA v
NATIONAL LABOR RELATIONS
COMMISSION, ROYALE
SECURITY AGENCY
(FORMERLY SCEPTRE
SECURITY AGENCY) and
CESAR S. TAN
G.R. No. 185280, January 18,
2012

Topic: Piercing the Corporate


Veil;
Computation
of
Backwages
Facts:
On
June
20,
2003,
TimoteoSarona, who was hired
by Sceptre as a security guard
sometime in April 1976, was
asked by theSceptres Operation
Manager,
to
submit
a
resignation letter as the same
was supposedly required for
applying for a position at
Royale. He was also asked to fill
up
Royales
employment
application form, which was
handed to him by Royales
General Manager. After several
weeks of being in floating
status, Royales Security Officer,
assigned
the
petitioner
at
Highlight
Metal
Craft,
Inc.
(Highlight Metal). Thereafter,
the petitioner was transferred
and assigned to Wide Wide
World Express, Inc. (WWWE,
Inc.). He was once again
assigned at Highlight Metal.
Subsequently, he was informed
that he would no longer be
given any assignment per the
instructions of Aida SabalonesTan (Aida), general manager of
Sceptre.
Sarona files a case for
illegal dismissal. Royale claims
that it is not liable since it is
Sceptre, not Royale that was its
employer.
Issue:
1.
Whether
Royales
corporate fiction should be
pierced for the purpose of
2011- 2012 Labor Law Case Digest Compilations
UC LLB III-B
Page 146

compelling it to recognize the


petitioners length of service
with Sceptre and for holding it
liable for the benefits that have
accrued to him arising from his
employment with Sceptre
2. Whether the petitioners
backwages should be limited to
his salary for three (3) months.
Ruling:
First Issue.Yes. Royale is a
continuation or successor of
Sceptre.The corporate mask
may be removed or the
corporate veil pierced when the
corporation is just an alter ego
of a person or of another
corporation. For reasons of
public policy and in the interest
of justice, the corporate veil will
justifiably be impaled only when
it becomes a shield for fraud,
illegality or inequity committed
against third persons. The
doctrine
of
piercing
the
corporate veil applies only in
three (3) basic areas, namely: 1)
defeat of public convenience as
when the corporate fiction is
used as a vehicle for the
evasion
of
an
existing
obligation; 2) fraud cases or
when the corporate entity is
used to justify a wrong, protect
fraud, or defend a crime; or 3)
alter ego cases, where a
corporation is merely a farce
since it is a mere alter ego or
business conduit of a person, or
where the corporation is so
organized and controlled and its
affairs are so conducted as to
make
it
merely
an
instrumentality, agency, conduit

or
adjunct
of
another
corporation.
Evidence abound showing
that
Royale
is
a
mere
continuation or successor of
Sceptre
and
fraudulent
objectives are behind Royales
incorporation
and
the
petitioners
subsequent
employment therein. Itwas Aida
who exercised control and
supervision over the affairs of
both Sceptre and Royale. Aida
took over as early as 1999 when
Roso assigned his license to
operate Sceptre on May 3, 1999.
She caused the registration of
the business name Sceptre
Security &Detective Agency
under her name with the DTI a
few
months
after
Roso
abdicated his rights to Sceptre
in her favor. As far as Royale is
concerned, has a hand in its
management and operation and
possesses
control
and
supervision of its employees,
including the petitioner. Aida
was the one who decided to
stop giving any assignments to
the petitioner and summarily
dismiss him is an eloquent
testament of the power she
wields insofar as Royales affairs
are concerned. The presence of
actual common control coupled
with the misuse of the corporate
form to perpetrate oppressive or
manipulative conduct or evade
performance of legal obligations
is patent; Royale cannot hide
behind its corporate fiction.
The fraudulent intent or
illegal purpose behind the
exercise of such control to
2011- 2012 Labor Law Case Digest Compilations
UC LLB III-B
Page 147

warrant the piercing of the


corporate veil was manifest
when the petitioner was made
to resign from Sceptre and how
he became an employee of
Royale. It was a scheme to
change
Timoteoslength
of
service with Sceptre
Effectively, the petitioner cannot
be deemed to have changed
employers
as
Royale
and
Sceptre are one and the same.
His separation pay should, thus,
be computed from the date he
was hired by Sceptre in April
1976 until the finality of this
decision. The intervening period
between the day an employee
was illegally dismissed and the
day the decision finding him
illegally dismissed becomes final
and
executory
shall
be
considered in the computation
of his separation pay as a period
of imputed or putative
service.
Second Issue:Backwages
should be computed from the
time
the
employee
was
terminated until the finality of
the
decision,
finding
the
dismissal unlawful.Therefore, in
accordance with R.A. No. 6715,
petitioners are entitled on their
full backwages, inclusive of
allowances and other benefits or
their monetary equivalent, from
the
time
their
actual
compensation was withheld on
them up to the time of their
actual reinstatement.
83. JONATHAN V. MORALES,
Petitioner, vs. HARBOUR

CENTRE PORT TERMINAL,


INC.
G.R. No. 174208, January 25,
2012
Topic: Constructive Dismissal
Facts:
Jonathan
V.
Morales
(Morales) was hired by Harbour
Centre
Port
Terminal,
Inc.
(HCPTI) to Division Manager of
the Accounting Department.
Later, Morales received an interoffice
memorandum,
reassigning him to Operations
Cost Accounting, tasked with the
duty
of
"monitoring
and
evaluating
all
consumables
requests, gears and equipment"
related to the corporations
operations and of interacting
with its sub-contractor, Bulk
Fleet
Marine
Corporation.
Morales
wrote
Singson,
protesting that his reassignment
was a clear demotion since the
position to which he was
transferred
was
not
even
included in HCPTIs plantilla. In
response to Morales grievance
that he had been effectively
placed on floating status and he
was told that transfer of
employees is a managerial
prerogative. For the whole of
the ensuing month Morales was
absent from work and/or tardy.
Singson gave two warnings
against
Morales
for
his
continuous absent. Morales did
not report to work so he was
dismissed.
Morales
filed
a
complaint
for
constructive
dismissal.
2011- 2012 Labor Law Case Digest Compilations
UC LLB III-B
Page 148

CA
reversed
reinstated LA.

NLRC,

Issue:
Whether the change in the
designation/position
of
petitioner
constituted
constructive dismissal.
Ruling:
Constructive
dismissal
exists where there is cessation
of work because "continued
employment
is
rendered
impossible,
unreasonable
or
unlikely, as an offer involving a
demotion in rank or a diminution
in pay" and other benefits. In
cases of a transfer of an
employee, the rule is settled
that the employer is charged
with the burden of proving that
its conduct and action are for
valid and legitimate grounds
such
as
genuine
business
necessity and that the transfer
is
not
unreasonable,
inconvenient or prejudicial to
the employee. If the employer
cannot overcome this burden of
proof, the employees transfer
shall be tantamount to unlawful
constructive dismissal.
Our perusal of the record
shows that HCPTI miserably
failed to discharge the foregoing
onus. The reassignment was a
demotion evident from Morales
new duties which, far from being
managerial in nature, were very
simply and vaguely described as
inclusive of "monitoring and
evaluating
all
consumables
requests, gears and equipments
related to [HCPTIs] operations"

as well as "close interaction with


[its] sub-contractor Bulk Fleet
Marine Corporation."
Although much had been
made about Morales supposed
refusal to heed his employers
repeated directives for him to
return to work, our perusal of
the record also shows that
HCPTIs theory of abandonment
of employment cannot bear
close scrutiny. As a just and
valid ground for dismissal, at
any rate, abandonment requires
the
deliberate,
unjustified
refusal of the employee to
resume
his
employment,
without
any
intention
of
returning. Since an employee
like Morales who takes steps to
protest his dismissal cannot
logically be said to have
abandoned his work, it is a
settled doctrine that the filing of
a complaint for illegal dismissal
is
inconsistent
with
abandonment of employment.
84. STOLT-NIELSEN
TRANSPORTATION GROUP,
INC. AND CHUNG GAI SHIP
MANAGEMENT, v SULPECIO
MEDEQUILLO, JR
G.R. No. 177498 January 18,
2012
Topic:
Employer-employee
Relationship; Money Claims
Of Migrant Workers For
Overseas
Deployment
Having
No
EmployerEmployee Relationship
Facts:
2011- 2012 Labor Law Case Digest Compilations
UC LLB III-B
Page 149

In
the
first
contract,
SullpecioMedequillowas hired by
Stolt-Nielsen Marine Servicesas
Third Assistant Engineer on
board
the
vessel
Stolt
Aspiration for a period of nine
(9) months. He joined the vessel
MV
Stolt
Aspiration.
On
February 1992 or for nearly
three (3) months of rendering
service and while the vessel was
at Batangas, he was ordered by
the ships master to disembark
the vessel and repatriated back
to Manila for no reason or
explanation.Upon his return to
Manila,
he
immediately
proceeded to the petitioners
office where he was transferred
employment with another vessel
named MV Stolt Pride under
the same terms and conditions
of the First Contract. A Second
Contract
was
noted
and
approved by the POEA, however
he was never deployed.
Issue:
1.
Whether
there
is
employeeemployer
relationship.
2. Whether Sulpecio can
claim monetary awards.
Ruling:
First Issue.There was no
employee-employer relationship
because the first contract was
novated by the second contract.
Since Sulpecio was no actual
deployed,
no
employeeemployer
relationship
commenced. However he can
still claim monetray awards
because of the breach of the

second contract.Even without


actual
deployment,
the
perfected contract gives rise to
obligations on the part of
petitioners.
We rule that distinction
must be made between the
perfection of the employment
contract
and
the
commencement
of
the
employer-employee relationship.
The perfection of the contract,
which in this case coincided with
the date of execution thereof,
occurred when petitioner and
respondent agreed on the object
and the cause, as well as the
rest of the terms and conditions
therein. The commencement of
the
employer-employee
relationship,
as
earlier
discussed, would have taken
place
had
petitioner
been
actually deployed from the point
of hire. Thus, even before the
start of any employer-employee
relationship, contemporaneous
with the perfection of the
employment contract was the
birth of certain rights and
obligations, the breach of which
may give rise to a cause of
action against the erring party.
Thus, if the reverse had
happened, that is the seafarer
failed or refused to be deployed
as agreed upon, he would be
liable for damages.
Second
Issue.Yes.
Following the law, the claim is
still cognizable by the labor
arbiters of the NLRC under
Section 10 of Republic Act No.
8042 (Migrant Workers Act)
2011- 2012 Labor Law Case Digest Compilations
UC LLB III-B
Page 150

which provides for money claims


by reason of a contract involving
Filipino workers for overseas
deployment. The existence of an
employer-employee relationship
is not essential for money
claims in case of OFWs working,
or about to work in foreign
vessels.
85. MANILA PAVILION HOTEL
owned and operated by
ACESITE (PHILS.) Hotel
Corporation vs. HENRY
DELADA
G.R. No. 189947
January 25, 2012
Topic: Grounds For
Dismissal- Insubordination
And Willful Disobedience
Facts:
Delada, union president,
filed a grievance against MPH
because he was reassigned as
Head Waiter from Rotisserie to
Seasons Coffee Shop at the
same hotel pursuant to a
reorganization
program.
He
asked for a grievance meeting
on the matter but no settlement
was reached, even until the
Grievance Committee level.
He
continued
to
report to Rotisserie soMPH sent
him several memoranda for him
to explain in writing why he
should not be penalized for
serious
misconduct;
willful
disobedience of lawful orders;
gross insubordination; gross and
habitual neglect of duties; and
willful breach of trust. MPH
initiated
administrative
proceedings
against
him.

Deladalodged a Complaint
before the NCMB. Whilethe
Complaint was pending before
the
Panel
of
Voluntary
Arbitrators, MPH placed himon a
30-day
preventive
suspension,and thenissued a
decisionimposing on hima 90day
suspension
for
insubordination. He opposed
arguing
that
MPH
cannot
proceed with the disciplinary
action since the matter had
been included in the voluntary
arbitration.
The
PVA
ruled
that
Deladas transfer was a valid
exercise
of
management
prerogative. It was done in the
interest
of
efficient
and
economic operations of MPH and
there was no bad faith, or
improper motive attendant at
the transfer. The mere fact that
he was the Union President did
not "put color or ill motive and
purpose" to his transfer. PVA
also found that the reason of
hisrefusalwas that he asked for
additional monetary benefits as
a condition. His transfer did not
prejudice him nor resulted in
diminution
of
salaries
or
demotion in rank. He had no
valid and justifiable reason to
refuse the order.
The PVA
also ruled that there was no
legal and factual basis on MPHs
imposition
of
preventive
suspension. The mere assertion
of MPH that Delada might
sabotage the food to be
prepared and served was more
imagined than real. MPH went
beyond the 30-day period of
2011- 2012 Labor Law Case Digest Compilations
UC LLB III-B
Page 151

preventive
suspension
prescribed by the Labor Code
when it imposed the 90-day
suspension. Also, MPH lost its
authority to continue with the
administrative proceedings for
insubordination
and
willful
disobedience and the imposition
of
the
90-day
suspension
because PVA acquired exclusive
jurisdiction when the issues
were submitted before it. The
joint submission of the issue on
the validity of the transfer order
encompassed, by necessary
implication,
the
issue
of
insubordination
and
willful
disobedience of the transfer
order.
MPH contended that the
specific issue of whether Delada
could be held liable for his
refusal to the transfer order was
not raised before the PVA, and
that the panels ruling was
limited to the validity of said
order. Thus, MPH maintains that
it cannot be deemed to have
surrendered its authority to
impose
the
penalty
of
suspension.
Issue:
Whether MPH retained the
authority to continue with the
administrative
case
against
Delada for insubordination and
willful disobedience of the
transfer order.
Ruling:
Yes. MPH did not lose its
authority to discipline
respondent for his continued

refusal to report to his new


assignment.
The PVA did not make a
ruling on the specific issue of
insubordination
and
willful
disobedience of the transfer
order. The PVA merely said that
its disagreement with the 90day penalty of suspension
stemmed from the fact that the
penalty went beyond the 30-day
limit for preventive suspension.
First, it must be pointed
out that the basis of the 30-day
preventive suspension imposed
on Delada was different from
that of the 90-day penalty of
suspension. The former was
imposed by MPH on the
assertion that Delada might
sabotage
hotel
operations
whilethe latter was imposed as
a form of disciplinary action. It
was the outcome of the
administrative
proceedings
conducted
against
him.
Preventive suspension is a
disciplinary measure resorted to
by
the
employer
pending
investigation of an alleged
malfeasance or misfeasance
committed by an employee. The
employer temporarily bars the
employee from working if his
continued employment poses a
serious and imminent threat to
the life or property of the
employer or of his co-workers.
On the other hand, the penalty
of suspension refers to the
disciplinary action imposed on
the employee after an official
investigation or administrative
hearing is conducted. The
employer exercises its right to
2011- 2012 Labor Law Case Digest Compilations
UC LLB III-B
Page 152

discipline
erring
employees
pursuant to company rules and
regulations. Thus, a finding of
validity of the penalty of 90-day
suspension will not embrace the
issue of the validity of the 30day preventive suspension.
In
Allied
Banking
Corporation vs CA, the SC ruled:
The refusal to obey a valid
transfer order constitutes willful
disobedience of a lawful order of
an employer. Employees may
object to, negotiate and seek
redress against employers for
rules or orders that they regard
as unjust or illegal. However,
until and unless these rules or
orders are declared illegal or
improper
by
competent
authority, the employees ignore
or disobey them at their peril.
Pursuant
to
Allied
Banking, unless the order of
MPH is rendered invalid, there is
a presumption of the validity of
that order. Since the PVA
eventually
ruled
that
the
transfer order was a valid
exercise
of
management
prerogative, we hereby reverse
the Decision and the Resolution
of the CA affirming the Decision
of the PVA in this respect. MPH
had the authority to continue
with
the
administrative
proceedings for insubordination
and willful disobedience against
Delada and to impose on him
the penalty of suspension. As a
consequence, MPH is not liable
to pay back wages and other
benefits
for
the
period
corresponding to the penalty of
90-day suspension.

86. HYPTE R. AUJERO,


Petitioner, vs. PHILIPPINE
COMMUNICATIONS
SATELLITE CORPORATION,
Respondent.
G.R. No. 193484
January 18, 2012
Topic: Validity Of Quitclaim;
Perfection Of Appeal
Facts:
Aujero started working for
Philcomsat as an accountant.
After 34 years of service, he
applied for early retirement and
was approved. It entitles him to
receive retirement benefits. At
that time, he was Philcomsat's
Senior Vice-President with a
monthly salary of P274, 805.00.
He executed a Deed of Release
and Quitclaim in Philcomsats
favor
and
receivedP9,
439,327.09.
After 3 years, he filed a
complaint for unpaid retirement
benefits, claiming that the
actual amount of his retirement
pay is P14, 015,055.00 and the
P9.4M he received as supposed
settlement for his claims is
unconscionable;
that
he
accepted a lesser amount as he
was in dire need thereof and he
signed the quitclaim as no
single
centavo
would
be
released to him if he did not
execute the Deed;that his right
to receive the full amount of his
retirement benefits is provided
under the Retirement Planwhich
plan was for the exclusive
benefit of its employees.
2011- 2012 Labor Law Case Digest Compilations
UC LLB III-B
Page 153

LA Lustria ruled thatthe


deductions
are illegal
and
directed Philcomsat to pay
Aujerothe
balance
of
his
retirement benefits and salary.
Philcomsat failed to prove that
the P9.4M received by Aujero
was a product of negotiations
between them.
NLRC reversed and set
aside LA LustriasDecisionand
ruled that the quitclaim is valid.
Aujero failed to allege that
Philcomsat employed means to
vitiate his consent to the
quitclaim. He even waited for
three years when he could
immediately raise his objections
at the earliest opportune.It is
clear that Aujero simply had a
change of mind.
On June 20, a copy of LA
Lustrias Decision was served on
Querubin, one of Philcomsats
executive assistants, who is not
the Secretary of Philcomsats
counsel. It was only on June 21
that Querubin gave the copy to
the executive assistant of the
counsel. This led Philcomsats
counsel to believe that it was on
June 21that the ten day-period
started to run.Having in mind
that the delay was only one day
and the explanation offered by
Philcomsats counsel, the NLRC
disregarded such procedural
lapse and proceeded to decide
the appeal on its merits.
CA ruled that the NLRC
was correct in upholding the
validity of Aujeros quitclaim.
Also, it ruled that NLRC did not
act with grave abuse of
discretion in giving due course

to the Philcomsats appeal


sinceSC has ruled that where a
copy of the decision is served on
a person who is neither a clerk
nor one in charge of the
attorneys office, such service is
invalid.
Issue:
1. Whether the quitclaim is valid
2. Whether the delay in the filing
of Philcomsats appeal and
posting of surety bond is
inexcusable
Ruling:
First Issue. Yes. Absent
any evidence that any of the
vices of consent is present and
considering Aujeros position
and education, the quitclaim
executed by Aujero constitutes a
valid and binding agreement
Aujero is not an ordinary
laborer. He is mature, intelligent
and educated with a college
degree, who cannot be easily
duped or tricked into performing
an act against his will. As no
proof was presented that the
said quitclaim was entered into
through
fraud,
deception,
misrepresentation, the same is
valid
and
binding. He
is
estopped from questioning the
said quitclaim
and
cannot
renege after accepting the
benefits thereunder. Aujero's
educational background and
employment stature render it
improbable
that
he
was
pressured,
intimidated
or
inveigled
into
signing
the
subject quitclaim. This Court
cannot permit Aujero to relieve
2011- 2012 Labor Law Case Digest Compilations
UC LLB III-B
Page 154

himself from the consequences


of his act, when his knowledge
and understanding thereof is
expected. Also, the period of
time that he allowed to lapse
before filing a complaint to
recover the supposed deficiency
in his retirement pay clouds his
motives,
leading
to
the
reasonable conclusion that his
claim of being aggrieved is a
mere afterthought, if not a mere
pretention.
Second Issue. No. CA did
not err in not finding grave
abuse of discretion in the NLRC's
decision to give due course to
Philcomsat's appeal despite its
being belatedly filed. A petition
for certiorari under Rule 65 of
the Rules of Court is confined to
the correction of errors of
jurisdiction and will not issue
absent a showing of a capricious
and
whimsical
exercise
of
judgment, equivalent to lack of
jurisdiction. Not every error in a
proceeding, or every erroneous
conclusion of law or of fact, is an
act in excess of jurisdiction or an
abuse
of
discretion.
The
prerogative of writ of certiorari
does not lie except to correct,
not every misstep, but a grave
abuse of discretion.
Procedural rules may be
relaxed to give way to the full
determination of a case on its
merits.
87. ABDULJUAHID R.
PIGCAULAN,* vs. SECURITY
and CREDIT INVESTIGATION,

INC. and/or RENE AMBY


REYES, Respondents.
G.R. No. 173648
January 16, 2012
Topic: Computation Of
Monetary Claims
Facts:
Canoy and Pigcaulan were
security guards of SCII. They
filed with the LA separate
complaints for underpayment of
salaries and non-payment of OT,
holiday, rest day, SIL and 13th
month pays. These complaints
were
consolidated.
They
submitted their daily time
records reflecting the number of
hours served and their wages.
SCII argued that Canoy
and Pigcaulan were paid just
salaries and other benefits
under the law as supported by
the payrolls; that their salaries
were
above
the
statutory
minimum
wage;and
that
Canoys
and
Pigcaulans
monetary claims should only be
limited to the past 3 years of
employment pursuant to the
rule on prescription of claims.
The LA held that the
payroll listings did not prove
that Canoy and Pigcaulan were
duly paid as itwas not signed by
them or by any SCII officer. The
LA ordered the payment of OT
pay, holiday pay, SIL pay and
13th month pay.
NLRC
dismissed
SCIIs
appeal and held that the
evidence show underpayment of
salaries as well as non-payment
of SIL benefit. It sustained the
LAs Decision.
2011- 2012 Labor Law Case Digest Compilations
UC LLB III-B
Page 155

CA set aside the rulings of


both the LA and the NLRC after
noting that there were no
factual
and
legal
bases
mentioned in the rulings to
support the conclusions made. It
dismissed all the monetary
claims
of
Canoy
and
Pigcaulanbecause:
(a)theLAsdecision
did
not
embody the detailed and full
amount awarded and did not
statethe
substance
of
the
evidence
given
by
petitionersand the laws that
would show that they are indeed
entitled to their claims;(b) the
payrolls from 1998 to 1999
indicate
Canoyssignatures;
(c)there is no proof that Gen.
Manager Reyes deliberately or
maliciously designed to evade
SCIIs
alleged
financial
obligation so he cannot be held
solidarily liable with SCII; SCII
has a corporate personality
separate and distinct from
Reyes.
The caption of the petition
reflects both the names of
Canoy
and
Pigcaulan
as
petitioners. It appears however
from its body that it is being
filed solely by Pigcaulan who
submits that the LA and the
NLRC are not strictly bound by
the rules. They assert that his
filing of the present petition
redounds likewise to Canoys
benefit since their complaints
were
consolidated.
They
maintain that any kind of
disposition made in favor or
against either of them would
inevitably apply to the other.

Hence, the institution of the


petition solely by Pigcaulan does
not render the assailed Decision
final as to Canoy.
SCII point out that since it
was only Pigcaulan who filed the
petition, the CA Decision has
already
become
final
and
binding upon Canoy.
Issues:
1. Whether
CA
Decision
is
considered final as to Canoy.
2. Whether CA erred in dismissing
the claims instead of remanding
the case to the LA for a detailed
computation of the judgment
award.
3. WhetherPigcaulans
monetary
claims should only be limited for
the period within three years
preceding the filing of the
complaint in 2000
Ruling:
First Issue.Yes. The SC
examined the petition and find
that same was filed by Pigcaulan
solely on his own behalf. This is
very clear from the petitions
prefatory.Also,
under
the
heading
"Parties",
only
Pigcaulan is mentioned as
petitioner and consistent with
this, the body of the petition
refers only to a "petitioner" and
never in its plural form. Aside
from
the
fact
that
the
Verification and Certification of
Non-Forum Shopping attached
to the petition was executed by
Pigcaulan alone, it was plainly
and particularly indicated under
the name of the lawyer who
prepared
the
same,
Atty.
2011- 2012 Labor Law Case Digest Compilations
UC LLB III-B
Page 156

Grageda, that he is the "Counsel


for
Petitioner
AdbuljuahidPigcaulan"
only.
Since no appeal from the CA
Decision was brought by Canoy,
same has already become final
and executory as to him.Canoy
cannot simply incorporate in his
affidavit a verification of the
contents and allegations of the
petition as he is not one of the
petitioners therein.
Second Issue. Yes. Indeed,
the
LA
failed
to
provide
sufficient basis for the monetary
awards granted. Such failure,
however, should not result in
prejudice to the substantial
rights of the party. While we
disallow the grant of overtime
pay and restday pay in favor of
Pigcaulan because nothing in
the records could substantially
support his contention that he
had rendered service beyond
eight hours and during Sundays,
he is entitled, as a matter of
right, to his holiday pay, SIL pay
and 13th month pay for year
2000. Hence, the CA is not
correct in dismissing Pigcaulans
claims in its entirety.
Since
the
records
are
insufficient to use as bases to
properly compute Pigcaulans
claims, the case should be
remanded to the Labor Arbiter
for a detailed computation of
the monetary benefits due to
him.
Third
Issue.
Yes.
Consistent with the rule that all
money claims arising from an

employer-employee relationship
shall be filed within three years
from the time the cause of
action accrued, Pigcaulan can
only demand the amounts due
him for the period within three
years preceding the filing of the
complaint in 2000.
88. MAGSAYSAY MARITIME
CORPORATION and/or
WASTFEL-LARSEN MGMT
vs.OBERTO S. LOBUSTA
G.R. No. 177578
January 25, 2012
Topic: Applicable Laws in
Disability Claims Of Migrant
Seafarers
Facts:
MMC
is
a
domestic
corporation and local manning
agent of MV "Fossanger" and
Wastfel-Larsen. Lobusta is a
seaman who has worked for
MMC since 1994. In 1998, he
was
hired
again
by
MMCforWastfel-Larsen.
The
contract provides for Lobusta's
basic salary and OT pay and
that the standard terms and
conditions
governing
the
employment of Filipino seafarers
on board ocean-going vessels,
as approved byDOLE and POEA
Standard Employment Contract,
shall be strictly and faithfully
observed.
Lobusta
boarded
MV
"Fossanger" in 1998. After two
months, he complained of
breathing difficulty and back
pain so he was admitted and
was diagnosed of severe acute
2011- 2012 Labor Law Case Digest Compilations
UC LLB III-B
Page 157

bronchial asthma. Lobusta was


referred to Dr. Lim whoissued
numerous medical reports on his
condition. Hehad undergone a
surgery. It wasthen opined that
Lobustas disease needs to be
monitored regularly and that he
needs to be on bronchodilator
indefinitely. Hence, he should be
declared disabled. He was
reexamined andwas diagnosed
to have a moderate obstructive
pulmonary disease.
MMC
took
a
second
opinion and the parties agreed
on Dr. Roa whostated that
Lobusta is not physically fit to
resume his normal work due to
the
persistence
of
his
symptoms; that his asthma will
remain
active
and
needs
multiple controller medications.
As the parties failed to reach a
settlement as to the amount to
which
Lobusta
is
entitled,
Lobusta filed a complaintfor
disability/medical
benefits
against MMC before the NLRC.
In 2000, the partiesagreed
on another medical examination
by Dr. David whose findings
would be considered final.She
foundthat Lobustais not fit to
return to work as an Able
Seaman; that while the degree
of impairment is mild, he should
no longer be considered as
capable of gainful employment
as a seafarer but he may
perform adequately in another
capacity, given a land-based
assignment.
No
settlement
was
reached despite the findings so
the LA ordered the parties to

filetheir position papers. The LA


rendered a decision ordering
MMC to pay Lobusta medical
allowance
for
120
days,
disability
benefits,
and
attorneys fees.The LA held that
provisions of the Labor Codeon
permanent total disability do not
apply to overseas seafarers.
Lobusta appealed. The NLRC
affirmed the LAs decision ruling
that Lobustas condition is only
permanent
partial
disability
since his impairment is only
mild.
The CA declared that
Lobusta
is
suffering
from
permanent total disability and
increased the award of disability
benefits in his favor. Lobusta's
disability brought about by his
bronchial asthma is permanent
and total as he had been unable
to work for more than 120 days,
and because Dr. David found
him not fit to return to his work.
Issue:
Whether CA erred in
applying the provisions of the
Labor Code instead of the
provisions of the POEA contract
in
determining
Lobustas
disability.
Ruling:
No.
Petitioners
are
mistaken that it is only the POEA
Standard Employment Contract
that must be considered in
determining
Lobusta's
disability.In Palisoc v. Easways
Marine, we said that whether
the Labor Codes provision on
permanent
total
disability
2011- 2012 Labor Law Case Digest Compilations
UC LLB III-B
Page 158

applies to seafarers is already a


settled matter. We cited the
earlier case of Remigio v. NLRC
where we said (1) that the
standard employment contract
for seafarers was formulated by
the POEA pursuant to its
mandate under E.O. No. 247 "to
secure the best terms and
conditions of employment of
Filipino contract workers and
ensure compliance therewith,"
and "to promote and protect the
well-being of Filipino workers
overseas"; (2) that Section 29 of
the
1996
POEA
Standard
Employment
Contract
itself
provides that all rights and
obligations of the parties to the
contract, including the annexes
thereof, shall be governed by
the laws of the Republic of the
Philippines,
international
conventions,
treaties
and
covenants where the Philippines
is a signatory; and (3) that even
without this provision, a contract
of labor is so impressed with
public interest that the Civil
Code expressly subjects it to the
special laws on labor unions,
collective bargaining, strikes
and lockouts, closed shop,
wages,
working
conditions,
hours of labor and similar
subjects.
In affirming the Labor
Code concept of permanent
total disability, Remigio further
stated:
Thus,
the
Court
has
applied the Labor Code concept
of permanent total disability to
the case of seafarers. In
Philippine Transmarine Carriers

v. NLRC, seaman Nietes was


found to be suffering from
congestive heart failure and
cardiomyopathy
and
was
declared as unfit to work by the
company-accredited physician.
The Court affirmed the award of
disability
benefits
to
the
seaman, citing ECC v. Sanico,
GSIS v. CA, and Bejerano v. ECC
that "disability should not be
understood more on its medical
significance but on the loss of
earning capacity. Permanent
total
disability
means
disablement of an employee to
earn wages in the same kind of
work, or work of similar nature
that [he] was trained for or
accustomed to perform, or any
kind of work which a person of
[his] mentality and attainment
could do. It does not mean
absolute
helplessness."
It
likewise cited Bejerano v. ECC,
that
in
a
disability
compensation, it is not the
injury which is compensated,
but rather it is the incapacity to
work resulting in the impairment
of ones earning capacity.
The same principles were
cited in the more recent case of
Crystal
Shipping,
Inc.
v.
Natividad. In addition, the Court
cited GSIS v. Cadiz and Ijares v.
CA that "permanent disability is
the inability of a worker to
perform his job for more than
120 days, regardless of whether
or not he loses the use of any
part of his body."
In
Vergara
v.
Hammonia
Maritime Services, Inc., we also
said that the standard terms of
2011- 2012 Labor Law Case Digest Compilations
UC LLB III-B
Page 159

the POEA Standard Employment


Contract
agreed
upon
are
intended to be read and
understood in accordance with
Philippine
laws,
particularly,
Articles 191 to 193 of the Labor
Code, as amended, and the
applicable implementing rules
and regulations in case of any
dispute, claim or grievance.
Thus, the CA was correct
in applying the Labor Code
provisions in Lobustas claim for
disability benefits. The Labor
Arbiter erred in failing to apply
them.
89. LOLITA S. CONCEPCION,
Petitioner, vs. MINEX
IMPORT
CORPORATION/MINERAMA
CORPORATION, KENNETH
MEYERS, SYLVIA P.
MARIANO, and VINA
MARIANO
G.R. No. 153569
January 24, 2012
Topic: Illegal Dismissal;
Labor Due Process
Facts:
Minex is engaged in the
retail of semi-precious stones. It
employed Concepcion as a
salesgirl. It later made her a
supervisor
without
salary
increase.Vinaassigned her to the
SM kiosk and gave her the keys.
Working under her were Calung
and Baquilar.
On
Nov.
9,
1997,
Concepcion and her salesgirls
had
P39,
194.50
sales.
Theydetermined the sales for
three
days
to
be
P50,

912.00.Concepcion wrapped the


money in a plastic bag and put
it in the drawer of the locked
wooden cabinet.The next day,
she phoned Vina to report that
the money was missing. Vina
and Sylvia, her superiors,came
with
a
policeman
andarrestedConcepcion.
She
was detained for a day.
Concepcion
complained
for illegal dismissal in the
DOLE.Minexfiled a complaint for
qualified
theft
against
Concepcion. Sheinsisted on her
innocence butCalung averred
that she had left Concepcion
alone in the kiosk that night
because the latter had still to
change her clothes and that was
the first time that Concepcion
had ever asked to be left
behind. Vina declared that
Concepcion did not call Minexto
pick-up the cash sales on
Sunday violating the standard
operating procedure requiring
cash proceeds exceeding P10K
to be reported for pick-up if it
could not be deposited in the
bank.
The Asst. Prosecutor found
probable cause for qualified
theft and Concepcion was
charged. She appealed to the
DOJ butwas denied. Meanwhile,
LAruled that the dismissal is
illegal and orderedConcepcions
reinstatement
with
full
backwages.Minexisalso ordered
to pay her monetary claims as
well as moral and exemplary
damages and 10%of the total
award for attorneys fees.

NLRC reversed the LAs


decision
declaring
that
Concepcion
had
not
been
dismissed but had abandoned
her job after being found to
have stolen the proceeds of the
sales; and that even if she had
been dismissed, her dismissal
would be justifiable for loss of
trust and confidence. It deleted
the awards of backwages and
other
monetary
claims.Concepcion appealed to
the CA via certiorari, claiming
that the NLRC committed grave
abuse of discretion amounting
to excess of jurisdiction. The CA
sustained NLRC because of the
finding of probable cause for
qualified theft.
Admittedly, there is no
direct evidence that Concepcion
took the money from the
drawer. But direct evidence that
she took the money is not
required for her to be lawfully
dismissed.
Circumstantial
evidence is sufficient on which
to anchor a factual basis for the
dismissal of Concepcion for loss
of confidence.
ISSUE:
1. Whether
Concepcion
was
illegally dismissed
2. Whether Minex complied with
the requirements of due process
prior to the termination

2011- 2012 Labor Law Case Digest Compilations


UC LLB III-B
Page 160

Ruling:
First
Issue.No.Concepcions argument
that there was no evidence to
her guiltsince she had not yet
been
found
guilty
beyond

reasonable doubt of the crime,


is not novel. It has been raised
and rejected many times before
on the basis that neither
conviction beyond reasonable
doubt for a crime against the
employer nor acquittal after
criminal
prosecution
was
indispensable. Nor was a formal
charge in court for the acts
prejudicial to the interest of the
employer a pre-requisite for a
valid dismissal.
In National Labor Union v.
Standard Vacuum Oil, the Court
expressly
stated
that
the
conviction of an employee in a
criminal
case
is
not
indispensable to warrant his
dismissal by his employer. If
there is sufficient evidence to
show that the employee has
been guilty of a breach of trust,
or that his employer has ample
reason to distrust him, it cannot
justly deny to the employer the
authority
to
dismiss
such
employee.
Second
Issue.No.
Concepcion demonstrated how
quickly and summarily her
dismissal
was
carried
out
without first requiring her to
explain anything in her defense
as demanded under the Labor
Code. Instead, her superiors
forthwith had her arrested and
investigated by the police
authorities for qualified theft.
This, we think, was a denial of
her right to due process of law,
consisting in the opportunity to
be heard and to defend herself.
In fact, their decision to dismiss
2011- 2012 Labor Law Case Digest Compilations
UC LLB III-B
Page 161

her was already final even


before the police authority
commenced an investigation of
the theft, the finality being
confirmed by no less than Sylvia
herself telling the petitioner
during their phone conversation
that she (Sylvia) "no longer
wanted to see" her.
Despite their obligation
under the Labor Code to give
her a reasonable opportunity to
explain her side; to set a
hearing or conference with the
assistance of counsel if she so
desires; and to serve her a
written notice of termination
indicating
that
upon
due
consideration
of
all
the
circumstances, grounds have
been established to justify her
termination,
they
wittingly
shunted aside the tenets that
mere accusation did not take
the
place
of
proof
of
wrongdoing,
and
that
a
suspicion or belief, no matter
how sincere, did not substitute
for factual findings carefully
established through an orderly
procedure.
Where the dismissal is for
a just cause, as in the instant
case, the lack of statutory due
process should not nullify the
dismissal, or render it illegal, or
ineffectual.
However,
the
employer should indemnify the
employee for the violation of his
statutory rights.
90. MANSION PRINTING
CENTER and CLEMENT

CHENG, Petitioners,
vs.DIOSDADO BITARA, JR.
Respondent.
G.R. No. 168120
January 25, 2012
Topic: Illegal Dismissal
Facts:
MPC is engaged in the
printing of quality self-adhesive
labels, brochures, posters and
the like. It hiredBitara as a
kargador and later promoted
him as the companys sole
driver tasked to pick-up raw
materials,
collect
account
receivables and deliver the
products to the clients.MPC
noted Bitaras habitual tardiness
and absenteeism.
Petitioners then issued a
Memorandum requiring Bitara to
explain in writing why he should
not
be
punished.
Months
passed,Bitaras attention was
again called. He apologized and
promised to come on time. He
however continued to disregard
attendance
policies.
This
adversely affected the business
operations.
Gen.Manager Davis issued
another
Memo
(Notice
to
Explain)requiring
Bitara
to
explain why he should not be
dismissed.
He
personally
handed the Memo butBitara did
not sign it nor submit any
explanation.Davis
personally
served another Memo (Notice of
Termination) and informedBitara
that he was found to be grossly
negligent of his duties.
Bitararequested
for
reconsideration
but
after
2011- 2012 Labor Law Case Digest Compilations
UC LLB III-B
Page 162

hearing
his
position,
the
management
decided
to
terminate
him.
The
management, out of generosity,
offered
Bitara
P6,
110.00equivalent to his one
month
salary
butBitarademanded an amount
equivalent to two months
salary.
The
management
declined.
Bitara filed a complaint for
illegal dismissal before the LA
praying for his reinstatement
and for the payment of full
backwages and other money
claims. He was summoned by
Davis who introduced him to a
lawyer, who informed him that
he will no longer be admitted to
work because of his 5-day
unauthorized absences. Bitara
explained
that
he
was
compelled to leave due to the
urgency of theirfamily problem
and allegedly, his wife informed
the office of his absence. His
explanation was notacceptable.
He was offered an amount
equivalent to his one month
salary as separation pay but he
refused.
The LA dismissed the
complaint for lack of merit. The
NLRCaffirmed the findings of the
LA. The material facts as found
are all in accordance with the
evidence presented during the
hearing as shown by the record.
The CA reversed the
findings of the NLRC and
declared Bitara to have been
illegally dismissed, thus, entitled
to reinstatement or separation
pay, backwages.

Bitaraclaimed that he was


denied due process because the
company did not observe the
two-notice rule. He maintained
that the Notice of Explanation
and the Notice of Termination,
both of which he allegedly
refused to sign, were never
served upon him.
ISSUES:
Whether there was illegal
dismissal.
Ruling:
None.In order to validly
dismiss
an
employee,
the
employer is required to observe
both substantive and procedural
aspects the termination of
employment must be based on
a just or authorized cause of
dismissal and the dismissal
must be effected after due
notice and hearing.
Petitioners
have
repeatedly called the attention
of respondent concerning his
habitual
tardiness.
The
Memorandum dated 23 June
1999 of Cheng required him to
explain his tardiness. Also in
connection
with
a
similar
infraction,
respondent
even
wrote Cheng a letter dated 29
November 1999 where he
admitted that his tardiness has
affected the delivery schedules
of the company, offered an
apology, and undertook to
henceforth report for duty on
time. Despite this undertaking,
he continued to either absent
himself from work or report late
during the first quarter of 2000.
2011- 2012 Labor Law Case Digest Compilations
UC LLB III-B
Page 163

The imputed absence and


tardiness of the complainant are
documented. He faltered on his
attendance 38 times of the 66
working days. His last absences
on 11, 13, 14, 15 and 16 March
2000 were undertaken without
even notice/permission from
management. These attendance
delinquencies
may
be
characterized as habitual and
are sufficient justifications to
terminate the complainants
employment.
Procedural due process
entails compliance with the twonotice rule in dismissing an
employee: (1) employer must
inform the employee of the
specific acts or omissions for
which his dismissal is sought;
(2) after the employee has been
given the opportunity to be
heard, the employer must
inform him of the decision to
terminate his employment.
In Bughaw v. Treasure
Island Industrial Corporationthis
Court, in verifying the veracity
of
the
allegation
that
respondent refused to receive
the
Notice
of
Termination,
essentially
looked
for
the
following: (1) affidavit of service
stating the reason for failure to
serve the notice upon the
recipient; and (2) a notation to
that effect, which shall be
written on the notice itself.
Thus:
xxx Bare and vague
allegations as to the manner of
service and the circumstances
surrounding the same would not
suffice. A mere copy of the

notice of termination allegedly


sent
by
respondent
to
petitioner, without proof of
receipt, or in the very least,
actual service thereof upon
petitioner, does not constitute
substantial evidence. It was
unilaterally prepared by the
petitioner and, thus, evidently
self-serving and insufficient to
convince even an unreasonable
mind.
Davis
Chengdid
both.
First, he indicated in the notices
the notation that respondent
"refused to sign" together with
the corresponding dates of
service. Second, he executed an
Affidavit dated 29 July 2000
stating that: (1) he is the
General
Manager
of
the
company; (2) he personally
served
each
notice
upon
respondent, when respondent
went to the office/factory on 17
March 2000 and 21 March 2000,
respectively; and (3) on both
occasions, after reading the
contents of the memoranda,
respondent
refused
to
acknowledge receipt thereof. We
are, thus, convinced that the
notices
have
been
validly
served.
91. MANILA ELECTRIC
COMPANY (MERALCO) VS
MA. LUISA BELTRAN
GR No. 189947, January 25,
2012
Topic:Grounds
For
Valid
Dismissal- Loss Of Trust And
Confidence
2011- 2012 Labor Law Case Digest Compilations
UC LLB III-B
Page 164

Facts:
Beltran, who was a Senior
Branch Clerk at MERALCOs
Pasig
Branch,
accepted
P15,164.48
from
Collection
Route Supervisor Berlin Marcos,
which the latter received from
customer Andy Chang.
Beltran, however, was
only able to remit Changs
payment four months after such
receipt.
Thus,
she
was
subsequently
placed
under
preventive suspension pending
completion of an investigation.
MERALCO
considered
as
misappropriation or withholding
of company funds her failure to
immediately remit said payment
in violation of its Code on
Employee
Discipline.
Investigation thereafter ensued.
Beltran, on her part,
admitted
receipt
of
said
payment; however explained
that due to marital woes and
coupled with her worries for her
ailing child, distracted her into
forgetting Changs payment.
Beltran
denied
having
personally used the money.
Beltran
was
subsequently
terminated effective March 13,
1997,
hence
she
filed
a
complaint for illegal dismissal
against MERALCO.
The Labor Arbiter ruled in
favor of Beltran and ordered
MERALCO
to
reinstate
the
complainant to her former
position without backwages.
Upon appeal, the NLRC reversed
the Labor Arbiters Decision and
dismissed Beltrans complaint
against MERALCO. It ruled that

MERALCO
validly
dismissed
Beltran from the service in the
exercise of its inherent right to
discipline its employees. When
brought to the CA, the latter
reversed said decision and
instead held that the penalty of
dismissal is harsh considering
the infraction committed and
Beltrans
nine
years
of
unblemished
service
with
MERALCO.
Issue:
Whether
Beltran
was
validly dismissed due to loss of
trust and confidence.
Ruling:
Yes. For loss of trust and
confidence to be a valid ground
for dismissal, it must be based
on a willful breach of trust and
founded on clearly established
facts. A breach is willful if it is
done intentionally, knowingly
and
purposely,
without
justifiable
excuse,
as
distinguished from an act done
carelessly,
thoughtlessly,
heedlessly or inadvertently. In
addition, loss of trust and
confidence
must
rest
on
substantial grounds and not on
the employers arbitrariness,
whims, caprices or suspicion. In
the case at bench, Beltran
attributed her delay in turning
over Changs payment to her
difficult family situation as she
and her husband were having
marital problems and her child
was suffering from an illness.
Admittedly, she was reminded
of Changs payment by her
2011- 2012 Labor Law Case Digest Compilations
UC LLB III-B
Page 165

supervisor on January 7, 1997


but denied having been ordered
to remit the money on that day.
She then reasoned that her
continued delay was caused by
an inevitable need to take a
leave of absence for her to
attend to the needs of her child
who was suffering from asthma.
Undoubtedly, Beltran was
remiss in her duties for her
failure to immediately turn over
Changs
payment
to
the
company.
Such
negligence,
however, is not sufficient to
warrant
separation
from
employment. To justify removal
from service, the negligence
should be gross and habitual.
Gross negligence x x x is the
want of even slight care, acting
or omitting to act in a situation
where there is duty to act, not
inadvertently but willfully and
intentionally, with a conscious
indifference to consequences
insofar as other persons may be
affected. Habitual neglect, on
the
other
hand,
connotes
repeated failure to perform
ones duties for a period of time,
depending
upon
the
circumstances.
No concrete
evidence was presented by
MERALCO to show that Beltrans
delay in remitting the funds was
done intentionally. Neither was it
shown that same is willful,
unlawful and felonious contrary
to MERALCOs finging as stated
in the letter of termination it
sent to Beltran.
Surely,
Beltrans single and isolated act
of negligence cannot justify her
dismissal from service.

92. NORMAN YABUT vs


MERALCO and MANUEL M.
LOPEZ
GR No. 190436, January 16,
2012
Topic:Illegal Dismissal.
Facts:
When Meralco's Inspection
Office and Investigation-Legal
Office conducted an inspection
regarding
illegal
service
connections, they discovered at
the
petitioners
(one
of
MERALCOs
Branch
Field
Representatives since 1989)
residence
shunting
wires
installed on his meter base.
These wires allegedly allowed
power transmission to the
petitioner's residence despite
the fact that Meralco had earlier
disconnected
his
electrical
service due to his failure to pay
his electric bills.
MERALCO
was
then
constrained to issue a notice of
dismissal upon the petitioner
based on the following grounds:
(a) Serious misconduct by the
employee in connection with his
work; (b) Fraud or willful breach
by the employee of the trust
reposed in him by his employer
or
representative;
(c)
Commission of a crime or
offense by the employee against
his employer; and (d) Other
causes
analogous
to
the
foregoing.
Aggrieved, Yabut filed
with the NLRC a complaint for
illegal dismissal and money
claims. The Labor Arbiter ruled
2011- 2012 Labor Law Case Digest Compilations
UC LLB III-B
Page 166

that the petitioner was illegally


dismissed from the service and
the act imputed upon Yabut was
not related to the performance
of his duties as a Meralco
employee, but as a customer of
the company's electric business.
NLRC upheld said decision.
When brought to the CA,
the latter reversed NLRCs
decision
declaring
the
petitioner's dismissal as lawful
and
attributed
unto
Yabut
authorship
of
the
meter
tampering and illegal use of
electricity acts which it
regarded as serious misconduct.
Issue:
Whether
Yabut
illegally dismissed.

was

Ruling:
No. The dismissal of the
petitioner was founded on just
causes under Article 282 of the
Labor Code of the Philippines.
The requirement for a just
cause was satisfied in this case.
We note that the petitioner's
employment was terminated by
the herein respondents for
violation of Section 7, par. 3 of
Meralco's Company Code on
Employee Discipline, and for the
existence of just cause under
Article 282 (a), (c), (d) and (e) of
the Labor Code. The petitioner's
violation of the company rules
was evident. While he denies
any
involvement
in
the
installation of the shunting wires
which Meralco discovered, it is
significant
that
said
SIN
708668501 is registered under

his name, and its meter base is


situated within the premises of
his
property.
Said
meter
registered electric consumption
during the time his electric
service
was
officially
disconnected by Meralco. It was
the petitioner and his family
who could have benefited from
the illegal connection, being the
residents of the area covered by
the service. His claim that he
failed to know or even notice
the shunted wires fails to
persuade as we consider the
meter located in the front of his
house, the nature of his work as
branch field representative, his
long-time
employment
with
Meralco and his familiarity with
illegal connections of this kind.
Significantly, (t)ampering
with electric meters or metering
installations of the Company or
the installation of any device,
with the purpose of defrauding
the Company is classified as an
act of dishonesty from Meralco
employees, expressly prohibited
under company rules. It is
reasonable that its commission
is classified as a severe act of
dishonesty,
punishable
by
dismissal even on its first
commission, given the nature
and gravity of the offense and
the fact that it is a grave wrong
directed against their employer.
Considering the foregoing, this
Court agrees that there were
just causes for the petitioner's
dismissal. We emphasize that
dismissal
of
a
dishonest
employee is to the best interest
not only of the management but
2011- 2012 Labor Law Case Digest Compilations
UC LLB III-B
Page 167

also of labor. As a measure of


self-protection
against
acts
inimical to its interest, a
company has the right to
dismiss its erring employees. An
employer cannot be compelled
to
continue
employing
an
employee guilty of acts inimical
to the employers interest,
justifying loss of confidence in
him.
93. PRUDENTIAL BANK (NOW
BANK OF THE PHILIPPINE
ISLANDS) VS ANTONIO S.A.
MAURICIO
GR No. 183350, January 18,
2012
Topic: Valid Dismissal
Facts:
Respondent Mauricio was
the
Branch
Manager
of
Prudential Banks Magallanes
Branch.
However,
he
was
consequently terminated due to
loss of trust and confidence.
Charges
brought
against
Mauricio involve violations of
specific
orders
and
memorandum, especially Order
No. 1516 which enjoin approving
officers from encashing U.S.
Treasury
Warrants
(USTWs)
whenever the presenter is not
the payee of the check,
prohibition on Drawing against
Uncollected
Deposit;
and,
commission of imprudent acts
prejudicial to the interests of the
bank.This was in result to an
investigation conducted by the
Bank regarding the alleged
irregularities on the transactions

made between Mauricio and


Spouses Marcelo and Corazon
Cruz. It was claimed that the
said spouses opened in the said
Branch a dollar savings account
and
subsequently,
they
executed a real estate mortgage
in favor of the said bank.
However, it was found out later
that credits to the said account
were being allowed immediate
withdrawals by Mauricio despite
the fact that Spouses Cruz were
not
the
payees
of
said
instruments and neither of them
endorsed the same. The dollar
checks were likewise returned
by their drawee banks for
having forged endorsements,
alterations
to
the
stated
amounts,
or
being
drawn
against
insufficient
funds,
among other reasons.
Cash
withdrawals were allowed even
if the account did not have
sufficient balance to cover the
withdrawals at the time they
were made.
The bank was
alleged to have been exposed to
losses
amounting
to
$774,561.58.
Mauricio filed with the
NLRC a complaint for illegal
dismissal. Mauricio maintained
that there was no irregularity in
the transactions made between
him and the Spouses Cruz .
Issue:
Whether Mr. Mauricio was
dismissed for a just cause and
was afforded due process
Ruling:
2011- 2012 Labor Law Case Digest Compilations
UC LLB III-B
Page 168

No. As correctly held by


the CA, Mauricio cannot be held
to have abused the discretion he
was clothed with absent some
semblance of parameters.
In
the absence of such guidelines,
the validity of Mauricios acts
can be tested by determining
whether they were justified
under
the
circumstances.
Mauricio was faced with a
dilemma
whether
to
accommodate the request for
immediate encashment and/or
withdrawals against USTWs by a
valued client, knowing that
under the Banks rules refund of
any returned check shall be the
personal accountability of the
approving officer. In exercising
his discretion to allow the
questioned
withdrawals,
Mauricio took into consideration
the fact that the Spouses Cruz
have substantial deposit and
security,
and
enjoyed
a
favorable credit standing with
the Bank. And, as found by the
RTC, no malice can be inferred
from Mauricios acts who tried to
collect from the Spouses Cruz
and reported all the transactions
to the head office; in fact, the
Bank never called his attention
to any irregularity in the
transactions but even continued
to credit the account of the
spouses for the value of the
returned checks.
Under the
circumstances, Mauricio indeed
fully considered the interest of
his employer before approving
the questioned transactions.
The
Bank
should
be
reminded that for a dismissal

based on loss of trust and


confidence to be valid, the
breach of trust must be willful,
meaning it must be done
intentionally, knowingly, and
purposely, without justifiable
excuse.
Loss of trust and
confidence stems from a breach
of trust founded on dishonest,
deceitful or fraudulent act. This
is obviously not the case here.

94. REPUBLIC OF THE


PHILIPPINES, REPRESENTED
BY THE CIVIL SERVICE
COMMISSION vs MINERVA
M.P. PACHEO
GR No. 178021, January 31,
2012
Topic: Constructive Dismissal
Facts:
Pacheo was a Revenue
Attorney IV, Assistant Chief of
the Legal Division of the BIR in
Revenue Region No. 7, Quezon
City. Then, she was ordered to
be transferred/reassigned in San
Fernando, Pampanga by virtue
of
a
Revenue
Assignment
Order(RTAO). The BIR cited
exigencies of the revenue
service as basis for the issuance
of the said RTAO.
Pacheo questioned the
reassignment averring that the
transfer would mean economic
dislocation and physical burden
on her part as she would be
compelled
to
travel
from
Quezon City to San Fernando,
Pampanga.
In
sum,
she
considered her transfer from
2011- 2012 Labor Law Case Digest Compilations
UC LLB III-B
Page 169

Quezon City to Pampanga as


amounting to a constructive
dismissal.
Due to the inaction of the BIR,
Pacheo filed a complaint before
the
CSCNational
Capital
Region,
praying
for
the
nullification of RTAO No. 252002. The CSC-NCR dismissed
the same for failure to comply
with Administrative Rules.
The
BIR
subsequently
denied Pacheos protest for lack
of merit. Pacheo appealed her
case before the CSC. The latter
ruled in favor of Pacheo and
held that her reassignment was
not valid as it shall cause
serious
detriment
to
the
petitioner, being merely a rank
and file employee, as his salary
shall be significantly reduced as
a result of her reassignment.
The CA reversed the CSC
Resolution and held that while
petitioners reassignment was
not valid, she was likewise
constructively dismissed as a
result thereof.
Issue:
Whether
the
assailed
decision is legally correct in
declaring that respondent was
constructively dismissed and
entitled
to
back
wages,
notwithstanding
respondents
refusal to comply with BIR RTAO
no.
25-2002
which
is
immediately executory pursuant
to section 24 (f) of P.D. 807.
Ruling:

It appears undisputed that


the reassignment of Pacheo was
not valid.
The Court agrees with the
CA
that
Pacheo
was
constructively dismissed.
While
a
temporary
transfer
or
assignment
of
personnel is permissible even
without the employee's prior
consent, it cannot be done when
the transfer is a preliminary step
toward his removal, or a scheme
to lure him away from his
permanent position, or when it
is
designed
to
indirectly
terminate his service, or force
his resignation. Such a transfer
would in effect circumvent the
provision which safeguards the
tenure of office of those who are
in the Civil Service.
Significantly, Section 6,
Rule III of CSC Memorandum
Circular No. 40, series of 1998,
defines constructive dismissal
as
a
situation
when
an
employee
quits
his
work
because of the agency heads
unreasonable, humiliating, or
demeaning actuations which
render
continued
work
impossible.
Hence,
the
employee is deemed to have
been illegally dismissed. This
may occur although there is no
diminution or reduction of salary
of the employee. It may be a
transfer from one position of
dignity to a more servile or
menial job.
Anent the first argument
of CSC, the Court cannot sustain
the proposition. It was legally
impossible for Pacheo to report
2011- 2012 Labor Law Case Digest Compilations
UC LLB III-B
Page 170

to
her
original
place
of
assignment in Quezon City
considering that the subject
RTAO
No.
25-2002
also
reassigned
Amado
Rey
B.
Pagarigan as Assistant Chief,
Legal Division, from RR4, San
Fernando, Pampanga to RR7,
Quezon City, the very same
position Pacheo formerly held.
The reassignment of Pagarigan
to the same position palpably
created an impediment to
Pacheos return to her original
station.
Reassignments involving a
reduction in rank, status or
salary violate an employees
security
of
tenure,
which
is assured by the Constitution,
the Administrative Code of
1987, and the Omnibus Civil
Service Rules and Regulations.
Security of tenure covers not
only
employees
removed
without cause, but also cases of
unconsented
transfers
and
reassignments,
which
are
tantamount
to
illegal/constructive removal.
The Court is not unaware
that the BIR is authorized to
assign or reassign internal
revenue officers and employees
as the exigencies of service may
require. This authority of the
BIR,
however,
should
be
prudently
exercised
in
accordance with existing civil
service rules.
95. TIMOTEO H. SARONA vs
NLRC, ROYALE SECURITY
AGENCY (FORMERLY

SCEPTRE SECURITY AGENCY)


and CESAR S. TAN
GR No. 185280, January 18,
2012
Topic:
Piercing
The
Corporate Veil; Backwages
Facts:
Petitioner
Sarona
was
hired by Sceptre as a security
guard sometime in April 1976,
however he was subsequently
hired/transferred to Royales
Security Agency. After several
weeks of being in floating status
for three months and being
assigned
to
various
establishments
for
short
periods/intervals, he was later
informed that he would no
longer
be
given
any
assignments per the instruction
of Sceptres General Manager.
This prompted him to file a
complaint for illegal dismissal.
The Labor Arbiter ruled that
petitioner
was
illegally
dismissed and there was no
abandonment of employment as
evident on the petitioners filing
of the said complaint three days
after he was informed that he
would no longer be given any
assignments.
He
however
refused to rule on the piercing of
Royales corporate personality.
The petitioner claimed that
Royale and Sceptre are not
separate legal persons for
purposes of computing the
amount of his separation pay
and other benefits under the
Labor Code. The piercing of
Royales corporate personality is
justified by several indicators
2011- 2012 Labor Law Case Digest Compilations
UC LLB III-B
Page 171

that Royale was incorporated for


the sole purpose of defeating his
right to security of tenure and
circumvent payment of his
benefits to which he is entitled
under the law.
The petitioner did not
appeal Labor Arbiters Decision
but opted to raise the validity of
the latters adverse findings
with respect to piercing Royales
corporate personality.
The CA, in consideration of
substantial justice proceeded to
review the evidence on record
and held that errors not
assigned on appeal may be
reviewed
as
technicalities
should not serve as bar to the
full adjudication of cases.
Issue:
1.
Whether
Royales
corporate fiction should be
pierced for the purpose of
compelling it to recognize the
petitioners length of service
with Sceptre and for holding it
liable for the benefits that have
accrued to him arising from his
employment with Sceptre
2. Whether or not the
petitioners backwages should
be limited to his salary for three
(3) months.
Ruling:
First Issue.Yes. Royale is a
continuation or successor of
Sceptre.
The doctrine of piercing
the corporate veil applies only in
three (3) basic areas, namely: 1)
defeat of public convenience as
when the corporate fiction is

used as a vehicle for the


evasion
of
an
existing
obligation; 2) fraud cases or
when the corporate entity is
used to justify a wrong, protect
fraud, or defend a crime; or 3)
alter ego cases, where a
corporation is merely a farce
since it is a mere alter ego or
business conduit of a person, or
where the corporation is so
organized and controlled and its
affairs are so conducted as to
make
it
merely
an
instrumentality, agency, conduit
or
adjunct
of
another
corporation.
In this regard, this Court
finds cogent reason to reverse
the CAs findings. Evidence
abound showing that Royale is a
mere continuation or successor
of
Sceptre
and
fraudulent
objectives are behind Royales
incorporation
and
the
petitioners
subsequent
employment therein. These are
plainly suggested by events that
the respondents do not dispute
and which the CA, the NLRC and
LA Gutierrez accept as fully
substantiated
but
misappreciated as insufficient to
warrant the use of the equitable
weapon of piercing. As correctly
pointed out by the petitioner, it
was Aida who exercised control
and supervision over the affairs
of both Sceptre and Royale.
Contrary to the submissions of
the respondents that Roso had
been the only one in sole control
of
Sceptres
finances
and
business affairs, Aida took over
as early as 1999 when Roso
2011- 2012 Labor Law Case Digest Compilations
UC LLB III-B
Page 172

assigned his license to operate


Sceptre on May 3, 1999. As
further
proof
of
Aidas
acquisition of the rights as
Sceptres sole proprietor, she
caused the registration of the
business
name
Sceptre
Security & Detective Agency
under her name with the DTI a
few
months
after
Roso
abdicated his rights to Sceptre
in her favor. As far as Royale is
concerned, the respondents do
not deny that she has a hand in
its management and operation
and possesses control and
supervision of its employees,
including the petitioner. As the
petitioner correctly pointed out,
that Aida was the one who
decided to stop giving any
assignments to the petitioner
and summarily dismiss him is an
eloquent testament of the
power she wields insofar as
Royales affairs are concerned.
The presence of actual common
control coupled with the misuse
of the corporate form to
perpetrate
oppressive
or
manipulative conduct or evade
performance of legal obligations
is patent; Royale cannot hide
behind its corporate fiction.
It is well-settled, even
axiomatic, that if reinstatement
is not possible, the period
covered in the computation of
backwages is from the time the
employee
was
unlawfully
terminated until the finality of
the decision finding illegal
dismissal.

Second Issue.
No.With
respect to the petitioners
backwages, this Court cannot
subscribe to the view that it
should be limited to an amount
equivalent to three (3) months
of his salary. Backwages is a
remedy affording the employee
a way to recover what he has
lost by reason of the unlawful
dismissal.
In
awarding
backwages,
the
primordial
consideration is the income that
should have accrued to the
employee from the time that he
was dismissed up to his
reinstatement and the length of
service prior to his dismissal is
definitely inconsequential. In
case separation pay is awarded
and reinstatement is no longer
feasible, backwages shall be
computed from the time of
illegal dismissal up to the
finality of the decision should
separation pay not be paid in
the
meantime.
It
is
the
employees actual receipt of the
full amount of his separation
pay
that
will
effectively
terminate the employment of an
illegally dismissed employee.
Otherwise,
the
employeremployee relationship subsists
and the illegally dismissed
employee
is
entitled
to
backwages, taking into account
the
increases
and
other
benefits, including the 13th
month pay, that were received
by his co-employees who are
not dismissed.

2011- 2012 Labor Law Case Digest Compilations


UC LLB III-B
Page 173

96. JONATHAN V. MORALES


vs HARBOUR CENTRE PORT
TERMINAL, INC.
GR No. 17428, January 25,
2012
Topic: Constructive Dismissal
Facts:
Petitioner was first hired
by respondent Harbour Centre
Port Terminal, Inc. (HCPTI) as an
Accountant and Acting Finance
Officer ,then was promoted to
Division
Manager
of
the
Accounting Department. Then,
he was subsequently reassigned
to Operations Cost Accounting,
tasked
with
the
duty
of
monitoring and evaluating all
consumables requests, gears
and equipment related to the
corporations operations.
Morales protested that his
reassignment
was
a
clear
demotion since the position to
which he was transferred was
not even included in HCPTIs
plantilla. In response, the latter
stated that the transfer was just
an exercise of its management
prerogative.
Later, Morales filed a
complaint
for
constructive
dismissal against HCPTI. The
Labor Arbiter dismissed Morales
complaint for lack of merit and
ruled that his reassignment was
a valid exercise of HCPTIs
management prerogative which
cannot
be
construed
as
constructive dismissal absent
showing that the same was
done in bad faith and resulted in
the diminution of his salary and

benefits.
decision.

CA

affirmed

said

Issue:
Whether the change in the
designation/position
of
petitioner
constituted
constructive dismissal.
Ruling:
No. Constructive dismissal
exists where there is cessation
of work because "continued
employment
is
rendered
impossible,
unreasonable
or
unlikely, as an offer involving a
demotion in rank or a diminution
in pay" and other benefits. Aptly
called a dismissal in disguise or
an act amounting to dismissal
but made to appear as if it were
not, constructive dismissal may,
likewise, exist if an act of clear
discrimination, insensibility, or
disdain
by
an
employer
becomes so unbearable on the
part of the employee that it
could foreclose any choice by
him except to forego his
continued employment. In cases
of a transfer of an employee,
the rule is settled that the
employer is charged with the
burden of proving that its
conduct and action are for valid
and legitimate grounds such as
genuine business necessity and
that
the
transfer
is
not
unreasonable, inconvenient or
prejudicial to the employee. If
the employer cannot overcome
this burden of proof, the
employees transfer shall be
tantamount
to
unlawful
constructive dismissal.
2011- 2012 Labor Law Case Digest Compilations
UC LLB III-B
Page 174

Our perusal of the record


shows that HCPTI miserably
failed to discharge the foregoing
onus. While there was a lack of
showing that the transfer or
reassignment
entailed
a
diminution
of
salary
and
benefits, one fact that must not
be lost sight of was that Morales
was already occupying the
position of Division Manager at
HCPTIs Accounting Department
as a consequence of his
promotion to said position on 22
October
2002.
Concurrently
appointed as member of HCPTIs
Management
Committee
(MANCOM) on 2 December
2002, Morales was subsequently
reassigned by HCPTI from
managerial
accounting
to
Operations Cost Accounting on
27 March 2003, without any
mention of the position to which
he
was
actually
being
transferred.
That
the
reassignment was a demotion
is,
however,
evident
from
Morales new duties which, far
from
being
managerial
in
nature, were very simply and
vaguely described as inclusive
of monitoring and evaluating
all consumables requests, gears
and equipments related to
HCPTIs operations as well as
close interaction with its subcontractor Bulk Fleet Marine
Corporation.
97.SEBASTIAN OASAY vs
PALACIO del GOBERNADOR
CONDOMINIUM
CORPORATION and OMAR
CRUZ

GR No. 194306, February 6,


2012
Topic: Illegal Dismissal;
Facts:
Sebastian Oasay was the
building administrator of Palacio
del Gobernador Condominum
Corporation (PDGCC).
This
corporation is a governmentowned
and
controlled
corporation organized for the
purpose
of
owning
and
arranging the common areas of
Palacio
Del
Gobernador
Condominium where all the
units therein was acquired by
the government, houses various
government agencies such as
the Commission on Elections
(COMELEC), Bureau of Treasury
and
the
Intramuros
Administration.
PDGCCs
president, Cruz, asked Oasay to
submit a written report on
compensation he receives as a
building administrator because
it was known to the president
that Oasay has been receiving
compensation from COMELEC.
Oasay complied with the written
report and admitted that he
receives compensation from
COMELEC because he was
tasked
to
supervise
the
rehabilitation being done by the
COMELEC on the 8th floor of the
building. Further, PDGCC asked
Oasay to explain why he hired
EGB Security Investigation and
General Services, Inc., despite
its lack of license to operate as
a security agency, to the
detriment of PDGCC. Oasay
further
answered
that
the
2011- 2012 Labor Law Case Digest Compilations
UC LLB III-B
Page 175

security agency was not his own


doing but was done by the Bids
and Awards Committee. It was
eventually found out that Oasay
was a member of the Bidding
Award and maneuvered the
others to favor the security
agency.
Oasay then was
dismissed. He filed a complaint
for constructive dismissal at the
NLRC and claims for service
incentive
leave
and
other
money claims. PDGCC on the
other hand claims that his
dismissal was just because of
the loss of trust and confidence
for receiving unlawful additional
compensation for work rendered
without its authority.
Issue:
1. Whether Oasay was
illegally dismissed.
2. Whether his right to
procedural due process was
violated
Ruling:
First Issue. No, Oasay was
not illegally dismissed. In the
present case, the Supreme
Court found no reason to depart
from these principles since the
Labor Arbiter found that there
was substantial evidence to
conclude
that
Oasay
had
breached
the
trust
and
confidence
of
Palacio
Del
Gobernador
Condominium
Corporation.
To stress, the
question
of
whether
the
petitioner
was
illegally
dismissed is a question of fact
as the determination of which
entails an evaluation of the

evidence
on
record.
Wellentrenched is the rule in our
jurisdiction that only questions
of law may be entertained by
this Court in a petition for
review on certiorari.
Second Issue. His right to
due process was not violated.
The validity of an employees
dismissal from service hinges on
the satisfaction of the two
substantive requirements for a
lawful termination. These are,
first, whether the employee was
accorded due process the basic
components of which are the
opportunity to be heard and to
defend himself. This is the
procedural aspect. And second,
whether the dismissal is for any
of the causes provided in the
Labor Code of the Philippines.
This constitutes the substantive
aspect. The substantive aspect
of the case was for a cause
provided in the Labor Code,
Article 282, paragraph c which
states (c) Fraud or willful
breach by the employee of the
trust reposed in him by his
employer or duly authorized
representative.
Here, it is
indubitable that the petitioner
holds a position of trust and
confidence. The position of
Building Administrator, being
managerial
in
nature,
necessarily enjoys the trust and
confidence of the employer. As
for the procedural aspect, there
was
substantial
compliance
because here, PDGCC complied
with the two-notice rule stated
above. PDGCC complied with
the first notice requirement2011- 2012 Labor Law Case Digest Compilations
UC LLB III-B
Page 176

notice informing the petitioner


of his infractions, as shown by
the
following:
(1)
the
Memorandum dated September
27, 2005 sent by Cruz to the
petitioner requiring the latter to
explain and to submit his report
on the additional compensation
he received from COMELEC; and
(2) the letter dated December 9,
2005 sent by Cruz to the
petitioner requiring him to
explain why he allowed the EGB
Security
Investigation
and
General Services, Inc. to render
services to the condominium.
98. ANTONIO B. SALENGA,
ET AL. VS COURT OF
APPEALS, ET AL.
GR No. 174941, February 1,
2012
Topic: Intracorporate Affairs;
Applicability Of Civil Service
Laws On Retirement
Facts:
Salenga filed a complaint
against
Clark
Development
Corporation (CDC) for illegally
dismissing him as the head
executive
assistant.
CDC
through its CEO, Culayco said
that the position held by
Salenga
was
redundant.
Salenga went to the NLRC but
this
was
questioned
by
CDCalleging that the NLRC had
no jurisdiction to entertain the
case on the ground that
petitioner was a corporate
officer and, thus, his dismissal
was an intra-corporate matter
falling
properly
within
the
jurisdiction of the Securities and

Exchange Commission (SEC).


NLRC ruled for the petitioner
contending
that
they
had
jurisdiction because Salenga
was a managerial employee and
not a corporate officer.
It
further contended that Salenga
was dismissed without due
process bcause CDC failed to
explain
why
Salenga
was
terminated and further failed to
prove that his position was
redundant.
The LA awarded
Salenga damages for being an
employee of CDC for 4o years.
CDC appealed, through Atty.
Roman
and
Attty
Mallari,
attorneys
for
OGCC.
Accompanying
the
memorandum of appeal was a
superseades bond covering the
amount due to the petitioner,
but the Memorandum of Appeal
and the Joint Affidavit of
Declaration
were
not
accompanied
by
a
board
resolution from respondents
board of directors authorizing
either Timbol-Roman or Atty.
Mallari, or both, to pursue the
case or to file the appeal on
behalf of respondent. Salenga
was then reinstated pending the
appeal. The CA based on the
testimonies ruled that Salenga
was a corporate officer so it is
an intra corportate dispute.
Issue:
1.Whether the NLRC has
jurisdiction to entertain an
appeal filed by the respondents.
2. Whether a government
owned
and
controlled
2011- 2012 Labor Law Case Digest Compilations
UC LLB III-B
Page 177

corporation is subject to civil


service laws on retirement.
Ruling:
First Issue. The NLRC had
no jurisdiction to entertain the
appeal. A corporation can only
exercise its powers and transact
its business through its board of
directors and through its officers
and agents when authorized by
a board resolution or its
bylaws. The
power
of
a
corporation to sue and be sued
is exercised by the board of
directors. The physical acts of
the corporation, like the signing
of documents, can be performed
only by natural persons duly
authorized for the purpose by
corporate bylaws or by a
specific act of the board. The
purpose of verification is to
secure an assurance that the
allegations in the pleading are
true and correct and have been
filed in good faith. Thus, The
OGCC failed to produce any
valid authorization from the
board
of
directors
despite
petitioner Salengas repeated
demands. It had been given
more than enough opportunity
and time to produce the
appropriate board resolution,
and yet it failed to do so. In fact,
many
of
its
pleadings,
representations,
and
submissions
lacked
board
authorization.
II. CDC is not under the civil
service laws. It is not at all
disputed that while Respondent
and
LUSTEVECO
are
government-owned
and

controlled corporations, they


have no original charters; hence
they are not under the Civil
Service Law. Hence, petitioner
Salenga is entitled to receive
only his retirement benefits
based only on the number of
years he was employed with the
corporation under the conditions
provided under its retirement
plan, as well as other benefits
given to him by existing laws.
99. Skippers United Pacific,
Inc & Skippers Maritimer
Services, Inc. Ltd vs Doza, et
al
GR 175558, February 8, 2012
Topic:
Illegal
Dismissal;
Evidence Needed To Prove
Illegal Dismissal
Facts:
This is a consolidated case
filed by seafarers De Gracia,
Lata, Doza, and Aprosta against
the petitioners, their employer.
Doza, et al worked on vessel MV
Wisdom Star under a contract
except for Doza who has none.
They were let go, accordingly by
Skippers, because they caused
trouble in the ship because of
they were not satisfied by the
conditions of the ship and with
this,
they
voluntarily
preterminated their contract.
Doza, et al on the other hand
claim for payment of home
allotment, salaries due them
based on their contract and
damages. The LA and the NLRC
favored Skippers.
The CA
declared that they committed
2011- 2012 Labor Law Case Digest Compilations
UC LLB III-B
Page 178

grave abuse of discretion when


they relied upon the telex
message of the captain of the
vessel that Doza,et al preterminated
their
contracts.
According to the CA, this does
not
amount to substantial
evidence. Hence, this appeal.
Issue:
1. Whether the CA erred in
not giving credit to the masters
telex message.
2.
Whether
the
respondents
were
illegally
dismissed.
Ruling:
The
Supreme
Court
affirms the decision of the Court
of Appeals.
First and Second Issue. As
to the issue of illegal dismissal,
the respondents were illegally
dismissed.
For a workers
dismissal to be considered valid,
it must comply with both
procedural and substantive due
process. The legality of the
manner of dismissal constitutes
procedural due process, while
the legality of the act of
dismissal
constitutes
substantive due process. In this
case, there was no written
notice furnished to De Gracia, et
al. regarding the cause of their
dismissal. Cosmoship furnished
a written notice (telex) to
Skippers, the local manning
agency, claiming that De Gracia,
et al. were repatriated because
the
latter
voluntarily
preterminated their contracts. This

telex was given credibility and


weight by the Labor Arbiter and
NLRC in deciding that there was
pre-termination
of
the
employment contract akin to
resignation and no illegal
dismissal. However, as correctly
ruled by the CA, the telex
message is a biased and selfserving document that does not
satisfy the requirement of
substantial
evidence.
If,
indeed,
De Gracia,
et
al.
voluntarily pre-terminated their
contracts, then De Gracia, et al.
should have submitted their
written resignations.
100. EASTERN
TELECOMMUNICATIONS
PHILIPPINES, INC (ETPI)VS
EASTERN TELECOMS
EMPLOYEES UNION (ETPU)
GR No, 185665, February 8,
2012
Topic: Demandability And
Enforceability Of Bonuses
Facts:
ETPI is a corporation
engaged in telecommunications.
ETEU is the certified and
exclusive
bargaining
representative of the rank and
file employees of the company.
It has an existing CBA and a side
agreement in between.
EPTI
decided
to
postpone
the
payment of bonuses agreed
upon in the side agreement
because
of
continuing
deterioration of the business.
The union opposed this and
alleged that ETPI has been
giving out bonuses despite not
2011- 2012 Labor Law Case Digest Compilations
UC LLB III-B
Page 179

realizing profits and such is


expressly contained in the side
agreements. ETPI on the other
hand claims that giving out
bonuses is a management
prerogative and it has been an
act of generosity dependent on
the financial standing of the
company. Further, it said that
the
side
agreement
was
discretionary to the company.
The NLRC favored ETPI but the
CA favored the union and said
that the side agreements create
a contractual obligation of ETPI
to pay the bonuses.
Issue:
Whether the employees
are subject to the bonuses
agreed upon in the Side
Agreements.
Ruling:
Yes, they are.
From a
legal point of view, a bonus is a
gratuity or act of liberality of the
giver which the recipient has no
right to demand as a matter of
right. The grant of a bonus is
basically
a
management
prerogative which cannot be
forced upon the employer who
may not be obliged to assume
the onerous burden of granting
bonuses or other benefits aside
from the employees basic
salaries or wages.
A bonus,
however,
becomes
a
demandable
or
enforceable
obligation when it is made part
of the wage or salary or
compensation of the employee.
In the case at bench, it is
indubitable that ETPI and ETEU

agreed on the inclusion of a


provision for the grant of 14 th,
15th and 16th month bonuses in
the
1998-2001
CBA
Side
Agreement, as well as in the
2001-2004
CBA
Side
Agreement, which was signed
on September 3, 2001. The
provision, which was similarly
worded, states:
xxxEmployment-Related
Bonuses
The Company confirms
that
the
14th,
15th and
th
16 month bonuses (other than
the
13th month
pay)
are
granted.xxx
A reading of the above
provision reveals that the same
provides for the giving of 14th,
15th and
16th month
bonuses without
qualification.
The wording of the provision
does not allow any other
interpretation. There were no
conditions specified in the CBA
Side Agreements for the grant of
the benefits contrary to the
claim of ETPI that the same is
justified only when there are
profits earned by the company.
Terse and clear, the said
provision does not state that the
subject bonuses shall be made
to depend on the ETPIs financial
standing or that their payment
was
contingent
upon
the
realization of profits. Neither
does it state that if the company
derives no profits, no bonuses
are to be given to the
employees. In fine, the payment
of these bonuses was not
related to the profitability of
business operations.
2011- 2012 Labor Law Case Digest Compilations
UC LLB III-B
Page 180

101. JULIES BAKESHOP


AND/OR EDGAR REYES VS
HENRY ARNAIZ, ET AL
GR No. 173882, February 15,
2012
Topic: Weight/Credibility Of
Findings Of Facts OF Labor
Arbiters
And
NLRC;
Constructive Dismissal
Facts:
Reyes hired Arnaiz, et. al as
chief bakers in his three franchise
branches of Julies Bakeshop in
Sibalom
and
San
Jose,
Antique. On January 26, 2000,
respondents filed a consolidated
complaint against petitioners for
underpayment of wages, payment
of premium pay for holiday and
rest day, service incentive leave
pay, 13th month pay, cost of living
allowance (COLA) and attorneys
fees. Reyes, then, reassigned
respondents as utility/security
personnel tasked to clean the
outside vicinity of his bakeshops
and to maintain peace and order
in the area. The respondents
refused to sign the memo and
perform their functions. The LA
dismissed Arnaiz and the others
complaints because of their failure
to prove their allegations. The CA
found that the respondents were
constructively dismissed since
their designation from chief
bakers
to
utility/security
personnel is undoubtedly a
demotion in rank which involved
a drastic change in the nature
of
work
resulting
to
a
demeaning
and
humiliating
work
condition. Further,

respondents could not be held


guilty of abandonment of work
as this was negated by their
immediate filing of complaints
to
specifically
ask
for
reinstatement.
Issue:
1.
Whether
the
CA
commited
grave
abuse
of
discretion when it reversed the
findings of facts of the NLRC and
the LA.
2. Whether the transfer of
respondents to another position
tantamount
to
constructive
dismissal
Ruling:
First Issue. No, the CA
correctly reviewed the findings
of the NLRC. Indeed, factual
findings of labor officials who
are deemed to have acquired
expertise in matters within their
respective
jurisdictions
are
generally accorded not only
respect, but even finality.[ It is
a well-entrenched rule that
findings of facts of the NLRC,
affirming those of the Labor
Arbiter, are accorded respect
and due consideration when
supported
by
substantial
evidence. We, however, find
that
the doctrine
of
great
respect and finality has no
application to the case at
bar. As stated, the Labor Arbiter
dismissed
respondents
complaints
on
mere
technicality. The NLRC, upon
appeal, then came up with three
divergent rulings. At first, it
remanded the case to the Labor
2011- 2012 Labor Law Case Digest Compilations
UC LLB III-B
Page 181

Arbiter. However,
in
a
subsequent
resolution,
it
decided to resolve the case on
the merits by ruling that
respondents were constructively
dismissed. But later on, it again
reversed itself in its third and
final resolution of the case and
ruled
in
petitioners
favor. Therefore,
contrary
to
Reyess claim, the NLRC did not,
on any occasion, affirm any
factual findings of the Labor
Arbiter. The CA is thus correct in
reviewing the entire records of
the case to determine which
findings of the NLRC is sound
and
in
accordance
with
law. Besides, the CA, at any
rate, may still resolve factual
issues by express mandate of
the law despite the respect
given to administrative findings
of fact.
Second Issue. There was
constructive
dismissal.
Petitioners claim that respondents
abandoned their job stands on
shallow
grounds. Respondents
cannot be faulted for refusing to
report for work as they were
compelled to quit their job due to
a demotion without any just
cause. Moreover,
we
have
consistently held that a charge of
abandonment is inconsistent with
the filing of a complaint for
constructive
dismissal. Respondents demand
to maintain their positions as chief
bakers by filing a case and asking
for the relief of reinstatement
belies abandonment. As the
transfer proves unbearable to

respondents as to foreclose any


choice on their part except to
forego continued employment,
same amounts to constructive
dismissal for which reinstatement
without loss of seniority rights, full
backwages,
inclusive
of
allowances, and other benefits or
their
monetary
equivalent,
computed from the time their
compensation was withheld up to
the
time
of
their
actual
reinstatement,
should
be
granted. The CA, therefore, did
not err in awarding the reliefs
prayed for by the respondents as
they were, without a doubt,
constructively dismissed.
102. Wuerth Philippines, Inc
vs Rodante Ynson
GR No. 175932, February 15,
2012
Topic: Valid Dismissal
Facts:
Wuert
Philippines,
a
subsidiary of Wuerth Germany
hired Ynson as its National Sales
Manager for Automotive. As an
NSM, he was required to travel
to different parts of the country
to supervise sales and the like.
But on January 2003, he
suffered a stroke. He sent an
email to Germany requesting
the
latter
to
give
him
administrative work while he is
still on therapy but it was
denied. Ricanor, the CEO of
Wuerth Philippines requested
Ynson through letters to go to
Manila for investigative reasons
concerning his job. Ynson did
not comply because of his
2011- 2012 Labor Law Case Digest Compilations
UC LLB III-B
Page 182

condition and informed Ricanor


of such.
Ricanor dismissed
Ynson
on
the
ground
of
continued
absences
without
filing a leave of absence. The
LA and the NLRC favored Ynson
and
awarded
him
of
a
substantial
amount
in
backwages and damages.
Issue:
Whether Ynsons dismissal
was valid due to his gross
dereliction of his duties as NSM
Ruling:
Yes, it was. As of June 4,
2003, respondent would have
been capable of returning to
work. However, despite notices
sent by the petitioner letter
dated June 12, 2003, requiring
respondent
to
attend
an
investigation set on July 14,
2003; letter dated July 4, 2003,
requiring respondent to appear
on
July
25,
2003
for
investigation; and letter dated
July
31,
2003,
requiring
respondent to appear for the
hearing and investigation on
August 18, 2003, respondent
refused to report to his office,
either to resume work or attend
the investigations set by the
petitioner. Even considering the
directive of respondent's doctor
to continue with his present
regimen for at least another
month and a half, it could be
safely deduced that, counted
from June 4, 2003, respondent's
rehabilitation regimen ended on
July 19, 2003. Despite the
completion of his treatment,

respondent failed to attend the


investigations set on July 25,
2003
and
August
18,
2003. Thus, his unexplained
absence in the proceedings
should be construed as waiver
of his right to be present therein
in order to adduce evidence that
would
have
justified
his
continued absence from work.
Being
the
NSM,
respondent
should
have
reported back to work or
attended
the
investigations
conducted
by
petitioner
immediately
upon
being
permitted to work by his
doctors,
knowing
that
his
position remained vacant for a
considerable length of time.
During his absence, nobody was
performing the duties of NSM,
which included, among others,
supervising and monitoring of
respondent's sales area which is
vital to the companys orderly
operation and viability. He did
not even show any sincere effort
to return to work.
Clearly, since there is no
more hindrance for him to
return to work and attend the
investigations set by petitioner,
respondent's failure to do so
was without any valid or
justifiable reason. Respondent's
conduct shows his indifference
and utter disregard of his work
and his employer's interest, and
displays his clear, deliberate,
and gross dereliction of duties.
103. Galang vs. Cityland
Shaw Tower, Inc., et al.
2011- 2012 Labor Law Case Digest Compilations
UC LLB III-B
Page 183

G.R. No. 173291, February


08, 2012
Topic: Applicability Of Laws
On Evidence Of Regular
Courts To Labor Arbiter And
NLRC Proceedings; Illegal
Dismissal
Facts:
Romeo A. Galang filed a
complaint for illegal dismissal
with several money claims,
including
damages
and
attorneys fees, against the
respondents
Cityland
Shaw
Tower, Inc. (Cityland) and its
Building
Manager,
Virgilio
Baldemor. Galang alleged on
compulsory arbitration that after
the
expiration
of
his
employment contracts with the
agencies providing maintenance
services to Cityland, he was
absorbed as a janitor by
Cityland with a promise of
regular employment after the
completion of his six-month
probation. He claimed that even
after the lapse of the period, he
continued working for Cityland
although he had no idea about
his employment status. He did
not know his status for certain
until he was shown a document
on May 21, 2002 informing him
that his employment would be
terminated effective May 20,
2002. On the other hand,
Cityland alleged that it absorbed
Galang as a casual employee
after the expiration of his
contract
with Gayren
Maintenance
Services.
They
alleged
that
during
his
employment with them, they

found him to be remiss in the


performance of his job and he
failed to conduct himself as a
good employee. On May 20,
2002
Cityland
terminated
Galangs services for gross
insubordination
after
a
comprehensive examination of
the accusation.
On compulsory arbitration,
the Labor Arbiter found that
Galang
had
been
illegally
dismissed and that Cityland
failed to present evidence to
support Galangs dismissal for
cause after observance of due
process. The Labor Arbiter
ordered Cityland to immediately
reinstate Galang or if not
feasible, pay separation pay,
backwages,
13th
month
differential,
and
service
incentive leave pay. The claim
for damages and attorneys fees
was denied.
The NLRC affirmed the
Labor Arbiters decision.
On appeal to the CA, the
decision was reversed declaring
that Galang was dismissed for a
just cause. It however ordered
Cityland
to
pay
nominal
damages of P30,000.00 for its
violation of Galangs right to
procedural due process, in
accordance
with Agabon
v.
NLRC. The CA upheld Galangs
dismissal on the strength of
affidavits
of
employees
executed and notarized in 2005
which were not presented
during the proceedings with the
Labor Arbiter and the NLRC.
Issue:
2011- 2012 Labor Law Case Digest Compilations
UC LLB III-B
Page 184

1. Whether the affidavits


of
Citylands
witnesses
constitute new evidence and, as
such, not admissible.
2. Whether the Agabon or
the Serrano doctrine should be
applied in this case.
Ruling:
First Issue. The rules of
evidence prevailing in courts of
law are not controlling in labor
cases. The affidavits were
intended
to
elucidate,
corroborate
or
bolster
the
evidence already presented to
the labor arbiter. Also, Galang
did not present any evidence
denying such claims of Cityland,
hence, the Court is more
inclined to give credence to the
evidence
pointing
to
the
conclusion
that
[Galangs]
employment
was
actually
severed for a just cause.
Second Issue. The Serrano
doctrine provides that failure to
comply
with
the
notice
requirement
in
employee
dismissals for cause entitles the
employee to full backwages
while the Agabon doctrine
provides that nominal damages
should be paid for violation of
procedural due process. The
decision of the NLRC did not
attain finality as it was brought
to the CA on a petition for
certiorari and was overturned.
Galang simply did not have the
benefit of any final arbiter or
NLRC
decision
to
which
the Serrano ruling
could
be
applied. When the CA ruled on
the case, this Court had

abandoned the Serrano doctrine


in favor of Agabon. Thus, the CA
committed
no
error
in
applying Agabon to the case.

104. Negros Slashers, Inc.,


et al. vs. Alvin Teng
G.R. No. 187122, February
22, 2012
Topic: Perfection
of
Appeal/Liberality Of Rules;
Forum
Shopping;
When
Dismissal Is Deemed Too
Harsh
Facts:
Alvin
Teng
is
a
professional basketball player
who played in the played in the
Metropolitan
Basketball
Association (MBA). On February
4, 1999, Teng signed a 3-year
contract
with
the
Laguna
Lakers. Before the expiration of
his contract the Lakers traded
and/or transferred Teng to
petitioner Negros Slashers, with
the
latter
assuming
the
obligations of Laguna Lakers
under
Tengs
unexpired
contract. On Game Number 4 of
the MBA Championship Round
for the year 2000 season, Teng
had
a
below-par
playing
performance. Because of this,
the coaching staff decided to
pull him out of the game. Teng
then sat on the bench, untied
his shoelaces and donned his
practice jersey. On the following
game, Game Number 5 of the
Championship
Round,
Teng
called-in sick and did not play.
2011- 2012 Labor Law Case Digest Compilations
UC LLB III-B
Page 185

On November 21, 2000, Vicente


Tan, Finance Head of Negros
Slashers, wrote Teng requiring
him to explain in writing why no
disciplinary action should be
taken against him for his
precipitated absence during the
crucial Game 5 of the National
Championship Round. A formal
investigation was conducted by
Negros Slashers which included
some hearings. On March 16,
2001, the management of
Negros Slashers came up with a
decision,
and
through
its
General Manager,
petitioner
Rodolfo
Alvarez,
wrote Teng
informing him of his termination
from the team. On July 28, 2001,
Teng filed a complaint before
the Office of the Commissioner
of the MBA pursuant to the
provision of the Uniform Players
Contract which the parties had
executed.
Subsequently, on
November 6, 2001, Teng also
filed an illegal dismissal case
with the Regional Arbitration
Branch No. VI. The Labor Arbiter
issued a decision finding Tengs
dismissal illegal and ordering
petitioner Negros Slashers, Inc.
to
pay
Teng
P2,530,000
representing his unpaid salaries,
separation pay and attorneys
fees. The Labor Arbiter ruled
that the penalty of dismissal
was not justified since the
grounds
relied
upon
by
petitioners did not constitute
serious misconduct or willful
disobedience or insubordination
that would call for the extreme
penalty
of
dismissal
from
service.

On appeal to the NLRC the


decision was set aside and the
complaint was dismissed for
being premature since the
arbitration proceedings before
the Commissioner of the MBA
were still pending when Teng
filed his complaint for illegal
dismissal. Teng filed a motion for
reconsideration, but it was
denied for being filed beyond
the
ten-day
reglementary
period.
Teng filed a petition for
certiorari with the CA which
abandoned the NLRC decision
and reinstated the decision of
the Labor Arbiter. The Negros
Slashers filed a motion for
reconsideration
which
was
denied.
Issue:
1. Whether the CA erred in
giving due course to respondent
Tengs petition for certiorari
despite its late filing.
2. Whether Teng violated
the rule on forum shopping
when he filed a complaint for
illegal
dismissal
with
the
Regional Arbitration Branch of
the NLRC while a similar
complaint was pending in the
Office of the Commissioner of
the MBA.
3. Whether the CA erred in
ruling that Tengs dismissal from
the Negros Slashers Team was
unjustified
and
too
harsh
considering his misconduct.
Ruling:
2011- 2012 Labor Law Case Digest Compilations
UC LLB III-B
Page 186

First Issue. The CA did not


commit a reversible error in
giving due course to Tengs
petition for certiorari although
said petition was filed late.
Ordinarily, rules of procedure
are strictly enforced by courts in
order to impart stability in the
legal system. However, in not a
few instances, the Court relaxed
the rigid application of the rules
of procedure to afford the
parties the opportunity to fully
ventilate their cases on the
merits. In Ong Lim Sing, Jr. v.
FEB
Leasing
and
Finance
Corporation,the Court ruled:
Courts have the prerogative to
relax procedural rules of even
the most mandatory character,
mindful of the duty to reconcile
both the need to speedily put an
end to litigation and the parties
right to due process the Court
has allowed liberal construction
of the rules when to do so would
serve
the
demands
of
substantial justice and equity.
Furthermore, no intent to delay
the administration of justice
could be attributed to Teng.
Second Issue. There is no
merit in petitioners claim that
respondents act of filing a
complaint with the Labor Arbiter
while the same case was
pending with the Office of the
Commissioner
of
the
MBA
constituted forum shopping. For
forum shopping to exist, it is
necessary that (a) there be
identity of parties or at least
such parties that represent the
same interests in both actions;

(b) there be identity of rights


asserted and relief prayed for,
the relief being founded on the
same facts; and (c) the identity
of the two preceding particulars
is such that any judgment
rendered in one action will,
regardless of which party is
successful,
amount
to res
judicata in the other action. The
third requisite of forum shopping
is missing in this case. Any
judgment or ruling of the Office
of the Commissioner of the MBA
will not amount to res judicata.
The Office of the Commissioner
of the MBA is not a court of
competent
jurisdiction
as
contemplated by law with
respect to the application of the
doctrine of res judicata.
Third Issue. The penalty of
dismissal handed out against
Teng was indeed too harsh.
Infractions committed by an
employee should merit only the
corresponding
penalty
demanded by the circumstance.
The
penalty
must
be
commensurate with the act,
conduct or omission imputed to
the employee and must be
imposed in connection with the
disciplinary authority of the
employer
105. Bitoy Javier vs. Fly Ace
Corporation
G.R. No. 192558, February
15, 2012
Topic:
Employer-Employee
Relationship
Facts:
2011- 2012 Labor Law Case Digest Compilations
UC LLB III-B
Page 187

On May 23, 2008, Javier


filed a complaint before the
NLRC for underpayment of
salaries
and
other
labor
standard benefits. He alleged
that he was an employee of Fly
Ace, performing various tasks at
the respondents warehouse
such as cleaning and arranging
the canned items before their
delivery to certain locations,
except in instances when he
would be ordered to accompany
the companys delivery vehicles,
as pahinante; that he reported
for work from Monday to
Saturday from 7:00 oclock in
the morning to 5:00 oclock in
the afternoon; that during his
employment, he was not issued
an
identification
card
and
payslips by the company; that
on May 6, 2008, he reported for
work but he was no longer
allowed to enter the company
premises by the security guard
upon the instruction of Ruben
Ong (Mr. Ong), his superior. After
several minutes of begging to
the guard to allow him to enter,
he
saw
Ong
whom
he
approached and asked why he
was being barred from entering
the premises; that Ong replied
by saying, Tanungin mo anak
mo; that he then went home
and discussed the matter with
his family; that he discovered
that Ong had been courting his
daughter Annalyn after the two
met at a fiesta celebration in
Malabon
City; that
Annalyn
tried to talk to Ong and
convince him to spare her father
from trouble but he refused to

accede; that thereafter, Javier


was
terminated
from
his
employment
without
notice; and that he was neither
given the opportunity to refute
the cause/s of his dismissal from
work. For its part, Fly Ace
averred that it was engaged in
the business of importation and
sales of groceries. In December
2007, Javier was contracted by
its employee, Mr. Ong, as extra
helper on a pakyaw basis. Mr.
Ong contracted Javier roughly 5
to 6 times only in a month
whenever the vehicle of its
contracted
hauler,
Milmar
Hauling
Services,
was
not
available. Fly Ace submitted a
copy of its agreement with
Milmar Hauling Services and
copies
of
acknowledgment
receipts evidencing payment to
Javier for his contracted services
bearing
the
words,
daily
manpower (pakyaw/piece
rate
pay)
and
the
latters
signatures/initials. The
LA
dismissed the complaint for lack
of merit on the ground that
Javier failed to present proof
that he was a regular employee
of Fly Ace. On appeal with the
NLRC, Javier was favored. It
ruled that the LA skirted the
argument
of
Javier
and
immediately concluded that he
was not a regular employee
simply because he failed to
present proof. It was of the view
that
a pakyaw-basis
arrangement did not preclude
the existence of employeremployee relationship. The CA
annulled the NLRC findings and
2011- 2012 Labor Law Case Digest Compilations
UC LLB III-B
Page 188

reinstated the dismissal of


Javiers complaint as ordered by
the LA. The CA declared that the
facts alleged by Javier did not
pass the control test. Javier
moved for reconsideration but
was denied.
Issue:
Whether there exists an
employer-employee relationship
between Javier and Fly Ace.
Ruling:
The
SC
affirms
the
assailed CA decision. As the
records bear out, the LA and the
CA found Javiers claim of
employment with Fly Ace as
wanting and deficient. Labor
officials are enjoined to use
reasonable means to ascertain
the
facts
speedily
and
objectively with little regard to
technicalities or formalities but
nowhere in the rules are they
provided a license to completely
discount evidence, or the lack of
it. The quantum of proof
required, however, must still be
satisfied.
The onus
probandi falls on petitioner to
establish or substantiate such
claim by the requisite quantum
of evidence. Whoever claims
entitlement to the benefits
provided by law should establish
his or her right thereto x x x.
On Javier lies the burden to pass
the
well-settled
tests
to
determine the existence of an
employer-employee
relationship, viz:
(1)
the
selection and engagement of
the employee; (2) the payment

of wages; (3) the power of


dismissal; and (4) the power to
control
the
employees
conduct. Of these elements, the
most important criterion is
whether the employer controls
or has reserved the right to
control the employee not only
as to the result of the work but
also as to the means and
methods by which the result is
to be accomplished. The Courts
decision does not contradict the
settled rule that payment by
the piece is just a method of
compensation and does not
define the essence of the
relation. Payment on a piecerate basis does not negate
regular employment. The term
wage is broadly defined in
Article 97 of the Labor Code as
remuneration
or
earnings,
capable of being expressed in
terms of money whether fixed or
ascertained on a time, task,
piece or commission basis.
Payment by the piece is just a
method of compensation and
does not define the essence of
the relations.
106. C.F. Sharp and Co., Inc.,
et al. vs. Pioneer Insurance,
et al.
G.R. No. 179469, February
15, 2012
Topic: Awards For Damages
Facts:
Responding
to
a
newspaper advertisement of a
job opening for sandblasters and
painters in Libya, respondents
Wilfredo
C.
Agustin
and
2011- 2012 Labor Law Case Digest Compilations
UC LLB III-B
Page 189

Hernando G. Minimo applied


with C.F. Sharp sometime in
August 1990. After passing the
interview, they were required to
submit
their
passports,
seamans book, National Bureau
of
Investigation
clearance,
employment
certificates,
certificates
of
seminars
attended, and results of medical
examination. Upon submission
of the requirements, a Contract
of Employment was executed
between respondents and C.F.
Sharp. They were then advised
to
prepare
for
immediate
deployment and to report to C.F.
Sharp to ascertain the schedule
of their deployment. After a
month, respondents were yet to
be deployed prompting them to
request for the release of the
documents they had submitted
to
C.F.
Sharp. C.F.
Sharp
allegedly refused to surrender
the documents which led to the
filing
of
a
complaint
by
respondents
before
the
Philippine
Overseas
Employment
Administration
(POEA) on 21 January 1991.
POEA issued an Order finding
C.F. Sharp guilty of violation of
Article 34(k) of the Labor Code,
which makes it unlawful for any
entity to withhold or deny
travel documents from applicant
workers before departure for
monetary
or
financial
considerations... On 10 March
1995,
respondents
filed
a
Complaint for breach of contract
and damages against C.F. Sharp
and its surety, Pioneer Insurance
and Surety Corporation (Pioneer

Insurance), before the Regional


Trial Court (RTC) of Pasay
City. Respondents claimed that
C.F. Sharp falsely assured them
of deployment and that its
refusal to release the disputed
documents on the ground that
they were already bound by
reason of the Contract of
Employment,
denied
respondents
of
employment
opportunities abroad and a
guaranteed
income. Respondents
also
prayed for damages. Pioneer
Insurance filed a cross claim
against C.F. Sharp and John J.
Rocha, the executive vicepresident of C.F. Sharp, based
on an Indemnity Agreement
which substantially provides
that the duo shall jointly and
severally
indemnify
Pioneer
Insurance for damages, losses,
and costs which the latter may
incur as surety. The trial court
ruled that there was a violation
of the contract when C.F. Sharp
failed to deploy and release the
papers
and
documents
of
respondents, hence, they are
entitled to damages. The trial
court likewise upheld the cause
of action of respondents against
Pioneer Insurance, the former
being the actual beneficiaries of
the surety bond. On appeal, C.F.
Sharp and Rocha raise a
jurisdictional issue that the
RTC has no jurisdiction over the
instant case pursuant to Section
4(a) of Executive Order No. 797
which vests upon the POEA the
jurisdiction over all cases,
including money claims, arising
2011- 2012 Labor Law Case Digest Compilations
UC LLB III-B
Page 190

out of or by virtue of any


contract involving workers for
overseas
employment. C.F.
Sharp and Rocha refuted the
findings of the trial court and
maintained that the perfection
and effectivity of the Contract of
Employment depend upon the
actual
deployment
of
respondents. The Court of
Appeals upheld the jurisdiction
of the trial court by ruling that
petitioners are now estopped
from raising such question. The
Court of Appeals further held
that since there is no perfected
employment contract between
the parties, it is the RTC and not
the POEA which has jurisdiction.
Despite the finding that no
contract was perfected between
the parties, the Court of Appeals
adjudged C.F. Sharp and Rocha
liable for damages pursuant to
Article 21 of the Civil Code
amounting to P100,000 in
temperate damages, P100,000
in moral damages, and P50,000
award of exemplary damages .
Rocha filed the instant petition
on the submission that there is
no basis to hold him liable for
damages under Article 21 of the
Civil Code because C.F. Sharp
has signified its intention to
return the documents and had
in fact informed respondents
that they may, at any time of
the business day, withdraw their
documents. Further,
respondents failed to establish
the basis for which they are
entitled
to
moral
damages. Rocha refuted the
award of exemplary damages

because the act of


respondents to sign a
prior to the release
documents
could
considered bad faith.

requiring
quitclaim
of their
not
be

Issue:
Whether there should be
an award for damages.
Ruling:
An employment contract,
like any other contract, is
perfected at the moment (1) the
parties come to agree upon its
terms; and (2) concur in the
essential elements thereof: (a)
consent of the contracting
parties, (b) object certain which
is the subject matter of the
contract and (c) cause of the
obligation. The perfection of the
contract, which in this case
coincided with the date of
execution
thereof,
occurred
when petitioner and respondent
agreed on the object and the
cause, as well as the rest of the
terms and conditions therein.
The commencement of the
employer-employee relationship,
as earlier discussed, would have
taken place had petitioner been
actually deployed from the point
of hire. Thus, even before the
start of any employer-employee
relationship, contemporaneous
with the perfection of the
employment contract was the
birth of certain rights and
obligations, the breach of which
may give rise to a cause of
action against the erring party.
C.F.
Sharp
committed
an
actionable
wrong
when
it
2011- 2012 Labor Law Case Digest Compilations
UC LLB III-B
Page 191

unreasonably
withheld
documents,
thus
preventing
respondents
from
seeking
lucrative
employment
elsewhere. That
C.F.
Sharp
arbitrarily imposed a condition
that the documents would only
be released upon signing of a
quitclaim is tantamount to bad
faith because it effectively
deprived respondents of resort
to legal remedies.
The SC likewise affirms
the
award
of
exemplary
damages
and
attorneys
fees. Exemplary damages may
be awarded when a wrongful act
is accompanied by bad faith or
when the defendant acted in a
wanton, fraudulent, reckless,
oppressive,
or
malevolent
manner which would justify an
award of exemplary damages
under Article 2232 of the Civil
Code. Since
the
award
of
exemplary damages is proper in
this case, attorneys fees and
cost of the suit may also be
recovered as provided under
Article 2208 of the Civil Code.
107. LYNVIL FISHING
ENTERPRISES, INC. VS.
ANDRES ARIOLA, ET AL.
G.R. No. 181974, February
01, 2012
Topic: Finding Of Probable
Cause By A Prosecutor As
Ground
For
Dismissal;
Regular Employees; Labor
Due
Process;
Solidary
Liability Of A Corporate
Officer
Facts:

Lynvil Fishing Enterprises,


Inc. (Lynvil) is a company
engaged in deep-sea fishing,
operating along the shores of
Palawan and other outlying
islands
of
the
Philippines.
According to Lynvil, on 1 August
1998, it received a report that
the respondents conspired with
one another and stole eight (8)
tubs
of
"pampano"
and
"tangigue" fish and delivered
them to another vessel, to the
prejudice of Lynvil. The said
employees were engaged on a
per trip basis or "por viaje"
which terminates at the end of
each trip. By reason of the
report
and
after
initial
investigation, Lynvil's General
Manager Rosendo S. De Borja
(De
Borja)
summoned
respondents to explain within
five (5) days why they should
not
be
dismissed
from
service. Failing to explain as
required,
respondents'
employment was terminated.
Lynvil filed a criminal complaint
against
the
dismissed
employees for violation of P.D.
532, or the Anti-Piracy and AntiHighway Robbery Law of 1974.
The Assistant City Prosecutor
found probable cause for the
indictment of the dismissed
employees for the crime of
qualified theft. On the other
hand, respondents allege that
on 31 July 1998, they arrived at
the Navotas Fishport on board
Analyn
VIII
loaded
with
1,241 baeras of different kinds
of fishes. These baeras were
delivered to a consignee named
2011- 2012 Labor Law Case Digest Compilations
UC LLB III-B
Page 192

SAS
and
Royale.
The following day, the private
respondents reported back to
Lynvil office to inquire about
their new job assignment but
were told to wait for further
advice. They were not allowed
to
board
any
vessel.
Respondents,
according
to
them, then went to the Lynvil
office where they learned of
their termination. Aggrieved, the
employees
filed
with
the
Arbitration
Branch
of
the
National
Labor
Relations
Commission a complaint for
illegal dismissal with claims for
backwages, salary differential
reinstatement, service incentive
leave, holiday pay and its
premium and 13th month pay
from 1996 to1998. They also
claimed for moral, exemplary
damages and attorney's fees for
their dismissal with bad faith.
The Labor Arbiter found merit in
complainants' charge of illegal
dismissal. The Labor Arbiter
also ruled that the contractual
provision that the employment
terminates upon the end of each
trip
does
not
make
the
respondents' dismissal legal.
The Labor Arbiter found that the
procedural due process was not
complied with. On appeal before
the National Labor Relations
Commission, NLRC reversed and
set aside the Decision of the
Labor Arbiter. The Court of
Appeals found merit in the
petition and reinstated the
Decision of the Labor Arbiter. It
also reversed the finding of the
NLRC
that
the
dismissed

employees
were
merely
contractual
employees
and
added that they were regular
ones performing activities which
are
usually
necessary
or
desirable in the business and
trade of Lynvil. It ruled that the
two-notice rule provided by law
and jurisprudence is mandatory
and non-compliance therewith
rendered the dismissal of the
employees illegal.
Issues:
1. Whether the CA erred in
failing to consider the doctrine
in Nasipit Lumberholding that
the filing of a criminal case
constitutes sufficient basis for a
valid termination of employment
on the grounds of serious
misconduct or loss of trust and
confidence.
2.
Whether
the
respondents
are
merely
contractual employees.
3.
Whether
the
respondents
were
accorded
procedural due process.
4.
Whether
the
respondents are entitled to their
money claims.
5. Whether the corporate
officer, Borja, is solidarily liable
there being no bad faith.
Ruling:
First Issue. Lynvil cannot
argue that since the Office of
the Prosecutor found probable
cause for theft the Labor Arbiter
must follow the finding as a
valid reason for the termination
of respondents' employment.
However, there was valid cause
2011- 2012 Labor Law Case Digest Compilations
UC LLB III-B
Page 193

for respondents' dismissal. In


illegal dismissal cases, the
employer bears the burden of
proving that the termination was
for a valid or authorized cause.
Loss of confidence as a just
cause
for
termination
of
employment is premised on the
fact
that
the
employee
concerned holds a position of
responsibility,
trust
and
confidence or that the employee
concerned is entrusted with
confidence with respect to
delicate matters, such as the
handling or care and protection
of the property and assets of the
employer. Breach of trust is
present in this case.
Second Issue. They are
regular employees. Art. 280 of
the Labor Code provides that
an
employment
shall
be
deemed to be regular where the
employee has been engaged to
perform activities which are
usually necessary or desirable in
the usual business or trade of
the employer, except where the
employment has been fixed for
a specific project or undertaking
the completion or termination of
which has been determined at
the time of the engagement of
the employee or where the work
or service to be performed is
seasonal in nature and the
employment is for the duration
of the season The purpose of
the provision is to prevent
circumvention of the employee's
right to be secure in his tenure.
The agreement of the parties
continued for more than ten

years, the clear intention is to


go around the security of tenure
of the respondents as regular
employees.
Third Issue. No. It is
required that the employer
furnish the employee with two
written notices: (1) a written
notice served on the employee
specifying
the
ground
or
grounds for termination, and
giving
to
said
employee
reasonable opportunity within
which to explain his side; and
(2)
a
written
notice
of
termination served on the
employee indicating that upon
due consideration of all the
circumstances, grounds have
been established to justify his
termination. From the records,
there was only one written
notice. No final written notice or
notices of termination were sent
to the employees. The SC
awarded P50,000 as nominal
damages for failure to follow
such procedural requirement.
Fourth Issue. Given the
fact that their dismissal was for
just cause, backwages and
separation
pay
cannot
be
granted.
However,
the
SC
granted 13th month pay and
salary
differential
of
the
dismissed employees.
Fifth Issue. In labor cases,
the corporate directors and
officers are solidarily liable with
the
corporation
for
the
termination of employment of
employees done with malice or
in bad faith. Bad faith being
2011- 2012 Labor Law Case Digest Compilations
UC LLB III-B
Page 194

absent, Borja is not solidarily


liable.
108. Garden of Memories
Park and Life Plan, Inc. and
Paulina Requino vs. NLRC, et
al.
G.R. No. 160278, February
08, 2012
Topic:
Labor-Only
Contracting;
EmployerEmployee
Relationship;
Abandonment
Facts:
Petitioner
Garden
of
Memories is engaged in the
business
of
operating
a
memorial park situated at
Pateros,
Metro-Manila
and
selling
memorial
Plan and
services.
Respondent
Cruz
worked at the Garden of
Memories Memorial Park as a
utility worker from August 1991
until her termination in February
1998. On March 13, 1998, Cruz
filed a complaint for illegal
dismissal,
underpayment
of
wages, non-inclusion in the
Social Security Services, and
non-payment of legal/special
holiday, premium pay for rest
day, 13th month pay and
service incentive leave pay
against Garden of Memories
before the Department of Labor
and Employment (DOLE).Upon
motion of Garden of Memories,
Requio was impleaded as
respondent on the alleged
ground that she was its service
contractor and the employer of
Cruz. Cruz averred that she
worked as a utility worker of

Garden of Memories with a


salary of P115.00 per day. As a
utility worker, she was in
charge, among others, of the
cleaning and maintenance of
the ground facilities of the
memorial park. She had a
misunderstanding with a coworker regarding the use of a
garden water hose. When the
misunderstanding came to the
knowledge of Requio, the latter
instructed them to go home and
not to return anymore. After
three (3) days, Cruz reported for
work but she was told that she
had been replaced by another
worker. Cruz argued that as a
regular employee of the Garden
of Memories, she could not be
terminated without just or valid
cause. Garden of Memories
denied liability for the claims of
Cruz and asserted that she was
not its employee but that of
Requio, its independent service
contractor, who maintained the
park for a contract price. The LA
ruled that Requio was not an
independent contractor but a
labor-only contractor and that
her
defense
that
Cruz
abandoned
her
work
was
negated by the filing of the
present case. The LA declared
both Garden of Memories and
Requio, jointly and severally,
liable for the monetary claims of
Cruz. The NLRC affirmed the
ruling of the LA, stating that
Requio had no substantial
capital or investments in the
form
of
tools,
equipment,
machineries,
and
work
premises, among others, for her
2011- 2012 Labor Law Case Digest Compilations
UC LLB III-B
Page 195

to qualify as an independent
contractor. It declared the
dismissal
of
Cruz
illegal
reasoning out that there could
be no abandonment of work on
her part since Garden of
Memories and Requio failed to
prove
that
there
was
a
deliberate
and
unjustified
refusal on the part of the
employee to go back to work
and resume her employment.
The NLRC denied the motion for
reconsideration of Garden of
Memories. On appeal to the CA,
the decision of the NLRC was
affirmed.
Issues:
1. Whether Requino is
engaged
in
labor-only
contracting.
2.
Whether
there
is
employer-employee relationship
between Garden of Memories
and Cruz.
3.
Whether
Cruz
abandoned her work.
Ruling:
First and Second Issue.
Requino is engaged in labor-only
contracting. There is laboronly contracting where the
person supplying workers to an
employer
does
not
have
substantial capital or investment
in the form of tools, equipment,
machineries, work premises,
among others, and the workers
recruited and placed by such
persons
are
performing
activities which are directly
related to the principal business
of such employer. Aside from

the capitalization requirement,


the power of control on the part
of Requio is also wanting. the
presumption
is
that
the
contractor
is
a
labor-only
contracting
unless
such
contractor
overcomes
the
burden of proving that it has the
substantial capital, investment,
tools and the like. In the present
case,
though
Garden
of
Memories is not the contractor,
it has the burden of proving that
Requio has sufficient capital or
investment since it is claiming
the supposed status of Requio
as
independent
contractor.
Garden of Memories, however,
failed
to
adduce evidence
purporting to show that Requio
had sufficient capitalization.
Requio is engaged in
labor-only contracting, and is
considered merely an agent of
Garden of Memories. As such,
the workers she supplies should
be considered as employees of
Garden
of
Memories.
Consequently, the latter, as
principal
employer,
is
responsible to the employees of
the labor-only contractor as if
such employees have been
directly employed by it.
Third Issue. Cruz did not
abandon her work. Garden of
Memories has the burden of
proof to show the employee's
deliberate
and
unjustified
refusal
to
resume
his
employment
without
any
intention
of
returning.
For
abandonment to exist, two
factors must be present: (1) the
2011- 2012 Labor Law Case Digest Compilations
UC LLB III-B
Page 196

failure to report for work or


absence
without
valid
or
justifiable reason; and (2) a
clear
intention
to
sever
employer-employee relationship,
with the second element as the
more determinative factor being
manifested by some overt acts.
It
has
been
said
that
abandonment of position cannot
be lightly inferred, much less
legally presumed from certain
equivocal acts.Mere absence is
not
sufficient.
109.FEDIRICO
S.
ROBOSA,
ET
AL.
VS
NATIONAL LABOR RELATIONS
COMMISSION ET AL.
G.R. NO. 176085. FEBRUARY
08, 2012
Topic: NLRCs Power To Cite
A Person In Contempt
Facts:
Federico S. Robosa,
Rolando E. Pandy, Noel D.
Roxas,
Alexander
Angeles,
Veronica Gutierrez, Fernando
Embat and Nanette H. Pinto
(petitioners) were rank-and-file
employees
of
respondent
Chemo-Technische
Manufacturing, Inc. (CTMI), the
manufacturer and distributor of
Wella products. They were
officers and members of the
CTMI
Employees
Union-DFA
(union). Respondent Procter and
Gamble Philippines, Inc. (P &
GPI) acquired all the interests,
franchises and goodwill of CTMI
during the pendency of the
dispute. Sometime in the first
semester of 1991, the union

filed a petition for certification


election at CTMI. On June 10,
1991,
Med-Arbiter
Rasidali
Abdullah of the Office of the
Department
of
Labor
and
Employment in the National
Capital
Region
(DOLE-NCR)
granted the petition. The DOLENCR
conducted
a
consent
election on July 5, 1991, but the
union failed to garner the votes
required to be certified as the
exclusive bargaining agent of
the company. On July 15, 1991,
CTMI, through its President and
General Manager Franklin R. de
Luzuriaga,
issued
a
memorandum.
On the same
day,
CTMI
issued
another
memorandum informing
the
companys sales representatives
and sales drivers of the new
system in the Salon Business
Groups selling operations. The
union asked for the withdrawal
and
deferment
of
CTMIs
directives, branding them as
union busting acts constituting
unfair labor practice. CTMI
ignored the request. Instead, it
issued on July 23, 1991 a notice
of termination of employment to
the sales drivers, due to the
abolition of the sales driver
positions.
On August 1, 1991, the
union and its affected members
filed a complaint for illegal
dismissal
and
unfair
labor
practice, with a claim for
damages, against CTMI, De
Luzuriaga
and
other
CTMI
officers. The union also moved
for the issuance of a writ of
preliminary injunction and/or
2011- 2012 Labor Law Case Digest Compilations
UC LLB III-B
Page 197

temporary
restraining
order
(TRO).
The
labor
arbiter
handling the case denied the
unions motion for a stay order
on the ground that the issues
raised by the petitioners can
best be ventilated during the
trial on the merits of the case.
This prompted the union to file
on August 16, 1991 with the
National
Labor
Relations
Commission (NLRC), a petition
for the issuance of a preliminary
mandatory injunction and/or
TRO. On August 23, 1991, the
NLRC issued a TRO.
Issue:
1. Whether the NLRC has
contempt powers
2. Whether the dismissal
of a contempt charge is
appealable
3. Whether the NLRC
committed grave abuse of
discretion in dismissing the
contempt charge against the
respondents.
Ruling:
First, Second and Third
Issue.
We
note
that
the
petitioners, in assailing the CA
main decision, claim that the
appellate
court
committed
grave abuse of discretion in not
ruling on the dismissal by the
NLRC of the contempt charges.
They also charge the NLRC of
having
gravely
abused
its
discretion
and
having
committed reversible errors. In
view of the grave abuse of
discretion allegation in this case,
we deem it necessary to look

into the NLRCs dismissal of the


contempt charges against the
respondents. As the charges
were
rooted
into
the
respondents
alleged
noncompliance with the NLRC
directives contained in the TRO
and the writ of preliminary
injunction, we first inquire into
what really happened to these
directives. An act of a court or
tribunal may only be considered
as committed in grave abuse of
discretion
when
it
was
performed in a capricious or
whimsical exercise of judgment
which is equivalent to lack of
jurisdiction.
The
abuse
of
discretion must be so patent
and gross as to amount to an
evasion of a positive duty
enjoined by law, or to act at all
in contemplation of law, as
where the power is exercised in
an
arbitrary
and
despotic
manner by reason of passion or
personal hostility.
It rightly
avoided delving into issues
which would clearly be in excess
of its jurisdiction for they are
issues involving the merits of
the case which are by law within
the
original
and
exclusive
jurisdiction of the labor arbiter.
To be sure, whether payroll
reinstatement of some of the
petitioners is proper; whether
the resignation of some of them
was compelled by dire economic
necessity;
whether
the
petitioners are entitled to their
money claims; and whether
quitclaims are contrary to law or
public policy are issues that
should be heard by the labor
2011- 2012 Labor Law Case Digest Compilations
UC LLB III-B
Page 198

arbiter in the first instance. The


NLRC can inquire into them only
on appeal after the merits of the
case
shall
have
been
adjudicated by the labor arbiter.
The NLRC correctly dismissed
the contempt charges against
the
respondents.
The
CA
likewise committed no grave
abuse of discretion in not
disturbing the NLRC resolution.
In light of the above discussion,
we find no need to dwell into the
other issues the parties raised.
110. CANADIAN
OPPORTUNITIES UNLIMITED,
INC. VS. BART Q. DALANGIN,
JR.
G.R. NO. 172223. FEBRUARY
06, 2012
Topic: Illegal Dismissal
Facts:
On November 20, 2001,
respondent Bart Q. Dalangin, Jr.
filed a complaint for illegal
dismissal,
with
prayer
for
reinstatement and backwages,
as well as damages (moral and
exemplary) and attorneys fees,
against
petitioner
Canadian
Opportunities Unlimited, Inc.
(company).
The
company,
based in Pasong Tamo, Makati
City, provides assistance and
related services to applicants for
permanent residence in Canada.
Dalangin was hired by
the company only in the
previous month, or in October
2001, as Immigration and Legal
Manager, with a monthly salary
of P15,000.00. He was placed on

probation for six months. He


was to report directly to the
Chief Operations Officer, Annie
Llamanzares Abad. His tasks
involved principally the review
of the clients applications for
immigration
to
Canada
to
ensure
that
they
are
in
accordance with Canadian and
Philippine laws.
Through a memorandum
dated October 27, 2001, signed
by
Abad,
the
company
terminated
Dalangins
employment,
declaring
him
unfit and unqualified to
continue as Immigration and
Legal Manager. In his decision
dated April 23, 2003, Labor
Arbiter Eduardo G. Magno
declared Dalangins dismissal
illegal,
and
awarded
him
backwages of P75,000.00, moral
damages of P50,000.00 and
exemplary
damages
of
P50,000.00, plus 10% attorneys
fees. The labor arbiter found
that
the
charges
against
Dalangin, which led to his
dismissal, were not established
by clear and substantial proof.
On appeal by the
company, the National Labor
Relations Commission (NLRC)
rendered a decision on March
26, 2004 granting the appeal,
thereby reversing the labor
arbiters
ruling.
It
found
Dalangins dismissal to be a
valid exercise of the companys
management
prerogative
because Dalangin failed to meet
the
standards
for
regular
employment. Dalangin moved
for reconsideration, but the
2011- 2012 Labor Law Case Digest Compilations
UC LLB III-B
Page 199

NLRC
denied
the
motion,
prompting him to go to the CA
on a petition for certiorari under
Rule 65 of the Rules of Court.
In its now assailed decision, the
CA held that the NLRC erred
when it ruled that Dalangin was
not illegally dismissed. As the
labor arbiter did, the CA found
that the company failed to
support,
with
substantial
evidence, its claim that Dalangin
failed to meet the standards to
qualify as a regular employee.
Issue:
Whether
Dalangin,
a
probationary employee, was
validly dismissed.
Ruling:
Dalangin was barely a
month on the job when the
company
terminated
his
employment. He was found
wanting in qualities that would
make him a proper and
efficient employee or, as the
company put it, he was unfit
and unqualified to continue as
its
Immigration
and
Legal
Manager. Dalangins dismissal
was viewed differently by the
NLRC and the CA. The NLRC
upheld the dismissal as it was, it
declared, in the exercise of the
companys
management
prerogative. On the other hand,
the CA found that the dismissal
was
not
supported
by
substantial evidence and that
the company did not allow
Dalangin to prove that he had
the qualifications to meet the
companys standards for his

regular employment. The CA


did
not
believe that the
company could fully assess
Dalangins performance within a
month. It viewed Dalangins
dismissal
as
arbitrary,
considering that the company
had very little time to determine
his fitness for the job. The
essence of a probationary
period
of
employment
fundamentally
lies
in
the
purpose or objective of both the
employer and the employee
during the period. While the
employer observes the fitness,
propriety and efficiency of a
probationer
to
ascertain
whether he is qualified for
permanent employment, the
latter seeks to prove to the
former
that
he
has
the
qualifications
to
meet
the
reasonable
standards
for
permanent employment. The
trial period or the length of
time the probationary employee
remains on probation depends
on the parties agreement, but it
shall not exceed six (6) months
under Article 281 of the Labor
Code, unless it is covered by an
apprenticeship
agreement
stipulating a longer period. We,
therefore, disagree with the CA
that the company could not
have
fully
determined
Dalangins performance barely
one month into his employment.
However, the companys noncompliance with the notice
requirement entitles Dalangin to
indemnity, in the form of
nominal damages in an amount
subject to our discretion. Under
2011- 2012 Labor Law Case Digest Compilations
UC LLB III-B
Page 200

the circumstances, we consider


appropriate an award of nominal
damages of P10,000.00 to
Dalangin.
111. BANK OF LUBAO, INC.
VS. ROMMEL J. MANABAT, ET
AL.
G.R. NO. 188722. FEBRUARY
01, 2012
Topic: Separation Pay
Facts:
Sometime in 2001, Rommel J.
Manabat (respondent) was hired
by petitioner Bank of Lubao, a
rural
bank,
as
a
Market
Collector. In November 2004,
an initial audit on the Bank of
Lubaos Sta. Cruz Extension
Office
conducted
by
the
petitioner revealed that there
was a misappropriation of funds
in the amount of P3,000,000.00,
more or less. On November 17,
2004, the respondent, through a
memorandum sent by the
petitioner, was asked to explain
in writing the discrepancies that
were discovered during the
audit. On December 11, 2004,
an administrative hearing was
conducted
by
the
banks
investigating committee where
the respondent was further
made to explain his side.
Subsequently, the investigating
committee concluded that the
respondent
conspired
with
Lingad in making fraudulent
entries
disguised
as
error
corrections
in
the
banks
computer. On August 9, 2005,
the petitioner filed several
criminal complaints for qualified

theft against Lingad and the


respondent with the Municipal
Trial Court (MTC) of Lubao,
Pampanga. Thereafter, citing
serious misconduct tantamount
to willful breach of trust as
ground,
it
terminated
the
respondents
employment
effective September 1, 2005.
On September 26, 2005,
the respondent filed a Complaint
for illegal dismissal with the
Regional Arbitration Branch of
the National Labor Relations
Commission (NLRC) in San
Fernando City, Pampanga. In the
said complaint, the respondent,
to bolster his claim that there
was no valid ground for his
dismissal, averred that the
charge against him for qualified
theft was dismissed for lack of
sufficient basis to conclude that
he conspired with Lingad. The
respondent sought an award for
separation pay, full backwages,
13th month pay for 2004 and
moral and exemplary damages.
On February 28, 2007,
the Labor Arbiter (LA) rendered
a
decision
sustaining
the
respondents claim of illegal
dismissal thus ordering the
petitioner
to
reinstate
the
respondent
to
his
former
position and awarding the latter
backwages in the amount of
P111,960.00 and 13th month pay
in the amount of P6,220.00. The
petitioner
appealed
the
foregoing disposition to the
NLRC, submitting a new audit
report dated April 30, 2007.
Pending appeal, the petitioner
sent the respondent a letter
2011- 2012 Labor Law Case Digest Compilations
UC LLB III-B
Page 201

dated April 30, 2007 requiring


him to report for work on May 4,
2007
pursuant
to
the
reinstatement order of the LA.
The said letter was served to
the respondent on May 3, 2007
but he refused to receive the
same. On July 21, 2008, the
NLRC rendered a Decision
affirming the February 28, 2007
Decision of the LA. The NLRC
held that it was sufficiently
established that only Lingad was
the one responsible for the said
misappropriations. Further, the
NLRC asserted that the February
14, 2006 and April 30, 2007
audit reports presented by the
petitioner could not be given
evidentiary weight as the same
were
executed
after
the
respondent had already been
dismissed. The petitioner sought
reconsideration of the said July
21, 2008 Decision but it was
denied by the NLRC in its
Resolution dated September 22,
2009.
Issues:
1.Whether the CA erred in
ordering the petitioner to pay
the respondent separation pay
in lieu of reinstatement
2.Whether the respondent
is entitled to payment of
backwages.
Ruling:
First
Issue.This
Court
notes that the LA, the NLRC and
the CA unanimously ruled that
the respondent was illegally
dismissed. Factual findings of
quasi-judicial bodies like the

NLRC,
if
supported
by
substantial
evidence,
are
accorded respect and even
finality by this Court, more so
when they coincide with those
of the LA. Such factual findings
are given more weight when the
same are affirmed by the CA.
We find no reason to depart
from the foregoing rule. At the
outset, it should be stressed
that a determination of the
applicability of the doctrine of
strained relations is essentially a
factual question and, thus, not a
proper subject in the instant
petition.
Under
the
law
and
prevailing
jurisprudence,
an
illegally dismissed employee is
entitled to reinstatement as a
matter of right. However, if
reinstatement
would
only
exacerbate the tension and
strained relations between the
parties,
or
where
the
relationship
between
the
employer and the employee has
been unduly strained by reason
of
their
irreconcilable
differences, particularly where
the illegally dismissed employee
held a managerial or key
position in the company, it
would be more prudent to order
payment of separation pay
instead of reinstatement.
Second
Issue.
Yes.
The
arguments
raised
by
the
petitioner with regard to the
issue of backwages, essentially,
attacks the factual findings of
the CA, the NLRC and the LA. As
stated earlier, subject to well2011- 2012 Labor Law Case Digest Compilations
UC LLB III-B
Page 202

defined
exceptions,
factual
questions may not be raised in a
petition for review on certiorari
under Rule 45 as this Court is
not a trier of facts. The
petitioner failed to assert any
circumstance which would impel
this Court to disregard the
findings of fact of the lower
tribunals on the propriety of the
award of backwages in favor of
the respondent. However, the
backwages that should be
awarded to the respondent
should be modified. Employees
who are illegally dismissed are
entitled to full backwages,
inclusive of allowances and
other benefits or their monetary
equivalent, computed from the
time their actual compensation
was withheld from them up to
the
time
of
their
actual
reinstatement.
But
if
reinstatement is no longer
possible, the backwages shall be
computed from the time of their
illegal termination up to the
finality of the decision.
112. BRIGHT MARITIME
CORPORATION
(BMC)/DESIREE P. TENORIO
VS. RICARDO B. FANTONIAL.
G.R. NO. 165935. FEBRUARY
08, 2012
Topic:Employer-Employee
Relationship;
Evidence
Needed To Prove Disability
Of
Migrant
Workers
Deployed In Vessels
Facts:
On January 15, 2000, a
Contract of Employment was

executed by petitioner Bright


Maritime Corporation (BMC),
andrespondent
Ricardo
B.
Fantonial, which contract was
verified and approved by the
Philippine
Overseas
Employment
Administration
(POEA). Respondent was made
to
undergo
a
medical
examination at the Christian
Medical
Clinic,
which
was
petitioners accredited medical
clinic. Respondent was issued a
Medical
Certificate
dated
January
17,
2000,
which
certificate had the phrase FIT
TO WORK stamped on its lower
and upper portion. But the
respondent was not allowed to
leave the country on the ground
that he is unfit to work.
Respondent
went
to
petitioners
office
for
an
explanation, but he was merely
told to wait for their call, as he
was being lined-up for a flight to
the ship's next port of call.
However, respondent never got
a call from petitioners. On May
16, 2000, respondent filed a
complaint against petitioners for
illegal dismissal, payment of
salaries
for
the
unexpired
portion of the employment
contract and for the award of
moral, exemplary, and actual
damages as well as attorneys
fees
before
the
Regional
Arbitration Branch No. 7 of the
NLRC in Cebu City. Petitioners
argued that they cannot be held
liable for illegal dismissal as the
contract of employment had not
yet commenced based on
Section 2 of the Standard Terms
2011- 2012 Labor Law Case Digest Compilations
UC LLB III-B
Page 203

and Conditions Governing the


Employment
of
Filipino
Seafarers on Board Ocean-Going
Vessels (POEA Memorandum
Circular
No.
055-96).
On
September 25, 2000, Labor
Arbiter
Ernesto
F.
Carreon
rendered a Decision in favor of
respondent. On May 31, 2001,
the NLRC, Fourth Division,
rendered a Decision reversing
the decision of the Labor Arbiter.
The NLRC held that the
affidavit of Dr. Lyn dela Cruz-De
Leon proved that respondent
was declared fit to work only on
January 21, 2000, when the
vessel was no longer at the port
of
Germany.
Hence,
respondents failure to depart
on January 17, 2000 to join the
vessel M/V AUK in Germany was
due to respondents health. The
NLRC
stated
that
as
a
recruitment agency, petitioner
BMC has to protect its name and
goodwill, so that it must ensure
that
an
applicant
for
employment abroad is both
technically
equipped
and
physically fit because a labor
contract affects public interest.
Respondent filed a motion for
reconsideration of the NLRC
decision, which motion was
denied in a Resolution dated July
23, 2001. Respondent filed a
petition for certiorari before the
Court of Appeals, alleging that
the NLRC committed grave
abuse of discretion in rendering
the Decision dated May 31,
2001and the Resolution dated
July 23, 2001. The Court of
Appeals held that the NLRC,

Fourth Division, acted with


grave abuse of discretion in
reversing the decision of the
Labor Arbiter who found that
respondent
was
illegally
dismissed.
Issues:
1. Whether there is an
employer employee relationship
2.
Whether
the
respondent is fit to work
Ruling:
First and Second Issue.The
Court has carefully reviewed the
records of the case, and agrees
with the Court of Appeals that
respondents Medical Certificate
dated
January
17,
2000,
stamped with the words FIT TO
WORK, proves that respondent
was medically fit to leave Manila
on January 17, 2000 to join the
vessel M/V AUK in Germany.
The Affidavit of Dr. Lyn dela
Cruz-De Leon that respondent
was declared fit to work only on
January
21,
2000
cannot
overcome the evidence in the
Medical
Certificate
dated
January 17, 2000, which already
stated that respondent had
Class-B
Non-Infectious
Hepatitis-B, and that he was fit
to work. The explanation given
by Dr. Lyn dela Cruz-De Leon in
her affidavit that the Medical
Certificate was dated January
17, 2000, since it carries the
date when they started to
examine
the
patient
per
standard operating procedure,
does not persuade as it goes
against
logic
and
the
2011- 2012 Labor Law Case Digest Compilations
UC LLB III-B
Page 204

chronological
recording
of
medical procedures. The Medical
Certificate
submitted
as
documentary evidence is proof
of its contents, including the
date thereof which states that
respondent
was
already
declared fit to work on January
17, 2000, the date of his
scheduled deployment. Hence,
in this case, the employment
contract was perfected on
January 15, 2000 when it was
signed
by
the
parties,
respondent and petitioners, who
entered into the contract in
behalf of their principal, Ranger
Marine S.A., thereby signifying
their consent to the terms and
conditions
of
employment
embodied in the contract, and
the contract was approved by
the POEA on January 17, 2000.
However,
the
employment
contract did not commence,
since petitioners did not allow
respondent to leave on January
17, 2000 to embark the vessel
M/V AUK in Germany on the
ground that he was not yet
declared fit to work on the day
of departure, although his
Medical
Certificate
dated
January 17, 2000 proved that
respondent was fit to work.
113. MA. MELISSA A.
GALANG VS. JULIA
MALASUGI.
G.R. NO. 174173. MARCH 07,
2012
Topic:
Employer-Employee
Relationship;
Constructive
Dismissal

Facts:
On 26 June 1993, Julia
Malasugui (Malasugui) was hired
by Ma. Melissa A. Galang
(Galang) to take care, oversee
and man the premises of the
Davao Royal Garden Compound
(Pangi Property) the main
compound of Galang where the
orchids and other ornamental
plants used for the business
were nursed and propagated.
Aside from taking care of the
plants, she was required by
Galang to be present at the
premises at seven thirty in the
morning until five thirty in the
afternoon every day, including
Saturdays,
Sundays
and
Holidays without any day-offs.
Petitioner Galang narrated
that she is the owner of Davao
Royal
Garden,
a
sole
proprietorship engaged in the
retailing of ornamental plants,
consisting of receiving of cutflowers
from
farmers
or
suppliers, packing them for
shipment, and shipping them to
the buyers. However, Galang
did
not
hire
respondent
Malasugui. Sometime in 1995,
Malasugui
visited
Solis,
a
relative by affinity, in the Pangi
property. She told Solis of her
intention to find a job in the city
but she had no place to stay in
the meantime.
There was no
need for another employee
since Solis was already taking
care of Elsas orchid collection
and Galangs orchid business.
However,
Malasugui
was
allowed
to
stay
in
the
bunkhouse occupied by Solis.
2011- 2012 Labor Law Case Digest Compilations
UC LLB III-B
Page 205

During these times, Malasugui


did not look for any job as
initially intended because she
was welcomed to stay in the
Galng premises.
They did not require
Malasugui to pay for rentals,
electricity, water and other
utilities. Solis, on the other
hand, asked Malasugui to help
out in her tasks of weeding,
watering, spraying chemicals on
the orchids as well as cleaning
the Pangi property. Malasugui
left without any apparent reason
and thereafter, Malasugui filed a
complaint for illegal dismissal
before
the
National
Labor
Relations Commission, Regional
Arbitration Branch No. XI of
Davao City on 8 February 1999
claiming
underpayment
of
wages, holiday pay, separation
pay and 13th month differential.
Galang appealed before
the NLRC assailing the finding of
the Labor Arbiter that there was
an
employer-employee
relationship between her and
Malasugui, the NLRC granted
the
appeal.
Aggrieved,
Malasugui filed a Special Civil
Action for Certiorari under Rule
65 of the Revised Rules of Civil
Procedure
before
the
CA
alleging
grave
abuse
of
discretion on the part of NLRC.
The CA granted the petition filed
by Malasugui. It ruled that
respondent
was
illegally
dismissed
by
Galang.
It
reinstated the award of salary
differential to Malasugui in
addition to the 13th month pay.
Further, because of the ruling of

illegal dismissal against Galang,


the appellate court awarded
separation pay to Malasugui for
every year of continuous service
and full backwages from the
time of her dismissal up to the
time of the finality of the
judgment.
Issues:
1.Whether Malasugui is an
employee of Galang.
2. If she is an employee,
whether
Malasugui
was
constructively dismissed.
Ruling:
First and Second Issue.All three,
Labor Arbiter, the NLRC and the
CA ruled that there was an
employer-employee relationship
between Galang and Malasugui.
We do not see any reason to
rule otherwise.
The Labor
Arbiter and the NLRC both ruled
that there was no illegal
dismissal, but the Court of
Appeals reversed such findings.
We find a need to look into the
decision of the CA. When
supported
by
substantial
evidence, the findings of fact of
the CA are conclusive and
binding on the parties and are
not reviewable by this Court.,
unless the case falls under any
of the following recognized
exceptions: (1)
When
the
conclusion is a finding grounded
entirely
on
speculation,
surmises and conjectures; (2)
When the inference made is
manifestly mistaken, absurd or
impossible; (3) Where there is a
grave abuse of discretion; (4)
2011- 2012 Labor Law Case Digest Compilations
UC LLB III-B
Page 206

When the judgment is based on


a misapprehension of facts; (5)
When the findings of fact are
conflicting; (6) When the Court
of Appeals, in making its
findings, went beyond the issues
of the case and the same is
contrary to the admissions of
both appellant and appellee; (7)
When the findings are contrary
to those of the trial court [in this
case the administrative bodies
of Labor Arbiter and NLRC]; (8)
When the findings of fact are
conclusions without citation of
specific evidence on which they
are based; (9) When the facts
set forth in the petition as well
as in the petitioners' main and
reply briefs are not disputed by
the respondents; and (10) When
the findings of fact of the Court
of Appeals are premised on the
supposed absence of evidence
and
contradicted
by
the
evidence on record. That said
and done, we conclude that
there was indeed an illegal
dismissal of the respondent by
the petitioner.
114. MANILA ELECTRIC
COMPANY VS. JAN CARLO
GALA.
G.R. NO. 191288. MARCH 07,
2012
Topic: Illegal Dismissal
Facts:
Respondent Jan Carlo Gala
commenced employment with
the petitioner Meralco Electric
Company
(Meralco)
as
a
probationary lineman. On July
27, 2006, barely four months on

the job, Gala was dismissed for


alleged complicity in pilferages
of Meralcos electrical supplies,
particularly, for the incident
which took place on May 25,
2006. On that day, Gala and
other Meralco workers were
instructed to replace a worn-out
electrical pole at the Pacheco
Subdivision in Valenzuela City.
Unknown to Gala and the rest of
the crew, a Meralco surveillance
task force was monitoring their
activities
and
recording
everything with a Sony video
camera. The task force was
composed of Joseph Aguilar,
Ariel Dola and Frederick Riano.
Meralco called for an
investigation of the incident and
asked Gala to explain. Gala
denied involvement in the
pilferage, contending that even
if his superiors might have
committed a wrongdoing, he
had no participation in what
they did. He claimed that: (1) he
was at some distance away from
the trucks when the pilferage
happened; (2) he did not have
an inkling that an illegal activity
was taking place since his
supervisors were conversing
with Llanes, giving him the
impression that they knew him;
(3) he did not call the attention
of his superiors because he was
not in a position to do so as he
was a mere lineman; and (4) he
was just following instructions in
connection with his work and
had no control in the disposition
of
company
supplies
and
materials.
2011- 2012 Labor Law Case Digest Compilations
UC LLB III-B
Page 207

In a decision dated
September 7, 2007, Labor
Arbiter Teresita D. Castillon-Lora
dismissed the complaint for lack
of merit. She held that Galas
participation in the pilferage of
Meralcos property rendered him
unqualified to become a regular
employee.
Gala appealed to
the National Labor Relations
Commission
(NLRC).
In
its
decision of May 2, 2008, the
NLRC
reversed
the
labor
arbiters ruling.
Issue:
Whether there is illegal
dismissal
Ruling:
Contrary
to
the
conclusions of the CA and the
NLRC,
there
is
substantial
evidence supporting Meralcos
position that Gala had become
unfit
to
continue
his
employment with the company.
Gala was found, after an
administrative investigation, to
have
failed
to
meet
the
standards expected of him to
become a regular employee and
this failure was mainly due to
his undeniable knowledge, if
not
participation,
in
the
pilferage activities done by their
group, all to the prejudice of the
Companys interests.
Gala misses the point. He
forgets that as a probationary
employee,
his
overall
job
performance and his behavior
were being monitored and
measured in accordance with
the standards (i.e., the terms

and conditions) laid down in his


probationary
employment
agreement. Under paragraph 8
of the agreement, he was
subject to strict compliance
with, and non-violation of the
Company Code on Employee
Discipline, Safety Code, rules
and regulations and existing
policies.Par. 10 required him to
observe at all times the highest
degree
of
transparency,
selflessness and integrity in the
performance of his duties and
responsibilities, free from any
form of conflict or contradicting
with his own personal interest.
The evidence on record
established Galas presence in
the worksite where the pilferage
of company property happened.
It also established that it was
not only on May 25, 2006 that
Llanes, the pilferer, had been
seen
during
a
Meralco
operation.
He
had
been
previously noticed by Meralco
employees,
including
Gala
(based on his admission), in
past operations. If Gala had
seen Llanes in earlier projects or
operations of the company, it is
incredulous for him to say that
he did not know why Llanes was
there or what Zuiga and Llanes
were talking about. To our
mind, the Meralco crew (the
foremen and the linemen)
allowed or could have even
asked Llanes to be there during
their operations for one and only
purpose to serve as their
conduit for pilfered company
supplies to be sold to ready
2011- 2012 Labor Law Case Digest Compilations
UC LLB III-B
Page 208

buyers
worksites.

outside

Meralco

115 NORKIS DISTRIBUTORS,


INC. AND ALEX D. BUAT,
PETITIONERS, VS. DELFIN S.
DESCALLAR, RESPONDENT.
G.R. No. 185255, March 14,
2012
Topic: Grounds For Valid
Dismissal- Loss Of Trust And
Confidence
Facts:
Delfin
Descallar
was
promoted
as
branch
administrstor
of
Norkis
Distributors in its Iligan City
branch. He has supervision and
control over all the employees
and held responsible for sales
and collection. On June 20,
2002, Descallar was required to
explain within 48 hours why he
should not be sanction for being
AWOL and a written explanation
was submitted the following day.
However,
a
separate
investigation
revealed
that
Descallars absence was not
justified. As s result Descallar
was suspended for 15 days.
Pending his suspension, internal
auditors conducted a random
operational audit and found out
some anomalies.
Later,
Branch
Control
Officer Rosanna Lanzador was
informed
that
there
was
shortage in its TNT fund and
there
was
an
irregular
disbursement
of
sales
commission.
Furthermore,
Descallar has not reach his
quota and his performance was

not satisfactory. As a result,


Descallar was placed under
preventive suspension for 15
days without pay. On August 21,
2002, petitioners terminated
respondents services for loss of
trust and confidence and gross
inefficiency.The labor arbiter
found
that
Descallar
was
illegally
terminated.
Such
decision was reversed by NLRC.
Respondents
motion
for
reconsideration having been
denied, he filed with the CA a
petition
for
certiorari.
Issue:
1. Whether Decallar was validly
dismissed on the ground of loss
of trust and confidence.
2. Whether failure of respondent
in
reaching
his
quota
is
tantamount to breach of trust.
Ruling:
First Issue. We dismiss the
petition for lack of merit. Loss of
trust and confidence as a
ground for termination of an
employee under Article 282 of
the Labor Code requires that the
breach of trust be willful,
meaning it must be done
intentionally, knowingly, and
purposely, without justifiable
excuse. The basic premise for
dismissal on the ground of loss
of confidence is that the
employees concerned hold a
position
of
trust
and
confidence. It is the breach of
this trust that results in the
employers loss of confidence in
the
employee.
2011- 2012 Labor Law Case Digest Compilations
UC LLB III-B
Page 209

Now, petitioners terminated his


employment on the ground of
loss of trust and confidence for
supposedly
committing
acts
inimical
to
the
companys
interests.
However,
in
termination cases, the burden of
proof rests upon the employer
to show that the dismissal is for
a just and valid cause and
failure
to
do
so
would
necessarily mean that the
dismissal was illegal.
The
employers case succeeds or
fails on the strength of its
evidence and not on the
weakness of the employees
defense. If doubt exists between
the evidence presented by the
employer and the employee, the
scales of justice must be tilted
in favor of the latter.
Second
Issue.
The
quantum of proof required in
determining the legality of an
employees dismissal is only
substantial evidence or such
relevant
evidence
as
a
reasonable mind might accept
as adequate to support a
conclusion, even if other minds,
equally
reasonable,
might
conceivably opine otherwise.
Thus, it is incumbent upon
petitioners
to
prove
by
substantial evidence that valid
grounds exist for terminating
respondents employment on
the ground of loss of trust and
confidence.
However,
our
review of the records of this
case reveals that the CA
correctly held that petitioners

failed to discharge this burden.


To our mind, the failure to reach
the monthly sales quota cannot
be considered an intentional and
unjustified act of respondent
amounting to a willful breach of
trust on his part that would call
for his termination based on loss
of confidence. This is simply not
the willful breach of trust and
confidence
contemplated
in
Article 282(c) of the Labor
Code. Indeed, the low sales
performance could be attributed
to several factors which are
beyond respondents control. To
be a valid ground for an
employees dismissal, loss of
trust and confidence must be
based on a willful breach. To
repeat, a breach is willful if it is
done intentionally, knowingly
and
purposely,
without
justifiable
excuse.
116. RONILO TIROL, ET. AL
VS NLRC ANDA BENTHEL
DEVELOPMENT CORP.
G.R. No. 174792, March 14,
2012
Topic: Project Employees;
Backwages;
Validity
of
Quitclaims
Facts:
The Employees of Benthel
Development Corp. including
the petitioners filed a complaint
for illegal dismissal with various
money claims and damages.
The labor arbiter decided in
favor of the petitioners and
ordered Benthel to pay its 36
employees their separation pay.
2011- 2012 Labor Law Case Digest Compilations
UC LLB III-B
Page 210

They appealed from the said


decision to NLRC and it was
affirmed with modification which
includes backwages from the
date of dismissal until finality of
decision.
Private respondent, unsatisfied
with the modification made by
the NLRC, filed a motion for
reconsideration
with
the
contention that, since it has
been found by the Labor Arbiter
and affirmed in the assailed
decision that the employees
were project employees, the
computation
of
backwages
should be limited to the date of
the completion of the project
and not to the finality of the
decision. The NLRC, however,
denied the motion ruling that
private respondent failed to
establish the date of the
completion
of
the
project.Aggrieved,
private
respondent filed a Petition for
Certiorari with the CA but was
denied
The employees, including
the petitioners, upon the finality
of this Court's resolution, filed a
Motion for Execution before the
Labor Arbiter
and a writ of
execution
was
issued.
Respondent appeal the order
and prayed for the issuance of
temporary restraining order with
NLRC but the said motion was
denied.
Fifteen (15) employees
have executed Affidavits of Full
Settlement after having settled
amicably
with
the
private
respondent.
Labor Arbiter
Violeta Ortiz-Bantug issued an

Order dated July 31, 2003 for


the issuance of a writ of
execution only for the payment
of the claims of the twenty-one
(21) remaining employees in the
total amount of P4,383,225.00,
which included attorney's fees
equivalent to ten (10%) percent
of
the
sum
received
as
settlement by the fifteen (15)
employees who had earlier
settled
with
the
private
respondent.
Issue:
1. Whether are project employees.
2. To what extent they may claim
backwages.
3. Whether quitclaims are invalid
considering that it was executed
without the assistance of the
counsel.
Ruling:
Fist Issue.The issue as to
whether petitioners were project
employees or regular employees
is factual in nature. It is wellsettled in jurisprudence that
factual
findings
of
administrative or quasi-judicial
bodies, which are deemed to
have acquired expertise in
matters within their respective
jurisdictions,
are
generally
accorded not only respect but
even finality, and bind the Court
when supported by substantial
evidence. Consistent therewith
is the doctrine that this Court is
not a trier of facts, and this is
strictly adhered to in labor
cases. We [this Court] may take
cognizance of and resolve
factual issues, only when the
2011- 2012 Labor Law Case Digest Compilations
UC LLB III-B
Page 211

findings of fact and conclusions


of law of the Labor Arbiter or the
NLRC are inconsistent with those
of the CA. In the present case,
the NLRC and the CA have
opposing views.
Second Issue. Therefore,
being
project
employees,
petitioners are only entitled to
full backwages, computed from
the date of the termination of
their employment until the
actual completion of the work.
Illegally dismissed workers are
entitled to the payment of their
salaries corresponding to the
unexpired
portion
of
their
employment
where
the
employment is for a definite
period. In this case, as found by
the CA, the Cordova Reef Village
Resort
project
had
been
completed in October 1996 and
private respondent herein had
signified its willingness, by way
of concession to petitioners, to
set the date of completion of the
project as March 18, 1997;
hence, the latter date should be
considered as the date of
completion of the project for
purposes of computing the full
backwages of petitioners.
Third Issue. As to the issue
that the CA committed grave
abuse of discretion in refusing to
rule on the invalidity of the
release and quitclaims executed
by some of the employees other
than the petitioners, such is
inconsequential
as
those
employees are not parties in the
present case.

P203,726.30. Petitioner
appealed the decision of DOLE
director but was denied. The
case was elevated to the CA.
The Court of Appeals held that
petitioner was not deprived of
due process as the essence
thereof is only an opportunity to
be heard, which petitioner had
when it filed a motion for
reconsideration with the DOLE
Secretary. Petitioner sought
reconsideration of the decision
but its motion was denied.The
case was further elevated to the
Supreme Court contending that
the NLRC and not the DOLE has
jurisdiction over its claims and
that evidence presented on the
existence of employee-employer
was not considered by the CA.

117. PEOPLE'S
BROADCASTING (BOMBO
RADYO PHILS., INC.), vs. THE
SECRETARY OF THE
DEPARTMENT OF LABOR AND
EMPLOYMENT, THE
REGIONAL DIRECTOR, DOLE
REGION VII, and JANDELEON
JUEZAN,
G.R. No. 179652
May 8, 2009
Topic: Jurisdiction Of
Secretary of Labor/ Visitorial
Power of Secretary Of Labor
Or Regional Directors
Facts:
Jandeleon Juenzan filed a
case against Peoples
Broadcasting Services ( Bombo
Radio) for illegal deduction, nonpayment of service incentive
leave, and other benefits to
which an employee is entitled.
before the Department of Labor
and Employment (DOLE)
Regional Office No. VII, Cebu
City.2 The DOLE conducted plant
level inspection. It was found
out that Juanzan was hired on a
per drama participation basis
hence no employer employee
relationship. They (mgt.) has
[sic] not control of the talent if
he ventures into another
contract w/ other broadcasting
industries.
Notwithstanding the
finding, the Regional Director
ruled that respondent is an
employee of petitioner, and that
the former is entitled to his
money claims amounting to

Issue:
1. Whether the Secretary of Labor
has jurisdiction over the case.
2. Whether the DOLE Director and
the CA committed a grave
abuse of discretion.
3. Whether
there
exists
an
employer-employee relationship

2011- 2012 Labor Law Case Digest Compilations


UC LLB III-B
Page 212

Ruling:
Fist Issue. It can be
assumed that the DOLE in the
exercise of its visitorial and
enforcement power somehow
has to make a determination of
the existence of an employeremployee relationship. Such
prerogatival
determination,
however, cannot be coextensive
with
the
visitorial
and
enforcement
power
itself.
Indeed, such determination is
merely preliminary, incidental

and collateral to the DOLEs


primary function of enforcing
labor standards provisions. The
determination of the existence
of
employer-employee
relationship is still primarily
lodged with the NLRC.
Second Issue and Third
Issue: Before the DOLE may
exercise its powers under Article
128, two important questions
must be resolved: (1) Does the
employer-employee relationship
still exist, or alternatively, was
there
ever
an
employeremployee relationship to speak
of; and (2) Are there violations
of the Labor Code or of any
labor law?
In the instant case, save
for respondents self-serving
allegations and self-defeating
evidence, there is no substantial
basis to warrant the Regional
Directors
finding
that
respondent is an employee of
petitioner.
Interestingly,
the
Order of the Secretary of Labor
denying
petitioners
appeal
dated 27 January 2005, as well
as the decision of the Court of
Appeals dismissing the petition
for certiorari, are silent on the
issue of the existence of an
employer-employee relationship,
which further suggests that no
real and proper determination
the
existence
of
such
relationship was ever made by
these tribunals. Even the dissent
skirted away from the issue of
the existence of employeremployee
relationship
and
conveniently ignored the dearth
2011- 2012 Labor Law Case Digest Compilations
UC LLB III-B
Page 213

of
evidence
presented
by
respondent.
The Regional Director,
therefore, committed grievous
error in ordering petitioner to
answer for respondents claims.
Moreover, with the conclusion
that
no
employer-employee
relationship has ever existed
between
petitioner
and
respondent, it is crystal-clear
that the DOLE Regional Director
had
no
jurisdiction
over
respondents complaint. Thus,
the improvident exercise of
power by the Secretary of Labor
and
the
Regional
Director
behooves the court to subject
their actions for review and to
invalidate all the subsequent
orders they issued.
The
most
important
consideration for the allowance
of the instant petition is the
opportunity for the Court not
only to set the demarcation
between the NLRCs jurisdiction
and the DOLEs prerogative but
also the procedure when the
case involves the fundamental
challenge
on
the
DOLEs
prerogative based on lack of
employer-employee relationship.
As exhaustively discussed here,
the DOLEs prerogative hinges
on the existence of employeremployee relationship, the issue
is which is at the very heart of
this case. And the evidence
clearly
indicates
private
respondent has never been
petitioners employee. But the
DOLE did not address, while the
Court of Appeals glossed over,
the issue. The peremptory

dismissal of the instant petition


on a technicality would deprive
the Court of the opportunity to
resolve the novel controversy.
118. ERNESTO G. YMBONG,
versus - ABS-CBN
BROADCASTING
CORPORATION, VENERANDA
SY AND DANTE LUZON,
G.R. No. 184885, March 7,
2012
Topic:
Validity
Of
Prohibitions On Requiring
Employees To Resign Before
Running For A Public Office
Facts:
Ernesto
Ymbong
and
Leandro Patalinghug are both
employees of ABS-CBN. ON
January 1, 1996, ABS-CBN head
office issued a policy requiring
any employees to file his
resignation if he intends to run
for public office and leave of
absence in case of intent to
participate in any political
campaign. Ymbong told Dante
Luzon,
Assistant
Station
Manager of DYAB that he would
leave radio for a couple of
months
because
he
will
campaign for the administration
ticket. After the election it was
found out that Ymbong actually
run for public office. Patalinghug
also
filed
his
resignation
because he will run as councilor
for Naga Cebu. Unfortunately,
both lost in the election and
wished to come back to work for
ABS-CBN. ON September 14,
1998
Ymbong
received
a
memorandum terminating his
2011- 2012 Labor Law Case Digest Compilations
UC LLB III-B
Page 214

services. Thus, he filed a case


for illegal dismissal against ABSCBN. Patalinghug likewise filed
an illegal dismissal complaint
against
ABS-CBN.
ABS-CBN
contended that they are not
employees
but
talents
as
evidenced
by
their
talent
contracts. Labor arbiter ruled in
favor of the complainants.
The NLRC modified the
LAs decision and ordered the
reinstatement of Ymbong and to
pay
his
full
backwages
computed from 15 September
1998 up to the time of his actual
reinstatement. ABS-CBN moved
to reconsider the NLRC decision,
but the same was denied.
Imputing
grave
abuse
of
discretion on the NLRC, ABSCBN filed a petition for certiorari
before
the
CA.
The
CA
concluded that there is no illegal
dismissal to speak of in the
instant case as Ymbong is
considered resigned when he
ran for an elective post pursuant
to the subject company policy.
Issue:
1. Whether Policy No. HRER-016 is valid
2. Whether the March 25,
1998 Memorandum issued by
Luzon superseded Policy No. HRER-016
3. Whether Ymbong, by
seeking an elective post, is
deemed to have resigned and
not dismissed by ABS-CBN.
Ruling:
First Issue.Policy No. HRER-016 is valid.

We have consistently held


that so long as a companys
management prerogatives are
exercised in good faith for the
advancement of the employers
interest and not for the purpose
of defeating or circumventing
the rights of the employees
under special laws or under
valid agreements, this Court will
uphold them. In the instant
case, ABS-CBN validly justified
the implementation of Policy No.
HR-ER-016. It is well within its
rights
to
ensure
that
it
maintains its objectivity and
credibility and freeing itself from
any appearance of impartiality
so that the confidence of the
viewing and listening public in it
will not be in any way eroded.
Even as the law is solicitous of
the welfare of the employees, it
must also protect the right of an
employer to exercise what are
clearly
management
prerogatives. The free will of
management to conduct its own
business affairs to achieve its
purpose cannot be denied.
Second Issue.Policy No.
HR-ER-016 was not superseded
by
the
March
25,
1998
Memorandum.
Clearly,
the
March 25, 1998 Memorandum
issued by Luzon which only
requires employees to go on
leave if they intend to run for
any elective position is in
absolute
contradiction
with
Policy No. HR-ER-016 issued by
the ABS-CBN Head Office in
Manila
which
requires
the
resignation, not only the filing of
2011- 2012 Labor Law Case Digest Compilations
UC LLB III-B
Page 215

a leave of absence, of any


employee who intends to run for
public office.
Having been
issued beyond the scope of his
authority, the March 25, 1998
Memorandum is therefore void
and did not supersede Policy
No.HR-ER-016.
Also worth noting is that
Luzon in his Sworn Statement
admitted the inaccuracy of his
recollection of the company
policy when he issued the March
25, 1998 Memorandum and
stated therein that upon doublechecking of the exact text of the
policy
statement
and
subsequent confirmation with
the ABS-CBN Head Office in
Manila, he learned that the
policy required resignation for
those who will actually run in
elections because the company
wanted
to
maintain
its
independence. Since the officer
who himself issued the subject
memorandum
acknowledged
that it is not in harmony with the
Policy issued by the upper
management, there is no reason
for it to be a source of right for
Ymbong.
Third
Issue.Ymbong
is
deemed resigned when he ran
for councilor.
As Policy No.
HR-ER-016 is the subsisting
company policy and not Luzons
March 25, 1998 Memorandum,
Ymbong is deemed resigned
when he ran for councilor.
We find no merit in
Ymbongs argument that [his]
automatic termination x x x was
a blatant [disregard] of [his]

right to due process as he was


never asked to explain why he
did not tender his resignation
before he ran for public office as
mandated
by [the subject
company policy]. Ymbongs
overt
act
of
running
for
councilor of Lapu-Lapu City is
tantamount to resignation on his
part. He was separated from
ABS-CBN not because he was
dismissed but because he
resigned. Since there was no
termination to speak of, the
requirement of due process in
dismissal cases cannot be
applied to Ymbong. Thus, ABSCBN is not duty-bound to ask
him to explain why he did not
tender his resignation before he
ran
for
public
office
as
mandated
by
the
subject
company policy.
119. C. ALCANTARA & SONS,
INC., vs. COURT OF APPEALS,
LABOR ARBITER et al.
G.R. No. 155109, September
29, 2010
NAGKAHIUSANG MAMUMUO
SA ALSONS-SPFL (NAMAALSPFL), et. Al vs.
C. ALCANTARA & SONS, INC.,
et al
G.R. No. 155135
NAGKAHIUSANG MAMUMUO
SA ALSONS-SPFL (NAMAALSPFL), et al vs.
C. ALCANTARA & SONS, INC.,
Respondent.
G.R. No. 179220
Topic: Validity Of Strikes;
Validity Of Dismissal Of
Union Members Participating
In
An
Illegal
Strike;
2011- 2012 Labor Law Case Digest Compilations
UC LLB III-B
Page 216

Separation
Monetary
Participants
Strike

Pay
And
Claims
Of
In An Illegal

Facts:
C. Alcantara & Sons, Inc.,
(the Company) is a domestic
corporation engaged in the
manufacture and processing of
plywood.
Nagkahiusang
Mamumuo sa Alsons-SPFL (the
Union)
is
the
exclusive
bargaining
agent
of
the
Companys
rank
and
file
employees. The other parties to
these cases are the Union
officers
and
their
striking
members.
The Company and the
Union entered into a Collective
Bargaining Agreement (CBA)
that bounds them to hold no
strike and no lockout in the
course of its life. At some point
the parties began negotiating
the economic provisions of their
CBA but this ended in a
deadlock, prompting the Union
to file a notice of strike. After
efforts at conciliation by the
(DOLE)
failed,
the
Union
conducted a strike vote that
resulted in an overwhelming
majority of its members favoring
it. The Union went on strike.
During the strike, the
Company filed a petition for the
issuance of a writ of preliminary
injunction with prayer for the
issuance
of
a
temporary
restraining order (TRO) Ex Parte
with the NLRC to enjoin the
strikers
from
intimidating,
threatening, etc non-striking

employees. The NLRC first


issued a 20-day TRO and, after
hearing, a writ of preliminary
injunction, enjoining the Union
and its officers and members
from
performing
the
acts
complained of. But several
attempts to implement the writ
failed.
The Company, on the
other hand, filed a petition with
the Regional Arbitration Board
to declare the Unions strike
illegal.
The
Labor
Arbiter
declared the strike illegal and
held that the Union officers
should be deemed to have
forfeited their employment. With
respect to the striking Union
members, finding no proof that
they actually committed illegal
acts during the strike, the Labor
Arbiter
ordered
their
reinstatement
without
backwages. The Labor Arbiter
denied the Unions counterclaim
for lack of merit.
Terminated Union members filed
motion for their reinstatement
but both the Labor arbiter and
the company did not act on it.
Both parties appealed7 the
Labor Arbiters decision to the
NLRC. The Company impugned
the Labor Arbiters decision
insofar as it ordered the
reinstatement of the terminated
Union members. The Union, on
the other hand, questioned the
declaration of illegality of the
strike as well as the dismissal of
its officers and the order for
them to pay damages.
The NLRC affirmed the
decision that of the Labor
2011- 2012 Labor Law Case Digest Compilations
UC LLB III-B
Page 217

Arbiter insofar as the latter


declared the strike illegal,
ordered the Union officers
terminated, and directed them
to
pay
damages
to
the
Company. The NLRC ruled,
however,
that
the
Union
members involved, should also
be
terminated
for
having
committed prohibited and illegal
acts.
The Union filed a petition
for certiorari with the CA. The
CA annulled the NLRCs decision
and affirmed that of the Labor.
The Company and the Union
with its officers and members
filed separate petitions for
review of the CA decision in G.R.
155109
and
155135,
respectively.
During the pendency of
these cases, the affected Union
members filed with the Labor
Arbiter
a
motion
for
reinstatement pending appeal
by
the
parties
and
the
computation of their backwages
based on the CA decision. After
hearing, the Labor Arbiter
approved the computation and
payment of their separation pay.
Both parties appealed the
Labor Arbiters resolution to the
NLRC. The NLRC declared the
decision void but order the
company
to
pay
affected
employees accrued wages ad
13th month. Nut such decision
was later modified.
Upon the Unions petition
for certiorari with the CA,
questioning the NLRCs denial of
the terminated Union members
claim
for
separation
pay,

accrued wages, and other


benefits, the CA dismissed the
petition.
Issue:
1. Whether the Union staged an
illegal strike
2. .
Assuming the strike to be
illegal, whether the impleaded
Union
members
committed
illegal acts during the strike,
justifying their termination from
employment
3. Whether the terminated Union
members are entitled to the
payment of backwages on
account of the Companys
refusal
to
reinstate
them,
pending appeal by the parties,
from the Labor Arbiters decision
of June 29, 1999; and
4. Whether the terminated Union
members are entitled to accrued
backwages and separation pay.
Ruling:
First Issue. Yes. A strike
may be regarded as invalid
although the labor union has
complied
with
the
strict
requirements for staging one as
provided in Article 263 of the
Labor Code when the same is
held contrary to an existing
agreement, such as a no strike
clause or conclusive arbitration
clause. Here, the CBA between
the parties contained a no
strike, no lockout provision that
enjoined both the Union and the
Company from resorting to the
use of economic weapons
available to them under the law
and to instead take recourse to
voluntary arbitration in settling
2011- 2012 Labor Law Case Digest Compilations
UC LLB III-B
Page 218

their disputes. No law or public


policy prohibits the Union and
the Company from mutually
waiving their respective right to
strike and lockout, which are
otherwise available to them
under the law, in favor of
voluntary arbitration.
Second Issue.Since the
Unions strike has been declared
illegal, the Union officers can be
terminated from employment
for their actions. This includes
the shop stewards who cannot
be shielded from the coverage
of Article 264 of the Labor Code
since the Union appointed them
as such and placed them in
positions of leadership and
power over the men in their
work units. As regards the rank
and file Union members, Article
264 provides that termination
from
employment
is
not
warranted by the mere fact that
a union member has taken part
in an illegal strike. It must be
shown that such union member,
clearly identified, performed an
illegal act or acts during the
strike.The
striking
Union
members allegedly committed
the following prohibited acts:
a. They threatened, coerced,
and intimidated non-striking
employees, officers, suppliers
and
customers;
b. They
obstructed the free ingress to
and egress from the company
premises; and c. They resisted
and defied the implementation
of the writ of preliminary
injunction issued against the
strikers. The mere fact that the

criminal
complaints
against
them
were
subsequently
dismissed does not extinguish
their liability under the Labor
Code. Nor does such dismissal
bar the admission of the
affidavits,
documents,
and
photos presented to establish
their identity and guilt during
the hearing of the petition to
declare the strike illegal.
Separation pay may be given as
a form of financial assistance
when a worker is dismissed in
cases such as the installation of
labor-saving
devices,
redundancy, retrenchment to
prevent
losses,
closing
or
cessation of operation of the
establishment, or in case the
employee was found to have
been suffering from a disease
such
that
his
continued
employment is prohibited by
law. It is a statutory right
defined as the amount that an
employee receives at the time
of his severance from the
service and is designed to
provide the employee with the
wherewithal during the period
that he is looking for another
employment. It is oriented
towards the immediate future,
the transitional period the
dismissed
employee
must
undergo
before
locating
a
replacement job. As a general
rule, when just causes for
terminating the services of an
employee exist, the employee is
not entitled to separation pay
because lawbreakers should not
benefit from their illegal acts.
2011- 2012 Labor Law Case Digest Compilations
UC LLB III-B
Page 219

The rule, however, is subject to


exceptions.
Here, not only did the
Court declare the strike illegal,
rather, it also found the Union
officers to have knowingly
participated in the illegal strike.
Worse, the Union members
committed
prohibited
acts
during the strike. Thus, as the
Court has concluded in other
cases it has previously decided,
such Union officers are not
entitled to the award of
separation pay in the form of
financial assistance.
Third Issue and Fourth
Issue. Yes. While it is true that
generally
the
grant
of
separation pay is not available
to employees who are validly
dismissed, there are certain
circumstances that warrant the
grant of some relief in favor of
the terminated Union members
based on equity. Bitter labor
disputes,
especially
strikes,
always generate abhorrence
that
result
in
unpleasant
consequences. Considering this,
the striking employees breach
of restrictions imposed on their
concerted actions cannot be
regarded
as
so
inherently
wicked that the employer can
totally disregard their long years
of service prior to such breach.
The records fail to disclose any
past infractions committed by
the dismissed Union members.
Taking these circumstances in
consideration, the Court regards
the award of financial assistance
to these Union members in the

form of one-half month salary


for every year of service to the
company up to the date of their
termination as equitable and
reasonable..

latters
performance.
Aliling
tendered
his
resignation
effective October 15, 2004."
WWWEC took no action so
Aliling demanded reinstatement
claiming that San Mateo forced
him to resign.
On October 6, 2004,Alilng was
advised of his termination
effective as of that date owing
to
his
"non-satisfactory
performance"
during
his
probationary period.
Earlier, however, or on October
4, 2004, Aliling filed a Complaint
for illegal dismissal due to
forced resignation, nonpayment
of salaries as well as damages
with the NLRC against WWWEC.
The Labor Arbiter issued a
Decision
declaring
Alilings
termination as unjustified. Both
parties appealed the above
decision to the NLRC, which
affirmed the Decision in toto.
The
separate
motions
for
reconsideration
were
also
denied by the NLRC. Alilings
motion for reconsideration was
rejected by the CA.

120. ARMANDO ALILING,


Petitioner, vs. JOSE B.
FELICIANO, MANUEL F. SAN
MATEO III, JOSEPH R.
LARIOSA, and WIDE WIDE
WORLD EXPRESS
CORPORATION,
Respondents.
G.R. No. 185829, April 25,
2012
Topic:
Illegal
Dismissat;
Failure To Meet Quota As A
Ground For Just Cause To
Terminate Employment
Facts:
Wide Wide World Express
Corporation (WWWEC) offered to
employ
petitioner
Armando
Aliling (Aliling) as "Account
Executive (Seafreight Sales).
The offer came with a six (6)month
probation
period
condition with this express
caveat: "Performance during
[sic] probationary period shall
be
made
as
basis
for
confirmation to Regular or
Permanent Status." However,
instead of a Seafreight Sale
assignment,
WWWEC
asked
Aliling to handle Ground Express
(GX), a new company product.
Barely a month after, Manuel F.
San Mateo III (San Mateo),
WWWEC Sales and Marketing
Director, emailed Aliling to
express dissatisfaction with the

Issue:
1. Whether the dismissal
is valid.
2. Whether failure to meet quota is
tantamount to gross inefficiency
and just cause to terminate
employee.
3. Is petitioner a probationary
employee.

2011- 2012 Labor Law Case Digest Compilations


UC LLB III-B
Page 220

Ruling:
First Issue. No. Here, the
first
and
second
notice
requirements have not been

properly observed. The adverted


memo
would
have
had
constituted the charge sheet,
sufficient to answer for the first
notice requirement, but for the
fact that there is no proof such
letter had been sent to and
received by him. Neither was
there compliance with the
imperatives of a hearing or
conference. Suffice it to point
out that the record is devoid of
any showing of a hearing or
conference
having
been
conducted. And the written
notice of termination itself did
not
indicate
all
the
circumstances
involving
the
charge to justify severance of
employment.For
violating
petitioners right to due process,
the Supreme Court ordered the
payment to petitioner of the
amount of P30,000 as nominal
damages.
Second Issue. In fine, an
employees failure to meet sales
or work quotas falls under the
concept of gross inefficiency,
which in turn is analogous to
gross neglect of duty that is a
just cause for dismissal under
Article 282 of the Code.
However, in order for the quota
imposed to be considered a
valid productivity standard and
thereby validate a dismissal,
managements prerogative of
fixing the quota must be
exercised in good faith for the
advancement of its interest. The
duty to prove good faith,
however, rests with WWWEC as
part of its burden to show that
2011- 2012 Labor Law Case Digest Compilations
UC LLB III-B
Page 221

the dismissal was for a just


cause. WWWEC must show that
such quota was imposed in good
faith. This WWWEC failed to do,
perceptibly because it could not.
The fact of the matter is that the
alleged imposition of the quota
was a desperate attempt to lend
a semblance of validity to
Alilings illegal dismissal.
The aforequoted Section 6
of the Implementing Rules of
Book VI, Rule VIII-A of the Code
specifically
requires
the
employer
to
inform
the
probationary employee of such
reasonable standards at the
time of his engagement, not at
any time later; else, the latter
shall be considered a regular
employee. Thus, pursuant to the
explicit provision of Article 281
of the Labor Code, Section 6(d)
of the Implementing Rules of
Book VI, Rule VIII-A of the Labor
Code and settled jurisprudence,
petitioner Aliling is deemed a
regular employee as of June 11,
2004,
the
date
of
his
employment contract.
The letter-offer to Aliling
states that the regularization
standards or the performance
norms to be used are still to be
agreed upon by him and his
supervisor. Moreover, Aliling was
assigned to GX trucking sales,
an activity entirely different to
the Seafreight Sales for which
he was originally hired and
trained for. In the present case,
there was no proof that Aliling
was informed of the standards
for his continued employment,

such as the sales quota, at the


time of his engagement.

affirmed the decision of NLRC.


CA concluded that although
Logarta was given a 30-day
notice of his termination, there
was no showing that (DOLE) was
also sent a copy of the said
notice .

121. INTERNATIONAL
MANAGEMENT
SERVICES/MARILYN C.
PASCUAL, PETITIONER, VS.
ROEL P. LOGARTA,
RESPONDENT.
G.R. No. 163657, April 18,
2012
Topic:Illegal Dismissal;
day Notice Rule

Pascual argues that the


30-day notice of termination, is
not applicable because Logarta
was given the 30-day notice.
That (R.A.) No. 8042, or the
Migrant Workers and Overseas
Filipino Act of 1995 nor its
Implementing Rules do not
require the sending of notice to
the DOLE, 30 days before the
effectivity of a retrenchment of
an Overseas Filipino Worker
(OFW) based on grounds under
Article 283 of the Labor Code
and that Logarta received his
separation pay.

30-

Facts:
International Management
Services (IMS), a recruitment
agency, owned by Marilyn
Pascual, deployed Roel Logarta
to work for Petrocon Arabia
Limited (Petrocon) in
Saudi
Arabia, in connection with
general engineering services of
Petrocon for the Saudi Arabian
Oil Company (Saudi Aramco).
Logarta was employed for two
(2) years. He started to work as
Piping Designer. Due to drastic
reduction in Petrocons 1998
work allocation, Petrocon was
constrained
to
reduce
its
personnel, one of whom was
Logarta. Petrocon gave him a
30-day notice of termination.

Issue:
1. Whether
Logarta
and
his
employer are subject to the
Provisions of the Labor Code.
2. Whether Logarta was illegally
dismissed.
3. Whether the 30-day notice to
DOLE prior to retrenchment is
not applicable.

Upon his return in the


Philippines, He filed a complaint
with NLRC against IMS for illegal
dismissal. Labor arbiter favored
Logarta and ordered IMS to pay
Logarta. He filed a motion for
reconsideration, but it was
denied.
Then
he
sought
recourse before the CA which
2011- 2012 Labor Law Case Digest Compilations
UC LLB III-B
Page 222

Ruling:
First Issue. Even if Logarta
was employed by Petrocon as an
OFW in Saudi Arabia, still both
he and his employer are subject
to the provisions of the Labor
Code when applicable.
The
basic policy in this jurisdiction is

that all Filipino workers, whether


employed locally or overseas,
enjoy the protective mantle of
Philippine labor and social
legislations.
Second
Issue.
Notwithstanding the fact that
Logartas
termination
was
procedurally infirm, having not
complied
with
the
notice
requirement, still it remains to
be for a just, valid and
authorized
cause,
i.e.,
retrenchment as a valid exercise
of
management
prerogative. Court ruled that
when the dismissal is for a just
cause, the absence of proper
notice should not nullify the
dismissal or render it illegal or
ineffectual.
Instead,
the
employer should indemnify the
employee for violation of his
statutory rights
Third Issue. It is applicable.
Proper notice to the DOLE within
30 days prior to the intended
date
of
retrenchment
is
necessary
and
must
be
complied with despite the fact
that Logarta is an overseas
Filipino
worker.
Although
Logarta was duly notified of his
termination by Petrocon 30 days
before
its
effectivity,
no
allegation
or
proof
was
advanced
by
Pascual
to
establish that Petrocon ever
sent a notice to the DOLE 30
days
before
Logarta
was
terminated.

2011- 2012 Labor Law Case Digest Compilations


UC LLB III-B
Page 223

122. Ma. Corina C. Jiao, et al.


Vs. Global Business Bank,
Inc., et al.
[G.R. No. 182331, April 18,
2012]
Topic: Separation Pay
Facts:
Corina, et al., were regular
employees of the Philbank.
Philbank established a Gratuity
Pay Plan (Old Plan) and again
implemented a new Gratuity
plan. Philbank merged with
Globalbank, but operated under
the name Global Business Bank,
Inc. As a result of the merger,
complainants
respective
positions became redundant. As
their positions were included in
the redundancy declaration,
they availed of the SSP, signed
acceptance letters and executed
quitclaims in Globalbanks favor
in consideration of their receipt
of separation pay equivalent to
150% of their monthly salaries
for every year of service.
Corina,
et
al.,
filed
separate complaints for nonpayment of separation pay
before the NLRC. They asserted
that under the Old Plan, they
were entitled to an additional
50% of their gratuity pay on top
of 150% of one months salary
for every year of service they
had already received. They
insisted that 100% of the 150%
rightfully belongs to them as
their separation pay. Thus, the
remaining 50% was only half of
the gratuity pay that they are

entitled to under the Old Plan.


They argued that even if the
New Gratuity Plan were to be
followed, the computation would
be
the same. Labor Arbiter
dismissed the complaint and
ruled that Corina et al., were not
entitled to the additional 50% in
gratuity pay that they were
asking for. The 150% rate used
by Globalbank could legally
cover both the separation pay
and
the
gratuity
pay
of
complainants. The New Gratuity
Plan must be deemed to have
superseded the Old Plan. The LA
also ruled that the minimum
amount due to the petitioners
under the New Gratuity Plan, in
relation to Article 283was one
months pay for every year of
service.
Corina et al., appealed but
the NLRC dismissed the appeal
and affirmed the LAs decision.
The petitioners elevated the
case to the CA via a Petition for
Certiorari under Rule 65. CA
dismissed
the
case.
The
petitioners then moved for the
reconsideration,
which
was
denied in the second assailed
Resolution, noting the absence
of an explanation for their
failure to file a motion for
reconsideration of the assailed
NLRC decision in their petition
for certiorari.

before applying for a writ of


certiorari in the CA is enough to
deny such application.

2.

3. Whether the Issuance of SSP


revoked or supersede the new
Plan.

4. Whether
it
is
within
the
management prerogatives of
employers to come up with
separation packages that will be
given in lieu of what is provided
under the Labor Code.

Issues:
1.

Whether the separation pay


received by employees complies
with the Labor Code.

Whether
the
petitioners
unexplained failure to move for
MFR of the NLRCs resolution

2011- 2012 Labor Law Case Digest Compilations


UC LLB III-B
Page 224

Ruling:
First Issue. Yes. Court held
that he who seeks a writ of
certiorari must apply for it only
in the manner and strictly in
accordance with the provisions
of the law and the Rules. To
dispense with the requirement
of
filing
a
motion
for
reconsideration, they must show
a concrete, compelling, and
valid reason for doing so

Second Issue.Yes. Corina et


als., receipt of separation pay
equivalent to their one and a
half months salary for every

year of service as provided in


the SSP and the New Gratuity
Plan more than sufficiently
complies with the Labor Code,
which only requires the payment
of separation pay at the rate of
one month salary for every year
of service.
Third Issue.The SSP was not
intended to supersede the New
Gratuity Plan. It was issued to
make the benefits under the
New Gratuity Plan available to
employees whose positions had
become redundant because of
the merger between Philbank
and Globalbank, subject to
compliance
with
certain
requirements such as age and
length of service, and to
improve
such
benefits
by
increasing or rounding it up to
an amount equivalent to the
affected employees one and a
half monthly salary for every
year of service. The benefits, to
which
the
redundated
employees are entitled to,
including the petitioners, are the
benefits under the New Gratuity
Plan, albeit increased by the
SSP.
Fourth Issue. Article 283 of
the Labor Codeprovides only the
required minimum amount of
separation
pay,
which
employees dismissed for any of
the authorized causes are
entitled to receive. Employers,
therefore, have the right to
create plans, providing for
separation pay in an amount
over and above what is imposed
2011- 2012 Labor Law Case Digest Compilations
UC LLB III-B
Page 225

by Article 283. There is nothing


therein that prohibits employers
and employees from contracting
on the terms of employment, or
from entering into agreements
on employee benefits, so long
as they do not violate the Labor
Code or any other law, and are
not contrary to morals, good
customs, public order, or public
policy.
123. CHARLIE JAO,
PETITIONER, VS. BCC
PRODUCTS SALES INC., AND
TERRANCE TY,
RESPONDENTS.
G.R. No. 163700, April 18,
2012
Topic:
Employer-Employee
Relationsihip
Facts:
BCC Product Sales Inc.
(BCC) and its President, Terrance
Ty (Ty), installed Charlie Jao as
its comptroller in BCC to oversee
and supervise SFCs collections
and the account of BCC to
protect SFCs interest. Later, He
was barred from entering BCCs
premises. He filed a complaint
for
illegal
dismissal,
reinstatement
with
full
backwages, non-payment of
wages, damages and attorneys
fees.
Labor Arbiter dismissed
Jaos
complaint
lack
of
employer-employee relationship

between
the
parties.
Jao
appealed and NLRC ruled that
Jao was illegally dismissed and
ordering BCC to pay the unpaid
salaries, backwages and 13th
month pay, separation pay and
attorneys fees.
MFR of BCC was denied.
An appeal to the CA. CA favored
BCC and founds that employeremployee relationship does not
exist.
Issue:
1. Whether or not an
employer-employee relationship
existed between BCC and Jao.
2. If yes, whether Jaw was
Illegally dismissed.
Ruling:
First and Second Issue. There
was
no
employer-employee
relationship. Jao was not BCCs
employee
but
SFCs
representative. Hence, Charlie
Jao was not illegally dismissed.
In determining the presence or
absence
of
an
employeremployee
relationship,
the
following must exist: (a) the
selection and engagement of
the employee; (b) the payment
of wages; (c) the power of
dismissal;
and
(d)
the
employers power to control the
employee on the means and
methods by which the work is
accomplished. The last element,
the so-called control test, is the
most important element.
2011- 2012 Labor Law Case Digest Compilations
UC LLB III-B
Page 226

Jao failed to present the


terms of his employment by
BCC. The failure to present such
agreement
on
terms
of
employment
may
be
understandable and expected if
he was a common or ordinary
laborer
who
would
not
jeopardize his employment by
demanding such document from
the employer, but may not
square well with his actual
status as a highly educated
professional.
124. D.M. CONSUNJI, INC.
AND/OR DAVID M. CONSUNJI,
PETITIONERS, VS. ESTELITO
L. JAMIN, RESPONDENT.
G.R. No. 192514, April 18,
2012
Topic: Regular Employee;
Illegal Dismissal
Facts:
Estelito Jamin was a
laborer of D.M. Consunji, Inc.
(DMCI), a construction company
for almost 31 years. His contract
was renewed many times but
terminated after the SM Manila
project. He filed for illegal
dismissal against DMCI and its
President/General
Manager,
David M. Consunji. DMCI denied
liability because Jamin was hired
on a project-to-project basis and
that it submitted a report to
DOLE everytime it terminated
Jamins services.
Labor Arbiter dismissed the
complaint and ruled that Jamin
is a project employee. On

appeal
by
Jamin,
NLRC
dismissed
the
appeal
and
affirmed the Labor Arbiters
decision. Jamin moved for
reconsideration, NLRC denied.
He sought relief from the CA for
petition for certiorari via Rule
65.CA held that Jamin was a
regular
employee.
CA
declared his dismissal illegal
and ordered Jamins immediate
reinstatement with backwages,
and without loss of seniority
rights and other benefits.
Issue:
1. Whether
Jamin
is
a
Regular employee;
2. Whether DMCIs President
is
liable
for
Jamins
dismissal

Neither had the parties brought


the matter up to the CA nor with
this Court. As there is no
express finding of Mr. Consunjis
involvement
in
Jamins
dismissal, we deem it proper to
absolve him of liability in this
case.
125. BILLY M. REALDA,
PETITIONER, VS. NEW AGE
GRAPHICS, INC. AND JULIAN
I. MIRASOL, JR.
RESPONDENTS.
G.R. No. 192190, April 25,
2012
Topic:
Illegal
Dismissal;
Overtime Pay;
Facts:

Ruling:
First Issue. Yes. The Court
held that once a project or work
pool employee has been: (1)
continuously, as opposed to
intermittently, rehired by the
same employer for the same
tasks or nature of tasks; and (2)
these tasks are vital, necessary
and indispensable to the usual
business or trade of the
employer, then the employee
must be deemed a regular
employee.

Billy Realda, a former


machine operator of Graphics,
Inc., filed a complaint for illegal
dismissal. He was terminated on
the
ground
of
repeated
violations of companys rules
and
regulations
namely:
insubordination,
deliberate
slowdown of work, habitual
tardiness,
absence
without
official leave, inefficiency and
refusal render overtime work
which contributed to losses
incurred by New Graphics.

Second Issue. No. While there


is no question that the company
is liable for Jamins dismissal, we
note that the CA made no
pronouncement
on
whether
DMCIs
President/General
Manager, a co-petitioner with
the company, is also liable.

Despite the warning issued by


Graphics, Inc.on his AWOLs
during the month of April and
May, and instead of reporting to
the company to deny or to
refute
the
basis
for
recommendation of dismissal,
he absented himself from Jun.

2011- 2012 Labor Law Case Digest Compilations


UC LLB III-B
Page 227

15 to Jul. 15, 2004, which


prompted to the termination of
his employment.
Issues:
1. Whether Graphics may compel
his
employees
to
perform
overtime work
2. Whether
there
was
willful
disobedience
3. if yes, Wether it was a valid
cause for dismissal
4. Whether there was violation of
due process
Ruling:
First Issue.Yes. Rendition of
overtime work is not mandatory
but Art. 89 of the Labor Code
empower the employer to
legally compel his employees to
perform overtime work against
their will to prevent serious loss
or damage. In here Realdas
refusal to render overtime work
when
required
upon
him,
contributed to losses incurred by
Graphics Inc. Since Graphics,
Inc. is a printing press whose
production
schedule
is
sometimes flexible and varying.
It is only reasonable that
workers are sometimes asked to
render overtime work in order to
meet production deadlines.
Second Issue. Yes. Realdas
arbitrary defiance to Graphics,
Inc.s order for him to render
overtime work constitutes willful
disobedience.
For
willful
disobedience to be a valid cause
for
dismissal,
these
two
2011- 2012 Labor Law Case Digest Compilations
UC LLB III-B
Page 228

elements must concur: (1) the


employees assailed conduct
must have been willful, that is,
characterized by a wrongful and
perverse attitude; and (2) the
order violated must have been
reasonable, lawful, made known
to the employee, and must
pertain to the duties which he
had been engaged to discharge.
Third Issue. Yes. Graphics,
Inc. failed to afford Realda with
a reasonable opportunity to be
heard
and
defend
itself.
Furthermore,
there
is
no
indication that Graphics, Inc.
issued
a
second
notice,
informing the Realda of his
dismissal.
Thus
requiring
Graphics Inc., to pay Realda for
nominal damges.
126. KAKAMPI AND ITS
MEMBERS, VICTOR
PANUELOS, ET AL.,
REPRESENTED BY DAVID
DAYALO, KAKAMPI VICE
PRESIDENT AND ATTORNEYIN-FACT, PETITIONER, VS.
KINGSPOINT EXPRESS AND
LOGISTIC AND/OR MARY ANN
CO, RESPONDENTS.
G.R. No. 194813, April 25,
2012
Topic: Valid Dismissal
Facts:
Victor Pauelos et,al. were
the former drivers of Kingspoint
Express and Logistic (Kingspoint
Express), a sole proprietorship

registered in the name of Mary


Ann Co (Co) and engaged in the
business of transport of goods.
They were dismissed from
service on January 20, 2006 on
the
grounds
of
serious
misconduct, dishonesty, loss of
trust
and
confidence
and
commission of acts inimical to
the
interest
of
Kingspoint
Express. Victor Pauelos et,al.
filed a complaint alleging that
the charges against them were
fabricated
and
that
their
dismissal was prompted by
Kingspoint Express aversion to
their union activities. The Labor
Arbiter decided that there was
no anti unionism committed by
Kingspoint. NLRC affirmed the
decision of the Labor Arbiter.
Victor Pauelos et,al. appealed
to CA, CA reversed the decision
of the NLRC. Kingspoint went to
SC.
Issues:
1. Whether
illegal dismissal.

there

was

2. Whether there was antiunionism.

2011- 2012 Labor Law Case Digest Compilations


UC LLB III-B
Page 229

Ruling:
Fist Issue.The dismissal
was valid. An employer may
terminate an employment on
the
ground
of
serious
misconduct
or
willful
disobedience by the employee
of the lawful orders of his
employer or representative in
connection with his work. Willful
disobedience
requires
the
concurrence of two elements:
(1) the employee's assailed
conduct must have been willful,
that is, characterized by a
wrongful and perverse attitude;
and (2) the order violated must
have been reasonable, lawful,
made known to the employee,
and must pertain to the duties
which he had been engaged to
discharge. Both elements are
present in this case.
Second Issue. There was
no anti-unionism, because the
dismissal was for a valid cause.

You might also like